Sei sulla pagina 1di 215

PRIME NOZIONI DI ALGEBRA

Lo studio dell’Algebra lineare e della Geometria richiede un’introduzione pre-


liminare delle strutture algebriche piú generali. In particolare daremo nel presente
capitolo introduttivo le definizioni di gruppo, anello e campo. Tali strutture saranno
in seguito utili per caratterizzare la maggior parte degli oggetti utilizzati, tra i quali,
per esempio, i vettori, le matrici, gli spazi vettoriali, gli omomorfismi, le trasfor-
mazioni piane.

1. Strutture algebriche. Sia G un insieme non vuoto ed ⊕ una operazione bi-


naria chiusa rispetto agli elementi di G, cioé:
∀x, y ∈ G x ⊕ y = z ∈ G.
La coppia (G, ⊕) é detta Gruppoide. Data la generalitá delle proprietá soddisfatte
da un gruppoide, é utile arricchire la sua struttura algebrica con ulteriori assiomi.
Diciamo Semigruppo, un gruppoide (G, ⊕) che sia associativo, cioé in cui valga la
seguente proprietá (associativa):
∀x, y, z ∈ G x ⊕ (y ⊕ z) = (x ⊕ y) ⊕ z.

Un Monoide (G, ⊕) é un semigruppo che sia dotato di un elemento neutro e ∈ G


rispetto all’operazione ⊕:
∃e ∈ G tale che ∀x ∈ G e ⊕ x = x ⊕ e = x.
Definizione 1. Un Gruppo (G, ⊕) é una struttura algebrica definita dagli assiomi
del monoide ai quali si aggiunge quello dell’esistenza dell’elemento inverso di ogni
elemento in G. Piú precisamente vale il seguente:
∀x ∈ G ∃y ∈ G tale che x⊕y =y⊕x=e
dove e é l’elemento neutro in G.
Un Gruppo é detto commutativo (o abeliano, in onore del matematico norvegese
N. Abel) se vale l’ulteriore seguente proprietá:
∀x, y ∈ G x ⊕ y = y ⊕ x.
Esempio 1. Gli insiemi Z , Q, l lR e C,
l rispettivamente dei numeri interi, razionali,
reali e complessi, formano un gruppo commutativo rispetto all’operazione di somma,
cioé nel caso ⊕ = +. I numeri IN naturali non sono un gruppo rispetto alla somma,
basti pensare al fatto che ogni numero 0 6= n ∈ IN non ammette un inverso.
Esempio 2. Gli insiemi Q l ∗ , lR∗ e C
l ∗ , rispettivamente dei razionali senza lo ’zero’,
reali senza lo ’zero’ e complessi senza lo ’zero’, formano un gruppo commutativo
rispetto all’operazione di prodotto, cioé nel caso ⊕ = ×. L’eliminazione dello ’zero’
é essenziale in quanto tale elemento non avrebbe l’inverso rispetto al prodotto.

1
2 PRIME NOZIONI DI ALGEBRA

Esempio 3. L’insieme di tutte le applicazioni biettive di un insieme in sé stesso


forma un gruppo non commutativo rispetto all’operazione di composizione tra ap-
plicazioni, cioé nel caso in cui l’operazione tra due applicazioni f, g é definita come
segue: (f ⊕ g)(x) = f (g(x)).
Definizione 2. Sia A un insieme dotato di due operazioni interne ⊕ e ⊗. Il
sistema (A, ⊕, ⊗) é detto Anello se valgono i seguenti assiomi:
1. (A, ⊕) é un gruppo commutativo;
2. (A, ⊗) é un semigruppo;
3. vale la proprietá distributiva dell’operazione ⊗ rispetto a ⊕:
∀x, y, z, ∈ A x ⊗ (y ⊕ z) = (x ⊗ y) ⊕ (x ⊗ z).
Osserviamo che non é richiesto che un anello possegga l’elemento neutro rispetto
alla seconda operazione ⊗ (se tale elemento esiste l’anello é detto unitario).
Inoltre non é detto che valga la proprietá commutativa rispetto a ⊗, se essa vale
l’anello é detto commutativo.
Ancora rispetto all’operazione ⊗, non é richiesta la validitá della legge di annulla-
mento del prodotto. In altre parole, il fatto che x ⊗ y = 0 non implica necessaria-
mente che x = 0 oppure y = 0.
Infine non é richiesta l’esistenza di un elemento inverso per ciascun x ∈ A, rispetto
all’operazione ⊗.
Esempio 4. L’insieme Z rispetto alle usuali operazioni di somma e prodotto, cioé
nel caso ⊕ = + e ⊗ = × é un anello unitario in cui vale la legge di annullamento del
prodotto, ma in cui non esiste l’elemento inverso rispetto alla seconda operazione.
Esempio 5. L’insieme mZ Z = {n ∈ Z : n sia multiplo di m} é un anello rispetto
alle usuali operazioni di somma e prodotto, ma non é unitario eccetto nel caso in
cui m ∈ {−1, +1}.
Esempio 6. Sia lR[X] l’insieme di tutti i polinomi a coefficienti reali in una vari-
abile e si introducano in lR[X] le usuali operazioni di somma e prodotto tra polinomi.
Rispetto a tali operazioni lR[X] é un anello commutativo ed unitario, nel quale vale
la legge di annullamento del prodotto.
Esempio 7. In seguito vedremo un esempio di anello in cui non vale la legge di
annullamento del prodotto rispetto alla seconda operazione: é l’insieme delle matrici
quadrate di un dato ordine, in cui vengano definite le opportune operazioni. Per la
comprensione di tale esempio, rimandiamo al Capitolo relativo alle Matrici.
Definizione 3. Sia (A, ⊕, ⊗) un anello. Indichamo con e ∈ A l’elemento neutro
rispetto alla prima operazione ⊕ e sia A∗ = A − {e}. Nel caso in cui (A∗ , ⊗) sia un
gruppo, diremo che (A, ⊕, ⊗) é un Corpo. Inoltre, se in (A∗ , ⊗) vale la proprietá
commutativa (cioé se (A∗ , ⊗) é un gruppo commutativo) allora (A, ⊕, ⊗) é detto
’corpo commutativo’ o equivalentemente ’campo’.
Esempio 8. Se definiamo le usuali operazioni di somma e prodotto in Q, l lR e C,l
essi hanno la struttura di corpi commutativi. Sono anche detti campi ’scalari’, ed
ogni loro elemento x puó essere chiamato ’scalare’.

2. Relazioni d’equivalenza. Siano A e B due insiemi non vuoti. Si definisce


prodotto cartesiano A × B, l’insieme di tutte le coppie ordinate di tipo (a, b), tali
PRIME NOZIONI DI ALGEBRA 3

che a ∈ A e b ∈ B:
A × B = {(a, b) : a ∈ A, b ∈ B}.
Nel caso in cui i due insiemi A e B posseggano un numero finito di elementi, la
cardinalitá (o potenza) dell’insieme A × B é data dal prodotto delle cardinalitá dei
due insiemi che intervengono nel prodotto cartesiano.
Definizione 4. Una relazione é una terna (A, B, R) in cui A e B siano due insiemi
non vuoti e R sia un sottoinsieme del prodotto cartesiano A × B. Usualmente per
indicare una relazione tra gli insiemi A e B si preferisce utilizzare il semplice simbolo
R (sottointendendo la terna (A, B, R)). Per indicare che due elementi a ∈ A e b ∈ B
sono in relazione tra loro, si scrive aRb oppure (a, b) ∈ R.
Esempio 9. Siano A = {2, 3, 7}, B = {1, 5, 7, 9}. Si definisca R nel seguente
modo:
(a, b) ∈ R ⇐⇒ a > b.
Le coppie ordinate di elementi che soddisfano la relazione sono le seguenti:
R = {(2, 1), (3, 1), (7, 1), (7, 5)}.
Esempio 10. Siano A = B = lR. Si definisca R nel seguente modo:
(a, b) ∈ R ⇐⇒ a + b − 1 = 0.
Le coppie ordinate che soddisfano la relazione sono chiaramente tutte le coppie di
coordinate del piano (x, y) che appartengano alla retta di equazione x + y − 1 = 0.
Ad esempio (2, −1) ∈ R, (−3, 4) ∈ R etc. etc.
Poniamoci nel caso in cui A = B e cosideriamo una qualsiasi relazione (A, A, R):
1. la relazione R si dice riflessiva se, per ogni elemento a ∈ A, la coppia (a, a) ∈
R; si noti ad esempio che nessuna delle due relazioni introdotte negli Esempi
9 e 10 soddisfano alla proprietá di riflessivitá.
2. R é detta simmetrica quando accade che, per ogni a, b ∈ A tali che (a, b) ∈ R
anche (b, a) ∈ R; la relazione definita nell’esempio 9 non é simmetrica, mentre
quella introdotta nell’esempio 10 lo é.
3. R é detta transitiva se, per ogni a, b, c ∈ A tali che (a, b) ∈ R ed anche
(b, c) ∈ R allora ne segue che (a, c) ∈ R; la relazione all’esempio 9 é transitiva,
mentre la relazione all’esempio 10 non lo é.
Definizione 5. Una relazione R su un insieme non vuoto A é detta equivalenza (o
relazione d’equivalenza) se essa soddisfa alle proprietá di riflessivitá, simmetricitá
e transitivitá.
Nessuna delle relazioni introdotte negli esempi 9 e 10 é una relazione d’equivalenza.
Ecco un classico esempio di relazione d’equivalenza:
Esempio 11. Sia A l’insieme di tutte le rette del piano. Diciamo che due rette r, s
sono in relazione tra loro se esse sono parallele. É evidente che ciascuna retta r é
parallela a se stessa (R é riflessiva). Inoltre se (r, s) ∈ R allora anche (s, r) ∈ R,
poiché nel parallelismo tra rette non é vincolante l’ordine in cui esse si considerino.
Infine se (r, s) ∈ R ed anche (s, t) ∈ R allora le rette r e t sono ancora parallele,
cioé (r, t) ∈ R. Si puó allora pensare di ripartire l’insieme A in sottoinsiemi, cias-
cuno dei quali contenga esclusivamente elementi (rette) tra loro in relazione. Tali
4 PRIME NOZIONI DI ALGEBRA

sottoinsiemi sono dette classi di equivalenza. Ciascuna classe puó essere rappresen-
tata da un qualsiasi elemento ad essa appartenente, inoltre due distinte classi non
hanno alcun elemento in comune. Infine l’unione di tutte le classi di equivalenza é
esattamente l’intero insieme A.
Quanto detto per il precedente esempio puó essere esteso ad ogni insieme in cui
venga introdotta una relazione di equivalenza. L’utilitá della partizione risiede nel
fatto che lo studio di un rappresentante di una qualsiasi classe, permette di ottenere
informazioni algebriche su tutti gli elementi di tale classe.

Siano A un insieme non vuoto e R una relazione di equivalenza definita su A. Ogni


classe di equivalenza [x] puó essere rappresentata da un suo qualsiasi elemento x,
[x] = {y ∈ A : xRy}. L’insieme di tutte le classi di equivalenza {[x] : x ∈ A} é una
A
partizione di A ed é detto insieme quoziente, denotato R .
Nell’esempio delle rette parallele, ogni classe é individuata da un angolo α, quello
formato dalla retta e dall’asse delle ascisse X. Le rette appartenenti ad una stessa
classe, cioé tra loro parallele, hanno chiaramente lo stesso coefficiente angolare e
quindi lo stesso rappresentante α.

Concludiamo con un ulteriore esempio di relazione di equivalenza:


Sia A = Z , e fissiano un n ∈ IN. Definiamo R come segue: due elementi a, b ∈ Z
sono in relazione se esiste un opportuno k ∈ Z tale che a − b = kn. Verifichiamo
che R é una equivalenza:
1. a − a = 0 · n, quindi aRa (riflessiva);
2. se aRb, allora esiste k ∈ Z tale che a − b = kn; quindi b − a = (−k)n, cioé
bRa (simmetrica);
3. se aRb e bRc, allora esistono k1 , k2 ∈ Z tali che a − b = k1 n e b − c = k2 n;
quindi a − c = a − b + b − c = (k1 + k2 )n, cioé aRc (transitiva).
Le classi di equivalenza sono esattamente n ed hanno come rappresentanti i
numeri naturali {0, 1, 2, . . . , n − 1}:
• [0] = {x ∈ Z : ∃k ∈ Z , x − 0 = kn}, cioé sono tutti i multipli di n;
• [1] = {x ∈ Z : ∃k ∈ Z , x − 1 = kn}, cioé sono i numeri interi che danno resto
1 dopo la divisione per n;
• [2] = {x ∈ Z : ∃k ∈ Z , x − 2 = kn}, cioé sono i numeri interi che danno resto
2 dopo la divisione per n;
• ..............................
• ..............................
• [n − 1] = {x ∈ Z : ∃k ∈ Z , x − (n − 1) = kn}, cioé sono i numeri interi che
danno resto n − 1 dopo la divisione per n;
• [n] = [0].
Ad esempio sia n = 3, allora aRb se a − b = 3k, per un opportuno k ∈ Z . Le
classi sono
• [0] = {x ∈ Z : ∃k ∈ Z , x − 0 = 3k} = {0, 3, 6, 9, . . . , . . .}, cioé sono tutti i
multipli di 3;
• [1] = {x ∈ Z : ∃k ∈ Z , x = 3k + 1} = {1, 4, 7, 10, . . . , . . .}, cioé sono i numeri
interi che danno resto 1 dopo la divisione per 3;
• [2] = {x ∈ Z : ∃k ∈ Z , x = 3k + 2} = {2, 5, 8, 11, . . . , . . .}, cioé sono i numeri
interi che danno resto 2 dopo la divisione per 3;
PRIME NOZIONI DI ALGEBRA 5

• [3] = [0].
A
In questo caso scriviamo R = {[0], [1], [2]} e per quanto detto: 26 = (8×3+2) ∈ [2],
−8 = (−3 × 3 + 1) ∈ [1].
I VETTORI GEOMETRICI.

1. Introduzione. Siano A, B due punti del piano (o dello spazio). Un segmento


orientato AB si dice anche vettore applicato in A ed é individuato da una direzione,
che é la retta su cui giace il segmento AB, un verso, che é quello che si osserva per-
correndo il segmento AB andando da A verso B, ed un modulo, che é il numero
reale non negativo che misura la lunghezza del segmento AB. Un vettore é detto
versore di una retta, se giace su tale retta ed é di modulo 1. Due segmenti orien-
tati AB e A0 B 0 sono equipollenti se hanno la stessa direzione, lo stesso verso e lo
stesso modulo. Tale relazione tra segmenti é una equivalenza nell’insieme di tutti i
segmenti orientati del piano (o dello spazio). Per cui tutti i segmenti orientati sono
ripartiti in classi di equivalenza. Un classe di equivalenza é del tipo [AB], composta
da tutti i segmenti orientati equipollenti al segmento AB. Identifichiamo tra loro
tutti i segmenti appartenenti ad una stessa classe di equivalenza. Ciascuna classe
é detta ”vettore geometrico” o ”vettore libero” e puó essere rappresentata da un
qualsiasi segmento ad essa appartenente. Ció vuol dire che, fissato un punto del
piano (o dello spazio) O , ciascuna classe di equivalenza possiede un rappresentante
che sia applicato in O.

Si noti che da ora in avanti, fissato un punto O, ciascun vettore geometrico puó
essere applicato in O, mantenendo la sua direzione ed il suo verso.

2. Somma e prodotto per uno scalare. Siano OP e OP1 due vettori aventi la
stessa direzione e stesso verso. Definiamo somma dei vettori OP + OP1 , il vettore
avente per direzione e verso quelli dei vettori addendi, e per modulo |OP + OP1 | la
somma dei moduli |OP | + |OP1 |.
Siano ora OP e OP1 due vettori aventi stessa direzione ma verso opposto. Defini-
amo somma dei vettori OP + OP1 il vettore avente direzione dei vettori addendi,
verso coincidente con quello del vettore addendo di modulo maggiore, e modulo
pari al valore assoluto della differenza dei moduli dei vettori addendi.
Siano infine OP e OP1 due vettori con direzioni differenti. Si costruisca il par-
allelogramma di lati OP e OP1 e si indichi con Q il quarto vertice del parallelo-
gramma. Definiamo somma OP + OP1 il vettore avente per direzione quella della
retta su cui giace la diagonale OQ, per verso quello di percorrenza da O a Q e per
modulo la lunghezza della diagonale OQ del parallelogramma (disegno 1).
Notiamo anche che dalla definizione di somma OP + OP1 = OQ si ricava al
contrario anche quella di differenza tra due vettori OQ − OP = OP1 , come lato del
parallelogramma avente per diagonale OQ e per secondo lato OP (disegno 1).
Indichiamo piú semplicemente v, v1 , v2 , v3 vettori del piano (o dello spazio). La
somma tra vettori gode delle seguenti proprietá:
1) Commutativa: v1 + v2 = v2 + v1 ;
2) Associativa : (v1 + v2 ) + v3 = v1 + (v2 + v3 );

1
2 I VETTORI GEOMETRICI.

3) Esistenza dell’elemento neutro : v + 0 = 0 + v = v, dove per vettore 0 si


intende il segmento OO;
4) Esistenza dell’elemento opposto : v + (−v) = 0.
Le proprietá 1)-2)-3)-4) rendono l’insieme dei vettori geometrici del piano (o
dello spazio), rispetto all’operazione di somma, un gruppo commutativo.

Siano v un vettore e α ∈ lR. Definiamo il vettore αv come quello avente la


direzione del vettore v, il modulo pari a |α||v| e verso coincidente con quello di v,
se α > 0, opposto a quello di v, se α < 0.
Il prodotto cosı́ definito gode delle seguenti proprietá:
1) (αβ)v = α(βv), per α, β ∈ lR;
2) 1 · v = v, dove 1 é l’unitá in lR;
3) (α + β)v = αv + βv;
4) α(v1 + v2 ) = αv1 + αv2 .
L’insieme di tutti i vettori geometrici, rispetto alle operazioni di somma e prodotto
per uno scalare da noi definite, viene detto spazio vettoriale geometrico.

3. Combinazioni lineari e componenti in un riferimento ortogonale. Siano


v1 , v2 , .., vr vettori e α1 , ..., αr ∈ lR. Diciamo combinazione lineare dei vettori
v1 , .., vr a coefficienti reali α1 , .., αr il vettore v risultante dalla seguente somma:
v = α1 v1 + α2 v2 + α3 v3 + ..... + αr vr .
Diremo anche che il vettore v é scomposto nella somma dei vettori α1 v1 , α2 v2 ,...,αr vr
e che α1 , ..., αr sono le componenti del vettore v rispetto ai vettori v1 , ..., vr .

Consideriamo un sistema di riferimento cartesiano ortogonale nello spazio, di


origine O e assi coordinati X, Y, Z. Un qualsiasi punto dello spazio sará individuato
da un terna di numeri reali (a, b, c), che sono le sue coordinate rispetto al sistema di
riferimento. Il vettore v = OP ha come componenti rispetto ai tre assi coordinati,
rispettivamente vx = a, vy = b, vz = c, cioé il vettore v si puó esprimere come
combinazione lineare dei tre versori i, j, k dei tre assi coordinati nel modo seguente:
v = vx i + vy j + vz k (disegno 2).
Diremo che due vettori v = (vx , vy , vz ) e w = (wx , wy , wz ) sono uguali se e solo
se hanno le stesse componenti rispetto ai versori degli assi coordinati.
Dalla definizione di componenti appena data, si puó facilmente q dedurre che il
modulo di un vettore v di componenti (vx , vy , vz ) é dato da |v| = vx2 + vy2 + vz2 .
Siano ora v = (vx , vy , vz ) e w = (wx , wy , wz ) due distinti vettori. Costruiamo la
loro somma v + w = (vx i + vy j + vz k) + (wx i + wy j + wz k) = (vx + wx )i + (vy +
wy )j + (vz + wz )k cioé v + w ha componenti (vx + wx , vy + wy , vz + wz ).
In modo del tutto analogo si osservi che se α ∈ lR, allora il vettore αv ha per
componenti la terna (αvx , αvy , αvz ).

4. Prodotto scalare e proiezione su una retta. Siano v = OP e w = OQ.


Per convenzione si definisce angolo ϕ compreso tra i due vettori quello interno al
triangolo P OQ (disegno 3).
Si definisce prodotto scalare il seguente
v × w = |v| · |w| · cos(ϕ).
I VETTORI GEOMETRICI. 3

Si noti che tale prodotto ha come risultato uno scalare (un numero reale), inoltre
esso é commutativo, cioé v × w = w × v ed infine esso é nullo solo quando uno dei
due vettori é nullo oppure se i due vettori sono tra loro ortogonali.
Nel caso i due vettori siano considerati in uno spazio dotato di riferimento carte-
siano, allora v = (vx , vy , vz ) e w = (wx , wy , wz ), ed il prodotto scalare si presenta
nella seguente forma:
v × w = (vx i + vy j + vz k) × (wx i + wy j + wz k) = vx wx + vy wy + vz wz .
Da quest’ultima ricaviamo anche che:
vx wx + vy wy + vz wz
cos(ϕ) = q q .
vx2 + vy2 + vz2 wx2 + wy2 + wz2

Applicando tale formula si ricavano anche i ”coseni direttori” di un vettore v, cioé


i coseni degli angoli che il vettore forma con i tre assi coordinati, o meglio con i tre
versori dei suddetti assi:
vx vy vz
cos(vX) = q , cos(vY ) = q , cos(vZ) = q .
vx2 + vy2 + vz2 vx2 + vy2 + vz2 vx2 + vy2 + vz2

Siano ora v = OQ e r una retta passante per O di versore u, tale che la di-
rezione del vettore e quella della retta formino un angolo ϕ (disegno 4). Diciamo
componente di v rispetto alla retta r, la lunghezza del segmento OQ0 , dove Q0 é
la proiezione ortogonale di Q su r. Applicando ora le regole trigonometriche sulla
risoluzione dei triangoli, si osserva facilmente che
|OQ0 | = |OQ| · cos(ϕ) = v × u.
Diremo invece vettore proiezione v 0 di v su r, quello avente per modulo |OQ0 | e per
direzione e verso quelli di u, cioé:
v 0 = |OQ0 | · u = (v × u) · u.
Si noti che pur non conoscendo il versore della retta r, esso si puó ricavare se é noto
w
un vettore w parallelo a r. In tal caso u = |w| ed otteniamo
w w w
v 0 = (v × )· = (v × w) · .
|w| |w| |w|2

5. Prodotto vettoriale e componente rispetto ad un piano. Siano v, w vet-


tori. Definiamo prodotto vettoriale v ∧ w quel vettore che abbia:
1) direzione ortogonale al piano individuato dai vettori v, w;
2) modulo pari a |v||w|sen(ϕ), dove ϕ é l’angolo compreso tra i due vettori;
3) verso tale che un osservatore posto dalla parte in cui é rivolto il vettore v ∧ w,
veda il vettore v sovrapporsi al vettore w in senso antiorario (disegno 5).
Il prodotto vettoriale é nullo solo quando uno dei due vettori é nullo oppure i
due vettori sono tra loro paralleli.
Notiamo che in base alla definizione, al contrario di quanto avviene per il prodotto
scalare, il prodotto vettoriale é anticommutativo cioé: v ∧ w = −w ∧ v. In-
oltre il prodotto vettoriale é distributivo rispetto alla somma tra vettori, cioé:
v ∧ (w1 + w2 ) = (v ∧ w1 ) + (v ∧ w2 ).
Consideriamo ora un parallelogramma che abbia come lati i vettori v, w. L’area di
tale parallelogramma é A = b·h, dove b é la base da noi scelta e h é l’altezza relativa
4 I VETTORI GEOMETRICI.

a tale base. Scegliamo ad esempio il vettore v come base (disegno 6). L’altezza
relativa a v forma con v e w un triangolo rettangolo, per cui h = |w|sen(ϕ), quindi
A = |v||w|sen(ϕ) = |v ∧ w|
concludendo che l’area del parallelogramma avente per lati v, w é pari al modulo
del prodotto vettoriale v ∧ w.
Poniamoci ora nel caso in cui i due vettori appartengano allo spazio cartesiano,
per cui v = (vx , vy , vz ) e w = (wx , wy , wz ). Calcoliamo ora il loro prodotto vettori-
ale:
v ∧ w = (vx i + vy j + vz k) ∧ (wx i + wy j + wz k) =
i(vy wz − vz wy ) + j(vz wx − vx wz ) + k(vx wy − vy wx ) =

i j k

vx vy vz .

wx wy wz
Osserviamo che lo sviluppo della formula precedente dipende dall’applicazione della
proprietá distributiva del prodotto vettoriale rispetto alla somma tra vettori ed
anche dalle seguenti identitá che derivano direttamente dalla definizione di prodotto
vettoriale:
i ∧ j = −j ∧ i = k, j ∧ k = −k ∧ j = i, i ∧ k = −k ∧ i = −j
i ∧ i = 0, j ∧ j = 0, k ∧ k = 0.
Dal controesempio che segue si deduce inoltre che per il prodotto vettoriale non
vale la proprietá associativa:
i ∧ (i ∧ k) = i ∧ (−j) = k
ma
(i ∧ i) ∧ k = 0.

Sia ora v = OP e π un piano passante per O, al quale appartengano i due vettori


u1 , u2 . La proiezione di OP su π é il vettore v 0 = OP 0 , dove P 0 é la proiezione
ortogonale del punto P sul piano. Se indichiamo w il vettore P P 0 allora otteniamo
v 0 + w = v, da cui ovviamente v 0 = v − w (disegno 7). Per ottenere la proiezione v 0 é
allora sufficiente determinare il vettore w. Tale vettore non é altro che la proiezione
di v lungo la retta normale al piano e passante per O. Per ottenere tale proiezione é
necessario conoscere il versore u normale al piano e, poiché sappiamo che il vettore
u1 ∧ u2 é normale al piano, deduciamo che u = |uu11 ∧u
∧u2 | . Infine otteniamo:
2

u1 ∧ u2
w = (v × u) · u = [v × (u1 ∧ u2 )] · .
|u1 ∧ u2 |2

6. Prodotto misto. Siano u, v, w tre vettori, definiamo prodotto misto quello


scalare che si ottiene dall’esecuzione, rispettando l’associazione tramite parentesi,
dei prodotti u × (v ∧ w).
Nel caso i tre vettori siano dati in uno spazio cartesiano ortogonale, allora u =
(ux , uy , uz ), v = (vx , vy , vz ), w = (wx , wy , xz ) e svolgendo il prodotto si ottiene:
u × (v ∧ w) = ux (v ∧ w)x + uy (v ∧ w)y + uz (v ∧ w)z =

vy vz vz vx vx vy
ux
+ uy
+ uz
=
wy wz wz wx wx wy
I VETTORI GEOMETRICI. 5

ux uy uz

vx vy vz .

wx wy wz
Si noti che il prodotto misto si annulla solo quando uno dei tre vettori é nullo
oppure quando i tre vettori sono complanari, cioé quando il vettore u é ortogonale
al vettore v ∧ w.
Consideriamo ora il parallelepipedo avente per spigoli i tre vettori u, v, w. Il
volume del parallelepipedo é V0 = b · h, dove indichiamo con b l’area di una base e
con h l’altezza relativa a tale base scelta (disegno 8).
Scegliamo come base quella avente per lati i vettori v, w e sappiamo per quanto
detto in precedenza che l’area cercata é pari al modulo |v ∧ w|.
L’altezza h é la componente ortogonale del terzo spigolo del parallelepipedo lungo
la normale al piano individuato da v e w, per cui
v∧w
h=u×
|v ∧ w|
da cui
v∧w
V0 = |v ∧ w| · u × = u × (v ∧ w).
|v ∧ w|

Esempio 1. Siano u = i − 2j + 3k e v = −3j vettori nello spazio cartesiano.


Determinare:
1. i moduli dei vettori;
2. il loro prodotto scalare;
3. il coseno dell’angolo da essi formato;
4. i coseni direttori dei due vettori.

Svolg.
√ √ √
1. |u| = 1 + 4 + 9 = 14, |v| = 9 = 3.
2. u × v= (-2)(-3)=6;q
6 2
3. cos(ϕ) = √
3· 14
= 7;
4. u × i = √1 , u
14
×j = √−2
14
, u×k = √3 ,
14
v × i = 0, v × j = −1, v × k = 0.

Esempio 2. Determinare a ∈ lR tale che i vettori u = (1, −2, 1) e v = (a +


1, −a, −1) formino un angolo di π3 .

Svolg.
1 π u×v
= cos( ) = ± =
2 3 |u| · |v|
a + 1 + 2a − 1 3a
±√ √ = ±√ √
2 2
6 a + 1 + 2a + a + 1 2
6 2a + 2a + 2
da cui
√ p
6 2a2 + 2a + 2 = ±6a.
√ √
Quando 6 2a2 + 2a + 2 = +6a, e poiché 2a2 + 2a + 2 > 0 per ogni a ∈ lR, avremo
allora a > 0. Risolvendo allora
12a2 + 12a + 12 = 36a2 (a > 0)
6 I VETTORI GEOMETRICI.

otteniamo infine 2a2 − a√


− 1√= 0, cioé a = 1.
Parallelamente, quando 6 2a2 + 2a + 2 = −6a, avremo a < 0, quindi risolvendo
12a2 + 12a + 12 = 36a2 (a < 0)
abbiamo ancora 2a − a − 1 = 0, ma in questo caso a = − 12 .
2

Esempio 3. Determinare a ∈ lR tale che i vettori u = (2, a, 1) e v = (1, 0, 2)


formino un angolo di π4 .

Svolg. √
2 π u×v
= cos( ) = ± =
2 4 |u| · |v|
4
±√ .
5a2 + 25
Poiché 5a2 +25 > 0 per ogni a ∈ lR, ne deriva che l’unica uguaglianza da considerare
é √
2 4
= +√
2 2
5a + 25
√ q
da cui 10a2 + 50 = 8 e quindi 10a2 + 50 = 64. Infine otteniamo a = ± 75 .

Esempio 4. Siano u = (1, 2, −2) e v = (3, 0, 1). Determinare la proiezione ortog-


onale di v rispetto alla retta r contenente u.

Svolg. La proiezione di v su r é data da


u 1
v× = .
|u| 3
Allora il vettore proiezione é dato da
 
u u 1
v× × = (i + 2j − 2k).
|u| |u| 9

Esempio 5. Determinare il vettore v 0 proiezione di v = 2i − j + 3k sul piano XY .

Svolg. Proiettando v su XY si avrebbe v = v 0 + w, dove w é il vettore (perpendi-


colare al piano XY ) che é proiezione ortogonale di v rispetto alla retta contenente
i ∧ j, cioé
i∧j
w = v × (i ∧ j) · = 3k
|i ∧ j|2
e v 0 = 2i − j.
Esempio 6. Determinare il vettore v 0 proiezione di v = i + 2j + k sul piano π
contenente i vettori u1 = 2i + j e u2 = i − k.

Svolg. Proiettando v su π si avrebbe v = v 0 +w, dove w é il vettore (perpendicolare


al piano π) che é proiezione ortogonale di v rispetto alla retta contenente u1 ∧ u2 .
Dapprima determiniamo il versore u perpendicolare a π. Esso é dato da
u1 ∧ u2 −i + 2j − k
u= = √ .
|u1 ∧ u2 | 6
I VETTORI GEOMETRICI. 7

Quindi
2 −i + 2j − k
w = (v × u)u = √ u =
6 3
ed infine
−i + 2j − k 4 4 4
v 0 = v − w = (i + 2j + k) − ( )= i+ j+ k
3 3 3 3

Esempio 7. Determinare il volume del parallelepipedo avente per spigoli i vettori


i + j, k, −i + k.

Svolg. Il volume é dato dal valore assoluto del determinante della matrice
 
1 1 0
 0 0 1 .
−1 0 1
Tale determinante é pari a −1, quindi il volume é 1.
8 I VETTORI GEOMETRICI.

7. Esercizi.
Esercizio 1. Siano u = i − 2j + 3k, v = −3j vettori dello spazio euclideo. Deter-
minare i loro moduli, il loro prodotto scalare, il coseno dell’angolo da essi formato,
i loro coseni direttori.
Esercizio 2. Ripetere l’esercizio precedente coi seguenti vettori: u = (1, 1, 0), v =
(2, 1, 1).
Esercizio 3. Determinare la proiezione del vettore v = i − j + k su una retta
parallela al vettore w = i + 2j − k.
Esercizio 4. Determinare la componente e il vettore proiezione di v = (3, 0, 1)
sulla retta contenente il vettore (1, 2, −2).
Esercizio 5. Determinare la proiezione del vettore (1, 2, 1) sulla retta contenente
il vettore (1, 1, 1).
Esercizio 6. Determinare la proiezione del vettore (1, 1, 1) sul piano contenente i
vettori (2, 1, 0) e (1, 0, 1).
Esercizio 7. Determinare la proiezione del vettore 2i − j + 3k sul piano XY .
Esercizio 8. Utilizzare i vettori (1, −2, 0), (0, 3, 4), (1, −1, 1) per dimostrare che il
prodotto vettoriale non é associativo.
Esercizio 9. Determinare il volume del parallelepipedo avente come spigoli i vettori
(1, 1, 0), (0, 0, 1) e (−1, 0, 1).
Esercizio 10. I seguenti vettori (2, −1, 3) e (1, 1, 0), sono reciprocamente paralleli,
perpendicolari o nessuna delle due?
Esercizio 11. Determinare se i vettori v = 2i − 3j + k e w = 53 i − 52 j + 56 k sono
paralleli, perpendicolari o nessuna delle due.
Esercizio 12. Determinare h1 e h2 tali che i vettori v = 2i + j − 3k e w =
i + h1 j + h2 k risultino paralleli.
Esercizio 13. Determinare il valore del parametro h in modo tale che il vettore
(2, h, 1 − h) sia complanare con i vettori (1, 2, 1) e (3, 1, 5).
Esercizio 14. Esprimere il vettore v = (2, −1, 1) come somma di un vettore v1
parallelo al vettore w1 = (0, 1, 1) e di un vettore v2 complanare coi vettori w2 =
(1, 2, 0) e w3 = (2, 0, 1).
Esercizio 15. Siano v1 = 2i + j e v2 = i + 3j vettori del piano euclideo. Deter-
minare le componenti del versore di v1 e del versore di v2 e l’angolo compreso tra
di essi.
Esercizio 16. Siano v1 = i + j e v2 = i − 2j vettori del piano. Determinare la
componente ortogonale di v1 secondo una retta parallela e concorde col versore di
v2 ed anche le componenti del vettore proiezione ortogonale di v1 su tale retta.
Esercizio 17. Dati i vettori v = i − j + k e w = −2i + k, determinare il loro
prodotto scalare e le componenti del loro prodotto vettoriale.
Esercizio 18. Siano v1 = (1, 0, 1), v2 = (0, 1, 0), v3 = (1, 1, 2) vettori dello spazio.
Determinare le componenti del vettore proiezione ortogonale di v1 sul piano conte-
nente v2 e v3 .
MATRICI.

1. Introduzione. Una matrice é un simbolo, una tabella nella quale si possono


individuare righe e colonne. Una matrice con m righe e n colonne é un insieme di
m · n elementi scelti in un campo (per noi usualmente il campo dei numeri reali),
che siano disposti in modo ordinato nelle righe e nelle colonne nel seguente modo:
 
a11 a12 ... a1n
 a21 a22 ... a2n 
 .
 ... ... ... ... 
am1 am2 ... amn
In altre parole, l’elemento aij é quello che occupa il posto che é individuato dall’incrocio
della riga i e dalla colonna j. Piú sinteticamente scriveremo una matrice col simbolo
A = [aij ], i = 1, .., m, j = 1, ..n. Chiameremo matrici quadrate quelle tabelle in cui
m = n e le chiameremo matrici di ordine n. In esse individueremo un particolare
insieme di elementi: {a11 , a22 , a33 , ..., ann } detta diagonale principale della matrice.
Denoteremo Mmn (lR) l’insieme di tutte le matrici con m righe e n colonne, i cui ele-
menti sono scelti nei reali. Introduciamo in Mmn (lR) una operazione, detta somma
tra matrici, nel modo seguente: siano A = [aij ] e B = [bij ] matrici in Mmn (lR), la
loro somma é ancora una matrice C = [cij ] di Mmn (lR) definita da
     
c11 c12 ... c1n a11 a12 ... a1n b11 b12 ... b1n
 c21 c22 ... c2n   a21 a22 ... a2n   b21 b22 ... b2n 
 = + =
 ... ... ... ...   ... ... ... ...   ... ... ... ... 
cm1 cm2 ... cmn am1 am2 ... amn bm1 bm2 ... bmn
 
a11 + b11 a12 + b12 ... a1n + b1n
 a21 + b21 a22 + b22 ... a2n + b2n 
 
 ... ... ... ... 
am1 + bm1 am2 + bm2 ... amn + bmn
quindi ogni elemento cij di C é la somma degli elementi omologhi in A e B.
Siano A, B, C matrici di Mmn (lR), rispetto all’operazione di somma sono soddis-
fatte le seguenti proprietá:
1) (A + B) + C = A + (B + C), proprietá associativa;
2) Esiste l’elemento neutro della somma, cioé la matrice
 
0 0 ... 0
 0 0 ... 0 
0= 
 ... ... ... ... 
0 0 ... 0
tale che A + 0 = 0 + A = A;
3) ogni matrice A = [aij ] possiede l’opposta B = −A = [−aij ], tale che A+B = 0;
4) A + B = B + A, proprietá commutativa.

1
2 MATRICI.

Siano ora α ∈ lR e A = [aij ] ∈ Mmn (lR). Moltiplicare lo scalare α per la


matrice A vuol dire moltiplicarlo per ogni elemento di A ottenendo cosı́ la matrice
αA = [αaij ].

2. Prodotto righe per colonne. Consideriamo ora due matrici A = [aij ] ∈


Mmn (lR) e B = [bij ] ∈ Mnq (lR). Si definisce prodotto righe per colonne di A e B
quella operazione che abbia come risultato la matrice C = [cij ] ∈ Mmq (lR), tale che
l’elemento di posto (r, s) di C sia uguale a
n
X
crs = ark bks = ar1 b1s + ar2 b2s + ar3 b3s + ... + arn bns
k=1

In sostanza l’elemento crs é la somma di tutti i prodotti degli elementi della riga r
della prima matrice, ciascuno con il corrispondente elemento della colonna s della
seconda matrice.
Per esempio:
 
  1 −1 0  
1 2 3 3 −2 5
· 1 1 1 = .
1 0 1 1 −2 1
0 −1 1
Tale prodotto non é definito se A ∈ Mmn e B = Mtq con n 6= t.
Il prodotto righe per colonne gode delle seguenti proprietá:
siano A ∈ Mmn (lR), B ∈ Mnn (lR), C ∈ Mnt (lR) e α ∈ lR.
1) A(BC) = (AB)C, associativa;
2) A(B + C) = AB + AC, (A + B)C = AC + BC, distributiva rispetto alla
somma;
3) α(AB) = A(αB).
Inoltre sia Mn (lR) l’insieme di tutte le matrici quadrate di ordine n. Esiste una
matrice I ∈ Mn (lR) che si comporta come elemento neutro per il prodotto righe
per colonne, cioé per ogni A ∈ Mn (lR) si ha AI = IA = A. Tale matrice é quella
che ha 1 su tutta la diagonale principale e zero altrove:
 
1 0 ... ... 0
 0 1 ... ... 0 
 
I=  0 0 1 ... ...  .

 ... ... ... ... ... 
0 0 ... ... 1
Si noti infine che non vale la proprietá commutativa per il prodotto righe per
colonne, cioé non é regola generale che AB = BA.

3. Determinanti. Sia A = [aij ] ∈ Mn (lR) una matrice quadrata di ordine n.


Esiste una corrispondenza ϕ : Mn (lR) −→ lR che associa ad ogni matrice uno
ed un solo valore in lR. Il valore ϕ(A) ∈ lR é detto determinante di PA ed indicato
ϕ(A) = det(A) = |A|. La corrispondenza ϕ é definita da: det(A) = σ (−1)σ a1σ(1) ·
a2σ(2) · · · anσ(n) al variare di σ tra le n! permutazioni di {1, 2, 3, .., n}. Il segno di
ogni addendo é (−1)σ , che é +1 se σ é una permutazione pari, ed é −1 se σ é
dispari.
A partire da tale definizione vogliamo introdurre alcuni metodi pratici per il
calcolo dei determinanti, in modo induttivo, da n = 1 fino ad un qualsiasi n.
n=1: A = [a], a ∈ lR ed in questo caso si definisce det(A) = a.
MATRICI. 3

 
a11 a12
n=2: , e det(A) = a11 ·a22 −a12 ·a21 cioé dapprima moltiplichiamo
a21 a22
tra loro gli elementi della diagonale principale. Quindi moltiplichiamo tra loro
gli elementi della diagonale secondaria cambiando il segno di questo ultimo
prodotto. Infine sommiamo i due prodotti ottenuti.
 
a11 a12 a13
n=3:  a21 a22 a23 . Per calcolare il determinante usiamo il seguente
a31 a32 a33
metodo, detto ’di Sarrus’. Ricopiamo le prime due colonne a destra della
terza:  
a11 a12 a13 a11 a12
 a21 a22 a23 a21 a22 
a31 a32 a33 a31 a32
Individuiamo cosı́ tre diagonali ”principali” e tre ”secondarie”. Eseguiamo i
prodotti degli elementi di ciascuna diagonale, cambiando il segno al prodotto
degli elementi di ogni diagonale ”secondaria”. Infine sommiamo i prodotti
ottenuti:
det(A) = a11 a22 a33 + a12 a23 a31 + a13 a21 a32 +
−a13 a22 a31 − a11 a23 a32 − a12 a21 a33 .
n ≥ 4: In questo caso abbiamo bisogno di introdurre alcune definizioni:
Sia A ∈ Mmn (lR), diciamo sottomatrice di A una qualsiasi matrice ottenuta
utilizzando gli elementi che appartengano contemporaneamente ad un numero
p di righe di A ed ad un numero q di colonne di A. Tale sottomatrice avrá
ordine (p, q).
Diremo minore di ordine p di A, una qualsiasi sottomatrice quadrata di A
ottenuta intersecando p righe e colonne di A.
Diremo minore (complementare) di A, un qualsiasi minore di ordine n − 1,
ottenuto quindi cancellando una riga ed una collonna in A.
Sia Mij il minore di ordine n − 1 di A ottenuto cancellando gli elementi della
riga i e della colonna j in A. Diremo complemento algebrico dell’elemento
aij ∈ A ∈ Mn (lR), il seguente scalare: (−1)i+j · det(Mij ).
 
1 0 1 0
Esempio 1. Sia A =  2 1 2 0 .
3 1 1 1

Svolg. Le seguenti sono sottomatrici di A:


 
1 0 1
2 1 2
ottenuta dalle prime 2 righe e dalle prime 3 colonne di A,
 
1 0
2 0
ottenuta dalle prime due righe e dalla terza e quarta colonna di A,
 
1 0
1 1
ottenuta dalla prima e terza riga e dalla terza e quarta colonna di A.
4 MATRICI.

Inoltre sono minori di ordine 2 i seguenti



1 0
= 0, 1 0 = 1.


2 0 1 1

Esempio 2.
 
1 2 3
A= 0 1 0 .
2 2 1

Svolg. Il complemento algebrico di a13 = 3 é



4 0 1

A13 = (−1) = −2.
2 2
Il complemento algebrico di a21 = 0 é

2 3
A21 = (−1)3 = 4.
2 1

Teorema 1. (Teorema di Laplace). Sia A una matrice quadrata di un


certo ordine n. Il determinante di A si ottiene moltiplicando gli elementi di
una qualsiasi riga (o colonna) di A per i rispettivi complementi algebrici e
sommando tutti i prodotti ottenuti.
Esempio 3.
 
1 2 1 0
 3 1 0 1 
A=
 1
.
1 1 1 
3 2 1 0

Svolg. Applicando il teorema di Laplace, rispetto alla prima riga, otteniamo:



1 0 1 3 0 1 3 1 1 3 1 0

det(A) = (+1) 1 1 1 +(−2) 1 1 1 +(+1) 1 1 1 +(0) 1 1 1
=

2 1 0 3 1 0 3 2 0 3 2 1

−2 + 10 − 4 + 0 = 4.

Come immediata conseguenza del teorema di Laplace, otteniamo che una matrice
quadrata che abbia una riga o una colonna composta da elementi tutti nulli, avrá
determinante nullo.
Una matrice con determinante nullo é detta matrice singolare (al contrario é
detta non singolare).
Si noti che il teorema di Laplace vale per tutte le matrici quadrate di qualsiasi
ordine, ovviamente per quelle di ordine 2 e 3 si preferisce usare i metodi preceden-
temente esposti, poiché piú veloci.
MATRICI. 5

4. Dipendenza ed indipendenza tra righe. Consideriamo la matrice


 
a11 a12 ... ... a1n
A =  ... ... ... ... ...  ∈ Mmn (lR).
am1 am2 ... ... amn
Isoliamo ciascuna riga della matrice come una n-upla ordinata di scalari:
u1 = (a11 , a12 , ..., a1n )
u2 = (a21 , a22 , ..., a2n )
......................
ui = (ai1 , ai2 , ..., ain )
......................
um = (am1 , am2 , ..., amn ).
Definiamo le seguenti operazioni:
1. sommare due righe (due n-uple) vuol dire sommare ciascun elemento di una
con il corrispondente dell’altra, ad esempio
u1 + u2 = (a11 + a21 , a12 + a22 , ..., a1n + a2n ).
2. moltiplicare una riga per uno scalare α ∈ lR vuol dire moltiplicare ciascun
elemento della riga per α, ad esempio
αu1 = (αa11 , αa12 , ..., αa1n ).
Siano u1 , .., ur righe di una matrice. Diremo che u1 , .., ur sono linearmente dipen-
denti se esistono α1 , .., αr ∈ lR non tutti nulli, tali che α1 u1 + α2 u2 + ... + αr ur =
(0, 0, 0, ..., 0) (la n-upla nulla). Supponiamo che sia αi 6= 0. Allora
αi ui = −α1 u1 − α2 u2 .. − αi−1 ui−1 − αi+1 ui+1 − ... − αr ur
da cui
α1 α2 αi−1 αi+1 αn
ui = − u1 − u2 − ... − ui−1 − ui+1 − ... − un
αi αi αi αi αi
cioé, se le righe u1 , .., ur sono linearmente dipendenti, allora almeno una di esse si
puó esprimere come combinazione lineare delle altre.
Diremo invece che le u1 , .., ur sono linearmente indipendenti se vale la seguente:
α1 u1 +α2 u2 +...+αr ur = (0, 0, 0, ..., 0) se e solo se α1 = α2 = α3 = ... = αr = 0.

 
1 2 0
 3 1 0 
Esempio 4. Sia A =   2 −1
 con righe u1 = (1, 2, 0), u2 = (3, 1, 0), u3 =
0 
0 0 1
(2, −1, 0), u4 = (0, 0, 1).

Svolg. Le righe u1 , u2 , u3 sono linearmente dipendenti infatti:


(−1)u1 + (1)u2 + (−1)u3 = (0, 0, 0)
da cui u3 = u2 − u1 , cioé u3 é combinazione lineare di u2 e u1 .
Le righe u2 , u3 , u4 sono linearmente indipendenti, infatti se supponiamo a2 u2 +
a3 u3 + a4 u4 = (0, 0, 0), otteniamo (3a2 + 2a3 , a2 − a3 , a4 ) = (0, 0, 0) e quindi a2 =
a3 = a4 = 0.
6 MATRICI.

5. Rango di una matrice. Sia A una matrice con m righe e n colonne. Diciamo
rango di A l’ordine massimo di un minore avente determinante non nullo. In altre
parole, diciamo che la matrice A ha rango p, e scriviamo rank(A) = p, se esiste
almeno un minore di ordine p di A che abbia determinante non nullo, ed inoltre
ogni mnore di ordine p + 1 abbia determinante nullo.
Ovviamente rank(A) ≤ min{m, n}.

Esempio 5.  
2 1 0
 3 1 1  ha rango 3
−1 0 1
 
2 1 0
 3 1 1  ha rango 2
1 0 1
 
2 1 1 0
ha rango 2
3 2 1 0
 
2 1 1 0
ha rango 1.
4 2 2 0

Diciamo orlato di un minore M di ordine k di A, quel minore di ordine k + 1 di


A che si ottiene aggiungendo a M gli elementi di una riga e di una colonna di A.
Vale il seguente:
Teorema 2. (Teorema degli orlati). Sia M un minore di ordine p di A con
determinante non nullo. Se tutti gli orlati di M hanno determinante nullo, allora
rank(A) = p.
Talvolta calcolare il rango di una matrice puó non essere facile come negli esempi
precedenti. Ci proponiamo ora di introdurre quelle che sono chiamate ”operazioni
elementari” sulle righe di una matrice. Tali operazioni, pur modificando la ma-
trice di partenza, ne lasciano invariato il rango. Lo scopo é quello di trasformare
una matrice in un’altra di piú facile lettura, al fine di determinare il rango della
trasformata, sicuri che esso sia anche il rango della matrice di partenza.

Le operazioni elementari sulle righe sono di tre tipi:


1. Scambio di posizione tra
 due righe;
  
1 2 0 1
per esempio la matrice  3 2  si trasforma nella  3 2  dopo lo scambio
0 1 1 2
tra la prima e la terza riga. Denoteremo lo scambio tra la riga i e la riga j
con il simbolo Ri ↔ Rj .
Nel caso di matrici quadrate, ció comporta che il valore assoluto del de-
terminante della matrice finale é identico a quello della matrice iniziale, ma i
segni dei due determinanti sono discordi.
2. Moltiplicazione di una riga peruno scalare α 6= 0;  
1 2 3 6
per esempio la matrice  3 2  si trasforma nella  3 2  dopo aver scam-
0 1 0 1
biato la prima riga con essa stessa moltiplicata per 3. Denoteremo lo scambio
MATRICI. 7

di una riga i con la stessa riga moltiplicata per lo scalare α, con il simbolo
Ri → αRi .
Nel caso di matrici quadrate, ció comporta che il determinante della matrice
finale é pari a quello della matrice iniziale, moltiplicato per α.
3. Sostituzione di un vettore riga con se stesso, sommato ad un altro vettore riga
che sia moltiplicato per  uno scalare
 non nullo;  
1 2 13 10
per esempio la matrice  3 2  si trasforma nella  3 2  dopo aver
0 1 0 1
scambiato la prima riga con la somma di essa stessa e della seconda riga
moltiplicata per 4. Denoteremo lo scambio della riga j con la somma della riga
j e della riga i moltiplicata per uno scalare α, con il simbolo Rj → Rj + αRi .
Nel caso di matrici quadrate, i determinanti della matrice finale e di quella
iniziale sono esattamente identici.
Indichiamo con R1 , . . . , Rn i vettori riga di una matrice A quadrata. In simboli
potremmo indicare A = (R1 , . . . , Rn ). Quanto detto in relazione al calcolo dei
determinanti dopo aver effettuato operazioni elementari su A puó essere riassunto
come segue:
1. det(R1 , . . . , Ri , . . . , Rj , . . . , Rn ) = −det(R1 , . . . , Rj , . . . , Ri , . . . , Rn );
2. dalla precedente abbiamo anche

det(R1 , . . . , Ri , . . . , Ri , . . . , Rn ) = −det(R1 , . . . , Ri , . . . , Ri , . . . , Rn )

per cui una matrice con 2 righe uguali ha determinante nullo;


3. det(R1 , . . . , Ri +Ri0 , . . . , Rn ) = det(R1 , . . . , Ri , . . . , Rn )+det(R1 , . . . , Ri0 , . . . , Rn );
4. det(R1 , . . . , λRi , . . . , Rn ) = λdet(R1 , . . . , Ri , . . . , Rn )
5. det(R1 , . . . , Ri , . . . , λRi + Rj , . . . , Rn ) = λdet(R1 , . . . , Ri , . . . , Ri , . . . , Rn ) +
det(R1 , . . . , Ri , . . . , Rj , . . . , Rn ) = det(A).
Diciamo elemento speciale in una riga, ogni elemento non nullo tale che nella
propria colonna,
 al di sotto di esso, vi siano solo elementi nulli. Per esempio nella
1 2 3
matrice  3 0 4  l’elemento a12 = 2 é speciale per la prima riga.
1 0 −1
Diremo che una matrice é ridotta per righe se in ogni riga vi é almeno un elemento
speciale (il controllo non va ovviamente fatto per gli elementi dell’ultima riga, poiché
essi non presentano altre righe al di sotto). Si dimostra che il rango di una matrice
ridotta per righe é pari al numero di righe non nulle della matrice stessa.
Quindi per calcolare il rango di una matrice potremmo operare nel modo seguente:
se la matrice é giá ridotta per righe, calcoliamo semplicemente il numero di righe
non nulle (esso é il rango della matrice); altrimenti, per prima cosa riportiamo la
matrice ad una forma ridotta, tramite le operazioni elementari sopra esposte, quindi
calcoliamo il numero di righe non nulle della forma ridotta ottenuta.
 
1 1 2 0 −1
Esempio 6. La matrice  0 9 2 −5 5  é ridotta per righe
0 4 0 8 7

Svolg. Infatti: a11 = 1 é l’elemento speciale della prima riga, a23 = 2 é l’elemento
speciale della seconda riga. Il rango della matrice é tre.
8 MATRICI.

 
1 1 2 0 −1
Esempio 7. La matrice  0 2 1 2 2  non é ridotta per righe.
1 3 3 2 1

Svolg. Operiamo nel modo seguente:  


1 1 2 0 −1
1) R3 → R3 − R1 e la matrice diventa  0 2 1 2 2 ;
0 2 1 2 2
 
1 1 2 0 −1
2) R3 → R3 − R2 e la matrice diventa  0 2 1 2 2  che presenta
0 0 0 0 0
l’elemento speciale a11 sulla prima riga e quello a22 sulla seconda. Quindi é ri-
dotta ed il suo rango é 2.

Si noti che la possibilitá di trasformare una riga della matrice di partenza in


una riga nulla, dipende esclusivamente dal fatto che tale riga sia una combinazione
lineare di altre righe della matrice (in caso contrario non riusciremmo in alcun
modo, attraverso le operazioni elementari, ad annullare tale riga). In tal senso
possiamo dare una ulteriore definizione di rango di una matrice: esso é il numero
massimo di righe tra loro linearmente indipendenti.
 
1 1 2 0 −1
Esempio 8. La matrice  −3 −3 −6 0 3  ha rango 1, poiché ciascuno
2 2 4 0 −2
dei vettori riga
 dipende dagli altri.

1 3 6 0 0
La matrice  0 2 1 0 2  ha rango 3 poiché le tre righe sono tutte tra loro
1 3 3 3 1
indipendenti.

Queste ultime considerazioni ci fanno comprendere come, nel caso di matrici


quadrate, si possa riconoscere in talune occasioni, se il determinante della matrice
é nullo oppure diverso da zero. In particolare sia A una matrice quadrata di ordine
n. Se una riga é tutta nulla, allora il suo determinante é nullo (il rango non potrá
essere n). Se una riga é proporzionale ad un’altra il determinante é ancora nullo
(con opportune trasformazioni sulle righe, una delle due tra loro proporzionali,
diventa nulla). Piú in generale se una riga é combinazione lineare di altre righe, il
determinante é nullo.
Quanto appena detto viene anche utilizzato per dimostrare il seguente
Teorema 3. (Secondo teorema di Laplace) In una matrice quadrata, molti-
plicando gli elementi di una riga (o di una colonna) per i complementi algebrici di
un’altra riga (o colonna) e sommando i prodotti ottenuti, si ottiene zero.

6. Trasposta di una matrice, matrici invertibili. Sia A una matrice con m


righe e n colonne, A ∈ Mmn (lR). Diciamo matrice trasposta di A, e la indichiamo
AT , quella matrice con n righe e m colonne, AT ∈ Mnm (lR), tale da avere come
righe e come colonne, rispettivamente le colonne e le righe di A. In altre parole,
se aij é l’elemento di A che occupa il posto di riga i e colonna j, esso sará anche
elemento di AT , ma occuperá il posto di riga j e colonna i.
MATRICI. 9

 
1 1 2 0 −1
Esempio 9. A =  0 2 1 2 2 ,
1 3 3 2 1
 
1 0 1
 1 2 3 
T
 
A = 2 1 3 .

 0 2 2 
−1 2 1

Una matrice A quadrata é detta simmetrica se A = AT . Inoltre, per ogni matrice


quadrata A si ha che det(A) = det(AT ).
Diciamo matrice aggiunta della matrice A, e la indichiamo Agg(A), quella ma-
trice che abbia come elemento di posto (i, j), il complemento algebrico dell’elemento
di A di posto (j, i).

 
1 2 1
Esempio 10. Sia A =  3 1 0 .
0 1 1
   
1 3 0 1 −1 −1
Svolg. Allora AT =  2 1 1  e Agg(A) =  −3 1 3 .
1 0 1 3 −1 −5

Diremo che una matrice A, quadrata di ordine n, é invertibile se esiste una


matrice quadrata B di ordine n tale che A · B = I, dove I é la matrice identica di
ordine n, avente 1 su tutta la diagonale principale e zero in ogni altra posizione.
Per esempio:
 
2 0 −1
A =  7 3 32  é invertibile, infatti esiste
1 3 4
 5   
−2 1 −1 1 0 0
B =  53 6 −3 10 3
 tale che A · B =  0 1 0 .
−6 2 −2 0 0 1
Le matrici quadrate invertibili sono tutte e sole quelle non singolari. Inoltre, sia
A invertibile tale che A · B = I. La matrice B é detta inversa della matrice A,
indicata anche con B = A−1 .
Osservazione 1. Siano A, B matrici quadrate di ordine n. Valgono le seguenti:
1. (A · B)T = B T · AT ;
2. (A + B)T = AT + B T ;
3. (A · B)−1 = (B)−1 · (A)−1 , nel caso entrambe A e B siano non singolari;
4. det(A) = det(AT ): infatti sia (ai1 , . . . , ain ) la i-esima riga di PnA e siano
Ai1 , . . . , Ain i rispettivi complementi algebrici. Allora det(A) = j=1 aij Aij ,
per il metodo di Laplace utilizzato rispetto alla i-esima riga. Consideriamo
ora la trasposta. Essa presenta nella colonna i-esima esattamante gli elementi
(ai1 , . . . , ain ) con i medesimi complementi algebrici Ai1 , . . . , Ain . Quindi, svol-
gendo il calcolo del determinante tramite il metodo di Laplace rispetto alla
i-esima colonna si ottiene l’asserto.
10 MATRICI.

6.1. Calcolo dell’inversa. Sia A una matrice quadrata non singolare di ordine
n. Per ottenere l’inversa di A si costruisca dapprima la matrice aggiunta di A.
Quindi si divida ciascun elemento dell’aggiunta per il determinante di A. La ma-
trice ottenuta é esattamente A−1 . Per dimostrarlo, costruiamo la matrice Agg(A)
det(A) e
−1
verifichiamo che essa coincide con A . Indichiamo con Aij il complemento alge-
brico dell’elemento aij in A. Allora l’aggiunta di A é:
 
A11 A21 . . . . . . An1
 A12 A22 . . . . . . An2 
 
 ... ... ... ... ... 
 
 ... ... ... ... ... 
A1n A2n . . . . . . Ann
e sfruttando entrambi i teoremi di Laplace abbiamo che
   
A11 A21 . . . . . . An1 a11 a12 . . . . . . a1n
 A12 A22 . . . . . . An2   a21 a22 . . . . . . a2n 
   
 ·  ... ... ... ... ...
 ... ... ... ... ...   =
 
 ... ... ... ... ...   ... ... ... ... ... 
A1n A2n . . . . . . Ann an1 an2 . . . . . . ann
 
det(A) 0 0 ... 0

 0 det(A) 0 ... 0 

 0 0 det(A) . . . 0  = det(A) · I
 
 ... ... ... ... ... 
0 0 0 . . . det(A)
Agg(A)
da cui A · det(A) = I.

 
1 1 0
Esempio 11. Sia A =  2 0 1 .
1 1 1

Svolg. Poiché det(A) = −2, essa é invertibile. Si ha che


   
1 2 1 −1 −1 1
AT =  1 0 1  , Agg(A) =  −1 1 −1  .
0 1 1 2 0 −2
Quindi
1 1
− 12
 
Agg(A)  2 2
A−1 = = 1
2 − 12 1
2

−2
−1 0 1
ed infatti
  1 1
− 12
   
1 1 0 2 2 1 0 0
A · A−1 = 2 0 1 ·
  1
2 − 12 1 
2 =  0 1 0 .
1 1 1 −1 0 1 0 0 1
MATRICI. 11

7. Le matrici elementari. Una matrice quadrata di ordine n é detta elementare


se essa é ottenuta dalla matrice identica di ordine n tramite una sola operazione
elementare. Per esempio la matrice elementare
 
1 0 0
E= 0 0 1 
0 1 0
é ottenuta dalla matrice identica I di ordine 3, scambiando tra di loro la seconda e
la terza riga.
Per quanto detto sulle operazioni elementari su matrici quadrate, segue facilmente
che ogni matrice elementare ha determinante non nullo, quindi é invertibile, e la
sua inversa é ancora una matrice elementare.
Le matrici elementari sono utilizzate per determinare in modo immediato le trasfor-
mazioni per righe su qualsiasi matrice (non necessariamente quadrata). Piú pre-
cisamente, se E é una matrice elementare di ordine n, ottenuta con una qualche
trasformazione elementare, e se A é una matrice con n righe, allora la matrice E · A
é la trasformata di A tramite la medesima operazione con la quale si é ottenuta E.
Per tornare alll’esempio precedente, sia
 
1 2 0 1
A= 1 1 1 2 
0 1 1 1
allora la matrice
     
1 0 0 1 2 0 1 1 2 0 1
E·A= 0 0 1 · 1 1 1 2 = 0 1 1 1 
0 1 0 0 1 1 1 1 1 1 2
si ottiene dalla A scambiando tra loro la seconda e la terza riga (R2 ←→ R3 ).
 
4 0 0
Esempio 12. La matrice elementare E =  0 1 0  si ottiene dalla matrice
0 0 1 
1 2 0 1
identitá moltiplicando la prima riga per 4. Se A =  1 1 1 2  allora la
     0 1 1 1
4 0 0 1 2 0 1 4 8 0 4
matrice E · A =  0 1 0  ·  1 1 1 2  =  1 1 1 2  si ottiene dalla
0 0 1 0 1 1 1 0 1 1 1
A sostituendo la prima riga con se stessa moltiplicata per 4 (R1 → 4R1 ).
 
1 0 4
Esempio 13. La matrice elementare E =  0 1 0  si ottiene dalla matrice
0 0 1
identitá sostituendo la prima riga con la somma di se stessae della terza riga
1 2 0 1
moltiplicata per 4 (R1 → R1 + 4R3 ). Se A =  1 1 1 2  allora la matrice
0 1 1 1
     
1 0 4 1 2 0 1 1 6 4 5
E · A =  0 1 0  ·  1 1 1 2  =  1 1 1 2  si ottiene dalla A
0 0 1 0 1 1 1 0 1 1 1
12 MATRICI.

sostituendo la prima riga con se stessa con la somma di se stessa e della terza riga
moltiplicata per 4 (R1 → R1 + 4R3 ).

Grazie alle proprietá delle operazioni elementari si puó facilmente notare che se
A é una matrice quadrata e E é una matrice elementare dello stesso ordine, allora
det(EA) = det(E)det(A).
Inoltre se E1 , E2 , ..., Ek sono matrici elementari, allora (E1 · E2 · .... · Ek )A é una
matrice che si ottiene dopo aver effettuato sulla A ordinatamente tutte le trasfor-
mazioni individuate dalle matrici Ek , Ek−1 ,...,E1 :
A → Ek A −→ Ek−1 Ek A −→ ..... −→ E1 E2 ....Ek A.

Osservazione 2. Sia A una qualsiasi matrice quadrata non singolare. Sappiamo


che operando sulle righe si puó ottenere la sua forma a gradini, che nel caso parti-
colare di matrici quadrate é esattamente una forma triangolare superiore:
 0
a11 a012 a013 ... a01n

...
 0 a022 ... ... ... a02n 
0
a03n 
 
 0 0 a33 ... ...
 .
 ... ... ... ... ... ... 
... ... a0n−1n−1 a0n−1n 
 
 0 0
0 0 0 ... ... a0nn
In generale non lo riteniamo utile (per i nostri fini), ma potremmo operare ulte-
riori trasformazioni con lo stesso procedimento, con lo scopo di annullare anche gli
elementi sopra la diagonale, ottenendo una forma diagonale per la matrice trasfor-
mata:

a0011
 
0 0 ... ... 0
 0 a0022 0 ... ... 0 
a0033
 

 0 0 0 ... 0 
.
 ... ... ... ... ... ... 
... a00n−1n−1
 
 0 0 ... 0 
0 0 0 ... ... a00nn
Poiché A é non singolare, segue che ogni aii é non nullo. Infine sostituendo ogni
riga i con se stessa moltiplicata per a−1 ii , otteniamo la matrice identitá come la
trasformata della matrice invertibile iniziale, in seguito ad un opportuno numero
di operazioni elementari sulle righe. In altre parole, esistono opportune matrici
elementari E1 , E2 , ..., Ek tali che (E1 · E2 · .... · Ek )A = I. Quindi A = (E1 · E2 · .... ·
Ek )−1 = Ek−1 · Ek−1
−1
· .... · E1−1 cioé ogni matrice non singolare é invertibile e si puó
esprimere come prodotto di opportune matrici elementari.
Osservazione 3. Una matrice quadrata A é invertibile se e solo se det(A) 6= 0.
Terminiamo il paragrafo con il seguente risultato (noto come Teorema di Binet):
Teorema 4. Siano A, B matrici quadrate di ordine n. Allora det(A · B) = det(A) ·
det(B).

Dim. Dividiamo la dimostrazione in due casi:


MATRICI. 13

1. Sia det(A) = 0; se fosse anche det(AB) = 0 avremmo concluso, visto che


0 = det(A)det(B) = det(AB). Supponiamo al contrario che sia det(AB) 6= 0,
quindi AB é una matrice quadrata invertibile, cioé (AB)(AB)−1 = I, da cui
A(B(AB)−1 ) = I. Questo significherebbe che A é invertibile, contro l’ipotesi
che det(A) = 0. Quindi effettivamente det(AB) = 0 ed abbiamo terminato.
2. Sia ora det(A) 6= 0; essendo in questo caso A invertibile, dalla nota precedente
esistono opportune matrici elementari E1 , .., Ek tali che A = E1 · E2 ..... · Ek ;
inoltre sappiamo anche che det(A) = det(E1 )det(E2 )...det(Ek ) (vedi consid-
erazioni iniziali sulle matrici elementari). Quindi
det(AB) = det(E1 E2 ...Ek B) = det(E1 )det(E2 )...det(Ek )det(B) = det(A)det(B).
Osservazione 4. Facile conseguenza del Teorema di Binet é la seguente:
Sia A invertibile tale che A · B = I, allora
1 = det(I) = det(A · B) = det(A) · det(B)
1
da cui det(B) = det(A) .

8. Matrici ortogonali e matrici simili. Sia A una matrice con m righe e n


colonne, A ∈ Mmn (lR). DA é detta ortogonale se AT = A−1 . Ció vuol dire che
1 = det(I) = det(A · A−1 ) = det(A · AT ) = det(A) · det(AT ) = det(A)2 .
Quindi condizione necessaria (ma non sufficiente) affinché una matrice sia ortogo-
nale é che il suo determinanate sia pari a +1 oppure −1.
 
cos(ϕ) sen(ϕ)
Esempio 14. La matrice A = é ortogonale.
−sen(ϕ) cos(ϕ)
Osservazione 5. Da semplici calcoli si ottiene una proprietá delle matrici ortog-
onali: se A e B sono entrambe ortogonali e dello stesso ordine, allora il prodotto
AB é ancora una matricie ortogonale.
8.1. Relazione di Similitudine. Diciamo che due matrici quadrate A, B, en-
trambe di ordine n, sono simili se esiste una matrice M quadrata di ordine n, non
singolare, tale che A = M −1 · B · M . Si noti che valgono le seguenti:
1. Ogni matrice quadrata A é simile a se stessa, infatti
A = I −1 · A · I = I · A · I.
2. Se la matrice A é simile alla B allora B é simile ad A, infatti
A = M −1 · B · M ⇒ B = (M −1 )−1 · A · (M −1 ).
3. Se A é simile a B e B é simile a C, allora A é simile a C, infatti
A = M −1 · B · M e B = N −1 · C · N
implica
A = (N · M )−1 · C · (N · M ).
Quindi la relazione di similitudine tra matrici soddisfa le proprietá riflessiva (1),
simmetrica (2) e transitiva (3), ed é una relazione di equivalenza.
L’insieme della matrici quadrate di ordine n é allora suddiviso in classi di equiv-
alenza, nel senso che tutte le matrici tra loro simili costituiscono un’unica classe,
avente come rappresentante una qualsiasi delle matrici che ne fanno parte. Inoltre
due distinte classi di equivalena non possono aver alcuna matrice in comune.
Le proprietá piú rilevanti sono le seguenti:
14 MATRICI.

1. Le matrici tra loro simili hanno lo stesso determinante.


2. Le matrici tra loro simili hanno lo stesso rango.
MATRICI. 15

9. Esercizi.
Esercizio 1. Determinare i ranghi delle seguenti matrici:
   
2 1 0 2 1 0    
 3 1 1 ; 3 1 1 ; 2 1 1 0 ; 2 1 1 0
.
3 2 1 0 4 2 2 0
−1 0 1 1 0 1
Esercizio 2. Ridurre le seguenti matrici nella loro forma a gradini:
   
1 1 2 0 −1 2 1 3
 0 2 1 2 2 ; 0 1 2 ;
1 3 3 2 1 0 3 −1
     
1 1 2 1 1 2 0 −1 3 1 4 2
 2 2 4 ; 0 9 2 −5 5  ;  6 0 7 5  .
−1 −1 −2 0 4 0 8 7 8 0 9 0
Esercizio 3. Determinare se le seguenti matrici sono ortogonali:
 
1 0 0  
 0 −1 1  ; cos(α) sen(α)
;
−sen(α) cos(α)
0 0 1
Esercizio 4. Calcolare l’inversa della seguente matrice:
 
1 1 0
 2 0 1 ;
1 1 1
Esercizio 5. Determinare una matrice triangolare superiore equivalente per righe
alla matrice  
1 1 1
 1 0 −1  .
0 1 1
Esercizio 6. Determinare una matrice triangolare superiore equivalente per righe
alla matrice  
1 1 2 2
 2 1 1 0 
 −1 −1 1 1  .
 

2 2 3 1
Esercizio 7. Calcolare il determinante della matrice
 
1 −1 2 3
 1 0 1 2 
 3 −1 −1 −2  .
 

0 1 1 2
Esercizio 8. Determinare i ranghi delle seguenti matrici:
 
    1 −3 −1 1 0
  1 1 1 2 3 0 1
1 2 −1  0 1 2 1 1 
;  2 −1 0  ;  1 2 −1 0  ;   2 −6 1 0 0
.
0 1 3 
3 0 1 −2 0 0 −1
−1 3 1 2 0
16 MATRICI.

Esercizio 9. Determinare il rango delle seguenti matrici, al variare del parametro


k:      
k −1 2 k 1 1 1 −1
 k −2 1 ; 1 0 1 ; 0 k ;
3 1 0 2 k 0 1 1
Esercizio 10. Determinare, se esistono, le matrici inverse delle seguenti:
   
  1 0 −1 2 1 0
1 1
;  0 1 0  ;  1 −1 1  ;
2 0
0 0 1 1 2 −1
Esercizio 11. Per quali valori del parametro reale k, le seguenti matrici sono
invertibili:    
1 0 −1 k k 1
 k 1 k ; 2 0 0 ;
1 2 1 −1 −1 k
Scegliere un valore di k per cui una di esse é invertibile e determinarne l’inversa.
I SISTEMI LINEARI.

1. Introduzione. Una equazione del tipo a1 x1 + a2 x2 + a3 x3 + ... + an xn = b,


con a1 , .., an , b ∈ lR, si dice lineare; x1 , .., xn sono le incognite, b é il termine noto,
a1 , .., an sono i coefficienti delle incognite. Risolvere una equazione lineare vuol
dire determinare una n-upla di valori in lR, (c1 , .., cn ), da attribuire alle incognite
x1 , .., xn , tali che a1 c1 + a2 c2 + ... + an cn = b. Un sistema lineare di m equazioni in
n incognite sul campo lR é un insieme di m equazioni lineari nelle stesse incognite
x1 , .., xn , cioé


 a11 x1 + a12 x2 + ... + a1n xn = b1
a21 x1 + a22 x2 + ... + a2n xn = b2

.

 ..........
am1 x1 + am2 x2 + ... + amn xn = bm

Una soluzione di un tale sistema é una n-upla di valori reali (c1 , .., cn ) da attribuire
alle incognite x1 , .., xn , tali che essi verifichino ciascuna delle m equazioni, cioé:


 a11 c1 + a12 c2 + ... + a1n cn = b1
a21 c1 + a22 c2 + ... + a2n cn = b2

.

 ...........
am1 c1 + am2 c2 + ... + amn cn = bm

Se consideriamo le seguenti matrici:


   
  x1 b1
a11 a12 ... a1n
 a21
 x2   b2 
a22 ... a2n     
A=
 ...
,X =  ... ,B =  ... 
... ... ...     
 ...   ... 
am1 am2 ... amn
xn bm
allora il sistema lineare si puó riscrivere in forma compatta nel modo seguente:
A · X = B.
 
a11 a12 ... a1n b1
 a21 a22 ... a2n b2 
Chiameremo A matrice incompleta e C = 
  matrice
... ... ... ... ... 
am1 am2 ... amn bm
completa del sistema lineare.

Siano A e C le matrici associate ad un sistema lineare. Supponiamo che la


matrice C non sia ridotta per righe. Dopo aver effettuato la riduzione della matrice
C, indichiamo con A0 e C 0 le matrici trasformate e ridotte. Ad esse viene associato
ancora un sistema lineare A0 · X = B 0 . Vale il seguente:

1
2 I SISTEMI LINEARI.

Teorema 1. Le soluzioni di un sistema lineare A · X = B sono le stesse di ciascun


sistema lineare A0 · X = B 0 ottenuto da esso attraverso operazioni elementari ef-
fettuete sulle righe delle matrici caratteristiche del sistema. Tali sistemi sono detti
equivalenti.

Esempio 1. Sia dato il sistema lineare



 x1 + x2 + x3 = 3
x1 + x2 − x3 = 2 .
x1 − x2 + x3 = 2

Svolg. Le matrici associate sono


   
1 1 1 1 1 1 3
A =  1 1 −1  , C =  1 1 −1 2  .
1 −1 1 1 −1 1 2
Riduciamo la matrice C:
 
1 1 1 3
R 3 → R 3 + R2 , C0 =  1 1 −1 2 
2 0 0 4
 
1 1 1 3
R2 → R2 − R 1 , C0 =  0 0 −2 −1  .
2 0 0 4
Il sistema associato a questa matrice é

 x1 + x2 + x3 = 3
−2x3 = −1
2x1 = 4

la cui soluzione é (x1 = 2, x2 = 12 , x3 = 12 ). Poiché il sistema trasformato é


equivalente a quello di partenza, quest’ultima é anche soluzione del primo sistema.

Ma non tutti i sistemi lineari hanno necessariamente una soluzione, per esempio
basti pensare al seguente:

x1 + x2 = 2
.
2x1 + 2x2 = 1
Diremo compatibili i sistemi lineari che ammettono soluzione ed incompatibili quelli
che non ne ammettono alcuna. Parallelamente, non é detto che se un sistema lineare
ha una soluzione, esso abbia solo quella, eccone un esempio:

 x1 + x2 + x3 = 2
2x1 + 2x2 + 2x3 = 4
x1 − x2 = 0

che ammette come soluzioni le infinite terne (α, α, 2 − 2α), per ogni α ∈ lR.
Diremo indeterminati i sistemi lineari che ammettono infinite soluzioni.
I SISTEMI LINEARI. 3

Teorema 2. (di Rouché-Capelli) Sia A · X = B un sistema lineare di m equazioni


in n incognite, con matrici associate A ∈ Mmn (lR), C ∈ Mmn+1 (lR). Esso é
compatibile se e solo se il rango della matrice incompleta A é uguale al rango della
matrice completa C. Inoltre se il sistema é compatibile, detto r il rango delle
matrici, le soluzioni sono in numero di ∞n−r , nel caso n > r. Infine, se n = r il
sistema ammette un’unica soluzione.

Dim. Sia A0 · X = B 0 il sistema lineare ottenuto da A · X = B dopo aver ridotto


per righe la matrice completa [A|B], in modo tale che rango(A) = rango(A0 ) e
rango([A|B]) = rango([A0 |B 0 ]) ed i due sistemi siano equivalenti.
Supponiamo prima che il sistema sia compatibile e per assurdo sia r = rango(A0 ) <
rango([A0 |B 0 ]). Ordiniamo le righe del sistema in modo che le prime r righe di A0
siano proprio quelle non nulle. Allora tutte le righe di A0 dalla r + 1-esima in poi
sono nulle. Poiché rango(A0 ) < rango([A0 |B 0 ]), esiste almeno una tra le righe di
([A0 |B 0 ]) comprese tra la r + 1-esima e la n-esima che non é nulla. Possiamo per es-
empio supporre che sia la riga di posto r + 1. Di conseguenza otteniamo la seguente
incongruenza:
0 = 0 · x1 + 0 · x2 + ...... + 0 · xn = br+1 6= 0.
Viceversa supponiamo ora che rango(A) = rango([A|B]) = r. Ció vuol dire che
la colonna dei termini noti B non incide nel calcolo dei ranghi, in altre parole
la colonna B é combinazione lineare delle colonne della matrice A. Allora ogni
elemento bi che si trovi su una qualsiasi riga i-esima di B é combinazione lineare di
tutti gli elementi che si trovano sulla riga i-esima di A. Quindi esistono α1 , ..., αn ∈
lR tali che:
a11 α1 + a12 α2 + ... + a1n αn = b1
a21 α1 + a22 α2 + ... + a2n αn = b2
a31 α1 + a32 α2 + ... + a3n αn = b3
....................................................
am1 α1 + am2 α2 + ... + amn αn = bm
cioé il sistema ammette almeno la soluzione (x1 , ..., xn )=(α1 , ..., αn ).
Passiamo alla dimostrazione dell’ultima parte del Teorema. Supponendo il sistema
compatibile, riscriviamo il sistema equivalente nella sua forma ridotta:


 a11 x1 + a12 x2 + a13 x3 + .... .... a1n xn = b1
a22 x2 + a23 x3 + .... .... a2n xn = b2



a33 x3 + .... .... a3n xn = ....
.... .... .... ....




arr xr + .... arn xn = br

Dall’ultima riga otteniamo


1
xr = (br − arr+1 xr+1 − arr+2 xr+2 − .... − arn xn ) = Fr (xr+1 , ...., xn )
arr
dove Fr é una funzione dei parametri xr+1 , ...., xn . Sostituendo nella precedente
1
xr−1 = (br−1 − ar−1r xr − ar−1r+1 xr+1 − .... − ar−1n xn ) =
ar−1r−1

1
br−1 − ar−1r (br − arr+1 xr+1 − arr+2 xr+2 − .... − arn xn )
arr
4 I SISTEMI LINEARI.


1
−ar−1r+1 xr+1 − .... − ar−1n xn = Fr−1 (xr+1 , ...., xn ).
ar−1r−1
In generale avremo che: per ogni indice di riga i ≤ r
1
xi = (bi − aii+1 xi+1 − aii+2 xi+2 − .... − ain xn ) = Fi (xr+1 , ...., xn )
aii
dove ogni Fi é una funzione dei parametri xr+1 , ...., xn . Concludiamo allora che si
puó ottenere una soluzione (F1 , F2 , .., Fr , xr+1 , .., xn ) del sistema lineare ogni volta
che si attribuiscono valori arbitrari ai parametri xr+1 , .., xn , da cui la seconda parte
del teorema.
Esempio 2. Sia dato il sistema

 x1 + x2 + x3 = 1
2x1 − 2x2 + 2x3 = 0 .
x1 − x3 = 1

 
1 1 1
Svolg. La matrice incompleta é A =  2 −2 2  che ha rango 3. Ovviamente
1 0 −1
anche la matrice completa avrá rango 3. Il sistema é compatibile ed ammette una
sola soluzione.
Esempio 3. Sia dato il sistema

 x1 + x2 − x3 + x4 = 0
3x1 + x2 − x3 = 1 .
2x1 − x4 = 1

 
1 1 −1 1
Svolg. La matrice incompleta é A =  3 1 −1 0  che ha rango 2. La
2 0 0 −1
 
1 1 −1 1 0
matrice completa é C =  3 1 −1 0 1  che ha ancora rango 2. Il sistema
2 0 0 −1 1
é compatibile ed ammette ∞4−2 = ∞2 soluzioni. Il rango 2 ci é dato dalle prime
due righe, le terza é combinazione lineare di esse. Allora un
 sistema equivalente
a
1 1 −1 1 0
quello dato é quello associato alla seguente matrice: C 0 =  3 1 −1 0 1  e
0 0 0 0 0
si scrive 
x1 + x2 = x3 − x4
.
3x1 + x2 = 1 + x3
Dalla prima otteniamo x1 = −x2 + x3 − x4 e sostituendo nella seconda:
−3x2 + 3x3 − 3x4 + x2 = 1 + x3 cioé − 2x2 = 1 − 2x3 + 3x4
1 − 2x3 + 3x4
x2 = −
2
ed ancora sostituendo il valore di x2 nella espressione di x1 :
1 − 2x3 + 3x4
x1 = + x3 − x4 .
2
I SISTEMI LINEARI. 5

Allora la generica soluzione del sistema lineare é


1 1 1 3
x1 = + β x2 = − + α − β x3 = α x4 = β
2 2 2 2
per ogni valore reale dei parametri α e β.
Esempio 4. Sia dato il sistema

 x1 + x2 − x3 = 2
x1 + 2x2 − x3 = 0 .
2x1 + 3x2 − 2x3 = 1

 
1 1 −1
Svolg. La matrice incompleta é A =  1 2 −1  che ha rango 2. La matrice
2 3 −2
 
1 1 −1 2
completa é C =  1 2 −1 0  che ha rango 3. Il sistema é incompatibile.
2 3 −2 1

2. Sistemi omogenei. Sia A ∈ Mmn (lR), matrice con m righe e n colonne.   Il


0
 0 
 
sistema lineare A · X = B é detto omogeneo se B = 0 cioé se B =   ...  é

 ... 
0
il vettore nullo di m componenti. Tali sistemi hanno sempre la soluzione banale,
(x1 = 0, .., xn = 0). Poiché in tale caso la matrice completa e quella incompleta
hanno sempre lo stesso rango, la soluzione banale é anche l’unica soluzione se e solo
se il rango della matrice A é pari al numero n di incognite. Al contrario, se il rango
di A é r < n allora le soluzioni sono in numero di ∞n−r .

Esempio 5. Sia dato il sistema



 x1 + x2 + 2x3 = 0
2x1 + 2x2 + 3x3 = 0 .
3x1 + x2 + 2x3 = 0

 
1 1 2
Svolg. La matrice associata é A =  2 2 3  che ha rango 3. Allora vi é la
3 1 2
sola soluzione banale.
Esempio 6. Sia dato il sistema

 x1 + x2 + 2x3 + x4 = 0
2x1 + 2x2 + 3x3 + 2x4 = 0 .
x1 + x2 + x3 + x4 = 0

 
1 1 2 1
Svolg. La matrice associata é A =  2 2 3 2  che ha rango 2. Allora vi
1 1 1 1
sono ∞2 soluzioni. Il rango é 2 poiché l’ultima riga é combinazione lineare delle
6 I SISTEMI LINEARI.

precedenti due. Quindi un sistema equivalente


 al precedente
 é quello che ha come
1 1 2 1
matrice associata la seguente: A0 =  2 2 3 2 . Tale sistema si scrive:
0 0 0 0

x1 + 2x3 = −x2 − x4
.
2x1 + 3x3 = −2x2 − 2x4
Dalla prima otteniamo x1 = −x2 − 2x3 − x4 . Sostituendo nella seconda otteniamo
−2x2 − 4x3 − 2x4 + 3x3 = −2x2 − 2x4 cioé x3 = 0.
Un generica soluzione del sistema é allora
x1 = −α − β x2 = α x3 = 0 x4 = β
al variare dei parametri α e β in lR.

Caso particolare é quello in cui r = n − 1. Verificheremo che le soluzioni non


banali del sistema lineare omogeneo sono date dalle n-uple proporzionali ai comple-
menti algebrici di ordine n − 1 della matrice A. Infatti se il rango é n − 1, possiamo
riscrivere il sistema cancellando le ultime m − n + 1 righe (dopo averle riordinate
in modo che le prime n-1 siano linearmente indipendenti):


 a11 x1 + a12 x2 + a13 x3 + .... + a1n xn = 0
a12 x1 + a22 x2 + a23 x3 + .... + a2n xn = 0


 ..............................................
an−11 x1 + an−12 x2 + an−13 x3 + .... + an−1n xn = 0

La matrice associata al sistema é quindi rettangolare (n − 1 righe, n colonne), cioé:


 
a11 a12 a13 .... .... ...... a1n
 a21 a22 a23 .... .... ...... a2n 
 
 a31 a32 a33 .... .... ...... a3n 
 
 a41 a42 a43 .... .... ...... a4n 
 
 ...... ...... ...... .... .... ...... ...... 
an−11 an−12 an−13 .... .... ...... an−1n
A partire dalla precedente, consideriamo ora la seguente matrice:
 
a11 a12 a13 .... .... ...... a1n
 a21 a22 a23 .... .... ...... a2n 
 
 a31 a32 a33 .... .... ...... a3n 
 
A=  a41 a42 a43 .... .... ...... a4n  
 ...... ...... ...... .... .... ...... ...... 
 
 an−11 an−12 an−13 .... .... ...... an−1n 
an1 an2 an3 .... .... ...... ann
nella quale abbiamo aggiunto un’ultima riga linearmente dipendente dalle prece-
denti, scelta arbitrariamente tra le m − n + 1 righe precedentemente scartate. Per
semplicitá abbiamo scelto di aggiungere la n-esima riga. Osserviamo subito che
det(A) = 0, qualunque sia la riga che scegliamo di aggiungere.
Se applichiamo il secondo teorema di Laplace alla matrice A, in ogni sua riga ad
esclusione dell’ultima, otteniamo
aj1 An1 + aj2 An2 + ... + ajn Ann = 0
I SISTEMI LINEARI. 7

per ogni indice j = 1, ..., n − 1. Inoltre, poiché det(A) = 0, abbiamo anche


an1 An1 + an2 An2 + ... + ann Ann = 0
ed a fortiori
α(aj1 An1 + aj2 An2 + ... + ajn Ann ) = 0
per ogni α ∈ lR e per j = 1, ..., n. Questo vuol dire che la n-upla α(An1 , An2 , ..., Ann )
é soluzione del sistema lineare omogeneo originario, al variare di α ∈ lR.

Esempio 7. Sia dato il sistema




 2x1 + 3x2 − x3 + x4 = 0
x1 − x2 + x3 = 0

.

 3x 1 − 2x3 + x4 = 0
2x2 + 2x3 = 0

 
2 3 −1 1
 1 −1 1 0 
Svolg. La matrice associata é A =   3 0 −2 1  che ha rango 3. In parti-

0 2 2 0
colare il rango é dato dalle prime tre righe della matrice, quindi le ∞1 soluzioni del
sistema sono proporzionali ai complementi algebrici degli elementi a41 = 0, a42 =
2, a43 = 2, a44 = 0, cioé
 
3 −1 1 2 −1 1 2 3 1 2 3 −1

α  −1 1 0 , − 1 1 0 , 1 −1 0 , − 1 −1 1 
0 −2 1 3 −2 1 3 0 1 3 0 −2
al variare di α parametro reale, cioé α(4, 2, −2, −16), ovvero β(2, 1, −1, −8) al vari-
are di β ∈ lR.

3. Metodo di Cramer per la risoluzione di un sistema lineare. Supponiamo


di aver un sistema al quale venga associata un matrice incompleta A quadrata di
ordine n e non singolare:
   
  x1 b1
a11 a12 ... a1n  x2   b2 
 a21 a22 ... a2n     
A=   , X =  ...  , B = 
 
 ...  .

... ... ... ...   ...   ... 
an1 an2 ... ann
xn bn
Il sistema sia A · X = B.
Sappiamo dal teorema di Rouché-Capelli che in tal caso il sistema ammette
un’unica soluzione. Essa puó essere determinata nel modo seguente:
A · X = B → A−1 · A · X = A−1 · B → X = A−1 · B
da cui
1 ∆1
x1 = (A11 b1 + A21 b2 + ... + An1 bn ) · =
det(A) det(A)
1 ∆2
x2 = (A12 b1 + A22 b2 + ... + An2 bn ) · =
det(A) det(A)
1 ∆3
x3 = (A13 b1 + A23 b2 + ... + An3 bn ) · =
det(A) det(A)
.............................
8 I SISTEMI LINEARI.

1 ∆n
xn = (A1n b1 + A2n b2 + ... + Ann bn ) · =
det(A) det(A)
dove Aij é il complemento algebrico dell’elemento aij ∈ A.
In sostanza, il termine al numeratore ∆1 = (A11 b1 + A21 b2 + ... + An1 bn ) é il
determinante della matrice
 
b1 a12 ... a1n
 b2 a22 ... a2n 
 
 ... ... ... ... 
bn an2 ... ann
ottenuta scambiando la prima colonna di A con la colonna dei termini noti.
In generale, il termine ∆i = (A1i b1 + A2i b2 + ... + Ani bn ) é il determinante della
matrice
 
a11 a12 ... b1 ... a1n
 a12 a22 ... b2 ... a2n 
 
 ... ... ... bj ... ... 
a1n an2 ... bn ... ann
ottenuta scambiando la colonna i di A con la colonna dei termini noti.

Supponiamo ora che la matrice A incompleta non sia quadrata e che il sistema sia
compatibile, diciamo r il rango del sistema. Consideriamo A0 la matrice di ordine
r che fornisce il rango al sistema, essa é ovviamente quadrata e non singolare.
Il sistema relativo alla matrice A0


 a11 x1 + a12 x2 + ... + a1r xr = b1 − a1(r+1) xr+1 − ... − a1n xn
a21 x1 + a22 x2 + ... + a2r xr = b2 − a2(r+1) xr+1 − ... − a2n xn


 ..........
ar1 x1 + ar2 x2 + ... + arr xr = bm − ar(r+1) xr+1 − ... − arn xn

é equivalente a quello di partenza, inoltre é risolvibile col metodo di Cramer. Le


soluzioni (x1 , .., xr ) saranno date in funzione dei parametri reali xr+1 , .., xn .

Esempio 8. Ripetiamo un esempio precedentemente esposto, ma applichiamo ora


il metodo di Cramer. Sia dato il sistema

 x1 + x2 + 2x3 + x4 = 0
2x1 + 2x2 + 3x3 + 2x4 = 0 .
x1 + x2 + x3 + x4 = 0

 
1 1 2 1
Svolg. La matrice associata é A =  2 2 3 2  che ha rango 2. Allora vi
1 1 1 1
sono ∞2 soluzioni. Il rango é 2 poiché l’ultima riga é combinazione lineare delle
precedenti due. Quindi un sistema equivalente
 al precedente
 é quello che ha come
1 1 2 1
matrice associata la seguente: A0 =  2 2 3 2 . Tale sistema si scrive:
0 0 0 0

x1 + 2x3 = −x2 − x4
.
2x1 + 3x3 = −2x2 − 2x4
I SISTEMI LINEARI. 9

Applichiamo
 ora
 il metodo di Cramer. La matrice incompleta del nuovo sistema é
1 2
A00 = , con det(A00 ) = −1. Le variabili x2 , x4 diventano parametri reali,
2 3
ai quali possiamo attribuire un qualsiasi valore in lR. Calcoliamo

−x2 − x4 2
∆1 = = x2 + x4
−2x2 − 2x4 3

1 −x2 − x4
∆3 = =0
2 −2x2 − 2x4
e quindi
∆1 ∆3
x1 = = −x2 − x4 x3 = =0
−1 −1
con x2 e x4 parametri reali liberi.

4. Metodo di eliminazione di Gauss. Concludiamo ora facendo vedere come un


qualsiasi sistema compatibile si possa risolvere utilizzando le operazioni elementari
sulle righe della matrice completa, associata ad esso, per riportarla in forma ridotta.
Siano A ∈ Mmn (lR), B ∈ Mm1 (lR), rispettivamente la matrice incompleta e la
colonna dei termini noti del sistema. Al solito indichiamo con C = [A|B] la matrice
completa del sistema.
Se operiamo sulle righe della matrice C, riportandola nella forma ridotta C 0 ,
automaticamente avremo ridotto anche la matrice A nella forma A0 . Il sistema
associato alle matrici A0 e C 0 é equivalente a quello iniziale, ma viene espresso in
una forma ridotta, quindi piú facilmente risolvibile, con il metodo della sostituzione.

Esempio 9. Sia dato il sistema




 x1 + 2x2 − x3 + x4 = 8
x1 + 3x2 − 5x3 − x4 = 9

.
 2x1 + 4x2 − 4x3 + x4 = 16

4x1 + 10x2 − 9x3 + x4 = 33

Svolg. La matrice completa associata é


 
1 2 −1 1 8
 1 3 −5 −1 9 
C= .
 2 4 −4 1 16 
4 10 −9 1 33
Cominciamo con le operazioni elementari sulle righe per rendere speciale l’elemento
di posto (1, 1):
 
1 2 −1 1 8
1  0 1 −3 − 3 1 
R2 → R2 − R3 C 0 =   2 4 −4 1 16 
2 
2
4 10 −9 1 33
 
1 2 −1 1 8
1  0 1 −3 − 3 1 
R3 → R 3 − R4 C 0 =  1 1
2 
2  0 −1
2 2 − 12 
4 10 −9 1 33
10 I SISTEMI LINEARI.

 
1 2 −1 1 8
0
 0 1 −3 − 3 1 
R4 → R4 − 4R1 C =
 0 −1 1 1
2 .
2 2 − 12 
0 2 −5 −3 1
Passiamo ora a determinare un elemento speciale sulla seconda riga, e come in
precedenza, scegliamo l’elemento sulla diagonale principale (quello di posto (2, 2)):
 
1 2 −1 1 8
 0 1 −3 − 3 1 
R3 → R3 + R2 , R4 → R4 − 2R2 , C 0 =   0 0 − 5 −1
2
1 .

2 2
0 0 1 0 −1
Infine passiamo alla terza riga:
 
1 2 −1 1 8
2 0
 0 1 −3 − 32 1 
R4 → R 4 + R3 C = 1 .

5  0 0 − 52 −1 2
0 0 0 − 25 − 45
La matrice é ora ridotta. Si noti che tanto il rango della incompleta che della
completa é 4, quindi il sistema é compatibile ed ammette una sola soluzione. Inoltre
il sistema é ora riscrivibile come segue:

 x1 +2x2 −x3 +x4 =8
−3x3 − 32 x4

3x2 =1

5 1 .

 − 2 x3 −x 4 = 2
− 25 x4 = − 45

Dall’ultima equazione si ha x4 = 2. Sostituendo nella terza otteniamo x3 = −1, e


continuando a sostituire nelle prime due, x2 = 1, x1 = 3.
Esempio 10. Sia dato il sistema

 2x1 − x2 + 7x3 = 1
3x1 − 3x2 + 10x3 = 0 .
4x1 − 5x2 + 13x3 = 7

 
2 −1 7 1
Svolg. La matrice completa associata é C =  3 −3 10 0 . Cominciamo
4 −5 13 7
con la prima riga:
 
2 −1 7 1
3
R2 → R2 − R1 C 0 =  0 − 32 − 12 − 32 
2
4 −5 13 7
 
2 −1 7 1
R3 → R3 − 2R1 C 0 =  0 − 32 − 12 − 32 
0 −3 −1 5
 
2 −1 7 1
R3 → R3 − 2R2 C 0 =  0 − 32 − 12 − 32  .
0 0 0 2
I SISTEMI LINEARI. 11

La matrice é ora ridotta. Si noti che il rango della incompleta é 2, mentre quello
della matrice completa é 3. Pertanto il sistema é incompatibile.
Esempio 11. Sia dato il sistema

 x1 − 2x2 + 3x3 − 4x4 = 4
x1 − x2 + 2x3 − 3x4 = 1 .
−x1 − x2 + x3 + 3x4 = −1

 
1 −2 3 −4 4
Svolg. La matrice completa associata é C =  1 −1 2 −3 1 .
−1 −1 1 3 −1
 
1 −2 3 −4 4
R2 → R2 − R 1 C 0 =  0 1 −1 1 −3 
−1 −1 1 3 −1
 
1 −2 3 −4 4
R3 → R3 + R1 C 0 =  0 1 −1 1 −3 
0 −3 4 −1 3
 
1 −2 3 −4 4
R3 → R3 + 3R2 C 0 =  0 1 −1 1 −3  .
0 0 1 2 −6
La matrice é ora ridotta. Si noti che tanto il rango della incompleta che della
completa é 3, quindi il sistema é compatibile ed ammette ∞1 soluzioni. Inoltre il
sistema é ora riscrivibile come segue:

 x1 −2x2 +3x3 −4x4 = 4
x2 −x3 +x4 = −3 .
x3 +2x4 = −6

Dall’ultima equazione si ha
x3 = −2x4 − 6.
Sostituendo nella seconda otteniamo
x2 = x3 − x4 − 3 = −2x4 − 6 − x4 − 3 = −3x4 − 9
ed infine dalla prima equazione:
x1 = 4 + 2x2 − 3x3 + 4x4 = 4 + 4x4 .
Quindi le soluzioni sono date dalle quaterne
(4 + 4α, −9 − 3α, −6 − 2α, α)
al variare del parametro α in lR.

5. Esercizi. Determinare, quando possibile, le soluzioni dei seguenti sistemi lin-


eari:

Esercizio 1. 
 2x + y − z = 1
x+z =0
x + 2y − z = 2

12 I SISTEMI LINEARI.

Esercizio 2. 
 2x + y − z = 1
x+z =0
3x + y = 1

Esercizio 3. 
 2x + y − z = 1
3x + y = 1
5x + 2y − z = 5

Esercizio 4. 
 x1 + x2 − 2x3 + x4 = 1
2x1 + x2 + x3 + x4 = 2
x1 + 2x2 − x4 = 7

Esercizio 5. 

 x+y+z+t=0
4y + 3t = 5


 2x + 5t = 4
−3z − 2t = 1

Esercizio 6. 
 2x1 + 3x2 − x3 + 4x4 = 12
3x1 + x2 + 2x3 − 6x4 = −6
x1 − 4x2 − x3 − 4x4 = −12

Esercizio 7. 
2x − 3y − 4z = 0
x + 4y − 2z = 0
Esercizio 8. 
 3x + y − z = 0
x + 3y + z = 0
x+y =0

Utilizzare il metodo di Gauss per determinare, quando possibile, le soluzioni dei


seguenti sistemi lineari:

Esercizio 9. 

 x + 2y − z + w = 8
x + 3y − 5z − w = 9


 2x + 4y − 4z + w = 16
4x + 10y − 9z + w = 33

Esercizio 10. 
 2x − y + 7z = 1
3x − 3y + 10z = 0
4x − 5y + 13z = 7

Esercizio 11. 
 x − 2y + 3z − 4w = 4
x − y + 2z − 3w = 1
−x − y + z + 3w = −1

I SISTEMI LINEARI. 13

Esercizio 12. 
 x + 2y + 3z = 1
4x + 5y + 6z = 2
7x + 8y + 9z = 3

Esercizio 13. 

 x − y + 3z = 0
−3x − y − 7z = 4


 5x + 2y + 9z = −10
2x + y + 2z = −6

Esercizio 14. 
 x + 2y − z + 5w = 7
2x − 4y − z − 2w = −1
5x − 6y − 3z + w = 0

Determinare, al variare del parametro k nei reali, quante soluzioni possiedono i


seguenti sistemi lineari:

Esercizio 15. 
 x + 2y + 3z = k + 1
4x + 5y + 6z = k
7x + 8y + 9z = k + 1

Esercizio 16. 
 5x − 3y = 1
2x + y = 7
8x + 3y = k 2

Esercizio 17. 
 x + 2y − z = −1
kx + 2z = 1
−x + 3z = 1

Esercizio 18. 
 kx + 3y + z = k + 4
4kx + y + 2z = 2k + 2
kx + kz = k − 1

Esercizio 19. 
 2kx + 3y − z = 4 − k
4kx + y + 2z = 2k
(k − 1)y = 2

Esercizio 20. 

 x−z =3
(k + 2)y + z = −1


 x + (k − 1)z = 5
(k + 4)z = k

Determinare i valori reali del parametro k per i quali i seguenti sistemi lineari
omogenei ammettono soluzioni non banali:
14 I SISTEMI LINEARI.

Esercizio 21. 
 2x + ky − z = 0
x + y − 3z = 0
kx − 2y + 2z = 0

Esercizio 22. 
 kx − y + 3z = 0
x + y − kz = 0
2x + 2y + kz = 0

GLI SPAZI VETTORIALI.

1. Introduzione. Siano K un campo e V un insieme non vuoto in cui sono definite


le seguenti operazioni:
+ : V ×V →V
· : K × V → V.
Diremo che (V, +, ·) é uno spazio vettoriale sul campo dei reali se valgono le seguenti:
1) (V, +) é un gruppo commutativo, cioé
i) (v1 + v2 ) + v3 = v1 + (v2 + v3 ), per ogni v1 , v2 , v3 ∈ V ;
ii) esiste e ∈ V tale che v + e = e + v = v, per ogni v ∈ V ;
iii) per ogni v ∈ V , esiste w ∈ V , tale che v + w = w + v = e;
iv) per ogni v1 , v2 ∈ V , v1 + v2 = v2 + v1 .

2) (a + b)v = av + bv, per ogni a, b ∈ K, v ∈ V .


3) a(v1 + v2 ) = av1 + av2 , per ogni a ∈ K, v1 , v2 ∈ V .
4) a(bv) = (ab)v = (ba)v = b(av), per ogni a, b ∈ K, v ∈ V .
5) 1K · v = v, per ogni v ∈ V .
Chiameremo vettori gli elementi di uno spazio vettoriale e scalari gli elementi
del campo K.

Esempio 1. L’insieme delle matrici Mmn (lR) é uno spazio vettoriale su lR, rispetto
alle operazioni di somma tra matrici e prodotto per uno scalare.
Esempio 2. L’insieme dei vettori geometrici in lR3 (o in lR2 ) é uno spazio vetto-
riale su lR, rispetto alle operazioni di somma tra vettori e prodotto per uno scalare.
Esempio 3. lR é uno spazio vettoriale su se stesso.
Esempio 4. L’insieme dei polinomi di grado minore o uguale ad un fissato n,
lR[X] = {a0 + a1 x + a2 x2 + a3 x3 + ... + an xn a0 , ..., an ∈ lR}
a coefficienti reali, é uno spazio vettoriale su lR, rispetto alle operazioni di somma
tra polinomi e di prodotto di un polinomio per uno scalare.
Esempio 5. Sia
lRn = {(x1 , x2 , .., xn ) /x1 , x2 , x3 , .., xn ∈ lR}.
Definiamo le seguenti operazioni
(x1 , .., xn ) + (y1 , .., yn ) = (x1 + y1 , .., xn + yn )
α(x1 , .., xn ) = (αx1 , .., αxn ).
n
Allora lR é uno spazio vettoriale sul campo dei reali. Ogni vettore é una n-upla
del tipo (x1 , .., xn ).

1
2 GLI SPAZI VETTORIALI.

Osservazione 1. Nel seguito ci occuperemo esclusivamente di spazi vettoriali


definiti sul campo K = lR dei numeri reali.

Sia W ⊆ V , sottoinsieme dello spazio vettoriale V . Diremo che W é un sot-


tospazio vettoriale di V , se é uno spazio vettoriale rispetto alle operazioni definite
in V , sul medesimo campo dei reali. Da tale definizione deriva che, condizione nec-
essaria e sufficiente affinché W sia sottospazio di V é che valgano le due seguenti:

w1 + w2 ∈ W
aw ∈ W
per ogni a ∈ lR, w, w1 , w2 ∈ W , e queste si possono compattare nell’unica condizione
aw1 + bw2 ∈ W
per ogni a, b ∈ lR, w1 , w2 ∈ W .
 
x y
Esempio 6. W = { x, y ∈ lR} é un sottospazio dello spazio vettoriale
0 0
delle matrici quadrate di ordine 2 su lR, M2 (lR).
Esempio 7. W = {(x, 0, z) x, z ∈ lR} é un sottospazio vettoriale di lR3 .
Esempio 8. lR é un sottospazio banale di se stesso.
Esempio 9. W = {a0 + a1 x1 + a2 x2 + ... + am xm , a0 , .., am ∈ lR}, insieme dei
polinomi di grado minore o uguale a m, con m ≤ n, é sottospazio vettoriale di
V = {a0 + a1 x1 + a2 x2 + ... + an xn , a0 , .., an ∈ lR}.

2. Intersezione, unione e somma di sottospazi. Siano U, W sottospazi dello


spazio vettoriale V . Consideriamo l’intersezione di U e W
U ∩ W = {v ∈ V : v∈U e v ∈ W}
esso é ancora un sottospazio di V .

Esempio 10. Siano U = {(x, y, 0), x, y ∈ lR} e W = {(x, 0, z), x, z ∈ lR}


sottospazi di lR3 . Allora un generico vettore che appartenga ad entrambi é dato
dalle componenti (x, 0, 0), quindi scriveremo
U ∩ W = {v ∈ V : v = (x, 0, 0), x ∈ lR}.
Esempio 11. Siano U = {(x, y, 0), x, y ∈ lR} e W = {(x, x, x), x ∈ lR} sot-
tospazi di lR3 . Allora l’unico vettore che appartenga ad entrambi é dato dalle com-
ponenti (0, 0, 0), quindi scriveremo
U ∩ W = {(0, 0, 0)}.
Al contrario definiamo l’unione di dei due sottospazi U e W
U ∪ W = {v ∈ V : v∈U oppure v ∈ W }.
non é detto che tale unione sia un sottospazio di V , e per dimostrarlo portiamo il
seguente controesempio: consideriamo
U = {(x, 0), x ∈ lR}
W = {(0, y), y ∈ lR}
GLI SPAZI VETTORIALI. 3

sottospazi di lR2 . Consideriamo il vettore (1, 0) ∈ U ed il vettore (0, 1) ∈ W ,


ovviamente entrambi appartengono a U ∪ W ma
(1, 0) + (0, 1) = (1, 1) ∈
/ U ∪ W.

Definiamo ora il seguente sottoinsieme dello spazio vettoriale V :


U + W = {v ∈ V : v = u + w, u∈U e w ∈ W }.
Esso é un sottospazio di V , detto somma di U e W , piú precisamente é il piú piccolo
sottospazio di V contenente U ∪ W .
Diremo che U + W é somma diretta se U ∩ W = {0}, il solo vettore nullo.

Esempio 12. Siano U = {(x, y, 0), x, y ∈ lR} e W = {(x, 0, z), x, z ∈ lR}


sottospazi di lR3 . Allora
U + W = {(x, y, z), x, y, z ∈ lR} = lR3
inoltre U ∩ W = {(x, 0, 0)}, quindi la somma non é diretta.
Esempio 13. Siano U = {(x, y, 0), x, y ∈ lR} e W = {(z, z, z), z ∈ lR} sot-
tospazi di lR3 . Allora
U + W = {(x + z, y + z, z), x, y, z ∈ lR} = lR3
inoltre U ∩ W = {(0, 0, 0)}, quindi la somma é diretta.
Esempio 14. Siano U = {(x, 0, z), x, z ∈ lR} e W = {(y, 0, y), y ∈ lR} sot-
tospazi di lR3 . Allora
U + W = {(x + y, 0, y + z), x, y, z ∈ lR}
inoltre U ∩ W = {(x, 0, x)}, quindi la somma non é diretta.
Consideriamo ancora un esempio di somma non diretta:
Esempio 15. Siano U = {(x+y, y, 0), x, y ∈ lR} e W = {(x+y, 0, y), x, y ∈ lR}
sottospazi di lR3 .

Svolg. Allora
U ∩ W = {(α, 0, 0), α ∈ lR}
quindi la somma non é diretta. Scegliamo un vettore che appartenga alla somma:
in particolare sia w = (6, 1, 1) ∈ U + W . Si noti che il vettore w si puó esprimere
in diversi modi come somma di due vettori scelti rispettivamente in U e W :
(6, 1, 1) = (3, 1, 0) + (3, 0, 1) con (3, 1, 0) ∈ U, (3, 0, 1) ∈ W
ed anche
(6, 1, 1) = (2, 1, 0) + (4, 0, 1) con (2, 1, 0) ∈ U, (4, 0, 1) ∈ W.
Questo accade per ogni vettore appartenente allo spazio somma, nel caso in cui
questa non sia diretta. Alla luce di ció, vale la seguente:
Proposizione 1. Siano U e W sottospazi vettoriali dello spazio V . La loro somma
é diretta se e solo se ogni vettore di essa si puó esprimere in modo unico come
somma di un vettore di U e di uno di W .

Dim. Rimandiamo la dimostrazione al paragrafo successivo.


4 GLI SPAZI VETTORIALI.

Ricordiamo che una combinazione lineare di vettori {v1 , .., vn } di V é una scrit-
tura del tipo
a1 v1 + a2 v2 + ... + an vn
per qualsiasi a1 , .., an scalari in lR.
Sia S ⊆ V , un sottoinsieme dello spazio V . Definiamo Span(S) =< S >, e
lo chiamiamo sottospazio generato da S, il sottospazio di V composto da tutte le
possibili combinazioni lineari di vettori di S e scalari in lR.
Esempio 16. Sia V = lR3 , S = {(1, 0, 0), (0, 1, 0)}. Allora Span(S) =< S >=
{(x, y, 0), x, y ∈ lR}.
Definizione 1. Siano v1 , .., vn vettori in V . Diremo che v1 , .., vn sono linearmente
dipendenti se esistono a1 , .., an ∈ lR, non tutti nulli tali che a1 v1 +a2 v2 +..+an vn =
0. Al contrario sono detti linearmente indipendenti se a1 v1 + a2 v2 + .. + an vn = 0
implica che a1 = a2 = ... = an = 0.
Esempio 17. v1 = (−1, 2, 3), v2 = (0, −1, 0), v3 = (1, 0, 1) vettori di lR3 sono
linearmente indipendenti.
Esempio 18. v1 = (1, 2, 1, 0), v2 = (1, −1, 0, 1), v3 = (−1, 2, −1, 0), v4 = (−1, 1, 0, −1),
v5 = (1, 1, 0, 1) vettori di lR4 sono linearmente dipendenti.
Esempio 19. v1 = (1, 2, 0), v2 = (0, 1, a), v3 = (1, a, −1) vettori di lR3 , con a
parametro reale, sono indipendenti per a 6= 1 e sono dipendenti per a = 1.

3. Basi e dimensione di uno spazio vettoriale. Sia V uno spazio vettoriale su


lR. Un insieme B di vettori é detta base di V se:
1. i vettori di B sono linearmente indipendenti;
2. Span(B) = V .
Esempio 20. Se V = lR, allora per ogni a ∈ lR, B = {a}.
 
x1 x2
Esempio 21. Sia V = { , x1 , x2 , x3 , x4 ∈ lR}.
x3 x4

Svolg. Allora una base per V é data da


       
1 0 0 1 0 0 0 0
B={ , , , }
0 0 0 0 1 0 0 1

Esempio 22. Sia V = lR3 , allora una base é data da


B = {(1, 0, 0), (0, 1, 0), (0, 0, 1)}
Esempio 23. Sia
V = lR[X] = {a0 + a1 x + a2 x2 + a3 x3 + ... + an xn a0 , ..., an ∈ lR}
allora una base é data da
B = {1, x, x2 , x3 , .., xn }
Teorema 1. Sia V uno spazio vettoriale e B = {e1 , e2 , .., en } una sua base. Siano
v1 , v2 , .., vr vettori linearmente indipendenti di V . Allora r ≤ n ed esistono n − r
vettori ej1 , ej2 , .., ejn−r di B tali che l’insieme B 0 = {v1 , v2 , .., vr , ej1 , ej2 , .., ejn−r }
costituisca una base per V .
GLI SPAZI VETTORIALI. 5

Dim. Dimostriamo dapprima che r ≤ n. Supponiamo al contrario che r > n.


Poiché B é una base di V , ciascuno dei vettori vi ammette n componenti rispetto
ad essa. Diciamo
v1 = (v11 , . . . , v1r ), . . . , vr = (vr1 , . . . , vrn ).
Costruiamo la matrice con r righe ed n colonne nella quale siano inserite le com-
ponenti degli r vettori vi :
 
v11 v12 ... . . . v1n
 v21 v22 ... . . . v2n 
 
 v31 . . . ... . . . v3n 
 
 ... ... ... ... ... 
vr1 vr2 ... . . . vrn
Tale matrice dovrebbe avere r righe indipendenti, quindi rango r, ma ció é impos-
sibile poiché n < r.
Passiamo ora alla seconda parte del teorema. Poiché B é una base di V , esistono
α1 , .., αn non tutti nulli tali che
v1 = α1 e1 + α2 e2 + ... + αn en
e supponiamo per esempio α1 6= 0. Segue che
1
e1 = (v1 − α2 e2 − ... − αn en ) .
α1
Quindi {v1 , e2 , e3 , ..., en } é un insieme di generatori di V . Supponiamo che esista
una combinazione lineare
β1 v1 + β2 e2 + ... + βn en = 0.
Se fosse β1 = 0 avremmo anche β2 = β3 = ... = βn = 0, poiché e2 , .., en sono
indipendenti tra loro. Nel caso in cui β1 6= 0 allora
1
v1 = −(β2 e2 + ... + βn en )
β1
cioé l’insieme {e2 , .., en } sarebbe un insieme di generatori di V , il che porta alla
falsa conclusione che il vettore e1 debba dipendere linearmente da {e2 , .., en }.
Ne segue che {v1 , e2 , e3 , ..., en } devono essere tra loro linearmente indipendenti,
quindi sono una base per V .
Completiamo la dimostrazione per induzione sul numero r di vettori v1 , v2 , .., vr :
supponiamo allora che il teorema sia vero per r−1 di essi, cioé {v1 , v2 , . . . , vr−1 , er , . . . , en }
sia una base di V .
Quindi devono esistere α10 , ..., αn0 non tutti nulli, tali che
vr = α10 v1 + α20 v2 + . . . + αr−1
0
vr−1 + αr0 er + . . . + αn0 en .
In particolare, poiché v1 , . . . , vr−1 sono indipendenti, almeno uno tra αr0 , ..., αn0 deve
essere non nullo. Supponiamo per esempio αr0 6= 0. Segue che
r−1 n
X X 1
er = (vr − αi0 vi − αj0 ej ) .
i=1 j=r+1
αr0

Quindi {v1 , . . . , vr , er+1 . . . , en } é un insieme di generatori di V . Come prima,


supponiamo ora che esista una combinazione lineare
β10 v1 + . . . + βr0 vr + βr+1
0
er+1 + ... + βn0 en = 0.
6 GLI SPAZI VETTORIALI.

Se fosse βr0 = 0 avremmo anche β10 = β20 = . . . = βr−1 = βr+1 = . . . = βn0 = 0,


poiché {v1 , v2 , . . . , vr−1 , er , . . . , en } é una base di V . Nel caso in cui βr0 6= 0 allora
r−1 n
X X 1
vr = (− βi0 vi − βj0 ej )
i=1 j=r+1
βr0

cioé l’insieme {v1 , . . . , vr−1 , er+1 . . . , en } sarebbe un insieme di generatori di V , che


porterebbe alla falsa conclusione che il vettore er debba dipendere linearmente da
{v1 , . . . , vr−1 , er+1 . . . , en }.
Teorema 2. Sia V uno spazio vettoriale di dimensione n e siano v1 , v2 , .., vr vet-
tori linearmente indipendenti di V , con r ≤ n. Allora esistono n − r vettori
vr+1 , vr+2 , .., vn di V tali che l’insieme B = {v1 , v2 , .., vr , vr+1 , vr+2 , .., vn } costi-
tuisca una base per V .

Dim. Indichiamo A1 il sottospazio generato da v1 , ..., vr . Essendo r < n, esiste un


vettore vr+1 ∈ V − A1 , quindi vr+1 é linearmente indipendente con v1 , ..., vr . Per
cui é lecito determinare il sottospazio A2 =< v1 , .., vr , vr+1 >. Se r+1 = n abbiamo
terminato, altrimenti sia vr+2 ∈ V − A2 . Continuando in tal senso si ottiene una
base per V , nel momento in cui la dimensione del sottospazio é pari ad n.

Dai precedenti due teoremi segue immediatamente che:


Teorema 3. Due distinte basi di uno spazio vettoriale contengono lo stesso numero
di elementi.

Dim. Siano {c1 , . . . , cr } e {e1 , . . . , en } due distinte basi di uno spazio vettoriale V .
Per il teorema del completamento di una base dovrá contemporaneamente essere
r ≤ n e n ≤ r, da cui r = n
Definiamo dimensione di uno spazio vettoriale V , e la indichiamo con
dim(V ), il numero di elementi di una qualsiasi base di V .

Esempio 24. Sia


W = {(x1 , x2 , x3 , x4 ) ∈ lR4 x1 − x4 = 0, x2 + x3 = 0}.
Il generico vettore di W é (x1 , x2 , −x2 , x1 ), quindi
dim(W ) = 2 e W =< (1, 0, 0, 1), (0, 1, −1, 0) > .
Esempio 25. Sia
W = {(x1 , x2 , x3 , x4 ) ∈ lR4 x4 − x2 + x3 = 0}.
Il generico vettore di W é (x1 , x2 , x3 , x2 − x3 ), quindi

dim(W ) = 3 e W =< (1, 0, 0, 0), (0, 1, 0, 1), (0, 0, 1, −1) > .

Supponiamo ora che lo spazio vettoriale V abbia dimensione n, indichiamo con


B = {e1 , .., en } una sua base. Diciamo componenti di un vettore v ∈ V rispetto
alla base B, gli scalari a1 , .., an tali che v = a1 e1 + a2 e2 + .. + an en .
GLI SPAZI VETTORIALI. 7

Esempio 26. Sia V = lR2 e consideriamo due distinte basi di V :


B1 = {(1, 0), (0, 1)}
B2 = {(1, −2), (4, 1)}.
Sia v ∈ V un vettore che abbia componenti (0, −1) rispetto alla base B1 , cioé
v = (0)(1, 0) + (−1)(0, 1) = (0, −1). Calcoliamo le sue componenti (a1 , a2 ) rispetto
alla base B2 :
v = a1 (1, −2) + a2 (4, 1)
cioé
(0, −1) = (a1 + 4a2 , −2a1 + a2 )
4
da cui a1 = 9 e a2 = − 19 .

Terminiamo questo paragrafo con la dimostrazione della Proposizione 1, enunci-


ata nel paragrafo precedente:

Dimostrazione della Proposizione 1.


Supponiamo dapprima che la somma U + V sia diretta. Indichiamo com {u1 , .., uh }
e {w1 , .., wk } rispettivamente una base per U ed una per W . Sia v ∈ U + W e
supponiamo che esso si possa esprimere in almeno due modi come somma di vettori
di U e W :
v =u+w
Ph 0
Ph
con u = i=1 αi ui ∈ U e u = i=1 βi ui ∈ U ed anche
v = u0 + w 0
Pk 0
Pk
con w = j=1 γj wj ∈ W e w = j=1 δj wj ∈ W , per opportuni scalari αi , βi , γj ,
δj . Quindi
h
X k
X h
X k
X
v= αi ui + γj wj = βi ui + δj wj
i=1 j=1 i=1 j=1
da cui
h
X k
X
(αi − βi )ui = (γj − δj )wj ∈ U ∩ W
i=1 j=1
poiché il primo membro appartiene a U ed il secondo a W . Dal fatto che la somma
é diretta segue che
Xh
(αi − βi )ui = 0
i=1
k
X
(γj − δj )wj = 0
j=1

cioé αi = βi per ogni indice i, γj = δj per ogni indice j. Quindi u = u0 e w = w0 .


Supponiamo ora che ogni vettore della somma sia esprimibile in modo unico e
scegliamo un qualsiasi vettore v ∈ U ∩ W . Esistono opportuni scalari αi , βj tali
che
X h
v= αi ui ∈ U
i=1
8 GLI SPAZI VETTORIALI.

ed anche
k
X
v= βj wj ∈ W.
i=1
Sottraendo le due espressioni otteniamo
h
X k
X
0= αi ui − βj wj ∈ U + W.
i=1 j=1

Poiché il vettore 0 ∈ U + W si deve esprimere in modo unico, esso deve essere


necessariamente la somma del vettore nullo in U e del vettore nullo in W . Ne segue
che
X h
αi ui = 0 ∈ U
i=1
e quindi αi = 0 per ogni indice i, ed anche
k
X
βj wj = 0 ∈ W
j=1

e quindi βj = 0 per ogni indice j. Da tutto ció otteniamo la conclusione v = 0.

4. Una nota sull’intersezione di sottospazi vettoriali. Alla luce delle definizioni


di base e dimensione (e per fugare ogni dubbio sul metodo da utilizzare), diamo ora
due esempi di come si possa determinare l’intersezione di spazi vettoriali nei casi
piú rilevanti:
1. il primo caso é quello in cui gli spazi siano descritti tramite le relazioni lineari
che intercorrono tra le componenti dei vettori appartenenti ad essi;
Ecco un esempio: siano A e B due sottospazi di lR4 definiti da
A = {(x1 , x2 , x3 , x4 ) : x1 − x4 = 0}
B = {(x1 , x2 , x3 , x4 ) : 2x1 + x3 = 0, x2 + x4 = 0}.
Per determinare A ∩ B, serve dapprima determinare la generica espressione
di un vettore (x1 , x2 , x3 , x4 ) ∈ lR4 che appartenga ad entrambi. Le compo-
nenti di un tale vettore devono soddisfare contemporaneamente a tutte le
condizioni lineari dettate da entrambe le definizioni di A e B. Quindi la qua-
terna (x1 , x2 , x3 , x4 ) deve essere una soluzione del sistema lineare omogeneo
in 4 incognite: 
 x1 − x4 = 0
2x1 + x3 = 0 .
x2 + x4 = 0

La matrice ad esso associata ha rango 3, per cui ci aspettiamo ∞1 soluzioni,


cioé la dimensione dell’intersezione é 1. Il generico vettore risolutivo del sis-
tema é dato da (α, −α, −2α, α), al variare di α ∈ lR, e concludiamo che una
base per l’intersezione é {(1, −1, −2, 1)}.
2. L’altro caso é quello in cui non venga data (almeno apparentemente) alcuna
relazione che leghi tra loro le componenti dei vettori degli spazi. Eccone un
esempio:
Siano ancora A e B due sottospazi di lR4 definiti da:
A = {(a, a + b, c, a) : a, b, c ∈ lR}
GLI SPAZI VETTORIALI. 9

B = {(d, e, e, e) : d, e ∈ lR}.
Ancora una volta dobbiamo dapprima determinare la generica espressione
del vettore che appartenga ad entrambi gli spazi: per un tale vettore deve
accadere che
(a, a + b, c, a) = (d, e, e, e)
quindi
a = d, a + b = e, c = e, a=e
che si riduce ancora ad un sistema lineare omogeneo nelle incognite a, b, c, d, e:


 a−d=0
a+b−e=0

.

 c−e=0
a−e=0

La matrice associata al sistema ha rango 4, quindi vi sono ∞1 soluzioni, per


cui lo spazio intersezione ha dimensione 1. Riscrivendo il sistema in funzione
del parametro e, otteniamo:


 a−d=0
a+b=e


 c=e
a=e

da cui a = d = c = e, b = 0. Sostituendo i valori ottenuti arbitrariamente nella


espressione dei vettori in A od in quella in B, si ottiene il vettore appartenente
all’intersezione: esso é (α, α, α, α), al variare del parametro α ∈ lR. Per cui la
base per lo spazio intersezione é {(1, 1, 1, 1)}.

5. Formula di Grassmann. Siano A, B sottospazi vettoriali dello spazio V . Vogliamo


considerare ora la relazione che intercorre tra le dimensioni di A, B, A + B e A ∩ B.
Vale la seguente (formula di Grassmann):
Teorema 4. dim(A + B) = dim(A) + dim(B) − dim(A ∩ B).

Dim. Dividiamo la dimostrazione in due casi:


1. Sia A ⊕ B somma diretta. Se EA = {c1 , .., cr }, EB = {d1 , .., ds } sono rispetti-
vamente basi per A e B, allora EA ∪ EB é un insieme di generatori per A ⊕ B.
É sufficiente quindi dimostrare che {c1 , .., cr , d1 , .., ds } sono linearmente in-
dipendenti. Consideriamo la combinazione lineare
α1 c1 + ... + αr cr + β1 d1 + ... + βs ds = 0.
Poiché la somma é diretta, il vettore 0 si puó esprimere in modo unico come
somma di un vettore di A e di uno in B. Certamente 0 si puó esprimere come
0 = 0 · c1 + .. + 0 · cr + 0 · d1 + ... + 0 · ds
Per cui, confrontando quest’ultima con la precedente combinazione lineare, si
ottiene
α1 = ... = αr = β1 = ... = βs = 0.
Quindi EA ∪EB é una base per A⊕B, per cui dim(A⊕B) = dim(A)+dim(B).
10 GLI SPAZI VETTORIALI.

2. Supponiamo ora che A ∩ B =< c1 , .., ck >, dim(A ∩ B) = k. Per il teorema


sul completamento di una base, esistono vettori indipendenti tra loro d1 , .., dq ,
tali che {c1 , .., ck , d1 , .., dq } sia una base per B. Consideriamo un vettore
v ∈< c1 , .., ck > ∩ < d1 , .., dq >:
k
X q
X
v= αi ci = βj dj
i=1 j=1

e quindi
k
X q
X
αi ci − βj dj = 0.
i=1 j=1
Poiché {c1 , .., ck , d1 , .., dq } sono indipendenti, segue che αi = βj = 0, per ogni
i, j. Allora v = 0, cioé < c1 , .., ck > ∩ < d1 , .., dq >= 0 e
B =< c1 , .., ck > ⊕ < d1 , .., dq >= (A ∩ B)⊕ < d1 , .., dq >
quindi per il punto precedente dim(B) = k + q. Inoltre, se w ∈ A∩ <
d1 , .., dq >, allora w ∈ A, w ∈ B e w ∈< d1 , .., dq >, quindi w ∈ (A ∩ B)∩ <
d1 , .., dq >, per cui w = 0. Alla luce di tutto ció:
A + B = A + ((A ∩ B)⊕ < d1 , .., dq >) = A⊕ < d1 , .., dq >
e passando alle dimensioni:
dim(A + B) = dim(A) + dim(< d1 , .., dq >) = dim(A) + (dim(B) − k) =
dim(A) + dim(B) − dim(A ∩ B).
Proposizione 2. Siano A e B sottospazi vettoriali dello spazio V , e siano CA e
CB rispettivamente una base di A ed una di B. Allora l’unione dei vettori delle due
basi, cioé CA ∪ CB , costituisce un insieme di generatori per il sottospazio A + B.
Inoltre i vettori di CA ∪CB che sono tra loro linearmente indipendenti costituiscono
una base per A + B.
Esempio 27. Siano V = lR4 ,
A = {(x, y, z, t) ∈ lR4 , y = 0, 2z − t = 0}
4
B = {(x, y, z, t) ∈ lR , x − t = 0, y + z = 0}
e calcoliamo dim(A + B).

Svolg. Il primo passo é quello di calcolare basi e dimensioni di A e B. Il generico


vettore di A si esprime (x, 0, z, 2z), al variare di x, z ∈ lR. Allora dim(A) = 2 ed
una sua base é la seguente
(1, 0, 0, 0), (0, 0, 1, 2).
Il generico vettore di B si esprime (x, y, −y, x), al variare di x, y ∈ lR. Allora
dim(B) = 2 ed una sua base é
(1, 0, 0, 1), (0, 1, −1, 0).
Quindi se v ∈ A∩B, esso deve essere esprimibile contemporaneamente in due modi,
cioé
v = (a, 0, b, 2b) = (c, d, −d, c) con a, b, c, d ∈ lR.
Uguagliando le due quaterne si ottiene
a=b=c=d=0
GLI SPAZI VETTORIALI. 11

che significa A ∩ B = {0} e dim(A ∩ B) = 0, da cui


dim(A + B) = dim(A) + dim(B) − dim(A ∩ B) = 2 + 2 − 0 = 4.
Concludiamo allora che A + B = lR4 , come somma diretta.
Esempio 28. Siano V = lR3 ,
A = {(a + b, b, a), a, b ∈ lR}
B = {(x, y, z), x − y = 0}.

Svolg. Si ha che dim(A) = 2 ed una sua base é data da


(1, 0, 1), (1, 1, 0).
Inoltre il generico vettore di B si esprime (x, x, z), quindi dim(B) = 2 ed una sua
base é
(1, 1, 0), (0, 0, 1).
Quindi se v ∈ A∩B, esso deve essere esprimibile contemporaneamente in due modi,
cioé
v = (a + b, b, a) = (c, c, d) con a, b, c, d ∈ lR.
Uguagliando le due terne si ottiene
a = d = 0 e b = c da cui v = (b, b, 0).
Ció vuol dire che dim(A ∩ B) = 1 ed una sua base é data dal vettore (1, 1, 0).
Applicando la formula di Grassmann otteniamo:
dim(A + B) = 2 + 2 − 1 = 3
3
quindi A + B = lR ma non come somma diretta.
Esempio 29. Siano U =< (0, 1, 1), (2, 0, 1) > e W =< (1, 1, 2) > sottospazi di lR3 .
Determiniamo dim(U + W ).

Svolg. Il generico vettore v ∈ U ∩ W si deve esprimere nei due seguenti modi


v = a(0, 1, 1) + b(2, 0, 1) = (2b, a, a + b) ∈ U
v = c(1, 1, 2) = (c, c, 2c) ∈ W.
Uguagliando le due terne otteniamo a = b = c = 0, cioé U ∩ W = {0}, quindi in
base alla formula di Grassmann dim(U + W ) = 2 + 1 − 0 = 3, e U + W = lR3 come
somma diretta.
Esempio 30. Siano
A =< (2, 0, 0, 1), (0, 0, −2, 0), (0, 0, 1, −1) >
B =< (0, 1, 0, 0), (1, 1, 0, 0) >
4
sottospazi di lR .

Svolg. Un generico vettore v ∈ A ∩ B si esprime nei due seguenti modi


v = a(2, 0, 0, 1) + b(0, 0, −2, 0) + c(0, 0, 1, −1) = (2a, 0, −2b + c, a − c) ∈ A
v = d(0, 1, 0, 0) + e(1, 1, 0, 0) = (e, d + e, 0, 0) ∈ B.
Uguagliando le due quaterne si ottiene
d e
a = 2b = c = − =
2 2
12 GLI SPAZI VETTORIALI.

quindi v = (e, 0, 0, 0), al variare di e ∈ lR. Per cui dim(A ∩ B) = 1 e dim(A + B) =


3 + 2 − 1 = 4, cioé A + B = lR4 , ma non come somma diretta.
Esempio 31. Siano
A =< (2, −1, 0, 1), (1, 3, 1, −1), (0, 1, −1, −1) >
B =< (2, 0, 1, 0), (1, 2, 2, 0) >
4
sottospazi di lR . Determiniamo dim(A + B).

Svolg. Un generico vettore v ∈ A ∩ B si esprime nei due seguenti modi


v = a(2, −1, 0, 1)+b(1, 3, 1, −1)+c(0, 1, −1, −1) = (2a+b, −a+3b+c, b−c, a−b−c) ∈ A
v = d(2, 0, 1, 0) + e(1, 2, 2, 0) = (2d + e, 2e, d + 2e, 0) ∈ B.
Uguagliando le due quaterne si ottiene
a=d=0 b = −c = e
quindi v = (e, 2e, 2e, 0), al variare di e ∈ lR. Per cui dim(A∩B) = 1 e dim(A+B) =
3 + 2 − 1 = 4, cioé A + B = lR4 , ma non come somma diretta.

6. Cambiamento di base in uno spazio vettoriale. Sia V uno spazio vettoriale


di dimensione n sul campo lR e siano B = {e1 , e2 , .., en } e B 0 = {e01 , e02 , .., e0n } due
distinte basi di V . Per ogni vettore v ∈ V avremo:
v = x1 e1 + x2 e2 + ... + xn en x1 , .., xn ∈ lR
v= x01 e01
+ x02 e02 + ... + x0n e0n x01 , .., x0n ∈ lR.
Indichiamo allora X = [x1 , .., xn ]T il vettore contenente le componenti di v rispetto
alla base B e X 0 = [x01 , .., x0n ]T quello contenente le componenti di v rispetto alla
0
base B .
In particolare anche i vettori e1 , .., en possono esprimersi come combinazione dei
vettori della base B 0 :

e1 = a11 e01 + a21 e02 + ... + an1 e0n




e2 = a12 e01 + a22 e02 + ... + an2 e0n


.
 ..........
0 0 0

en = a1n e1 + a2n e2 + ... + ann en

Da queste otteniamo:
v = x1 e1 +x2 e2 +...+xn en = x1 (a11 e01 +a21 e02 +...+an1 e0n )+x2 (a12 e01 +a22 e02 +...+an2 e0n )+
+.......................... + xn (a1n e01 + a2n e02 + ... + ann e0n ) =
e01 (a11 x1
+ a12 x2 + ... + a1n xn ) + e02 (a21 x1 + a22 x2 + ... + a2n xn )+
+........................... + e0n (an1 x1 + an2 x2 + ... + ann xn )
che deve essere uguale a v = x01 e01 + x02 e02 + ... + x0n e0n , cioé
 0
x = a11 x1 + a12 x2 + ... + a1n xn
 01


x2 = a21 x1 + a22 x2 + ... + a2n xn
.
 ...........................
 0

xn = an1 x1 + an2 x2 + ... + ann xn
Indichiamo con A la matrice dei coefficienti di questo sistema lineare, A = [aij ],
quindi possiamo riscrivere il sistema nel seguente modo:
X0 = A · X
GLI SPAZI VETTORIALI. 13

che costituiscono le formule di passaggio dalle componenti di un vettore in base B


a quelle del medesimo vettore in base B 0 .
La matrice A é detta matrice del cambiamento di base ed é costruita come segue:
- nella prima colonna vi sono le componenti del vettore e1 rispetto alla base B 0 ;
- nella seconda colonna vi sono le componenti del vettore e2 rispetto alla base
B0;
- In generale, nella colonna j vi sono le componenti del vettore ej della base B,
calcolate rispetto alla base B 0 .

Poiché gli n vettori di una base sono sempre linearmente indipendenti, il sistema
sopra citato ha rango massimo, cioé n, quindi la matrice A é invertibile, da cui
otteniamo le formule inverse per il cambiamento di base:
X = A−1 · X 0
che costituiscono le formule di passaggio dalle componenti di un vettore in base B 0
a quelle del medesimo vettore in base B.
La matrice A−1 é costruita come segue:
- nella prima colonna vi sono le componenti del vettore e01 rispetto alla base B;
- nella seconnda colonna vi sono le componenti del vettore e02 rispetto alla base
B;
- In generale, nella colonna j vi sono le componenti del vettore e0j della base B 0 ,
calcolate rispetto alla base B.
Esempio 32. Siano V = lR2 e B = {e1 = (1, 1), e2 = (0, 1)}, B 0 = {e01 =
(1, 0), e02 = (2, 1)} due basi di V . Determiniamo le formule di cambiamento di base
in entrambi i versi. Calcoliamo le componenti dei vettori della prima base rispetto
alla seconda.

Svolg. Le componenti di e1 = (1, 1) rispetto a B 0 sono (e1 )B 0 = (−1, 1) infatti


(1, 1) = (−1)(1, 0) + (1)(2, 1)
analogamente le componenti di e2 = (0,1) rispetto a B 0 sono (e2 )B 0 = (−2, 1).
−1 −2
Per cui, la matrice A = é quella che determina il passaggio dalla
1 1
0
base B a quella B , cioé
 0     
x1 −1 −2 x1
= ·
x02 1 1 x2
e le formule di passaggio sono
x01 = −x1 − 2x2

.
x02 = x1 + x2
Le formule inverse sono date da
     0 
x1 1 2 x1
= ·
x2 −1 −1 x02

x1 = x01 + 2x02

x2 = −x01 − x02
14 GLI SPAZI VETTORIALI.

 
1 2
in cui la matrice di passaggio é A−1 = . Per esempio consideriamo il
−1 −1
vettore v ∈ V che abbia componenti X = (x1 , x2 ) = (2, −3) rispetto alla base B.
Determiniamo le sue componenti X 0 = (x01 , x02 ) rispetto alla base B 0 :

x01 = −2 + 6 = 4

.
x02 = 2 − 3 = −1

Esempio 33. Siano V = lR3 ,


B = {(1, 1, 0), (1, 0, 1), (2, 0, 1)}, B 0 = {(0, 1, 1), (2, −1, 0), (1, 0, 2)}
due basi di V . Determiniamo le formule di cambiamento di base.

Svolg. Calcoliamo le componenti dei vettori della prima base B rispetto alla sec-
onda B 0 :
(1, 1, 0) → (2, 1, −1)B 0
1 1 1
(1, 0, 1) → ( , , )B 0
3 3 3
(2, 0, 1) → (1, 1, 0)B 0
per cui
x01 1
     
2 3 1 x1
 x02  =  1 1
1  ·  x2 
3
0
x3 −1 13 0 x3
 0
 x1 = 2x1 + 13 x2 + x3
x0 = x1 + 13 x2 + x3 .
 20
x3 = −x1 + 13 x2
Per esempio il vettore v di componenti (1, 0, 3) rispetto alla base B avrá componenti
(5, 4, −1) rispetto alla base B 0 .
Esempio 34. Siano V = lR3 e B = {e1 , e2 , e3 }, B 0 = {e01 , e02 , e03 } due basi di V tali
che  0
 e1 = e1 + 3e2 + 2e3
e02 = e1 + e3 .
e03 = e2

Svolg. In tale caso le formule di passaggio dalla base B 0 alla base B sono
     0 
x1 1 1 0 x1
 x2  =  3 0 1  ·  x02 
x3 2 1 0 x03
e calcolando l’inversa della matrice che compare nel sistema precedente:
 0     
x1 −1 0 1 x1
 x02  =  2 0 −1  ·  x2 
x03 3 1 −3 x3
GLI SPAZI VETTORIALI. 15

Esempio 35. Siano V = lR3 ,


B = {(1, 1, 0), (1, 0, 1), (2, 0, 1)}, B 0 = {(1, 1, 1), (0, 0, 1), (1, 0, 2)}
due basi di V . Sia v ∈ V un vettore di componenti (1, 1, 2) rispetto alla base
B. Indichiamo (a, b, c) le componenti di v rispetto alla base B 0 . Per determinare
(a, b, c) applichiamo ora esattamente la definizione di componenti:
(1)(1, 1, 0) + (1)(1, 0, 1) + (2)(2, 0, 1) = a(1, 1, 1) + b(0, 0, 1) + c(1, 0, 2)
cioé
(6, 1, 3) = (a + c, a, a + b + 2c)
da cui a = 1, b = −8, c = 5.

7. Esercizi.
Esercizio 1. Siano U = {(x, y, 0); x, y ∈ lR}, V = {(x, 0, z); x, z ∈ lR} e W =
{(x, x, x); x ∈ lR} sottospazi di lR3 . Determinare i sottospazi U ∩ V , U ∩ W , V ∩ W .
Esercizio 2. Siano U = {(x, y, 0); x, y ∈ lR}, V = {(x, 0, z); x, z ∈ lR} e W =
{(x, x, x); x ∈ lR} sottospazi di lR3 . Determinare i sottospazi U + V , U + W ,
V + W.
Esercizio 3. Siano v1 = (−1, 2, 3), v2 = (0, −1, 0), v3 = (1, 0, 1) vettori di lR3 .
Determinare la dimensione del sottospazio generato da v1 , v2 , v3 .
Esercizio 4. Ripetere l’esercizio precedente in lR4 con i vettori v1 = (1, 2, 1, 0),
v2 = (1, −1, 0, 1), v3 = (−1, 2, −1, 0), v4 = (−1, 1, 0, −1), v5 = (1, 1, 0, 1).
Esercizio 5. Siano v1 = (1, 2, 0, 0), v2 = (3, 1, 0, 1) vettori indipendenti in lR4 .
Determinare due vettori che uniti ai precedenti li completino ad una base di lR4 .
Esercizio 6. Siano B1 = {(1, 0), (0, 1)} e B2 = {(1, −2), (4, 1)} due basi di lR2 e
v un vettore di componenti (0, −1) rispetto a B1 . Determinare le componenti di v
rispetto alla base B2 .
Esercizio 7. Siano U = {(x, y, z, t) ∈ lR4 ; y = 0, 2z − t = 0} e V = {(x, y, z, t) ∈
lR4 ; x − t = 0, y + z = 0}. Determinare una base per U ∩ V ed una per U + V .
Esercizio 8. Siano U = {(h + k, k, h); h, k ∈ lR}, V = {(x, y, z); x − y = 0}
sottospazi di lR3 . Determinare una base per U ∩ V ed una per U + V .
Esercizio 9. Siano B1 = {(1, 1), (0, 1)} e B2 = {(1, 0), (2, 1)} due basi di lR2 e
v un vettore di componenti (1, 1) rispetto a B1 . Determinare le componenti di v
rispetto alla base B2 .
Esercizio 10. Siano
B1 = {(1, 1, 0), (1, 0, 1), (2, 0, 1)} e B2 = {(0, 1, 1), (2, −1, 0), (1, 0, 0)}
due basi di lR3 e v un vettore di componenti (1, 2, 3) rispetto a B1 . Determinare le
componenti di v rispetto alla base B2 .
Esercizio 11. Siano B1 = {e1 , e2 , e3 } e B2 = {e01 , e02 e03 } due basi di lR3 e v un vet-
tore di componenti (1, 2, 3) rispetto a B1 . Determinare le componenti di v rispetto
alla base B2 sapendo che valgono le seguenti relazioni tra i vettori delle due basi:
e01 = e1 + 3e2 + 2e3 , e02 = e1 + e3 , e03 = e2 .
16 GLI SPAZI VETTORIALI.

Esercizio 12. Siano


B1 = {(1, 0, 0), (0, 1, 0), (0, 0, 1)} e B2 = {(1, 4, 0), (1, 5, 0), (0, 0, −1)}
3
due basi di lR e v un vettore di componenti (−1, 0, 5) rispetto a B1 . Determinare
le componenti di v rispetto alla base B2 .
Esercizio 13. Determinare la dimensione e la base del sottospazio: W = {(x, y, z, t) ∈
lR4 ; 2x − t = 0, y + 3z = 0}.
Esercizio 14. Determinare la dimensione e la base del sottospazio: W = {(x, y, z, t) ∈
lR4 ; 2x − t + 2z = 0}.
Esercizio 15. Determinare la dimensione e la base del sottospazio: W = {(x, y, z, t) ∈
lR4 ; x − y = 0, y + 3z + t = 0}.
Esercizio 16. Siano U =< {(0, 1, 1), (2, 0, 1)} > e V =< {(1, 1, 2)} > sottospazi
di lR3 . Determinare la dimensione di U + V .
Esercizio 17. Siano
U =< {(2, 0, 0, 1), (0, 0, −2, 0), (0, 0, 1, −1)} >, V =< {(0, 1, 0, 0), (1, 1, 0, 0)} >
4
sottospazi di lR . Determinare la dimensione di U + V .
Esercizio 18. Determinare la dimensione della somma dei sottospazi
     
2 −1 1 3 0 1
A =< , , >
0 1 1 −1 −1 −1
   
2 0 1 2
B =< , >.
1 0 2 0
LE APPLICAZIONI LINEARI.

1. Introduzione. Siano U e V due spazi vettoriali sul campo K e sia f : U → V


una corrispondenza tra i vettori di U e quelli di V . Diremo che f é una applicazione
lineare se, per ogni a, b ∈ K, u1 , u2 ∈ U si ha:
f (au1 + bu2 ) = af (u1 ) + bf (u2 ).
Lo spazio vettoriale U é detto dominio di f , V é detto codominio di f .

Esempio 1. f : U → U , tale che f (u) = u, per ogni u ∈ U , é una applicazione


lineare, detta identitá di U .
Esempio 2. f : U → U , tale che f (u) = 0, vettore nullo di U , per ogni u ∈ U , é
una applicazione lineare, detta applicazione nulla in U .
Esempio 3. Sia a ∈ lR, f : U → U , tale che f (u) = au, per ogni u ∈ U , é una
applicazione lineare, detta omotetia in U di rapporto a.
Esempio 4. f : lR2 → lR3 , tale che, per ogni X = (x1 , x2 ) ∈ lR, f (X) =
f (x1 , x2 ) = (x1 , x1 + x2 , x1 − x2 ) ∈ lR3 , é una applicazione lineare.
Osservazione 1. Per uniformarci alla trattazione degli Spazi Vettoriali nei prece-
denti capitoli, ci occuperemo esclusivamente di applicazioni leneari tra spazi vetto-
riali definiti sul campo K = lR dei numeri reali.
Definizione 1. Sia f : U → V una applicazione lineare. Diciamo Immagine di f
il seguente sottoinsieme di V :
Im(f ) = {v ∈ V : ∃u ∈ U, f (u) = v}.
É facile osservare che Im(f ) é un sottospazio vettoriale di V .
Esempio 5. f : lR3 → lR4 sia definita da f (x1 , x2 , x3 ) = (x1 , 0, x3 , 0). Allora si ha
Im(f ) = {(a, 0, b, 0) : a, b ∈ lR} =< (1, 0, 0, 0), (0, 0, 1, 0) >
4
sottospazio di dimensione 2 di lR .
Esempio 6. f : lR4 → lR3 sia definita da f (x1 , x2 , x3 , x4 ) = (x1 + x2 , x2 − x3 , x1 +
x3 ). Sia w ∈ Im(f ), allora w = (a1 , a2 , a3 ) tali che

 x1 + x2 = a1
x2 − x3 = a2
x1 + x3 = a3

da cui ricaviamo che a1 = a2 +a3 , per cui w = (a2 +a3 , a2 , a3 ), per ogni a1 , a2 , a3 ∈
lR.
Im(f ) = {(a2 + a3 , a2 , a3 ) : a1 , a2 , a3 ∈ lR} =< (1, 1, 0), (1, 0, 1) >
3
sottospazio di dimensione 2 di lR .

1
2 LE APPLICAZIONI LINEARI.

Esempio 7. f : lR3 → lR4 sia definita da f (x1 , x2 , x3 ) = (x1 , x1 + x2 − x3 , 2x1 +


x2 − x3 , x2 − x3 ). Sia w ∈ Im(f ), allora w = (a1 , a2 , a3 , a4 ) tali che


 x1 = a1
x1 + x2 − x3 = a2


 2x1 + x2 − x3 = a3
x2 − x3 = a4

da cui ricaviamo che a2 = a1 + a4 e a3 = 2a1 + a4 per cui w = (a1 , a1 + a4 , 2a1 +


a4 , a4 ), per ogni a1 , a4 ∈ lR.
Im(f ) = {(a1 , a1 + a4 , 2a1 + a4 , a4 ) : a1 , a4 ∈ lR} =< (1, 1, 2, 0), (0, 1, 1, 1) >
4
sottospazio di dimensione 2 di lR .
Esempio 8. f : lR4 → lR3 sia definita da f (x1 , x2 , x3 , x4 ) = (x1 − x2 , 2x1 − x2 +
x4 , −x1 + x2 + x4 ). Sia w ∈ Im(f ), allora w = (a1 , a2 , a3 ) tali che

 x1 − x2 = a1
2x1 − x2 + x4 = a2
−x1 + x2 + x4 = a3

da cui ricaviamo che a2 = a1 + x1 + x4 e a3 = −a1 + x4 per cui, detti x1 = b1 e


x4 = b2 , w = (a1 , a1 + b1 + b2 , −a1 + b2 ), per ogni a1 , b1 , b2 ∈ lR.
Im(f ) = {(a1 , a1 +b1 +b2 , −a1 +b2 ) : a1 , b1 , b2 ∈ lR} =< (1, 1, 1), (0, 1, 0), (0, 1, 1) >
cioé Im(f ) = lR3 .
Definizione 2. Una applicazione lineare f é detta suriettiva quando l’immagine
di f coincide con tutto il codominio di f .
Definizione 3. Diciamo Nucleo di una applicazione lineare f : U → V , il seguente
sottoinsieme del dominio:
N (f ) = {u ∈ U : f (u) = 0V }
cioé l’insieme dei vettori di U che hanno come immagine in V il vettore nullo. Si
dimostra facilmente che N (f ) é un sottospazio vettoriale di U .
Esempio 9. f : lR3 → lR4 sia definita da f (x1 , x2 , x3 ) = (x1 , 0, x3 , 0). Sia u ∈
N (f ), allora f (u) = (0, 0, 0, 0) quindi

x1 = 0
x3 = 0
da cui u = (0, x2 , 0)
N (f ) = {(0, a, 0) : a ∈ lR} =< (0, 1, 0) >
3
sottospazio di dimensione 1 di lR .
Esempio 10. f : lR4 → lR3 sia definita da f (x1 , x2 , x3 , x4 ) = (x1 +x2 , x2 −x3 , x1 +
x3 ). Sia u ∈ N (f ), allora f (u) = (0, 0, 0) quindi

 x1 + x2 = 0
x2 − x3 = 0
x1 + x3 = 0

da cui x1 = −x2 = −x3 e u = (x1 , −x1 , −x1 , x4 )


N (f ) = {(a, −a, −a, b) : a, b ∈ lR} =< (1, −1, −1, 0), (0, 0, 0, 1) >
LE APPLICAZIONI LINEARI. 3

sottospazio di dimensione 2 di lR4 .


Esempio 11. f : lR3 → lR4 sia definita da f (x1 , x2 , x3 ) = (x1 , x1 + x2 − x3 , 2x1 +
x2 − x3 , x2 − x3 ). Sia u ∈ N (f ), allora f (u) = (0, 0, 0, 0) quindi


 x1 = 0
x1 + x2 − x3 = 0


 2x1 + x2 − x3 = 0
x2 − x3 = 0

da cui u = (0, x2 , x2 )
N (f ) = {(0, a, a) : a ∈ lR} =< (0, 1, 1) >
3
sottospazio di dimensione 1 di lR .
Esempio 12. f : lR4 → lR3 sia definita da f (x1 , x2 , x3 , x4 ) = (x1 − x2 , 2x1 − x2 +
x4 , −x1 + x2 + x4 ). Sia u ∈ N (f ), allora f (u) = (0, 0, 0, 0) quindi

 x1 − x2 = 0
2x1 − x2 + x4 = 0
−x1 + x2 + x4 = 0

da cui x1 = x2 = x4 = 0 e u = (0, 0, x3 , 0)
N (f ) = {(0, 0, a, 0) : a ∈ lR} =< (0, 0, 1, 0) >
4
sottospazio di dimensione 1 di lR .

Definizione 4. Una applicazione lineare f : U → V é detta iniettiva quando per


ogni u1 6= u2 in U si ha: f (u1 ) 6= f (u2 ) in V .

Teorema 1. Una applicazione lineare f : U → V é iniettiva se e solo se N (f ) =


{0}, cioé se il nucleo di f é il sottospazio banale di U .

2. Applicazioni lineari e matrici. Siano U e V spazi vettoriali su lR, tali da


avere dim(U ) = n e dim(V ) = m e
B = {e1 , .., en } una base per U
B 0 = {e01 , .., e0m } una base per V.
Sia f : U → V una applicazione lineare. Ogni vettore X ∈ U ha delle componenti
(x1 , .., xn ) rispetto alla base B, cioé X = x1 e1 + ... + xn en , ed ogni vettore Y ∈ V
ha delle componenti (y1 , .., ym ) rispetto alla base B 0 , cioé Y = y1 e01 + ... + ym e0m .
In particolare sia Y = f (X) ∈ V , quindi
Y = f (x1 e1 + ... + xn en ) = x1 f (e1 ) + x2 f (e2 ) + .. + xn f (en ).
Poiché i vettori f (e1 ), .., f (en ) sono in V , essi sono esprimibili per componenti
rispetto alla base B 0 :
f (e1 ) = a11 e01 + a21 e02 + ... + am1 e0m
f (e2 ) = a12 e01 + a22 e02 + ... + am2 e0m
...........
f (en ) = a1n e01 + a2n e02 + ... + amn e0m
4 LE APPLICAZIONI LINEARI.

da cui
Y = x1 (a11 e01 + a21 e02 + ... + am1 e0m ) + x2 (a12 e01 + a22 e02 + ... + am2 e0m )+
+.............................. + xn (a1n e01 + a2n e02 + ... + amn e0m ) =

e01 (a11 x1 + a12 x2 + ... + a1n xn ) + e02 (a21 x1 + a22 x2 + ... + a2n xn )+
+............................... + e0n (am1 x1 + am2 x2 + ... + amn xn ).
Se indichiamo
   
  y1 x1
a11 a12 ... a1n  y2   x2 
 a21 a22 ... a2n     
A=   , Y =  ...  , X = 
  ... 
... ... ... ... 
 ... 

 ... 

am1 am2 ... amn
ym xn
allora f (X) = Y si puó scrivere nel modo seguente:
A·X =Y
e la matrice A é la matrice associata all’applicazione lineare f rispetto alle basi B
e B 0 . Tale matrice é ottenuta nel seguente modo: la sua colonna j é formata dalle
componenti rispetto alla base B 0 dell’immagine f (ej ) del vettore ej della base B.
Concludiamo allora che ad ogni applicazione lineare f é associata una matrice che
dipende dalle basi scelte per il dominio e codominio di f .

Viceversa sia data una qualsiasi matrice A ∈ Mmn (lR) e siano al solito U e
V spazi vettoriali rispettivamente di dimensioni n e m. Costruiamo la seguente
corrispondenza tra i due spazi vettoriali:
f (X) = A · X ∈ V, per ogni X ∈ U.
Essa é ovviamente una applicazione lineare. Quindi ad ogni matrice é associata
una applicazione lineare.

Quanto detto fin’ora si puó riassumere nel modo seguente: esiste una corrispon-
denza che associa ad ogni matrice una applicazione lineare e viceversa.

Osservazione 2. Si noti che se Y ∈ f (U ), allora esso é generato dall’insieme


{f (e1 ), ..., f (en )}, che quindi é un generatore per l’immagine della f . Ció vuol dire
che le colonne della matrice associata ad una applicazione lineare costituiscono un
insieme di generatori per Im(f ). Allora una base dell’immagine si puó ricavare
facilmente, considerando i vettori colonna della matrice associata, che siano tra
loro indipendenti.
Teorema 2. Sia data una applicazione lineare f : U → V e sia S = {v1 , .., vr } ⊆ U
un sottoinsieme di S formato da vettori linearmente dipendenti. Allora l’insieme
f (S) = {f (v1 ), .., f (vr )} ⊆ V é anche esso formato da vettori linearmente dipen-
denti.

Dim. Siano {v1 , .., vr } vettori P linearmente dipendenti, quindi esistano α1 , . . . , αr


r
scalari non tutti P nulli tali che P i=1 αi vi = 0. Applichiamo la f alla precedente
r r
relazione: 0 = f ( i=1 αi vi ) = i=1 αi f (vi ) che implica la dipendenza lineare dei
vettori {f (v1 ), . . . , f (vr )}.
LE APPLICAZIONI LINEARI. 5

Teorema 3. Sia data una applicazione lineare f : U → V e sia S = {v1 , .., vr } ⊆ U


un sottoinsieme di S formato da vettori linearmente indipendenti. Allora l’insieme
f (S) = {f (v1 ), .., f (vr )} ⊆ V é anche esso formato da vettori linearmente indipen-
denti se e solo se f é iniettiva.

Dim. Supponiamo dapprima che f sia iniettiva.


Se {f (v1 ), . . . , f (vr )} sono linearmente Pr dipendenti allora esistono P opportuni scalari
α1 , . . . , αr non tutti nulli Pr tali che α
i=1 i f (vi ) = 0, cioé f ( i i i )) = 0. Questo
α (v
implica che il vettore i=1 αi (viP ) sia nel nucleo di f . Per ipotesi (f iniettiva con
r
nucleo banale) allora avremo che i=1 αi (vi ) = 0, cioé i vettori {v1 , .., vr } costituis-
cono un sottoinsieme di formato da vettori linearmente dipendenti, che contraddice
l’ipotesi.
Supponiamo ora che {f (v1 ), . . . , f (vr )} sono linearmente indipendenti. Per as-
surdo ipotizziamo che f non siainiettiva, quindi esista un abase per il suo nucleo
N =< c1 , . . . , ck >. Completiamo tale base ad una base dello spazio V : V =<
c1 , . . . , ck , b1 , . . . , bh >. Per l’ipotesi, l’insieme < f (c1 ), . . . , f (ck ), f (b1 ), . . . , f (bh ) >
é composto da vettori indipendenti, il che non puó accadere poiché f (c1 ) = f (c2 ) =
. . . = f (ck ) = 0.

Teorema 4. Sia data una applicazione lineare f : U → V . Allora dim(U ) =


dim(N (f )) + dim(Im(f )).

Dim. Indichiamo dim(U ) = n. Se N (f ) = 0 allora f é iniettiva e sappiamo


che dato un insieme di vettori indipendenti B = {e1 , ..., en } nel dominio, i vettori
{f (e1 ), ..., f (en )} sono indipendenti nel codominio. Inoltre se B é una base di U
allora é noto anche che {f (e1 ), ..., f (en )} sono generatori di Im(f ). Unendo le due
cose otteniamo che {f (e1 ), ..., f (en )} é esattamente una base per Im(f ), da cui
l’asserto.
Poniamoci allora nel caso in cui N (f ) 6= 0. Essendo N (f ) un sottospazio di
U , indichiamo con {b1 , ..., br } una sua base. Siano ora {cr+1 , ..., cn } vettori in-
dipendenti in U tali che {b1 , .., br , cr+1 , ..., cn } sia una base per U . Ancora richi-
amiamo il risultato per cui l’insieme {f (b1 ), ..., f (br ), f (cr+1 ), ..., f (cn )} é un in-
sieme di generatori per Im(f ). Poiché b1 , .., br appartengono al nucleo di f , allora
f (b1 ) = f (b2 ) = ... = f (br ) = 0, per cui l’insieme {f (cr+1 ), ..., f (cn )} é un in-
sieme di generatori per Im(f ). Dimostriamo ora che essi sono anche linearmente
indipendenti. Consideriamo la combinazione lineare

αr+1 f (cr+1 ) + ..... + αn f (cn ) = 0;

dalla linearitá di f abbiamo che

f (αr+1 cr+1 + ..... + αn cn ) = 0

cioé il vettore αr+1 cr+1 + ..... + αn cn appartiene al nucleo N (f ), che contraddice


il fatto che i vettori cr+1 , ..., cn debbano essere indipendenti con i vettori b1 , .., br .
Questo vuol dire che αr+1 cr+1 + ..... + αn cn = 0, e dalla indipendenza lineare dei
vettori cr+1 , ..., cn segue che αi = 0, per ogni i = r + 1, ..., n.
Infine, essendo {cr+1 , ..., cn } una base per Im(f ), alora dim(Im(f )) = n − r =
dim(U ) + dim(N (f )).
6 LE APPLICAZIONI LINEARI.

Esempio 13. f : lR3 → lR4 sia definita da f (x1 , x2 , x3 ) = (x1 , x1 + x2 − x3 , 2x1 +


x2 − x3 , x2 − x3 ). Abbiamo visto in un esempio precedente che
Im(f ) = {(a1 , a1 + a4 , 2a1 + a4 , a4 ) : a1 , a4 ∈ lR} =< (1, 1, 2, 0), (0, 1, 1, 1) >
sottospazio di dimensione 2 di lR4 . Ma anche
N (f ) = {(0, a, a) : a ∈ lR} =< (0, 1, 1) >
3
é sottospazio di dimensione 1 di lR .
Esempio 14. Sia f : lR2 → lR3 definita da
f (x1 , x2 ) = (x1 + 2x2 , 3x2 , x1 ).
Determiniamo la matrice associata alla f rispetto alle basi canoniche sia nel do-
minio che nel codominio.

Svolg.
f (1, 0) = (1, 0, 1)
f (0, 1) = (2, 3, 0)
 
1 2
A =  0 3 .
1 0
Il rango della matrice é 2, quindi dim(Im(f )) = 2 e Im(f ) =< (1, 0, 1), (2, 3, 0) >.
Perció dim(N (f )) = 0 e N (f ) = {0}. L’applicazione é iniettiva ma non suriettiva.
Esempio 15. Ripetiamo l’esempio precedente ma ora esprimiamo la matrice asso-
ciata a due basi differenti da quelle canoniche:
B2 = {(1, 1), (2, 1)} base nel dominio
B3 = {(1, 1, 0), (1, 0, 0), (1, 2, 1)} base nel codominio.

Svolg. Per costruire tale matrice dobbiamo calcolare le immagini dei vettori della
base B2 . Tali immagini si possono calcolare sfruttando la matrice A associata alla
f rispetto alle basi canoniche:
   
1 2   3
1
f (1, 1) =  0 3  · = 3 
1
1 0 1
   
1 2   4
2
f (2, 1) =  0 3  · =  3 .
1
1 0 2
Ora conosciamo le immagini,ma esse sono espresse rispetto alla base canonica di
lR3 , dobbiamo quindi convertirle rispetto alla base B3 :
(3, 3, 1) = (1, 1, 1)B3
(4, 3, 2) = (−1, 3, 2)B3
quindi la matrice associata alla f rispetto alle basi B2 e B3 é
 
1 −1
A0 =  1 3 
1 2
LE APPLICAZIONI LINEARI. 7

da cui    
1 −1   x1 − x2
x1
f (X) =  1 3  · =  x1 + 3x2 
x2
1 2 x1 + 2x2
é l’espressione dell’applicazione lineare rispetto a tali basi. Si noti che le dimen-
sioni di Nucleo e Immagine sono invarianti rispetto ad un cambiamento di basi nel
dominio e nel codominio.
Esempio 16. Siano B = {e1 , e2 , e3 } la base canonica di lR3 e f : lR3 → lR4 una
applicazione lineare tale che
f (e1 ) = (1, 1, 1, 0)
f (e2 ) = (1, 0, 1, 0)
f (e3 ) = (0, 0, 0, 1).

Svolg. La matrice associata a f rispetto alla base canonica in lR4 é allora


 
1 1 0
 1 0 0 
A=  1

1 0 
0 0 1
che ha rango 3, quindi dim(Im(f )) = 3 (f non é suriettiva) e dim(N (f )) = 0 (f é
iniettiva), inoltre l’espressione di f rispetto alle basi canoniche sia nel dominio che
nel codominio é
   
1 1 0   x1 + x2
 1 0 0  x 1  x2 
f (X) =   1 1 0  · x2 =  x1 + x2  .
    
x3
0 0 1 x3
Esempio 17. Sia f : lR5 → lR3 definito da
f (x1 , x2 , x3 , x4 , x5 ) = (x1 − x4 , x2 − x4 , x3 ).

Svolg. La matrice associata rispetto alle basi canoniche in lR5 e lR3 é


 
1 0 0 −1 0
A= 0 1 0 −1 0 
0 0 1 0 0
che ha rango 3. Quindi dim(Im(f )) = 3 (f é suriettiva) e dim(N (f )) = 2 (f non
é iniettiva).
Una base dell’immagine é data da
(1, 0, 0), (0, 1, 0), (0, 0, 1).
Il generico vettore del Nucleo, X = (x1 , x2 , x3 , x4 , x5 ) é dato da x3 = 0, x1 = x2 =
x4 , per cui
N (f ) = {(x1 , x1 , 0, x1 , x5 ), x1 , x5 ∈ lR} =< (1, 1, 0, 1, 0), (0, 0, 0, 0, 1) > .

Esempio 18. Sia f : lR3 → lR4 definito da


f (x1 , x2 , x3 ) = (5x1 + 4x2 − 9x3 , 4x1 + 5x2 − 9x3 , −9x1 − 9x2 + 9x3 , x1 + x2 + x3 ).
8 LE APPLICAZIONI LINEARI.

Svolg. La matrice associata rispetto alle basi canoniche in lR3 e lR4 é


 
5 4 −9
 4 5 −9 
A=  −9 −9 9  .

1 1 1
Determiniamo la matrice associata a f rispetto alla base
B = {(1, 1, 0), (1, 0, −1), (0, 1, −1)} di lR3
ed alla base canonica in lR4 .
Si devono calcolare le immagini dei vettori di B:
   
5 4 −9   9
1
 4 5 −9   9
· 1 =

f (1, 1, 0) = 
 −9

−9 9   −18 
0
1 1 1 2
ed in modo analogo
f (1, 0, −1) = (14, 13, −18, 0) f (0, 1, −1) = (13, 14, −18, 0).
Allora la matrice associata a f rispetto a tali basi é
 
9 14 13
9 13 14
A0 = 
 
.
 −18 −18 −18 
2 0 0

Esempio 19. Sia f : lR4 → lR2 con matrice associata


 
1 0 0 1
A=
−1 1 2 −1
rispetto alle basi
B4 = {(1, 1, 0, 0), (1, 0, 0, 0), (2, 0, 0, 1), (0, 0, 1, 0)} in lR4
B2 = {(1, 1), (1, 0)} in lR2 .
Determiniamo la matrice associata a f rispetto alle basi canoniche sia nel dominio
che nel codominio.

Svolg. Per prima cosa esprimiamo i vettori della base canonica di lR4 per compo-
nenti rispetto alla base B4 :
(1, 0, 0, 0) = (0, 1, 0, 0)B4
(0, 1, 0, 0) = (1, −1, 0, 0)B4
(0, 0, 1, 0) = (0, 0, 0, 1)B4
(0, 0, 0, 1) = (0, −2, 1, 0)B4 .
Quindi calcoliamo le immagini di tali vettori utilizzando la matrice A:
 
  0  
1 0 0 1  1  0
f (0, 1, 0, 0) = · =
 
−1 1 2 −1 0 1
0
LE APPLICAZIONI LINEARI. 9

ed analogamente
f (1, −1, 0, 0) = (1, −2)
f (0, 0, 0, 1) = (1, −1)
f (0, −2, 1, 0) = (0, 0).
Tali immagini sono espresse per componenti rispetto alla base B2 e quindi dobbiamo
ora convertirle per componenti rispetto alla base canonica C2 di lR2 :
(0, 1)B2 = (1, 0)C2
(1, −2)B2 = (−1, 1)C2
(1, −1)B2 = (0, 1)C2
(0, 0)B2 = (0, 0)C2 .
La matrice associata rispetto alle basi canoniche é allora
 
0 1 −1 0 0
A =
0 1 1 0
e l’espressione dell’applicazione lineare nelle basi canoniche di dominio e codomino
é  
  x1  
1 −1 0 0  x2  x1 − x2
f (x1 , x2 , x3 , x4 ) = ·
  = .
0 1 1 0 x3  x2 + x3
x4

Esempio 20. Siano f : lR4 → lR4 e


B = {(1, 0, 0, −1), (1, 0, 0, 0), (0, 1, 1, 0), (0, 0, 1, 0)}
4
un base di lR , tali che
f (1, 0, 0, −1) = (2, 0, 0, −1)
f (1, 0, 0, 0) = (−1, 0, 1, 0)
f (0, 1, 1, 0) = (1, 0, 1, −1)
f (0, 0, 1, 0) = (1, 1, 2, 0).
La matrice associata a f rispetto alla base B nel dominio e alla base canonica nel
codominio é  
2 −1 1 1
 0 0 0 1 
A=  0
.
1 1 2 
−1 0 −1 0
Determiniamo la matrice associata a f rispetto alla base B anche nel codominio.

Svolg. Si devono esprimere le immagini dei vettori di B per componenti rispetto


alla base B stessa, per cui, detta C la base canonica di lR4 , si ha:
(2, 0, 0, −1)C = (1, 1, 0, 0)B
(−1, 0, 1, 0)C = (0, −1, 0, 1)B
(1, 0, 1, −1)C = (1, 0, 0, 1)B
(1, 1, 2, 0)C = (0, 1, 1, 1)B
10 LE APPLICAZIONI LINEARI.

e la matrice associata é  
1 0 1 0
 1 −1 0 1 
A0 =   0 0 0 1 .

0 1 1 1
Infine determiniamo la matrice relativa ad f rispetto alla base canonica sia nel
dominio che nel codominio:
Per prima cosa esprimiamo i vettori della base canonica C per componenti
rispetto alla base B:
(1, 0, 0, 0) = (0, 1, 0, 0)B
(0, 1, 0, 0) = (0, 0, 1, −1)B
(0, 0, 1, 0) = (0, 0, 0, 1)B
(0, 0, 0, 1) = (−1, 1, 0, 0)B
quindi calcoliamo le immagini di tali vettori tramite la matrice A:
     
2 −1 1 1 0 −1
 0 0 0 1   1   0 
f (0, 1, 0, 0) =  · = 
 0 1 1 2   0   1 
−1 0 −1 0 0 0
ed analogamente
f (0, 0, 1, −1) = (0, −1, −1, −1)
f (0, 0, 0, 1) = (1, 1, 2, 0)
f (−1, 1, 0, 0) = (−3, 0, 1, 1)
ed é chiaro che, avendo usato la matrice A, i risultati sono giá espressi per compo-
nenti rispetto alla base canonica. Quindi la matrice associata a f rispetto alla base
canonica sia nel dominio che nel codominio é
 
−1 0 1 −3
 0 −1 1 0 
A00 = 
 1 −1 2 1 

0 −1 0 1
ed esprimiamo f come segue:
     
−1 0 1 −3 x1 −x1 + x3 − 3x4
 0 −1 1 0   x2   −x2 + x3 
f (x1 , x2 , x3 , x4 ) =  · =
  x1 − x2 + 2x3 + x4  .

 1 −1 2 1   x3
0 −1 0 1 x4 −x2 + x4

Esempio 21. Sia f : lR3 → lR3 associato alla matrice


 
1 1 2
A= 2 1 3 
3 1 4
rispetto alle basi
B = {(1, 1, 0), (1, 0, 1), (0, 0, 1)} nel dominio
C = {(1, 0, −1), (1, 0, 1), (0, 1, 0)} nel codominio.
Determiniamo una base per l’immagine in base canonica.
LE APPLICAZIONI LINEARI. 11

Svolg. Poiché il rango della matrice A é 2, la dimensione dell’immagine é appunto


2.
Le colonne (1, 2, 3) e (1, 1, 1) sono indipendenti, quindi esse rappresentano i 2
vettori che generano l’immagine.
Per definizione, essi compaiono nella matrice espressi per componenti rispetto
alla base C, quindi per ottenere i vettori che generano l’immagine dobbiamo es-
primerli rispetto alla base canonica:
(1, 2, 3)C = 1(1, 0, −1) + 2(1, 0, 1) + 3(0, 1, 0) = (3, 3, 1)
(1, 1, 1)C = 1(1, 0, −1) + 1(1, 0, 1) + 1(0, 1, 0) = (2, 1, 0).
Allora Im(f ) =< (3, 3, 1), (2, 1, 0) >.
Esempio 22. Siano f : lR3 → lR2 ,
B3 = {(1, 0, 1), (0, 2, 0), (1, 0, −1)} una base di lR3
B2 = {(1, 1), (2, 3)} una base di lR2
 
10 4 −2
A=
−3 −2 1
la matrice associata a f rispetto alla base B3 nel dominio ed alla base B2 nel
codominio. Determiniamo la matrice associata a f rispetto alle basi canoniche sia
nel dominio che nel codominio.

Svolg. Dapprima si devono esprimere i vettori della base canonica C3 di lR3 per
componenti rispetto alla base B3 :
1 1
(1, 0, 0) = ( , 0, )B3
2 2
1
(0, 1, 0) = (0, , 0)B3
2
1 1
(0, 0, 1) = ( , 0, − )B3 .
2 2
Di tali vettori calcoliamo adesso le immagini:
1 1
f ( , 0, ) = (4, −1)B2
2 2
1
f (0, , 0) = (2, −1)B2
2
1 1
f ( , 0, − ) = (6, −2)B2 .
2 2
Riportiamo tali immagini per componenti rispetto alla base canonica C2 di lR2 :
(4, −1)B2 = (2, 1)C2
(2, −1)B2 = (0, −1)C2
(6, −2)B2 = (2, 0)C2
per cui la matrice associata a f rispetto alle basi canoniche C3 e C2 é:
 
0 2 0 2
A = .
1 −1 0
12 LE APPLICAZIONI LINEARI.

Esempio 23. Siano V = lR2 [x], W = lR3 [x] rispettivamente gli spazi vettoriali
dei polinomi di secondo e terzo grado nella variabile x. Definiamo la seguente
applicazione lineare f : V → W , rispetto alle basi
BV = {1, x, x2 } nel dominio
BW = {1, x, x2 , x3 } nel codominio
f (p(x)) = x · p0|x+1
2

dove p0|x+1 indica la derivata prima del polinomio p(x) calcolata in x + 1.


1. Calcoliamo la matrice associata a f nelle basi scelte;
2. Determiniamo nucleo ed immagine di f ;
3. Determiniamo la matrice associata a f rispetto alle basi
CV = {x + 1, x2 + 1, x2 + x} nel dominio
CW = {x, x3 + 1, x + 1, x2 } nel codominio.

Svolg. Iniziamo calcolando le immagini dei vettori in BV :


f (1) = x2 · 0 = 0W = (0, 0, 0, 0)W
f (x) = x2 · 1 = x2 = (0, 0, 1, 0)W
f (x2 ) = x2 (2x + 2) = 2x3 + 2x2 = (0, 0, 2, 2)W .
La matrica associata é quindi
 
0 0 0
 0 0 0 
A=
 0
.
1 2 
0 0 2
Il nucleo di f si ricava risolvendo il sistema lineare omogeneo:
 
0 0 0  
 0 x1
0 0  

 0 · x2  = 0
1 2 
x3
0 0 2
da cui x1 = α, x2 = 0, x3 = 0. Quindi i polinomi di lR2 [x] che appartengono al
nucleo sono generati dal vettore di componenti (1, 0, 0) rispetto alla base BV , cioé
sono tutti e soli gli scalari in lR: Ker(f ) =< α >, α ∈ lR.
Segue inoltre che l’immagine ha dimensione 2 ed osservando le colonne della matrice
A, una base dell’immagine é individuata dai vettori di componenti:
(0, 0, 1, 0), (0, 0, 2, 2)
2 2 3
quindi Im(f ) =< x , 2x + 2x >.
Per determinare la matrice rispetto alle nuove basi, dapprima calcoliamo le com-
ponenti dei vettori di CV rispetto alla base BV :
x + 1 = (1, 1, 0)BV
x2 + 1 = (1, 0, 1)BV
x2 + x = (0, 1, 1)BV .
LE APPLICAZIONI LINEARI. 13

L’immagine di tali vettori tramite f é il prodotto di ciascuno di essi per la matrice


A associata:
 
0 0 0  
 0 0 0  1
f (1, 1, 0) =   ·  1  = (0, 0, 1, 0)B = x2
 0 1 2  W
0
0 0 2
 
0 0 0  
 0 0 0  1
f (1, 0, 1) =   ·  0  = (0, 0, 2, 2)B = 2x2 + 2x3
 0 1 2  W
1
0 0 2
 
0 0 0  
 0 0 0  0
f (0, 1, 1) =   ·  1  = (0, 0, 3, 2)B = 3x2 + 2x3
 0 1 2  W
1
0 0 2
Esprimiamo infine tali vettori per componenti rispetto alla base CW .
x2 = αx + β(x3 + 1) + γ(x + 1) + δx2
cioé
x2 = (β + γ) + (α + γ)x + δx2 + βx3
ed uguagliando i monomi simili, segue β = 0, γ = −β = 0, α = −γ = 0, δ = 1:
x2 = (0, 0, 0, 1)CW
Analogamente:
2x2 + 2x3 = (2, 2, −2, 2)CW
3x2 + 2x3 = (2, 2, −2, 3)CW
e la matrice associata é:  
0 2 2
 0 2 2 
A0 =  
 0 −2 −2 
1 2 3

3. Endomorfismi e cambiamento di base. Sia V uno spazio vettoriale di di-


mensione n. Una applicazione lineare f : V → V , in cui dominio e codominio
coincidono é detta endomorfismo dello spazio V . Sia B una base di V . Relativa-
mente ad essa, viene associata all’endomorfismo una matrice quadrata di ordine
n. Al variare della base B, varia la matrice che rappresenta f . Ci proponiamo di
osservare in quale modo tali matrici sono tra loro correlate.
Siano allora B e B 0 due distinte basi di V e A la matrice associata alla base B,
A quella associata a B 0 . Siano inoltre v, w ∈ V , tali che f (v) = w. Il vettore v avrá
0

componenti X = (x1 , .., xn ) rispetto alla base B e X 0 = (x01 , .., x0n ) rispetto a B 0 .
Analogamente diciamo che il vettore w avrá componenti Y = (y1 , .., yn ) rispetto
a B e Y 0 = (y10 , .., yn0 ) rispetto a B 0 . Per quanto detto nel capitolo dedicato agli
spazi vettoriali, esiste una matrice C di cambiamento di base che ci permette di
esprimere le componenti di un vettore in base B 0 in funzione di quelle in base B.
In particolare abbiamo che
X0 = C · X e Y 0 = C · Y.
14 LE APPLICAZIONI LINEARI.

Inoltre il fatto che w = f (v) significa


Y =A·X e Y 0 = A0 · X 0 .
Da X = C −1 · X 0 e Y = C −1 · Y 0 ne segue che
C −1 · Y 0 = A · C −1 · X 0 e Y 0 = (C · A · C −1 ) · X 0
cioé
A0 = C · A · C −1 .
Ció vuol dire che le matrici associate ad uno stesso endomorfismo, relativamente a
basi differenti, sono tra loro tutte simili.

Esempio 24. Siano V = lR3 ,


B = {(1, 1, 0), (0, 0, 1), (1, 0, 1)}
B 0 = {(0, 1, 0), (0, 1, 1), (1, 0, 0)}
due basi di lR3 e f : lR3 → lR3 un endomorfismo con matrice associata rispetto alla
base B  
1 0 1
A =  1 0 0 .
0 1 1
Rispetto a tale base l’endomorfismo si esprime:
     
1 0 1 x1 x1 + x3
f (X) =  1 0 0  ·  x2  =  x1 .
0 1 1 x3 x2 + x3
Determiniamo la matrice associata a f rispetto alla base B 0 , utilizzando sia il
metodo esposto nei paragrafi precedenti, che quello appena illustrato.

Svolg. Metodo 1. Per prima cosa esprimiamo i vettori di B 0 per componenti


rispetto alla base B:
(0, 1, 0) = (1, 1, −1)B
(0, 1, 1) = (1, 2, −1)B
(1, 0, 0) = (0, −1, 1)B
quindi calcoliamo le immagini di tali vettori tramite la matrice A:
w1 = f (1, 1, −1) = (0, 1, 0)
w2 = f (1, 2, −1) = (0, 1, 1)
w3 = f (0, −1, 1) = (1, 0, 0).
Ricordiamo che per definizione della matrice A, tali immagini sono espresse per
componenti rispetto alla base B, quindi
w1 = (0, 1, 0)B = (0, 0, 1)
w2 = (0, 1, 1)B = (1, 0, 2)
w3 = (1, 0, 0)B = (1, 1, 0)
e per ottenere la matrice A0 non resta altro che esprimere tali vettori per componenti
rispetto alla base B 0 :
w1 = (0, 0, 1) = (−1, 1, 0)B 0
w2 = (1, 0, 2) = (−2, 2, 1)B 0
LE APPLICAZIONI LINEARI. 15

w3 = (1, 1, 0) = (1, 0, 1)B 0 .


La matrice A associata a f in base B 0 é allora
0
 
−1 −2 1
A0 =  1 2 0 .
0 1 1
Metodo 2. Calcoliamo la matrice C di cambiamento di base da B a B 0 . Essa
ha per colonne le componenti dei vettori di B rispetto alla base B 0 :
   
1 −1 −1 1 1 0
−1
C= 0 1 1 , C =  1 2 −1 
1 0 1 −1 −1 1
da cui
     
1 −1 −1 1 0 1 1 1 0
A0 = C · A · C −1 = 0 1 1 · 1 0 0 · 1 2 −1  =
1 0 1 0 1 1 −1 −1 1
 
−1 −2 1
 1 2 0 .
0 1 1

Ci chiediamo allora se e quando esiste una base di V tale che un endomorfismo


f : V → V sia rappresentabile nel modo piú semplice possibile, cioé tramite una
matrice nella quale compaia il numero piú elevato possibile di elementi nulli. La
condizione ideale sarebbe quella in cui la matrice associata a f si presenti in forma
diagonale cioé
 
a11 0 0 ... ... ... 0

 0 a22 0 ... ... ... 0 


 0 0 a33 ... ... ... 0 

A=
 ... ... ... ... ... ... ... 


 ... ... ... ... ... ... ... 

 ... ... ... ... ... ... ... 
0 0 0 0 ... ... ann
in modo tale che
     
a11 0 0 ... ... ... 0 x1 a11 x1
 0 a22 0 ... ... ... 0   x2   a22 x2 
     
 0 0 a33 ... ... ... 0   x3   a33 x3 
     
·
 ...
f (X) =  ... ... ... ... ... ... ... ...
= .
    
 ... ... ... ... ... ... ...   ...   ... 
     
 ... ... ... ... ... ... ...   ...   ... 
0 0 0 0 ... ... ann xn ann xn
Per cui il problema che ci proponiamo di risolvere é il seguente : in quali casi esiste
una base di V rispetto alla quale la matrice associata ad un endomorfismo di V
sia diagonale? Osserviamo che nel caso ció sia possibile, la forma diagonale della
matrice é rappresentativa di ogni altra matrice associata a f in ogni altra base, e
tali matrici sono tra di esse tutte simili.
16 LE APPLICAZIONI LINEARI.

Esempio 25. Siano V = lR3 ,


B = {(0, 1, 0), (1, −1, −1), (0, 1, 1)}
B 0 = {(1, 1, 0), (1, 0, 0), (0, 1, 1)}
due basi di lR e f : lR → lR3 un endomorfismo con matrice associata rispetto alla
3 3

base B  
1 0 0
A =  −1 2 0  .
−1 1 1
Rispetto a tale base l’endomorfismo si esprime:
     
1 0 0 x1 x1
f (X) =  −1 2 0  ·  x2  =  −x1 + 2x2  .
−1 1 1 x3 −x1 + x2 + x3
Determiniamo la matrice associata a f rispetto alla base B 0 .

Svolg. Calcoliamo la matrice C di cambiamento di base da B a B 0 . Essa ha per


colonne le componenti dei vettori di B rispetto alla base B 0 :
   
1 0 0 1 0 0
C =  −1 1 0  , C −1 =  1 1 0 
0 −1 1 1 1 1
da cui
     
1 0 0 1 0 0 1 0 0
A0 = C · A · C −1 =  −1 1 0  ·  −1 2 0  ·  1 1 0 =
0 −1 1 −1 1 1 1 1 1
 
1 0 0
 0 2 0 
0 0 1
e l’endomorfismo rispetto alla base B 0 si esprime:
     
1 0 0 x1 x1
f (X) =  0 2 0  ·  x2  =  2x2  .
0 0 1 x3 x3

In quanto segue, dapprima esporremo un metodo per determinare in quali casi é


possibile ottenere una forma diagonale della matrice associata ad un endomorfismo
ed infine osserveremo quale sia tale forma.

4. Autovalori ed autovettori di un endomorfismo. Siano V uno spazio vet-


toriale di dimensione n sul campo K, f : V → V un endomorfismo di V e A = [aij ]
la matrice quadrata di ordine n associata a f in una certa base B. Indichiamo con
X ∈ V un generico vettore di V , f (X) = A · X.
Ci chiediamo se possano esistere vettori X ∈ V , la cui immagine f (X) sia un
vettore proporzionale a X, cioé f (X) = A · X = hX, per qualche opportuno h ∈ K.
Chiaramente il vettore nullo 0 ∈ V soddisfa tale proprietá, f (0) = h · 0, per cui ci
poniamo nel caso in cui la ricerca é fatta per vettori X 6= 0. La determinazione di
tali vettori si riduce alla risoluzione del sistema lineare A · X = h · X che si puó
scrivere come segue:
LE APPLICAZIONI LINEARI. 17



 a11 x1 + a12 x2 + ... + a1n xn = hx1
a21 x1 + a22 x2 + ... + a2n xn = hx2


 ..........
an1 x1 + an2 x2 + ... + ann xn = hxn

o meglio ancora, come sistema lineare omogeneo (A − hI)X = 0:


 (a11 − h)x1 + a12 x2 + ... + a1n xn = 0
a21 x1 + (a22 − h)x2 + ... + a2n xn = 0

.

 ..........
an1 x1 + an2 x2 + ... + (ann − h)xn = 0

Tale sistema ammette soluzioni non banali solo quando p(h) = det(A − hI) = 0. Il
polinomio p(h) = det(A − hI) é detto polinomio caratteristico di f (o di A) ed ha
grado n. Analogamente l’equazione p(h) = 0 é detta equazione caratteristica di f
(o di A) ed ha n soluzioni (in K o non in K, distinte o coincidenti).
Si osservi che da ora in poi ci riferiremo al polinomio caratteristico di un endo-
morfismo o di una matrice ad esso associata senza alcuna distinzione.
Le radici del polinomio caratteristico che appartengano al campo K, sono dette
autovalori di f . In corrispondenza di ciascun autovalore h0 , il sistema lineare omo-
geneo (A − h0 I)X = 0 ammette sempre soluzioni non banali. I vettori X che rico-
prono l’insieme V0 di tali soluzioni sono detti autovettori di f relativi all’autovalore
h0
V0 = {0 6= X ∈ V, (A − h0 I)X = 0}.
Il sottospazio vettoriale V0 ∪ {0} ricoperto da tali autovettori é detto autospazio di
f relativo all’autovalore h0 .
Osservazione 3. Sia A una matrice quadrata di ordine n. Se λ é un autovalore
di A, allora λn lo é per la matrice An .

Dim. É sufficiente osservare che da AX = λX, moltiplicando a sinistra ambo i


membri per A, si ottiene A2 X = λAX = λ2 X, cioé A2 ammette λ come autovalore.
Allo stesso modo lo si dimostra per ogni potenza della matrice.
Osservazione 4. Sia A una matrice quadrata di ordine n. Allora A e la sua
trasposta AT hanno gli stessi autovalori (nulla si puó concludere sugli autovettori
relativi).

Dim. Deriva direttamente dal fatto che


det(A − λI) = det((A − λI)T ) = det(AT − λI).
Esempio 26. Siano V = lR2 , e f : lR2 → lR2 con matrice associata rispetto alla
base canonica sia nel dominio che nel codominio:
 
0 −1
A= .
1 0

Svolg. Il polinomio caratteristico é p(h) = h2 + 1, il quale non ha radici reali.


Quindi l’endomorfismo reale da noi definito non possiede autovalori. Nel caso aves-
simo definito f : C 2 → C 2 , dove C é il campo dei numeri complessi, con la medesima
matrice associata A, avremmo avuto i due autovalori immaginari +i, −i.
18 LE APPLICAZIONI LINEARI.

Esempio 27. Siano V = lR3 e f : lR3 → lR3 con matrice associata rispetto alla
base canonica di lR3 :  
1 2 2
A =  1 3 1 .
2 2 1

Svolg. Gli autovalori della matrice A sono le soluzioni dell’equazione caratteristica:



1−h 2 2

1
3−h 1 = 0
2 2 1−h
cioé
(h + 1)(−h2 + 6h − 5) = 0.
Tali soluzioni sono: h1 = −1, h2 = 5, h3 = 1, ciascuna con molteplicitá algebrica 1
come radici del polinomio caratteristico.
Determiniamo gli autospazi relativi a ciascun autovalore:
Per h1 = −1,
 
2 2 2
A − h1 I =  1 4 1 
2 2 2
ed il sistema lineare omogeneo associato é:

 2x1 + 2x2 + 2x3 = 0
x1 + 4x2 + x3 = 0 .
2x1 + 2x2 + 2x3 = 0

Il rango di tale sistema é 2, quindi vi sono ∞1 soluzioni, cioé ∞1 autovettori relativi


a h1 = −1. Essi si ottengono risolvendo il sistema omogeneo: x1 = −x3 , x2 = 0.
Per cui il generico autovettore relativo a h1 é X = (−a, 0, a), al variare di a ∈ lR, e
l’autospazio associato a h1 é
V1 = {(−a, 0, a) ∈ lR3 , a ∈ lR}
con dim(V1 ) = 1.
Per h2 = 5,
 
−4 2 2
A − h2 I =  1 −2 1 
2 2 −4
ed il sistema lineare omogeneo associato é:

 −4x1 + 2x2 + 2x3 = 0
x1 − 2x2 + x3 = 0 .
2x1 + 2x2 − 4x3 = 0

Il rango di tale sistema é 2, quindi vi sono ∞1 soluzioni, cioé ∞1 autovettori relativi


a h2 = 5. Essi si ottengono risolvendo il sistema omogeneo: x1 = x3 , x2 = x3 . Per
cui il generico autovettore relativo a h2 é X = (a, a, a), al variare di a ∈ lR, e
l’autospazio associato a h2 é
V2 = {(a, a, a) ∈ lR3 , a ∈ lR}
con dim(V2 ) = 1.
LE APPLICAZIONI LINEARI. 19

Per h3 = 1,
 
0 2 2
A − h3 I =  1 2 1 
2 2 0
ed il sistema lineare omogeneo associato é:

 2x2 + 2x3 = 0
x1 + 2x2 + x3 = 0 .
2x1 + 2x2 = 0

Il rango di tale sistema é 2, quindi vi sono ∞1 soluzioni, cioé ∞1 autovettori relativi


a h3 = 1. Essi si ottengono risolvendo il sistema omogeneo: x1 = x3 , x2 = −x3 .
Per cui il generico autovettore relativo a h3 é X = (a, −a, a), al variare di a ∈ lR, e
l’autospazio associato a h3 é
V3 = {(a, −a, a) ∈ lR3 , a ∈ lR}
con dim(V3 ) = 1.

Definizione 5. Chiameremo molteplicitá algebrica di un autovalore, la sua molteplicitá


come radice del polinomio caratteristico e molteplicitá geometrica di un autovalore,
la dimensione dell’autospazio associato ad esso.

Osservazione 5. Una matrice diagonale ha come autovalori gli elementi presenti


sulla sua diagonale principale. Infatti se
 
a11 0 0 ... ... ... 0
 0 a22 0 ... ... ... 0 
 
 0 0 a33 ... ... ... 0 
 
A=  ... ... ... ... ... ... ... 

 ... ... ... ... ... ... ... 
 
 ... ... ... ... ... ... ... 
0 0 0 0 ... ... ann
allora
 
a11 − h 0 0 ... ... ... 0

 0 a22 − h 0 ... ... ... 0 


 0 0 a33 − h ... ... ... 0 

A − hI = 
 ... ... ... ... ... ... ... 


 ... ... ... ... ... ... ... 

 ... ... ... ... ... ... ... 
0 0 0 0 ... ... ann − h
e det(A − hI) = (a11 − h)(a22 − h) · · · (ann − h), le cui radici sono
h1 = a11 , h2 = a22 , ..., ..., hn = ann .
Teorema 5. Siano A, B ∈ Mn (lR) due matrici simili, cioé esista una matrice non
singolare C ∈ Mn (lR) tale che B = C −1 · A · C. Siano Yi autovettori di B. Allora
A e B hanno gli stessi autovalori h1 , ..., hm ed inoltre Xi = C · Yi sono autovettori
di A, per ogni autovettore Yi di B.
20 LE APPLICAZIONI LINEARI.

Dim. Siano h1 , ..., hm gli autovalori di A e X1 , .., Xm autovettori di A tali che


AXi = hi Xi , per i = 1, .., m. Poiché A = CBC −1 , segue che:
(CBC −1 )Xi = hi Xi

B(C −1 Xi ) = hi (C −1 Xi )
cioé h1 , .., hm sono anche autovalori di B, con autovettori corrispondenti Yi =
C −1 Xi .
Teorema 6. Sia f : V −→ V un endomorfismo su uno spazio vettoriale V di
dimensione n sul campo K, con matrice associata A, e sia h0 un autovalore di f
con molteplicitá algebrica r. Indichiamo con t la dimensione dell’autospazio V0 di
f relativo all’autovalore h0 . Allora t ≤ r.

Dim. Denotiamo p(h) il polinomio caratteristico di f (o equivalentemente di A).


Poiché dim(V0 ) = t, esiste un insieme di vettori {b1 , .., bt } che cosituiscono una base
per V0 .
Di conseguenza esistono {et+1 , .., en } vettori linearmente indipendenti tali che l’insieme
B = {b1 , .., bt , et+1 , ..., en } sia una base per V . Costruiamo la matrice A0 associata a
f rispetto alla base B, ricordando che le colonne di tale matrice saranno determinate
dalle componenti rispetto alla base B dei vettori  {f (b1 ), .., f (bt ), f (et+1 ), .., f (en )}.
0 D A1
Poiché f (bi ) = h0 bi , avremo che A = , con A1 matrice di ordine
0 A2
(t, n − t), A2 matrice di ordine (n − t, n − t) e D matrice diagonale di ordine
(t, t):
 
h0 0 0 0 0
 0 h0 0 0 0 
 
D=  0 0 h0 0 0  .
 0 0 0 ... 0 
0 0 0 0 h0
Poiché A e A0 sono simili, esse hanno gli stessi autovalori. In particolare il polinomio
caratteristico di A0 é : p(h) = (h−h0 )t ·µ(h), dove µ(h) é il polinomio caratteristico
della matrice A2 . Quindi h0 compare almeno t-volte come radice del polinomio
p(h), di conseguenza la sua molteplicitá algebrica é almeno t, anche come radice
del polinomio caratteristico di A.

5. Endomorfismi diagonalizzabili. Sia f : lRn → lRn un endomorfismo di


V = lRn e siano B e B 0 due distinte basi di lRn . Indichiamo con A la matrice
associata a f rispetto alla base B e con A0 quella associata a f rispetto a B 0 . Come
visto in precedenza, le matrici A e A0 sono simili e quindi esse hanno gli stessi
autovalori. Quindi, piú in generale, diciamo che tutte le matrici relative ad uno
stesso endomorfismo possiedono gli stessi autovalori, indipendentemente dalla base
rispetto alla quale sono costruite.
Diremo che l’endomorfismo f é diagonalizzabile se esiste una base di lRn , rispetto
alla quale la matrice associata a f sia diagonale.
Analogamente diremo che una matrice A é diagonalizzabile se é simile ad una
matrice diagonale, cioé se esiste una matrice non singolare P tale che P −1 · A · P
sia diagonale. La matrice P sará detta matrice diagonalizzante di A.
LE APPLICAZIONI LINEARI. 21

Teorema 7. Siano h1 , .., hm autovalori distinti di f e X1 , .., Xm autovettori di f


tali che ogni Xi sia autovettore relativo a hi , per ogni i = 1, ..., m. Allora i vettori
X1 , .., Xm sono linearmente indipendenti.

Dim. Possiamo facilmente dimostrarlo nel caso di 2 autovalori distinti h1 6= h2


(induttivamente si puó estendere al caso generico).
Siano X1 6= 0 autovettore di f relativo a h1 e X2 6= 0 autovettore di f relativo
a h2 . Supponiamo al contrario che essi siano linearmente dipendenti, quindi esista
un α 6= 0 tale che X1 = αX2 . Da f (X1 ) = h1 X1 segue che f (αX2 ) = h1 αX2 ,
cioé αf (X2 ) = αh1 X2 . Inoltre f (X2 ) = h2 X2 , quindi otteniamo αh2 X2 = αh1 X2 .
Essendo α 6= 0 e h1 6= h2 , segue l’assurdo che X2 = 0.
Supponiamo il teorema vero per ogni insieme di m − 1 autovalori. Sia
m
X
αi Xi = 0 (i).
i=1

Applicando f alla (i) otteniamo:


m
X m
X
αi f (Xi ) = αi hi Xi = 0 (ii).
i=1 i=1

Moltiplichiamo ora la (i) per h1 :


m
X
h1 αi Xi = 0 (iii).
i=1

Sottraendo dalla (iii) la (ii) otteniamo:


m
X
αi (h1 − hi )Xi = 0
i=2

e per l’ipotesi induttiva, i vettori X2 , ..., Xm sono indipendenti, quindi α(h1 − hi ) =


0, per ogni i = 2, ..., m. Poiché h2 , .., hm sono tutti distinti, concludiamo che
α2 = α3 = ... = αm = 0, da cui, sostituendo nella (i), anche α1 = 0. Se ne
deduce quindi che X1 , .., Xm sono linearmente indipendenti.
Vale allora il seguente:

Teorema 8. Sia f : lRn −→ lRn un endomorfismo con matrice associata A. Le


seguenti affermazioni sono equivalenti:
1. f é diagonalizzabile;
2. ogni autovalore di f possiede una molteplicitá algebrica ed una geometrica
coincidenti;
3. esiste una base dello spazio lRn che é costituita da n autovettori di f .

Dim. Siano h1 , . . . , hm gli autovalori distinti della matrice A associata all’endomorfismo


e siano (a1 , a2 , .., am ) e (g1 , . . . , gm ) rispettivamente le molteplicitá algebriche e
quelle geometriche di h1 , h2 , . . . , hm . Ricordiamo che a1 + a2 + ... + am = n.
22 LE APPLICAZIONI LINEARI.

(1) ⇒ (2): Nell’ipotesi in cui la matrice A associata all’endomorfismo sia diago-


nalizzabile, esisterá una matrice non sigolare C tale che
 
h1 0 0 0 0 ... ... 0
 0 ∗ 0 0 0 ... ... 0 
 
 0 0 ∗ 0 0 ... ... 0 
 
0 −1
 0 0 0 ∗ 0 ... ... 0 
A =C ·A·C =  
 0 0 0 0 ∗ 0 ... 0 

 ... 0 0 0 ... ... ... 0 
 
 0 0 0 0 ... ... ∗ 0 
0 0 0 0 ... ... 0 hm
sulla cui diagonale principale ogni autovalore compare tante volte quanto é la
sua molteplicitá algebrica.
Scegliamo un qualsiasi autovalore λ ∈ {h1 , . . . , hm }, con molteplicitá alge-
brica a. Poiché la matrice A0 − λI ha rango n − a, allora l’autospazio relativo
all’autovalore λ ha dimensione n − (n − a) = a, cioé la molteplicitá geometrica
di λ é esattamente pari alla molteplicitá algebrica (ed analogamente per ogni
altro autovalore di A).

(2) ⇒ (3): Supponiamo ora che per ogni autovalore hi , la sua molteplicitá alge-
brica ai coincida con la sua molteplicitá geometrica gi = dim(Vi ) (la dimen-
sione dell’autospazio relativo a hi ). Allora si ha che

dim(V1 ) + dim(V2 ) + ... + dim(Vm ) = n = dim(V )

cioé
V1 ⊕ V2 ⊕ ..... ⊕ Vm = V = lRn
ed inoltre l’unione delle basi di tutti gli autospazi costituisce una base di
V = lRn .

(3) ⇒ (1): Infine supponiamo che esista una base di lRn formata da autovettori
di A. Esistono allora X1 , .., Xn autovettori in lRn che siano tra loro linear-
mente indipendenti. Ricordiamo che gli autovalori sono in numero di n, anche
se non tutti necessariamente distinti. Indichiamo adesso λ1 , . . . , λn tali auto-
valori, m dei quali siano distinti (m ≤ n). Allora:

AX1 = λ1 X1 , AX2 = λ2 X2 , AX3 = λ3 X3 , .... ..., AXn = λn Xn .

Se indichiamo con B = {X1 , X2 , X3 , X4 , . . . , . . . , Xn } la base di V formata


dall’unione delle basi di tutti gli autospazi, allora

f (X1 ) = AX1 = λ1 X1 = (λ1 , 0, 0, 0, . . . , 0)B

f (X2 ) = AX2 = λ2 X2 = (0, λ2 , 0, 0, . . . , 0)B

f (X3 ) = AX3 = λ3 X3 = (0, 0, λ3 , 0, . . . , 0)B

............

f (Xn ) = AXn = λn Xn = (0, 0, 0, 0, . . . , λn )B


LE APPLICAZIONI LINEARI. 23

per cui la matrice associata all’endomorfismo rispetto alla base B in dominio


e codominio é diagonale:
 
λ1 0 0 0 0 ... ... 0
 0 λ2 0 0 0 ... ... 0 
 
 0 0 λ 3 0 0 ... ... 0 
 
0
 0 0 0 λ 4 0 ... ... 0 
A =  
 0 0 0 0 λ 5 0 ... 0 

 ... 0 0 0 ... ... ... 0 
 
 0 0 0 0 ... ... λn−1 0 
0 0 0 0 ... ... 0 λn
 
dove C = X1 X2 X3 X4 ... ... ... Xn é la matrice che prevede la
presenza nella colonna i delle componenti dell’autovettore Xi . Quindi rispetto
alla base di lRn formata dagli autovettori linearmente indipendenti, la matrice
associata all’endomorfismo é diagonale. Le colonne della matrice diagonaliz-
zante sono costituite proprio dagli autovettori di f che costituiscono una base
per lRn .
Esempio 28. Siano V = lR3 e f : lR3 → lR3 tale che f (x1 , x2 , x3 ) = (x1 , x2 , x1 +
4x3 ).

Svolg. La matrice associata a f rispetto alla base canonica di lR3 é:


 
1 0 0
A= 0 1 0 .
1 0 4
Gli autovalori della matrice A sono le soluzioni dell’equazione caratteristica:

1−h 0 0

0
1−h 0 = 0
1 0 4−h
cioé
(1 − h)2 (4 − h) = 0.
Tali soluzioni sono: h1 = 4, con molteplicitá algebrica a1 = 1 e h2 = 1, con
molteplicitá algebrica a2 = 2.
Determiniamo gli autospazi relativi a ciascun autovalore:
Per h1 = 4,  
−3 0 0
A − h1 I =  0 −3 0 
1 0 0
ed il sistema lineare omogeneo associato é:

 −3x1 = 0
−3x2 = 0 .
x1 = 0

Il rango di tale sistema é 2, quindi vi sono ∞1 soluzioni, cioé ∞1 autovettori relativi


a h1 = 4. Essi si ottengono risolvendo il sistema omogeneo: x1 = x2 = 0. Per cui il
generico autovettore relativo a h1 é X = (0, 0, a), al variare di a ∈ lR, e l’autospazio
associato a h1 é
V1 = {(0, 0, a) ∈ lR3 , a ∈ lR}
24 LE APPLICAZIONI LINEARI.

con dim(V1 ) = 1 e V1 =< (0, 0, 1) >.


Per h2 = 1,  
0 0 0
A − h2 I =  0 0 0 
1 0 3
ed il sistema lineare omogeneo associato é:

x1 + 3x3 = 0 .
Il rango di tale sistema é 1, quindi vi sono ∞ soluzioni, cioé ∞2 autovettori relativi
2

a h2 = 1. Essi si ottengono risolvendo il sistema omogeneo: x1 = −3x3 . Per cui


il generico autovettore relativo a h2 é X = (−3a, b, a), al variare di a, b ∈ lR, e
l’autospazio associato a h2 é
V2 = {(−3a, b, a) ∈ lR3 , a, b ∈ lR}
con dim(V2 ) = 2 e V2 =< (−3, 0, 1), (0, 1, 0) >.
Per ogni autovalore le molteplicitá algebrica e geometrica coincidono, quindi la
matrice é diagonalizzabile, l’unione delle basi degli autospazi é una base per lR3
B = {(0, 0, 1), (−3, 0, 1), (0, 1, 0)}
e la matrice associata a f rispetto a tale base é diagonale.
La matrice diagonalizzante é formata dagli autovettori che costituiscono la base
per lR3 :
 
0 −3 0
P = 0 0 1 
1 1 0
0 −1
 1
A= P · A ·P 
= 
3 0 1 1 0 0 0 −3 0
 − 13 0 0 · 0 1 0 · 0 0 1 =
0 1 0 1 0 4 1 1 0
 
4 0 0
 0 1 0 .
0 0 1

Esempio 29. Siano V = lR3 e f : lR3 → lR3 tale che f (x1 , x2 , x3 ) = (x1 + x3 , x2 +
x3 , x3 ).

Svolg. La matrice associata a f rispetto alla base canonica di lR3 é:


 
1 0 1
A= 0 1 1 .
0 0 1
Gli autovalori della matrice A sono le soluzioni dell’equazione caratteristica:

1−h 0 1

0
1−h 1 = 0
0 0 1−h
cioé
(1 − h)3 = 0.
Tali soluzioni sono: h1 = 1, con molteplicitá algebrica a1 = 3.
LE APPLICAZIONI LINEARI. 25

Determiniamo l’autospazio relativo all’unico autovalore:


 
0 0 1
A − h1 I =  0 0 1 
0 0 0
ed il sistema lineare omogeneo associato é:

x3 = 0 .
Il rango di tale sistema é 1, quindi vi sono ∞2 soluzioni, cioé ∞2 autovettori
relativi a h1 = 1. Essi si ottengono risolvendo il sistema omogeneo: x3 = 0. Per
cui il generico autovettore relativo a h1 é X = (a, b, 0), al variare di a, b ∈ lR, e
l’autospazio associato a h1 é
V1 = {(a, b, 0) ∈ lR3 , a, b ∈ lR}
con dim(V1 ) = 2 e V1 =< (1, 0, 0), (0, 1, 0) >. Le molteplicitá algebrica e geometrica
non coincidono, quindi la matrice non é diagonalizzabile, cioé non esiste alcuna
base rispetto alla quale l’endomorfismo sia rappresentabile attraverso una matrice
diagonale.

Esempio 30. Siano V = lR3 e f : lR3 → lR3 tale che f (x1 , x2 , x3 ) = (2x1 +
x3 , 2x2 − x3 , x1 − x2 + x3 ).

Svolg. La matrice associata a f rispetto alla base canonica di lR3 é:


 
2 0 1
A =  0 2 −1  .
1 −1 1
Gli autovalori della matrice A sono le soluzioni dell’equazione caratteristica:

2−h 0 1

0
2−h −1 = 0
1 −1 1−h
cioé
(2 − h)(h2 − 3h) = 0.
Tali soluzioni sono: h1 = 2, con molteplicitá algebrica a1 = 1, h2 = 0, con
molteplicitá algebrica a2 = 1, h3 = 3, con molteplicitá algebrica a3 = 1.
Determiniamo gli autospazi relativi a ciascun autovalore:
Per h1 = 2,  
0 0 1
A − h1 I =  0 0 −1 
1 −1 −1
ed il sistema lineare omogeneo associato é:

x3 = 0
.
x1 − x2 − x3 = 0
Il rango di tale sistema é 2, quindi vi sono ∞1 soluzioni, cioé ∞1 autovettori relativi
a h1 = 2. Essi si ottengono risolvendo il sistema omogeneo: x1 = x2 , x3 = 0. Per
cui il generico autovettore relativo a h1 é X = (a, a, 0), al variare di a ∈ lR, e
l’autospazio associato a h1 é
V1 = {(a, a, 0) ∈ lR3 , a ∈ lR}
26 LE APPLICAZIONI LINEARI.

con dim(V1 ) = 1 e V1 =< (1, 1, 0) >.


Per h2 = 0,  
2 0 1
A − h2 I =  0 2 −1 
1 −1 1
ed il sistema lineare omogeneo associato é:

 2x1 + x3 = 0
2x2 − x3 = 0 .
x1 − x2 + x3 = 0

Il rango di tale sistema é 2, quindi vi sono ∞1 soluzioni, cioé ∞1 autovettori relativi


a h2 = 0. Essi si ottengono risolvendo il sistema omogeneo: x1 = −x2 = − x23 . Per
cui il generico autovettore relativo a h2 é X = (− a2 , a2 , a), al variare di a ∈ lR, e
l’autospazio associato a h2 é
a a
V2 = {(− , , a) ∈ lR3 , a ∈ lR}
2 2
con dim(V2 ) = 1 e V2 =< (− 12 , 12 , 1) >.
Per h3 = 3,  
−1 0 1
A − h3 I =  0 −1 −1 
1 −1 −2
ed il sistema lineare omogeneo associato é:

 −x1 + x3 = 0
−x2 − x3 = 0 .
x1 − x2 − 2x3 = 0

Il rango di tale sistema é 2, quindi vi sono ∞1 soluzioni, cioé ∞1 autovettori relativi


a h3 = 3. Essi si ottengono risolvendo il sistema omogeneo: x1 = −x2 = x3 . Per
cui il generico autovettore relativo a h3 é X = (a, −a, a), al variare di a ∈ lR, e
l’autospazio associato a h3 é
V3 = {(a, −a, a) ∈ lR3 , a ∈ lR}
con dim(V3 ) = 1 e V3 =< (1, −1, 1) >.
Per ogni autovalore le molteplicitá algebrica e geometrica coincidono, quindi la
matrice é diagonalizzabile, l’unione delle basi degli autospazi é una base per lR3
1 1
B = {(1, 1, 0), (− , , 1), (1, −1, 1)}
2 2
e la matrice associata a f rispetto a tale base é diagonale:
 
2 0 0
A0 =  0 0 0  .
0 0 3
La matrice diagonalizzante é formata dagli autovettori che costituiscono la base
per lR3 :
1 − 12
 
1
1
P = 1 2 −1  .
0 1 1
LE APPLICAZIONI LINEARI. 27

Esempio 31. Siano V = lR3 e f : lR3 → lR3 tale che f (x1 , x2 , x3 ) = (3x1 +
x2 , 3x2 , 3x1 ).

Svolg. La matrice associata a f rispetto alla base canonica di lR3 é:


 
3 1 0
A= 0 3 0 .
3 0 0
Gli autovalori della matrice A sono le soluzioni dell’equazione caratteristica:

3−h 1 1

0
3 − h 0 = 0
3 0 −h
cioé
−h(3 − h)2 = 0.
Tali soluzioni sono: h1 = 0, con molteplicitá algebrica a1 = 1, h2 = 3, con
molteplicitá algebrica a2 = 2,
Determiniamo gli autospazi relativi a ciascun autovalore:
Per h1 = 0,  
3 1 0
A − h1 I =  0 3 0 
3 0 0
ed il sistema lineare omogeneo associato é:

 3x1 + x2 = 0
3x2 = 0 .
3x1 = 0

Il rango di tale sistema é 2, quindi vi sono ∞1 soluzioni, cioé ∞1 autovettori relativi


a h1 = 0. Essi si ottengono risolvendo il sistema omogeneo: x1 = x2 = 0. Per cui il
generico autovettore relativo a h1 é X = (0, 0, a), al variare di a ∈ lR, e l’autospazio
associato a h1 é
V1 = {(0, 0, a) ∈ lR3 , a ∈ lR}
con dim(V1 ) = 1 e V1 =< (0, 0, 1) >.
Per h2 = 3,  
0 1 0
A − h2 I =  0 0 0 
3 0 −3
ed il sistema lineare omogeneo associato é:

x2 = 0
.
3x1 − 3x3 = 0
Il rango di tale sistema é 2, quindi vi sono ∞1 soluzioni, cioé ∞1 autovettori relativi
a h2 = 3. Essi si ottengono risolvendo il sistema omogeneo: x2 = 0, x1 = x3 . Per
cui il generico autovettore relativo a h2 é X = (a, 0, a), al variare di a ∈ lR, e
l’autospazio associato a h2 é
V2 = {(a, 0, a) ∈ lR3 , a ∈ lR}
con dim(V2 ) = 1 e V2 =< (1, 0, 1) >. L’autovalore h2 = 3 ha le molteplicitá
differenti, quindi la matrice non é diagonalizzabile.
28 LE APPLICAZIONI LINEARI.

Osservazione 6. Se gli autovalori di un endomorfismo f sono tutti distinti tra


loro, cioé ciascuno di essi ha molteplicitá algebrica 1, allora f é certamente diago-
nalizzabile.

6. Matrici simmetriche reali. Ci occupiamo infine di una classe di matrici (e


quindi endomorfismi) certamente diagonalizzabili: sono le matrici simmetriche reali
(gli endomorfismi rappresentati da matrici simmetriche reali).
Vale il seguente:
Teorema 9. Sia f : lRn → lRn un endomorfismo. Esiste una base ortonormale
B = {v1 , .., vn } di autovettori di lRn se e solo se f é simmetrico.
In altre parole: Sia data la matrice A ∈ Mn (lR). Esiste una matrice non singolare
P ∈ Mn (lR), costituita da vettori colonna tra loro ortonormali, tale che P −1 · A · P
sia una matrice diagonale se e solo se A é simmetrica.
Forniremo la dimostrazione del teorema nel capitolo dedicato alle Forme quadratiche
reali.

Esempio 32. Siano V = lR3 e f : lR3 → lR3 tale che f (x1 , x2 , x3 ) = (x1 + x2 +
x3 , x1 , x1 ).

Svolg. La matrice associata a f rispetto alla base canonica di lR3 é:


 
1 1 1
A= 1 0 0 
1 0 0
che é simmetrica. Gli autovalori della matrice A sono le soluzioni dell’equazione
caratteristica:
1−h 1 1

1
−h 0 = 0
1 0 −h
cioé
h(h − 2)(h + 1) = 0.
Tali soluzioni sono: h1 = 0, con molteplicitá algebrica a1 = 1, h2 = 2, con
molteplicitá algebrica a2 = 1, h3 = −1 con molteplicitá algebrica a3 = 1. La
matrice é diagonalizzabile ed una sua forma diagonale é
 
2 0 0
A0 =  0 −1 0 
0 0 0
per ottenere la quale si utilizza la matrice diagonalizzante
 
2 1 0
P =  1 −1 1 
1 −1 −1
le cui colonne sono gli autovettori, tra loro ortogonali, che costituiscono una base
di lR3 rispetto alla quale A0 é la matrice associata a f . Per ottenere una base
ortgonormale, é sufficiente normalizzare i vettori della base.
LE APPLICAZIONI LINEARI. 29

7. Esercizi.
Esercizio 1. Sia f : lR3 −→ lR4 una applicazione lineare cosı́ definita: f (x1 , x2 , x3 ) =
(x1 , 0, x3 , 0). Determinare nucleo, immagine ed una loro base.
Esercizio 2. Sia f : lR4 −→ lR3 definita da f (x1 , x2 , x3 , x4 ) = (x1 + x2 , x2 −
x3 , x1 + x3 ). Determinare nucleo, immagine ed una loro base.
Esercizio 3. Sia f : lR3 −→ lR4 definita da f (x1 , x2 , x3 ) = (x1 , x1 + x2 − x3 , 2x1 +
x2 − x3 , x2 − x3 ). Determinare nucleo, immagine ed una loro base.
Esercizio 4. Sia f : lR4 −→ lR3 definita da f (x1 , x2 , x3 , x4 ) = (x1 − x2 , 2x1 − x2 +
x4 , −x1 + x2 + x4 ). Determinare nucleo, immagine ed una loro base.
Esercizio 5. Sia f : lR2 −→ lR3 definita da: f (x1 , x2 ) = (x1 + 2x2 , 3x2 , x1 ).
Determinare la matrice associata alla f rispetto alle seguenti basi:
B1 = {(1, 1), (2, 1)} di lR2
B2 = {(1, 1, 0), (1, 0, 0), (1, 2, 1)} di lR3 .
Esercizio 6. Sia f : lR3 −→ lR4 definita da: f (x1 , x2 , x3 ) = (x1 + x2 , x2 , x1 +
x2 , x3 ). Determinare la matrice associata alla f rispetto alle basi canoniche in lR3
e lR4 .
Esercizio 7. Sia f : lR5 −→ lR3 definita da: f (x1 , x2 , x3 , x4 , x5 ) = (x1 − x4 , x2 −
x4 , x3 ). Determinare la matrice associata alla f rispetto alle basi canoniche in lR3
e lR5 .
Esercizio 8. Sia f : lR3 −→ lR4 definita da: f (x1 , x2 , x3 ) = (5x1 + 4x2 − 9x3 , 4x1 +
5x2 − 9x3 , −9x1 − 9x2 + 9x3 , x1 + x2 + x3 ). Determinare la matrice associata alla
f rispetto alla base canonica in lR4 e B = {(1, 1, 0), (1, 0, −1), (0, 1, −1)} in lR3 .
Esercizio 9. Sia f : lR4 −→ lR2 una applicazione lineare avente matrice associata
 
1 0 0 1
−1 1 2 −1
rispetto alle basi
B1 = {(1, 1), (1, 0)} di lR2
B2 = {(1, 1, 1, 0), (1, 0, 0, 0), (2, 0, 0, 1), (0, 0, 1, 0)} di lR4 .
Determinare la matrice associata a f rispetto alle basi canoniche sia nel dominio
che nel codominio.
Esercizio 10. Sia f : lR4 −→ lR4 l’endomorfismo avente matrice associata
 
1 0 1 0
 0 −1 0 1 
 
 0 0 0 1 
0 1 1 1
rispetto alla base
B = {(1, 0, 0, −1), (1, 0, 0, 0), (0, 1, 1, 0), (0, 0, 1, 0)} di lR4
sia nel dominio che nel codominio. Determinare la matrice associata a f rispetto
alle basi canoniche sia nel dominio che nel codominio.
30 LE APPLICAZIONI LINEARI.

Esercizio 11. Sia f : lR3 −→ lR3 l’endomorfismo avente matrice associata


 
1 1 2
 2 1 3 
3 1 4
rispetto alle basi
B1 = {(1, 1, 0), (1, 0, 1), (0, 0, 1)} nel dominio
B2 = {(1, 0, −1), (1, 0, 1), (0, 1, 0)} nel codominio.
Determinare la matrice associata a f rispetto alle basi canoniche sia nel dominio
che nel codominio.
Esercizio 12. Determinare gli autovalori, una base per gli autospazi e, quando
possibile, la matrice diagonalizzante della seguente matrice:
 
1 2 2
 1 3 1 .
2 2 1
Esercizio 13. Determinare gli autovalori, una base per gli autospazi e, quando
possibile, la matrice diagonalizzante della seguente matrice:
 
1 0 0
 0 1 0 .
1 0 4
Esercizio 14. Determinare gli autovalori, una base per gli autospazi e, quando
possibile, la matrice diagonalizzante della seguente matrice:
 
1 0 1
 0 1 1 .
0 0 1
Esercizio 15. Determinare gli autovalori, una base per gli autospazi e, quando
possibile, la matrice diagonalizzante della seguente matrice:
 
2 0 1
 0 2 −1  .
1 −1 1
Esercizio 16. Determinare gli autovalori, una base per gli autospazi e, quando
possibile, la matrice diagonalizzante della seguente matrice:
 
2 0 0
 0 2 −1  .
1 −2 1
Esercizio 17. Determinare gli autovalori, una base per gli autospazi e, quando
possibile, la matrice diagonalizzante della seguente matrice:
 
1 0 1
 0 0 −2  .
0 −2 0
Esercizio 18. Determinare gli autovalori, una base per gli autospazi 
e, quando
3 1 0
possibile, la matrice diagonalizzante della seguente matrice:  0 3 0 .
3 0 0
LA FORMA CANONICA DI JORDAN.

1. Definizioni. Sia f : lRn −→ lRn un endomorfismo. Abbiamo in precedenza


analizzato in quali casi tale endomorfismo sia diagonalizzabile, cioé quando esista
una base di lRn composta da autovettori di f , rispetto alla quale la matrice associata
all’endomorfismo si presenti in forma diagonale. In sostanza abbiamo visto che,
quando f é diagonalizzabile, tutte le matrici che si possono ad esso associare, in
una qualsiasi base di lRn , sono tra loro simili e tutte simili ad una matrice diagonale.
Abbiamo concluso che non tutti gli endomorfismi, e quindi non tutte le matrici, sono
diagonalizzabili. Ci proponiamo ora di affrontare l’analisi di una classe di matrici
sufficientemente semplici, in modo tale che ogni matrice quadrata sui reali sia simile
ad una di esse, e di conseguenza ogni endomorfismo possa essere rappresentato, in
una qualche base, da una di queste matrici, dette forme canoniche. Fra tutte tali
forme, certamente la piú semplice é la forma canonica di Jordan.
Diciamo Blocco di Jordan di ordine p e relativo allo scalare α ∈ lR, la matrice
 
α 1 0 0 0 0 0 0
 0 α 1 0 0 0 0 0 
 
 0 0 α 1 0 0 0 0 
 
 0 0 0 α 1 0 0 0 
Jp (α) =   ... ... ... ... ... ... ... ... 

 
 ... ... ... ... ... ... ... ... 
 
 0 0 0 0 0 0 α 1 
0 0 0 0 0 0 0 α
cioé gli elementi aij della matrice sono

 α se i=j
aij = 1 se j =i+1
0 negli altri casi

Diremo che una matrice A ∈ Mn (lR) é in forma canonica di Jordan se essa


presenta la seguente forma diagonale a blocchi di Jordan
 
Jn1 (α1 ) 0 0 0 0 0 0 0

 0 Jn2 (α2 ) 0 0 0 0 0 0 


 0 0 J n 3 (α 3 ) 0 0 0 0 0 

 0 0 0 J n 4
(α 4 ) 0 0 0 0 
A=  
 ... ... ... ... ... ... ... ... 


 ... ... ... ... ... ... ... ... 

 0 0 0 0 0 0 Jnr−1 (αr−1 ) 0 
0 0 0 0 0 0 0 Jnr (αr )
dove ogni Jni (αi ) éPun blocco di Jordan di ordine ni relativo ad un qualche scalare
αi ed ovviamente i ni = n.

1
2 LA FORMA CANONICA DI JORDAN.

Esempio 1.  
3 1 0 0 0
 0 3 1 0 0   
 = J3 (3) 0
 
A1 =  0 0 3 0 0
  0 J2 (2)
 0 0 0 2 1 
0 0 0 0 2
Esempio 2.
 
2 1 0 0 0 0
 0 2 1 0 0 0   
  J3 (2) 0 0
 0 0 2 0 0 0 
= 0
A2 =  J2 (4) 0 
 0 0 0 4 1 0 
  0 0 J 1 (3)
 0 0 0 0 4 0 
0 0 0 0 0 3
Esempio 3.
   
3 0 0 0 J1 (3) 0 0 0
 0 2 0 0 
= 0 J1 (2) 0 0
 
A3 = 
 .
0 0 4 0   0 0 J1 (4) 0 
0 0 0 5 0 0 0 J1 (5)
Notiamo allora che le matrici diagonali sono delle particolari matrici di Jordan,
con un numero di blocchi pari all’ordine della matrice stessa ed ogni blocco ha
ordine 1.
Teorema 1. Sia A ∈ Mn (lR) una matrice il cui polinomio caratteristico abbia tutte
radici reali. Allora esiste una matrice non singolare C ∈ Mn (lR) tale che la matrice
C −1 AC sia in forma canonica di Jordan. In altre parole, per ogni endomorfismo f
di lRn avente tutti gli autovalori nel campo reale, esiste una base di lRn , rispetto alla
quale, la matrice associata a tale endomorfismo sia in forma canonica di Jordan.
Osservazione 1. Sia A la matrice, ed indichiamo
P (X) = (X − λ1 )r1 · (X − λ2 )r2 · · · ·(X − λh )rh
il suo polinomio caratteristico, in cui λ1 , λ2 , λ3 , ..., λh sono tutti gli autovalori di A,
tra loro distinti e tali che l’autovalore λi abbia
P molteplicitá algebrica, come radice
del polinomio caratteristico, pari a ri , con i ri = n. Sia A0 la forma canonica di
Jordan simile alla A. Gli scalari rispetto ai quali i blocchi di Jordan della A0 vengono
costruiti sono proprio gli autovalori della matrice A, cioé per ogni autovalore di A
esiste almeno un blocco di Jordan in A0 .
A questo punto, per poter effettivamente costruire la matrice nella sua forma
canonica di Jordan, é necessario rispondere alle due seguenti domande:
1. Quanti sono i blocchi di Jordan relativi a ciascun autovalore?
2. Quale deve essere l’ordine di ciascun blocco?

2. Numero totale di blocchi relativi ad uno stesso autovalore. Sia λ un


autovalore di A con molteplicitá geometrica pari a m, cioé sia dim(Vλ ) = m, dove
Vλ é l’autospazio associato a λ. Supponiamo che tutti gli autovalori di A siano reali,
quindi esiste una forma canonica di Jordan A0 di A. Allora il numero di blocchi di
Jordan in A0 relativi all’autovalore λ é proprio m.
LA FORMA CANONICA DI JORDAN. 3

In particolare, per ogni autovalore per il quale le molteplicitá algebrica e geo-


metrica coincidano (diciamo m per entrambi i valori), allora nella A0 compaiono m
blocchi relativi all’autovalore e tutti con ordine 1.

Esempio 4. Sia A ∈ M2 (lR) con autovalori λ1 , λ2 .

Svolg. Se i due autovalori sono distinti, allora sappiamo che la dimensione geo-
metrica di entrambi é 1, per cui esiste un blocco di Jordan per ognuno dei due
autovalori e ciascun blocco ha ordine 1, cioé la matrice
 é diagonalizzabile,
 la forma
λ1 0
di Jordan coincide con quella diagonale ed essa é .
0 λ2
Se i due autovalori coincidono (λ1 = λ2 = λ) e la dimensione dell’autospazio asso-
ciato é 2, allora esitono due blocchi di Jordan relativi all’autovalore, ciascuno con
ordine 1. Anche in questo caso la matrice é diagonalizzabile
  e la forma canonica di
λ 0
Jordan si riduce alla semplice forma diagonale .
0 λ
Se i due autovalori coincidono (λ1 = λ2 = λ) ma la dimensione dell’autospazio
associato é 1, allora esiste un solo blocco di Jordan relativo all’autovalore, ed esso
deve avere
  necessariamente ordine 2. La forma canonica di Jordan é in tale caso
λ 1
.
0 λ

Esempio 5. Sia A ∈ M3 (lR) e siano λ1 , λ2 , λ3 ∈ lR gli autovalori di A.

Svolg. Se i tre autovalori sono tutti distinti allora la matrice é diagonalizzabile.


Infatti ogni autospazio ha dimensione 1 e quindi esiste un solo blocco relativo ad
ogni autovalore ed ogni blocco ha ordine  1. La forma canonica di Jordan si riduce
λ1 0 0
a quella diagonale ed é  0 λ2 0 .
0 0 λ3
Supponiamo ora che λ1 = λ2 = λ e λ3 sia distinto dai primi due.
Se la molteplicitá geometrica di λ é 2, allora vi sono due blocchi di Jordan relativi
a λ ed ovviamente uno relativo a λ3 , per cui ciascun  blocco dovrá  avere ordine 1, e
λ 0 0
la matrice é diagonalizzabile, con forma canonica  0 λ 0 .
0 0 λ3
Al contrario, se la molteplicitá geometrica di λ é 1, allora esiste un solo blocco di
Jordan relativo a λ. Poiché il blocco relativo a λ3 deve necessariamente  avere ordine

λ 1 0
1, allora il blocco relativo a λ ha ordine due, e la forma canonica é  0 λ 0 .
0 0 λ3
Infine consideriamo il caso in cui i tre autovalori siano tutti coincidenti col valore
λ.
Se la molteplicitá geometrica dell’autovalore
 é anche
 3, allora la forma canonica di
λ 0 0
Jordan si riduce a forma diagonale  0 λ 0 .
0 0 λ
Se la molteplicitá geometrica dell’autovalore é 2 allora esistono due blocchi di Jordan
relativi ad esso, quindi uno di questi avrá ordine 2 e l’altro avrá ordine 1. La forma
4 LA FORMA CANONICA DI JORDAN.

 
λ 1 0
canonica sará  0 λ 0 . Se la molteplicitá geometrica dell’autovalore é 1, esiste
0 0 λ
 
λ 1 0
un solo blocco di Jordan relativo ad esso e la forma canonica é  0 λ 1 .
0 0 λ
Esempio 6. Sia A ∈ M6 (lR),
 
0 1 −1 −1 1 −1

 0 0 1 2 0 1  
 0 0 1 1 0 0 
A= .

 0 0 0 1 0 1  
 0 0 0 0 2 −1 
0 0 0 0 1 0
Il polinomio caratteristico é det(A − λI) = λ2 (λ − 1)4 , cioé λ1 = 0 é autovalore
con molteplicitá algebrica 2, mentre l’autospazio ad esso associato ha dimensione
1. Esiste allora un solo blocco di Jordan ad esso relativo.
Infine l’autospazio relativo all’autovalore λ2 = 1 ha dimensione 1, quindi vi é un
solo blocco di Jordan relativo a λ2 , necessariamente di ordine 4.
La forma canonica sará:
 
1 1 0 0 0 0
 0 1 1 0 0 0 
   
 0 0 1 1 0 0 
A0 =   = J4 (1) 0
.
 0 0 0 1 0 0  0 J2 (0)
 
 0 0 0 0 0 1 
0 0 0 0 0 0

Esempio 7. Sia A ∈ M3 (lR),


 
1 2 5
A =  0 14 39  .
0 −5 −14

Svolg. Il polinomio caratteristico é (λ − 1)(1 − λ)(1 + λ). Quindi un autovalore


é λ1 = −1 con molteplicitá algebrica e geometrica pari ad 1, per cui ad esso é
associato un solo blocco di Jordan di ordine 1.
L’altro autovalore é λ2 = 1 con molteplicitá algebrica 2. La dimensione dell’autospazio
ad esso associato é 1, quindi vi é un solo blocco ad esso corrispondente, ed ovvia-
mente deve avere ordine 2. La forma canonica della matrice é allora
 
1 1 0
A0 =  0 1 0  .
0 0 −1

3. Ordine massimo di un blocco in matrici con un unico autovalore.


Quanto detto fino ad ora é sufficiente per determinare una eventuale forma canonica
di una matrice con un ordine due o tre. Occupiamoci ora di come determinare gli
ordini dei blocchi in matrici che presentino un unico autovalore, nel caso in cui la
LA FORMA CANONICA DI JORDAN. 5

matrice A ∈ Mn (lR) da riportare in forma canonica A0 abbia un ordine superiore o


uguale a 4.
Sia λ l’autovalore della matrice A, ossia il polinomio caratteristico sia det(A −
λI)n e si consideri B = A − λI, dove I sia la matrice identica di ordine n. Sup-
poniamo che B m = 0 per qualche intero m (si dice in tal caso che la matrice B é
nilpotente). Indichiamo con
r = min{t ∈ N : B t = 0}
dove indichiamo con 0 la matrice nulla (con ogni elemento nullo). Chiamiamo r
indice di nilpotenza di B.
Allora esisterá certamente almeno un blocco di Jordan relativo a λ che abbia
ordine r ed ogni altro eventuale blocco di Jordan relativo a λ avrá un ordine minore
o al piú uguale a r.
Esempio 8. Sia A ∈ M6 (lR),
 
0 1 1 0 0 0

 0 1 0 0 1 −1 

 −1 1 2 0 0 0 
A= .

 0 0 0 1 0 0 
 1 0 −1 1 1 1 
0 0 0 1 0 1

Svolg. Il polinomio caratteristico é det(A − λI) = (λ − 1)6 , cioé λ = 1 é autovalore


con molteplicitá algebrica 6.
L’autospazio associato é
V = {(a − b, −b, a, 0, b, b), a, b ∈ lR}
che ha quindi dimensione 2. Vi sono allora due blocchi relativi all’autovalore λ = 1.
Calcolando l’indice di nilpotenza della matrice B = A − λI = A − I, si avrá B 5 = 0,
cioé uno dei due blocchi ha ordine 5, quindi l’altro deve avere necessariamente
ordine 1. La forma canonica della matrice é
 
1 1 0 0 0 0
 0 1 1 0 0 0 
   
 0 0 1 1 0 0 
A0 =   = J5 (1) 0
.
 0 0 0 1 1 0  0 J1 (1)
 
 0 0 0 0 1 0 
0 0 0 0 0 1

4. Numero di blocchi di uno certo ordine relativi ad uno stesso autoval-


ore. Sia al solito A ∈ Mn (lR) con autovalori h1 , ..., hm . Fissiamo un h0 autovalore
di A. Abbiamo precedentemente visto che il numero totale di blocchi di Jordan rel-
ativi a h0 é pari alla dimensione dell’autospazio associato a h0 , cioé la molteplicitá
geometrica di h0 .
Ci proponiamo ora di risolvere l’ultimo problema nella costruzione della forma
canonica di Jordan di una matrice: fissiamo l’intero k ≤ t. Ci chiediamo quanti
blocchi di ordine k sono associati all’autovalore h0 .
Useremo la seguente notazione:
ri = rango(A − h0 I)i , per ogni i = 1, 2, 3, ..., t.
6 LA FORMA CANONICA DI JORDAN.

Se k = 1 allora il numero di blocchi di ordine 1 relativi a h0 é pari a

n − 2r1 + r2 .

Per k ≥ 2 vale la seguente tabella, in cui sono riportati a sinistra gli ordini di tutti i
blocchi che eventualmente si possono presentare, e a destra il numero di tali blocchi:

ORDINE DEI BLOCCHI NUMERO DEI BLOCCHI


2 r1 − 2r2 + r3
3 r2 − 2r3 + r4
4 r3 − 2r4 + r5
5 r4 − 2r5 + r6
6 r5 − 2r6 + r7
... ........
... ........
... ........
k rk−1 − 2rk + rk+1

Esempio 9. Consideriamo la matrice

 
1 4 5 0 0 0 0 0

 0 1 3 0 0 0 0 0 


 0 0 1 0 0 0 0 0 

 0 0 0 2 3 0 0 0 
A= .

 0 0 0 0 2 0 0 0 


 0 0 0 0 0 2 2 1 

 0 0 0 0 0 0 1 3 
0 0 0 0 0 0 0 1

Svolg. L’equazione caratteristica é det(A − hI) = (1 − h)5 (2 − h)3 = 0, da cui


otteniamo gli autovalori h1 = 1 con molteplicitá algebrica 5, h2 = 2 con molteplicitá
algebrica 3.
L’autospazio relativo all’autovalore h1 = 1 ha dimensione 2, quindi vi sono 2 blocchi
di Jordan relativi a tale autovalore. Inoltre:
 
0 4 5 0 0 0 0 0

 0 0 3 0 0 0 0 0 


 0 0 0 0 0 0 0 0 

 0 0 0 1 3 0 0 0 
A − h1 I =  

 0 0 0 0 1 0 0 0 


 0 0 0 0 0 1 2 1 

 0 0 0 0 0 0 0 3 
0 0 0 0 0 0 0 0

che ha rango 6, quindi r1 = 6.


LA FORMA CANONICA DI JORDAN. 7

 
0 0 12 0 0 0 0 0

 0 0 0 0 0 0 0 0 


 0 0 0 0 0 0 0 0 

2
 0 0 0 1 6 0 0 0 
(A − h1 I) =  

 0 0 0 0 1 0 0 0 


 0 0 0 0 0 1 2 7 

 0 0 0 0 0 0 0 0 
0 0 0 0 0 0 0 0
che ha rango 4, quindi r2 = 4.
 
0 0 0 0 0 0 0 0

 0 0 0 0 0 0 0 0 


 0 0 0 0 0 0 0 0 

0 0 0 1 9 0 0 0
(A − h1 I)3 = 
 


 0 0 0 0 1 0 0 0 


 0 0 0 0 0 1 2 7 

 0 0 0 0 0 0 0 0 
0 0 0 0 0 0 0 0
che ha rango 3, quindi r3 = 3.
 
0 0 0 0 0 0 0 0

 0 0 0 0 0 0 0 0 


 0 0 0 0 0 0 0 0 

4
 0 0 0 1 12 0 0 0 
(A − h1 I) =  

 0 0 0 0 1 0 0 0 


 0 0 0 0 0 1 2 7 

 0 0 0 0 0 0 0 0 
0 0 0 0 0 0 0 0
che ha rango 3, quindi r4 = 3. Quindi

ORDINE DEI BLOCCHI NUMERO DEI BLOCCHI


1 0
2 1
3 1
c’é un blocco di ordine 2 ed uno di ordine 3 relativi all’autovalore h1 = 1.
Passiamo all’autovalore h2 = 2.
 
−1 4 5 0 0 0 0 0

 0 −1 3 0 0 0 0 0 


 0 0 −1 0 0 0 0 0 

 0 0 0 0 3 0 0 0 
A − h2 I =  

 0 0 0 0 0 0 0 0 


 0 0 0 0 0 0 2 1 

 0 0 0 0 0 0 −1 3 
0 0 0 0 0 0 0 −1
che ha rango 6, quindi r1 = 6 e vi sono in totale 2 blocchi relativi a tale autovalore.
8 LA FORMA CANONICA DI JORDAN.

 
1 −8 2 0 0 0 0 0

 0 1 −6 0 0 0 0 0 


 0 0 1 0 0 0 0 0 

0 0 0 0 0 0 0 0
(A − h2 I)2 = 
 


 0 0 0 0 0 0 0 0 


 0 0 0 0 0 0 −2 5 

 0 0 0 0 0 0 1 −6 
0 0 0 0 0 0 0 1
che ha rango 5, quindi r2 = 5.
 
−1 12 −21 0 0 0 0 0

 0 −1 9 0 0 0 0 0 

 0 0 −1 0 0 0 0 0 
3
 0 0 0 0 0 0 0 0 
(A − h2 I) =  

 0 0 0 0 0 0 0 0 


 0 0 0 0 0 0 2 −11 

 0 0 0 0 0 0 −1 10 
0 0 0 0 0 0 0 −1
che ha rango 5, quindi r3 = 5. Quindi

ORDINE DEI BLOCCHI NUMERO DEI BLOCCHI


1 1
2 1
c’é un blocco di ordine 1 ed uno di ordine 2 relativi all’autovalore h2 = 2.
Quindi una forma canonica di Jordan della matrice di partenza é la seguente:
 
1 1 0 0 0 0 0 0

 0 1 0 0 0 0 0 0 


 0 0 1 1 0 0 0 0 

0
 0 0 0 1 1 0 0 0 
A = .

 0 0 0 0 1 0 0 0 


 0 0 0 0 0 2 0 0 

 0 0 0 0 0 0 2 1 
0 0 0 0 0 0 0 2
Osservazione 2. L’ordine con cui i blocchi compaiono nella forma canonica di
Jordan non é fondamentale per l’individuazione della forma canonica stessa. Si dice
infatti che la forma canonica ottenuta é unica a meno di permutazioni dei blocchi
di Jordan di cui é composta. Permutando tali blocchi si ottiene comunque una
forma canonica che sia simile a quella precedente. Tale permutazione corrisponde
di fatto alla scelta di una differente base di lRn rispetto alla quale la matrice che
rappresenta l’endomorfismo é ancora espressa in forma canonica di Jordan.

5. Il polinomio minimo di una matrice (o di un endomorfismo). Sia A ∈


Mn (lR) una matrice quadrata di ordine n ad elementi reali e sia f : lRn −→ lRn
l’endomorfismo ad essa associato. Indichiamo al solito p(λ) = det(A − λI) il poli-
nomio caratteristico di A (o equivalentemente di f ). Sará utile per l’argomento che
ora tratteremo un richiamo al ben noto Teorema di Caley-Hamilton:
LA FORMA CANONICA DI JORDAN. 9

Teorema 2. Ogni matrice quadrata A di un certo ordine n é una radice del suo
polinomio caratteristico p(λ) = det(A − λI), in altre parole se
p(λ) = a0 + a1 λ + a2 λ2 + ... + an λn
allora
p(A) = a0 I + a1 A + a2 A2 + ... + an An = 0.
Alla luce di ció, vi sono chiaramente infiniti polinomi f (x) di grado superiore
ad n tali da avere la matrice A come radice, sono esattamente tutti i polinomi
che ammettano il polinomio p(x) come divisore, cioé quelli del tipo f (x) = p(x) ·
h(x), per ogni polinomio h(x). Ci chiediamo allora se esistono polinomi di grado
inferiore a n che ammettano la matrice A come radice. Essi possono esistere, ma
non necessariamente.
Definizione 1. Il polinomio minimo di A é il polinomio monico (con coefficiente
1 per il monomio di grado massimo) di grado minimo che ammetta A come radice.
Se ne deduce che il polinomio minimo di A deve essere cercato tra tutti i poli-
nomi divisori del polinomio caratteristico di A. Se nessuno di tali polinomi dovesse
ammettere A come radice, concluderemmo che il polinomio minimo e quello carat-
teristico coincidono. Vale inoltre il seguente:
Teorema 3. Il polinomio minimo m(λ) di A deve contenere tutti gli zeri del poli-
nomio caratteristico p(λ).

Dim. Il risultato é ovvio se p(λ) = m(λ). Supponiamo allora che il µ = grado(m) <
grado(p) e per assurdo ammettiamo che esista un autovalore λi tale che il monomio
(λ − λi ) non sia un divisore di m(λ). Avremmo che
m(λ) = (λ − λi )q(λ) + r
dore q(λ) é un polinomio di grado µ − 1 e r é il resto della divisione tra m(λ) e
(λ − λi ). Poiché m(A) = 0 allora
0 = (A − λi I)q(A) + rI

(A − λi I)q(A) = −rI.
Passando ai determinanti delle precedenti matrici otteniamo
det(A − λi I) · det(q(A)) = (−r)n .
Quindi, visto che det(A − λi I) = 0, deduciamo che (−r)n = 0, cioé r = 0 e
m(λ) = (λ − λi )q(λ), da cui la conclusione che ogni radice di p(λ) lo é anche per
m(λ).
Quindi un metodo per determinare il polinomio minimo m(λ) é quello di scom-
porre in fattori p(λ)
p(λ) = (λ − λ1 )h1 (λ − λ2 )h2 ......(λ − λr )hr
con h1 +h2 +...+hr = n, e considerare dapprima il polinomio composto dal prodotto
dei fattori lineari di p:
m1 = (λ − λ1 )(λ − λ2 )......(λ − λr ).
10 LA FORMA CANONICA DI JORDAN.

Se la matrice A é radice di m1 , allora esso é il polinomio minimo. In caso contrario


si aumentano gradualmente ed alternativamente tutti i gradi dei monomi (λ − λi ),
fino a quando non si ottiene un polinomio del tipo
m(λ) = (λ − λ1 )t1 (λ − λ2 )t2 ......(λ − λr )tr
per cui A sia una radice, con 1 ≤ ti ≤ hi , per ogni i = 1, ..., r. Inoltre ogni esponente
ti é il massimo ordine dei blocchi di Jordan in cui si distribuisce l’autovalore hi nella
forma canonica finale della matrice.
Appare chiaro che, per matrici di ordini relativamente grandi, costruire tutti i
possibili divisori di p(λ) e verificare quale sia il minimo, potrebbe risultare compli-
cato. É quindi consigliabile risalire al polinomio minimo di una matrice, dopo aver
determinato l’ordine massimo di ciascun blocco di Jordan che in essa compare.

6. Autospazi generalizzati. Abbiamo fin qui analizzato il comportamento di


un endomorfismo e della matrice ad esso associata in una data base, riferendoci
in particolare alla sua trasformazione in forma canonica, diagonale o di Jordan.
Nel caso la matrice sia diagonalizzabile, siamo anche in grado di determinare la
base rispetto alla quale essa assume una forma diagonale, tramite la costruzione
della matrice diagonalizzante, attraverso la determinazione degli autovettori della
matrice. Vogliamo ora costruire un algoritmo che ci permetta di determinare una
base di vettori, rispetto alla quale la matrice, non diagonalizzabile, sia espressa in
forma canonica di Jordan. A tal fine abbiamo la necessitá di richiamare alcuni
risultati e definizioni relativi ad endomorfismi e matrici.
Definizione 2. Un endomorfismo f : V −→ V di uno spazio vettoriale V di
dimensione n é detto invariante su di un sottospazio W ≤ V se f (w) ∈ W , per
ogni w ∈ W . In tale caso l’applicazione restrittiva fW : W −→ W é ancora un
endomorfismo.
Osservazione 3. Sia f : V −→ V un endomorfismo di uno spazio vettoriale V di
dimensione n su di un campo K e siano W1 , ..., Wr sottospazi di V invarianti sotto
l’azione di f , tali che
V = W1 ⊕ W2 ⊕ W3 ⊕ ..... ⊕ Wr .
Consideriamo B1 , ..., Br rispettivamente basi per W1 , ..., Wr . Sappiamo che l’insieme
B = B1 ∪ B2 ∪ ... ∪ Br é una base di V . Allora la matrice associata ad f rispetto
alla base B di V é una matrice a blocchi
 
A1 0 0 0 0 0 0 0
 0 A2 0 0 0 0 0 0 
 
 0 0 A3 0 0 0 0 
 
 0 0 0 ... 0 0 0 0 
A=  
 0 0 0 0 ... 0 0 0 
 0 0 0 0 0 ... 0 0 
 
 0 0 0 0 0 0 Ar−1 0 
0 0 0 0 0 0 0 Ar
in cui ogni blocco Ai é la matrice associata alla restrizione fWi rispetto alla base
Bi .
Definizione 3. Un endomorfismo f : V −→ V é detto nilpotente di indice t se f t
é l’omomorfismo nullo, cioé se f t (v) = 0, per ogni v ∈ V . Come giá visto, si dice
LA FORMA CANONICA DI JORDAN. 11

indice di nilpotenza il minimo t ≥ 1, tale che f t = 0 (analogamente t é l’indice di


nilpotenza della matrice A associata a f , cioé At = 0 e As 6= 0, per ogni s ≤ t).
Sia quindi f : V −→ V un endomorfismo sullo spazio vettoriale V di dimensione
n sul campo K. Consideriamo i seguenti endomorfismi di V :
f 0 = id, f 1 = f, f 2 = f (f ), .. f t = f (f t−1 ).
Ciascuno di essi ha un nucleo ed una immagine:
ker(f i ) = {v ∈ V : f i (v) = 0}, Im(f i ) = {w ∈ V : ∃v ∈ V, f i (v) = w}.
La prima osservazione che possiamo fare é certamente la seguente:
0 = ker(f 0 ) ⊂ ker(f ) ⊂ ker(f 2 ) ⊂ ......... ⊂ ker(f s ) ⊂ ....
é una catena di sottospazi di V , ma poiché quest’ultimo ha dimensione finita, essa
non puó essere infinita, quindi esiste un certo s ≥ 1 tale che ker(f s ) = ker(f s+1 ).
In particolare notiamo che nel caso f sia nilpotente di indice t, allora ker(f t ) = V
ed Im(f t ) = 0.

Teorema 4. Sia V uno spazio vettoriale di dimensione n sul campo K e sia


f : V −→ V un endomorfismo. Esso é nilpotente se e solo se il suo polinomio
caratteristico é p(λ) = (−1)n λn , cioé se ammette come unico autovalore lo 0 con
molteplicitá n.

Dim. Sia A la matrice associata all’endomorfismo f e supponiamo che essa sia


nilpotente, esista quindi t ≥ 1 tale che At = 0. Se λ é un autovalore di A, allora λt
lo é per la matrice At . Quindi esiste X 6= 0 tale che 0 = At X = λt X, cioé λ = 0.
Al contrario, supponiamo che A ammetta come unico autovalore λ = 0. In tale
caso il polinomio caratteristico di A é p(λ) = λn . Infine, An = 0 segue dal Teorema
di Caley-Hamilton.
Definizione 4. Sia V uno spazio vettoriale di dimensione n sul campo K e sia
f : V −→ V un endomorfismo nilpotente. Indichiamo con s ≥ 1 il minimo intero
tale che ker(f s ) = ker(f s+1 ). In base alla definizione di nucleo di un endomorfismo
sappiamo che
ker(f s ) = {X ∈ V : ∃i ≤ s, f i (X) = 0}.
Il sottospazio ker(f s ) é detto autospazio generalizzato di f relativo all’autovalore
0 ed i suoi elementi sono detti autovettori generalizzati relativi all’autovalore 0.
Teorema 5. Sia V uno spazio vettoriale di dimensione n sul campo K e sia f :
V −→ V un endomorfismo nilpotente. Sia vr ∈ V tale che vr ∈ ker(f r )−ker(f r−1 )
e costruiamo la seguente catena di vettori in V :
vr−1 = f (vr ) vr−2 = f (vr−1 ) = f 2 (vr )
vr−3 = f (vr−2 ) = f 3 (vr ), .... ....v1 = f (v2 ) = f r−1 (vr ) 6= 0.
Allora vr−i ∈ ker(f r−i ) − ker(f r−i−1 ), per ogni i = 1, ..., r − 1. Inoltre i vettori
{v1 , v2 , ...., vr } sono linearmente indipendenti.

Dim. Per la dimostrazione della prima parte del teorema é sufficiente provare che
vr−i ∈ ker(f r−i ) − ker(f r−i−1 ) per un fissato i ∈ {1, ..., r − 1}. Per semplicitá
prendiamo i = 1, dobbiamo allora dimostrare che vr−1 ∈ ker(f r−1 ) − ker(f r−2 ).
Poiché vr−1 = f (vr ), allora f r−1 (vr−1 ) = f r−1 f (vr ) = f r (vr ) = 0, per cui
12 LA FORMA CANONICA DI JORDAN.

vr−1 ∈ ker(f r−1 ).


Inoltre f r−2 (vr−1 ) = f r−2 f (vr ) = f r−1 (vr ) 6= 0, da cui vr−1 ∈ / ker(f r−2 ).
Passiamo a dimostrare l’indipendenza dei vettori. Supponiamo che esista una com-
binazione lineare
α1 v1 + α2 v2 + . . . + αr vr = 0
cioé
α1 f r−1 (vr ) + α2 f r−2 (vr ) + . . . + αr−1 f (vr ) + αr vr = 0 (A)
r−1 j
ed applichiamo ad essa la f . Poiché vj ∈ ker(f ), per ogni j = 1, .., r − 1, ne
segue 0 = αr f r−1 (vr ), da cui αr = 0, poiché vr ∈ / ker(f r−1 ).
Possiamo riscrivere la combinazione lineare (A):
α1 f r−1 (vr ) + α2 f r−2 (vr ) + . . . + αr−1 f (vr ) = 0 (A0 )
ed applicando ora f r−2 ripetiamo lo stesso ragionamento per dimostrare che anche
αr−1 = 0. Un passo alla volta dimostriamo quindi che ogni αi = 0, ed i vettori
sono linearmente indipendenti.
Definizione 5. L’insieme {v1 , v2 , ...., vr }, composto dai vettori definiti nel teorema
precedente, é chiamato catena di autovettori generalizzati relativi allo 0 di lunghezza
r.
Osservazione 4. Sia V uno spazio vettoriale di dimensione n sul campo K e sia
f : V −→ V un qualsiasi endomorfismo. Sia λ un autovalore di f e costruiamo
il seguente endomorfismo g di V : g(X) = f (X) − λX, per ogni X ∈ V . In
particolare, se A é la matrice associata a f , allora A − λI é la matrice associata
a g. Inoltre, se Y ∈ V é un autovettore relativo a λ rispetto all’endomorfismo f ,
avremo (A − λI)Y = 0 = 0 · Y , cioé Y é un autovettore relativo all’autovalore 0
rispetto all’endomorfismo g.
Definizione 6. Sia V uno spazio vettoriale di dimensione n sul campo K e sia
f : V −→ V un endomorfismo. Sia λi un autovalore di f e costruiamo il seguente
endomorfismo di V : fi = f − λi I. Come fatto in precedenza, sia s ≥ 1, tale che
ker(fis ) = ker(fis+1 ), detto indice di λi Il sottospazio
ker(fis ) = {X ∈ V : ∃j ≤ s, fij (X) = 0}
é detto autospazio generalizzato di f relativo all’autovalore λi ed i suoi elementi
sono detti autovettori generalizzati relativi all’autovalore λi .
Ripetiamo quanto detto per ogni autovalore λi ∈ {λ1 , ..., λp } nello spettro di f
(insieme di tutti gli autovalori) e denotiamo
Ei = ker(fisi ) = {X ∈ V : ∃j ≤ si , fij (X) = 0}
gli autospazi generalizzati relativi a ciascun λi con indice si .
Osservazione 5. Se X ∈ Ei , allora fisi (X) = 0, da cui 0 = fisi +1 (X) = fisi (fi (X)).
Ció implica che fi (X) ∈ Ei .
Inoltre fi (X) = f (X) − λi X, cioé f (X) = fi (X) + λi X ∈ Ei . Quindi ogni spazio
Ei é invariante sotto l’azione di f .
Infine notiamo che la restrizione fi : Ei → Ei é un endomorfismo nilpotente, di
indice si . Questo ci consente di utilizzare la trattazione precedentemente esposta, al
fine di ottenere una catena di autovettori generalizzati in Ei , che siano linearmente
indipendenti.
LA FORMA CANONICA DI JORDAN. 13

Teorema 6. Sia V uno spazio vettoriale di dimensione n sul campo K, sia f :


V −→ V un endomorfismo e sia λi un autovalore di f . Sia vr ∈ V tale che
vr ∈ ker(fir ) − ker(fir−1 ) e costruiamo la seguente catena di vettori in V :
vr−1 = fi (vr ), vr−2 = fi (vr−1 ) = fi2 (vr )

vr−3 = fi (vr−2 ) = fi3 (vr ), .... ....v1 = fi (v2 ) = fir−1 (vr ) 6= 0.


Allora vr−i ∈ ker(fir−i ) − ker(fir−i−1 ), per ogni i = 1, ..., r − 1. Inoltre i vettori
{v1 , v2 , ...., vr } sono linearmente indipendenti e sono chiamati autovettori general-
izzati relativi all’autovalore λi .
Inoltre valgono i seguenti:
Teorema 7. La dimensione di ciascun autospazio generalizzato Ei é pari alla
molteplicitá algebrica dell’autovalore λi cui esso si riferisce.
Teorema 8. La massima lunghezza di una catena di autovettori relativi ad un
autovalore λi di f é pari all’indice si di λi . Inoltre si é ≤ della molteplicitá algebrica
dell’autovalore.
Concludiamo, riassumendo i passi da eseguire per ottenere una base di autovet-
tori generalizzati rispetto alla quale l’endomorfismo si esprima in forma canonica:
1. Siano λ1 , . . . , λp ∈ K gli autovalori distinti di A, ciascuno con la sua molteplicitá
algebrica, e siano E1 , . . . , Ep i rispettivi autospazi generalizzati.
2. Puntiamo l’attenzione su un fissato autovalore λ ∈ {λ1 , . . . , λp } e sul cor-
rispondente autospazio generalizzato E ∈ {E1 , . . . , Ep }. Siano inoltre m
la molteplicitá algebrica di λ e B1 , . . . , Bk i blocchi nei quali si presenta
l’autovalore λ nella forma canonica finale A0 di Jordan. Se d1 , . . . , dk sono
le dimensioni dei blocchi B1 , . . . , Bk , allora d1 + . . . + dk = m.
3. Ad ogni blocco Bj corrisponde una catena w1 , . . . , wdj di autovettori gener-
alizzati (sono in numero pari alla dimensione del blocco).
4. Riordiniamo i blocchi Bj , a partire dal piú grande fino al piú piccolo e costru-
iamo per ciascuno di essi la corrispondente catena di autovettori generalizzati.
L’unione di tutti i vettori presenti nelle catene é un insieme di m autovettori
generalizzati, i quali costituiscono la base per l’autospazio generalizzato E.
5. Ripetiamo questo processo per ciascun autovalore λ1 , . . . , λp , determinando
quindi una base per ciascuno dei corrispondenti E1 , . . . , Ep autospazi gener-
alizzati.
6. Riuniamo ora in un unico insieme B i vettori che formano la base di ogni
autospazio generalizzato E1 , . . . , Ep . Avremo ottenuto in tal modo
V = E1 ⊕ E2 ⊕ . . . ⊕ Ep
ed una base B rispetto alla quale la matrice associata ad f si esprime in forma
canonica di Jordan.
Esempio 10. Sia f : lR4 −→ lR4 con matrice A ∈ M4 (lR) associata
 
0 1 1 0
 0 0 1 0 
A=  0 0 0
.
0 
0 0 0 0
14 LA FORMA CANONICA DI JORDAN.

Svolg. Il polimomio caratteristico é p(λ) = λ4 e quello minimo m(λ) = λ3 . L’unico


autovalore é λ = 0 e le catene di autovettori sono 2, una di lunghezza 3 e l’altra di
lunghezza 1. Per costruirle si devono calcolare i sottospazi ker(f ), ker(2 ), ker(f 3 ).
ker(f ) = {X ∈ lR4 : AX = 0} =< w1 = (1, 0, 0, 0), w2 = (0, 0, 0, 1) > .
 
0 0 1 0
2
 0 0 0 0 
Inoltre A =   , quindi
0 0 0 0 
0 0 0 0
ker(f 2 ) = {X ∈ lR4 : A2 X = 0} =< w3 = (1, 0, 0, 0), w4 = (0, 1, 0, 0), w5 = (0, 0, 0, 1) > .
Infine A3 = 0 da cui ker(f 3 ) = lR4 =< w6 = (1, 0, 0, 0), w7 = (0, 1, 0, 0), w8 =
(0, 0, 1, 0), w9 = (0, 0, 0, 1) >. Iniziamo dalla catena di ordine 3:
v3 = w8 ∈ ker(f 3 ) − ker(f 2 )
v2 = f (v3 ) = (1, 1, 0, 0) ∈ ker(f 2 ) − ker(f )
v1 = f (v2 ) = f 2 (v3 ) = (1, 0, 0, 0).
Passiamo ora alla catena di ordine 1, per la quale é sufficiente scegliere un vettore
v10 ∈ ker(f )− < v1 , v2 , v3 >, prendiamo v10 = w2 = (0, 0, 0, 1). Possiamo ora definire
la base rispetto alla quale la matrice A ammette la forma canonica di Jordan:
B = {(1, 0, 0, 0), (1, 1, 0, 0), (0, 0, 1, 0), (0, 0, 0, 1)}
e la matrice P tale che P −1 AP sia la matrice in forma di Jordan é:
 
1 1 0 0
 0 1 0 0 
P =  0 0 1
.
0 
0 0 0 1

Esempio 11. Sia f : lR3 −→ lR3 con matrice A ∈ M3 (lR) associata


 
2 0 0
A =  2 2 0 .
1 3 2

Svolg. Il polimomio caratteristico e quello minimo coincidono: p(λ) = m(λ) =


(λ − 2)3 . L’unico autovalore é λ = 2 con una sola catena di autovettori una di
lunghezza 3. Per costruirla si devono calcolare i sottospazi ker(f ), ker(f 2 ), ker(f 3 ).
ker(f2 ) = {X ∈ lR3 : (A − 2I)X = 0} =< w1 = (0, 0, 1) > .
 
0 0 0
Inoltre (A − 2I)2 =  0 0 0 , quindi
6 0 0
ker(f22 ) = {X ∈ lR3 : (A − 2I)2 X = 0} =< w2 = (0, 1, 0), w3 = (0, 0, 1) > .
Infine (A − 2I)3 = 0 da cui ker(f23 ) = lR3 =< w4 = (1, 0, 0), w5 = (0, 1, 0), w6 =
(0, 0, 1) >. Costruiamo la catena di ordine 3:
v3 = w4 ∈ ker(f23 ) − ker(f22 )
v2 = f2 (v3 ) = (0, 2, 1) ∈ ker(f22 ) − ker(f2 )
v1 = f2 (v2 ) = f22 (v3 ) = (0, 0, 6).
LA FORMA CANONICA DI JORDAN. 15

La base rispetto alla quale la matrice A ammette la forma canonica di Jordan:


B = {(0, 0, 6), (0, 2, 1), (1, 0, 0)}
e la matrice P tale che P −1 AP = A0 sia la matrice in forma di Jordan é:
   
0 0 1 2 1 0
P = 0 2 0  con A0 =  0 2 1 
6 1 0 0 0 2

Esempio 12. Sia f : lR5 −→ lR5 con matrice A ∈ M5 (lR) associata


 
1 1 1 1 1
 0 1 0 1 1 
 
A=  0 0 1 0 1  .
 0 0 0 1 0 
0 0 0 0 1

Svolg. Il polimomio caratteristico é p(λ) = (λ − 1)5 e la forma canonica finale sará


 
1 1 0 0 0
 0 1 1 0 0 
A0 = 
 
 0 0 1 0 0


 0 0 0 1 1 
0 0 0 0 1
quindi avremo per l’autovalore λ = 1 una catena di ordine 3 ed una di ordine 2.
Calcoliamo la sequenza di tutti i sottospazi generalizzati relativi a λ = 1:
ker(f1 ) = {X ∈ lR5 : (A − I)X = 0} =< w1 = (1, 0, 0, 0, 0), w2 = (0, 1, −1, 0, 0) > .
 
0 0 0 1 1
 0 0 0 0 0 
 
Inoltre (A − I)2 = 
 0 0 0 0 0 , quindi
 0 0 0 0 0 
0 0 0 0 0
ker(f12 ) = {X ∈ lR5 : (A − I)2 X = 0} =
< w3 = (1, 0, 0, 0, 0), w4 = (0, 1, 0, 0, 0), w5 = (0, 0, 1, 0, 0), w6 = (0, 0, 0, 1, −1) > .
Infine (A − I)3 = 0 da cui
ker(f13 ) = lR5 =
< w7 = (1, 0, 0, 0, 0), w8 = (0, 1, 0, 0, 0), w9 = (0, 0, 1, 0, 0),
w10 = (0, 0, 0, 1, 0), w11 = (0, 0, 0, 0, 1) > .
Iniziamo dalla catena di ordine 3:
v3 = w11 ∈ ker(f13 ) − ker(f12 )
v2 = f1 (v3 ) = (1, 1, 1, 0, 0)
v1 = f1 (v2 ) = f12 (v3 ) = (2, 0, 0, 0, 0).
Passiamo ora alla catena di ordine 2: il vettore di partenza v20 deve essere scelto in
modo che
v20 ∈ ker(f12 ) − ker(f1 )− < (0, 0, 0, 0, 1), (1, 1, 1, 0, 0), (2, 0, 0, 0, 0) >
v20 = w6
16 LA FORMA CANONICA DI JORDAN.

v10 = f1 (v60 ) = (0, 0, −1, 0, 0)


Possiamo ora definire la base rispetto alla quale la matrice A ammette la forma
canonica di Jordan:
B = {(2, 0, 0, 0, 0), (1, 1, 1, 0, 0), (0, 0, 0, 0, 1), (0, 0, −1, 0, 0), (0, 0, 0, 1, −1)}.

Esempio 13. Sia f : lR5 −→ lR5 con matrice A ∈ M5 (lR) associata


 
1 0 0 0 0
 1 2 1 1 1 
 
A=  0 0 2 0 1  .
 0 0 0 1 0 
1 0 0 1 1

Svolg. Il polimomio caratteristico é p(λ) = (λ − 2)2 (λ − 1)3 . quindi avremo un


autovalore λ1 = 1 ed uno λ2 = 2. Calcoliamo la sequenza di tutti i sottospazi
generalizzati relativi a λ = 1:
ker(f1 ) = {X ∈ lR5 : (A−I)X = 0} =< w1 = (1, 0, 0, −1, 0), w2 = (0, 0, 1, 0, −1) > .
Da questo ricaviamo anche che l’autospazio (in senso usuale) ha dimensione 2,
quindi vi sono due blocchi di Jordan per λ1 = 1, da cui riconosciamo
 la presenza 
di
0 0 0 0 0
 2 1 2 2 2 
 
una catena di ordine 2 ed una di ordine 1. Inoltre (A − I)2 = 
 1 0 1 1 1 ,

 0 0 0 0 0 
0 0 0 0 0
quindi
ker(f12 ) = {X ∈ lR5 : (A − I)2 X = 0} =
< w3 = (1, 0, 0, 0, −1), w4 = (0, 0, 1, 0, −1), w5 = (0, 0, 0, 1, −1) > .
Iniziamo dalla catena di ordine 2:
v2 = w5 ∈ ker(f12 ) − ker(f1 )
v1 = f1 (v2 ) = (0, 0, −1, 0, 1).
Passiamo ora alla catena di ordine 1: il vettore in ker(f1 )− < (0, 0, −1, 0, 1), (0, 0, 0, 1, −1) >:
v10 = w1 = (1, 0, 0, −1, 0).
Consideriamo ora l’autovalore λ2 = 2:
ker(f2 ) = {X ∈ lR5 : (A − 2I)X = 0} =< w6 = (0, 1, 0, 0, 0) >
quindi vi é un solo blocco relativo a tale autovalore, e sará di ordine 2. Vi é allora 
1 0 0 0 0
 0
 0 0 −1 0 

una catena di ordine 2 di autovettori generalizzati: (A−2I)2 =   1 0 0 0 −1 ,

 0 0 0 1 0 
−2 0 0 0 1
quindi
ker(f22 ) = {X ∈ lR5 : (A − 2I)2 X = 0} =< w7 = (0, 1, 0, 0, 0), w8 = (0, 0, 1, 0, 0) >
da cui le scelte per la catena sono:
u2 = w8 ∈ ker(f22 ) − ker(f2 )
LA FORMA CANONICA DI JORDAN. 17

u1 = f2 (u2 ) = (0, 1, 0, 0, 0).


Possiamo ora definire la base rispetto alla quale la matrice A ammette la forma
canonica di Jordan:
B = {(1, 0, 0, −1, 0), (0, 0, −1, 0, 1), (0, 0, 0, 1, −1), (0, 1, 0, 0, 0), (0, 0, 1, 0, 0)}.

Esempio 14. Sia f : lR3 −→ lR3 con matrice A ∈ M3 (lR) associata


 
3 1 1
A =  0 3 1 .
0 0 3

Svolg. Il polimomio caratteristico é p(λ) = (λ − 3)3 . L’unico autovalore é λ = 3:


ker(f3 ) = {X ∈ lR3 : (A − 3I)X = 0} =< w1 = (1, 0, 0) >
quindi vi é un solo blocco
 di Jordane quindi una sola catena, chiaramente di ordine
0 0 1
3. Inoltre (A − 3I)2 =  0 0 0 , quindi
0 0 0
ker(f32 ) = {X ∈ lR3 : (A − 3I)2 X = 0} =< w2 = (1, 0, 0), w3 = (0, 1, 0) > .
Infine (A − 3I)3 = 0 da cui ker(f33 ) = lR3 =< w4 = (1, 0, 0), w5 = (0, 1, 0), w6 =
(0, 0, 1) >. Costruiamo la catena di ordine 3:
v3 = w6 ∈ ker(f33 ) − ker(f32 )
v2 = f3 (v3 ) = (1, 1, 0) ∈ ker(f22 ) − ker(f2 )
v1 = f3 (v2 ) = f32 (v3 ) = (1, 0, 0).
La base rispetto alla quale la matrice A ammette la forma canonica di Jordan:
B = {(1, 0, 0), (1, 1, 0), (0, 0, 1)}.

Esempio 15. Sia f : lR3 −→ lR3 con matrice A ∈ M3 (lR) associata


 
1 1 −1
A =  0 1 0 .
9 2 7

Svolg. Il polimomio caratteristico: p(λ) = (λ − 1)(λ − 4)2 . Per l’autvalore λ1 = 1


non abbiamo alcun problema ad individuare l’autovettore (che in tal caso é auto-
valore in senso ordinario e generalizzato):
ker(f1 ) = {X ∈ lR3 : (A − I)X = 0} =< w1 = (8, −9, −9) > .
Passiamo all’autovalore λ4 = 4:
ker(f4 ) = {X ∈ lR3 : (A − 4I)X = 0} =< w2 = (1, 0, −3) >
quindi vi é un solo blocco di ordine 2 relativo all’autovalore 4, e quindi anche
 una
0 8 0
sola catena di autovettori di lunghezza 2. Inoltre (A − 4I)2 =  0 9 0 , quindi
0 9 0
ker(f42 ) = {X ∈ lR3 : (A − 4I)2 X = 0} =< w3 = (1, 0, 0), w4 = (0, 0, 1) > .
18 LA FORMA CANONICA DI JORDAN.

Costruiamo la catena di ordine 2:


v2 = w3 ∈ ker(f42 ) − ker(f4 )
v1 = f4 (v2 ) = (−3, 0, 9).
La base rispetto alla quale la matrice A ammette la forma canonica di Jordan:
B = {(−3, 0, 9), (1, 0, 0), (8, −9, −9)}.

Esempio 16. Sia f : lR3 −→ lR3 con matrice A ∈ M3 (lR) associata


 
2 0 0
A= 0 1 2 .
0 −2 −3

Svolg. Il polimomio caratteristico: p(λ) = (λ − 2)(λ + 1)2 . Per l’autvalore λ2 = 2


non abbiamo alcun problema ad individuare l’autovettore (che in tal caso é auto-
valore in senso ordinario e generalizzato):
ker(f2 ) = {X ∈ lR3 : (A − 2I)X = 0} =< w1 = (1, 0, 0) > .
Passiamo all’autovalore λ1 = −1:
ker(f1 ) = {X ∈ lR3 : (A + I)X = 0} =< w2 = (0, 1, −1) >
quindi vi é un solo blocco di ordine 2 relativo all’autovalore −1,
 e quindi anche
 una
1 0 0
sola catena di autovettori di lunghezza 2. Inoltre (A + I)2 =  0 0 0 , quindi
0 0 0
ker(f12 ) = {X ∈ lR3 : (A + I)2 X = 0} =< w3 = (0, 1, 0), w4 = (0, 0, 1) > .
Costruiamo la catena di ordine 2:
v2 = w3 ∈ ker(f12 ) − ker(f1 )
v1 = f1 (v2 ) = (0, 2, −2).
La base rispetto alla quale la matrice A ammette la forma canonica di Jordan:
B = {(0, 2, −2), (0, 1, 0), (1, 0, 0)}.
LA FORMA CANONICA DI JORDAN. 19

7. Esercizi.
Esercizio 1. Determinare la forma canonica di Jordan della seguente matrice:
1
 
3 4 .
−1 1
Esercizio 2. Determinare la forma canonica di Jordan della seguente matrice:
 
1 0 0
 0 1 0 .
1 0 2
Esercizio 3. Determinare la forma canonica di Jordan della seguente matrice:
 
2 3 0
 0 2 0 .
4 0 2
Esercizio 4. Determinare la forma canonica di Jordan della seguente matrice:
 
1 2 5
 0 14 39  .
0 −5 −14
Esercizio 5. Determinare la forma canonica di Jordan della seguente matrice:
 
0 1 1 0 0 0
 0 1 0 0 1 −1 
 
 −1 1 2 0 0 0 
 0 0 0 1 0 0 .
 
 
 1 0 −1 1 1 1 
0 0 0 1 0 1
Esercizio 6. Determinare la forma canonica di Jordan della seguente matrice:
 
0 1 −1 −1 1 −1
 0 0 1 1 0 0 
 
 0 0 1 1 0 0 
 .
 0 0 0 1 0 1 
 
 0 0 0 0 2 −1 
0 0 0 0 1 0
LE FORME BILINEARI E LE FORME QUADRATICHE REALI.

1. Introduzione. Siano K un campo e V , W due spazi vettoriali su K, rispettiva-


mente di dimensione n e m. La applicazione f : V × W −→ K che fa corrispondere
ad una coppia di vettori (v, w) ∈ V × W uno scalare f (v, w) ∈ K, é detta forma
bilineare se valgono le seguenti proprietá: per ogni v, v1 , v2 ∈ V , w, w1 , w2 ∈ W ,
α, β ∈ K
f (αv1 + βv2 , w) = αf (v1 , w) + βf (v2 , w)
f (v, αw1 + βw2 ) = αf (v, w1 ) + βf (v, w2 ).
In sostanza la f é lineare sia in V che in W .

Esempio 1. La applicazione f : lR2 ×lR3 −→ lR, definita da f ((x1 , x2 ), (y1 , y2 , y3 )) =


x1 (y1 + y2 ) + x2 (y1 − y3 ), é una forma bilineare.

Svolg. Per dimostrarlo é sufficiente verificare che per ogni a, b ∈ lR, (x1 , x2 ), (x01 , x02 ) ∈
lR2 , (y1 , y2 , y3 ), (y10 , y20 , y30 ) ∈ lR3 , si abbia
f (a(x1 , x2 )+b(x01 , x02 ), (y1 , y2 , y3 )) = af ((x1 , x2 ), (y1 , y2 , y3 ))+bf ((x01 , x02 ), (y1 , y2 , y3 ))
e
f ((x1 , x2 ), a(y1 , y2 , y3 )+b(y10 , y20 , y30 )) = af ((x1 , x2 ), (y1 , y2 , y3 ))+bf ((x1 , x2 ), (y10 , y20 , y30 )).

Osservazione 1. Siano 0V e 0W rispettivamente il vettore nullo in V e quello


nullo in W . Allora f (0V , w) = 0 e f (v, 0W ) = 0, per ogni v ∈ V , w ∈ W .

2. Forme bilineari e matrici. Sia f : V × W −→ K una forma bilineare e siano


rispettivamente B = {b1 , b2 , .., bn } e C = {c1 , c2 , .., cm } una base di V ed una di
W entrambe su K. Allora per ogni coppia di vettori v ∈ V e w ∈ W , essi si
possono esprimere come combinazione lineare dei vettori base tramite le proprie
componenti:
v = v1 b1 + v2 b2 + v3 b3 + ...... + vn bn
w = w1 c1 + w2 c2 + w3 c3 + ...... + wm cm .
Allora, sfruttando le proprietá di bilinearitá della f si ha:
f (v, w) = f (v1 b1 + v2 b2 + v3 b3 + ...... + vn bn , w1 c1 + w2 c2 + w3 c3 + ...... + wm cm ) =
v1 w1 f (b1 , c1 ) + v1 w2 f (b1 , c2 ) + v1 w3 f (b1 , c3 ) + .... + v1 wm f (b1 , cm )+
v2 w1 f (b2 , c1 ) + v2 w2 f (b2 , c2 ) + v2 w3 f (b2 , c3 ) + .... + v2 wm f (b2 , cm ) + .....
........ + vn w1 f (bn , c1 ) + vn w2 f (bn , c2 ) + vn w3 f (bn , c3 ) + .... + vn wm f (bn , cm ).
Indichiamo con aij = f (bi , cj ), al variare di tutti gli elementi bi e cj delle due
basi. Con tali scalari costruiamo una matrice A = (aij ) che viene detta la matrice
associata alla f rispetto alle basi B e C. Se denotiamo

1
2 LE FORME BILINEARI E LE FORME QUADRATICHE REALI.

   
v1 w1

 v2 


 w2 


 v3 


 w3 

v=
 ... ,w = 
  ... 


 ... 


 ... 

 ...   ... 
vn wm
allora possiamo osservare che f (v, w) = v T Aw. Quindi ad ogni forma bilineare é
associata una matrice relativa ad una scelta di basi.
Viceversa per ogni matrice A ∈ Mnm (K) esiste una forma bilineare f : V ×W −→
K, definita da f (X, Y ) = X T AY , per ogni vettore X ∈ V espresso per componenti
in una certa base B di V e per ogni vettore Y ∈ W espresso per componenti in una
certa base C di W .
In definitiva, la scelta delle basi é determinante al fine di definire una qualsiasi
forma bilineare associata ad una matrice.

Esempio 2. Sia f : lR2 × lR3 −→ lR, definita da f ((x1 , x2 ), (y1 , y2 , y3 )) = x1 (y1 +


y2 )+x2 (y1 −y3 ), determiniamo la matrice associata a f rispetto alle basi canoniche
B = {b1 , b2 } = {(1, 0), (0, 1)} di lR2 e C = {c1 , c2 , c3 } = {(1, 0, 0), (0, 1, 0), (0, 0, 1)}
di lR3 .

Svolg.
f (b1 , c1 ) = 1, f (b1 , c2 ) = 1, f (b1 , c3 ) = 0
f (b2 , c1 ) = 1, f (b2 , c2 ) = 0, f (b2 , c3 ) = −1
allora la f si puó rappresentare, rispetto alle basi canoniche, nel modo seguente:
 
  y1
  1 1 0
f (X, Y ) = x1 x2  y2  .
1 0 −1
y3
Consideriamo ora due basi diverse da quelle canoniche: D = {d1 , d2 } = {(1, 1), (2, 1)}
di lR2 e E = {e1 , e2 , e3 } = {(1, 1, 0), (0, 0, 1), (2, 0, 1)}. Calcoliamo gli elementi della
matrice associata in tali basi:
f ((x1 , x2 ), (y1 , y2 , y3 )) = x1 (y1 + y2 ) + x2 (y1 − y3 ),

f (d1 , e1 ) = 3, f (d1 , e2 ) = −1, f (d1 , e3 ) = 3


f (d2 , e1 ) = 5, f (d2 , e2 ) = −1, f (d2 , e3 ) = 5
allora la f si puó rappresentare, rispetto alle basi C e D, nel modo seguente:
 
  y1
  3 −1 3  y2  = x1 (3y1 −y2 +3y3 )+x2 (5y1 −y2 +5y3 ).
f (X, Y ) = x1 x2
5 −1 5
y3

3 2
Esempio 3.  Sia f: lR × lR −→ lR una forma bilineare, alla quale sia associata
1 1
la matrice  1 0  rispetto alle basi B = {b1 , b2 , b3 } = {(1, 1, 0), (0, 0, 1), (0, 1, 1)}
2 1
LE FORME BILINEARI E LE FORME QUADRATICHE REALI. 3

di lR3 e C = {c1 , c2 } = {(1, 1), (0, 1)} di lR2 . Determiniamo la matrice associata
a f rispetto alle basi D = {d1 , d2 , d3 } = {(0, 1, 0), (0, 1, 1), (2, 0, 1)} di lR3 e E =
{e1 , e2 } = {(1, 1), (2, 1)} di lR2 .

Svolg. L’espressione della f rispetto alle basi B e C é data da:


 
 1 1
 
 y1
f (X, Y ) = x1 x2 x3  1 0  = x1 (y1 + y2 ) + x2 y1 + x3 (2y1 + y2 ).
y2
2 1
Per poter determinare la matrice rispetto alle nuove basi, dovremmo calcolare i co-
efficienti f (di , ej ). Poiché l’unico strumento che abbiamo é la matrice data rispetto
alle basi B e C, siamo costretti inizialmente ad esprimere tutti i vettori di e ej per
componenti rispetto alle basi B e C, e solo allora potremo effettuare il calcolo degli
f (di , ej ).
d1 = (0, 1, 0) = (0, −1, 1)B , d2 = (0, 1, 1) = (0, 0, 1)B , d3 = (2, 0, 1) = (2, 3, −2)B
e1 = (1, 1) = (1, 0)C , e2 = (2, 1) = (2, −1)C .
f (d1 , e1 ) = 1, f (d1 , e2 ) = 1
f (d2 , e1 ) = 2, f (d2 , e2 ) = 3
f (d3 , e1 ) = 1, f (d3 , e2 ) = 2
allora la f si puó rappresentare, rispetto alle basi D e E, nel modo seguente:
 
 1 1  
 y1
f (X, Y ) = x1 x2 x3  2 3  = x1 (y1 +y2 )+x2 (2y1 +3y2 )+x3 (y1 +2y2 ).
y2
1 2

3. Forme quadratiche. Sia f : V × V −→ K una forma bilineare definita in


V × V , dove V sia uno spazio vettoriale di dimensione n sul campo K. Sia B =
{e1 , e2 , .., en } una base di V e sia A la matrice associata a f rispetto a B. Per quanto
detto fin’ora, una rappresentazione della f é la seguente: f (X, Y ) = X T AY , per
ogni X, Y vettori di V .
Diciamo che la forma bilineare é simmetrica se accade che f (X, Y ) = f (Y, X) per
ogni X, Y ∈ V . Poiché, da tale definizione, in particolare dovrá essere f (ei , ej ) =
f (ej , ei ), per ogni ei , ej vettori differenti della base B, allora la matrice A associata
alla f , sará una matrice simmetrica.
Viceversa, data una matrice A simmetrica di ordine n, si puó definire la forma
bilineare f (X, Y ) = X T AY , definita in V × V , dove V sia uno spazio vettoriale di
dimensione n su un campo K. Si osserva facilmente che tale f é una forma bilineare
simmetrica.
In definitiva tutte le forme bilineari simmetriche sono rappresentate da matrici
simmetriche ed inoltre, per ogni matrice simmetrica si puó definire una forma bi-
lineare simmetrica, relativamente ad una base B dello spazio vettoriale.

Esempio 4.  Sia f : lR3 × lR3 −→ lR una forma bilineare, alla quale sia associata
1 1 0
la matrice  1 0 2  rispetto alla base canonica di lR3 .
0 2 1
4 LE FORME BILINEARI E LE FORME QUADRATICHE REALI.

Svolg. La rappresentazione della f é allora la seguente

  
  1 1 0 y1
f (X, Y ) = x1 x2 x3  1 0 2   y2  = x1 y1 +x2 y1 +x1 y2 +2x3 y2 +2x2 y3 +x3 y3 .
0 2 1 y3

Essa é simmetrica, poiché associata ad una matrice simmetrica. Si noti che per
determinare la simmetricitá della f é sufficiente constatare che le coppie dei termini
misti xi yj e xj yi abbiano lo stesso coefficiente, per ogni scelta di i e j.

Introduciamo ora il concetto di forma quadratica associata ad una forma bi-


lineare simmetrica. Sia quindi f : V × V −→ K una forma bilineare simmetrica
definita in V × V , dove V sia uno spazio vettoriale di dimensione n sul campo K e
sia A la matrice associata alla f . Definiamo q : V −→ K la applicazione che agisce
nel modo seguente: per ogni X ∈ V , q(X) = f (X, X) = X T AX. La q é detta
forma quadratica (associata o polare alla forma f ).

Esempio 5. Come nell’esempio precedente, sia f : lR3 × lR3 −→ lR la forma


bilineare simmetrica definita da
  
  1 1 0 y1
f (X, Y ) = x1 x2 x3  1 0 2   y2  = x1 y1 +x2 y1 +x1 y2 +2x3 y2 +2x2 y3 +x3 y3
0 2 1 y3

rispetto alla base canonica di lR3 .

Svolg. Allora la forma quadratica associata a f é la q : lR3 −→ lR definita da


  
 1 1 0 x1
q(X) = f (X, X) = x1 x2 x3  1 0 2   x2  = x21 +2x1 x2 +4x2 x3 +x23 .

0 2 1 x3

Notiamo che, pur rimanendo invariata la matrice associata ad f e q , essa si


ottiene in modo differente se ricavata dai coefficienti dei monomi presenti nella
espressione di f oppure da quelli presenti nella espressione di q.
Infatti se volessimo costruire tale matrice, rispetto alle basi canoniche, a par-
tire dalla f , sarebbe sufficiente considerare il coefficiente del termine xi yj come
l’elemento di posto (i, j) della matrice stessa.
Se invece volessimo la matrice, a partire dalla q, dovremmo notare che il termine
misto xi xj scaturisce dalla somma dei due termini misti xi yj e xj yi presenti nella
b
f . Per cui, se bij é il coefficiente di xi xj allora 2ij é l’elemento di posto (i, j) nella
matrice.
Per quanto riguarda gli elementi della diagonale principale della matrice, essi
sono i coefficienti dei termini xi yi in f oppure, indifferentemente, dei termini x2i in
q.
LE FORME BILINEARI E LE FORME QUADRATICHE REALI. 5

Esempio 6. Sia f : lR3 −→ lR la forma quadratica definita da q(X) = 3x21 +


2x1 x2 − 4x2 x3 + 7x22 − x23 cioé
  
3 1 0 x1
x2  = 3x21 + 2x1 x2 − 4x2 x3 + 7x22 − x23
 
q(X) = x1 x2 x3  1 7 −2  
0 −2 −1 x3
rispetto alla base canonica di lR3 .

Svolg. Allora la forma bilineare simmetrica associata a q é la f : lR3 × lR3 −→ lR


definita da
  
  3 1 0 y1
f (X, Y ) = x1 x2 x3  1 7 −2   y2  =
0 −2 −1 y3
3x1 y1 + x1 y2 + 7x2 y2 + x2 y1 − 2x2 y3 − x3 y3 − 2x3 y2 .

Per quanto visto fin’ora, potremo da ora in poi confondere le proprietá relative
ad una forma bilineare simmetrica con quelle della forma quadratica ad essa as-
sociata. Infatti tali proprietá sono semplicemente caratteristiche della matrice che
rappresenta entrambe le f e q.

4. Cambiamento di base. Sia q : lRn −→ lR una forma quadratica reale. Siano


A1 la matrice associata alla q rispetto alla base B1 di lRn e A2 la matrice associata
rispetto alla base B2 di lRn . Indichiamo con C la matrice del cambiamento di base
da B1 a B2 . Quindi, se X ∈ lRn é un vettore, di componenti X1 rispetto alla base
B1 e componenti X2 rispetto alla base B2 , allora la legge che lega tra loro tali
componenti é X2 = CX1 .
Calcolando la q(X) rispetto alle due differenti basi otteniamo
q(X) = X1T A1 X1 e q(X) = X2T A2 X2
da cui
X1T A1 X1 = X2T A2 X2 = (CX1 )T A2 (CX1 ) = X1T C T A2 CX1
con
A1 = C T A2 C.
Questa é la relazione che lega tra loro le due matrici associate alla stessa forma
quadratica, ma rispetto a due basi differenti. Essa non é una relazione di similitu-
dine, eccetto quando la matrice di cambiamento di base C sia ortogonale. Diremo
che le matrici A1 e A2 sono congruenti. La congruenza garantisce che, se una delle
due é simmetrica, allora lo é anche l’altra, ma in generale esse hanno autovalori
differenti. Quindi due matrici rappresentano la stessa forma quadratica rispetto a
due basi diverse se e solo se esse sono congruenti.

5. Diagonalizzabilitá delle matrici simmetriche reali. Ci occupiamo ora della


principale proprietá delle matrici (e quindi endomorfismi) simmetriche (simmetrici):
esse sono sempre diagonalizzabili. In particolre rivolgiamo la nostra attenzione alle
matrici simmetriche reali (gli endomorfismi rappresentati da matrici simmetriche
reali).
Prima di dimostrare il principale risultato di questo paragrafo, consideriamo alcune
proprietá delle matrici simmetriche:
6 LE FORME BILINEARI E LE FORME QUADRATICHE REALI.

1. Siano λ1 6= λ2 autovalori distinti di una matrice A simmetrica e X1 , X2 due


autovettori corrispondenti rispettivamnte a λ1 eλ2 . Allora X2T AX1 = 0 (cioé
i due autovettori sono ortogonali).

Dim. Consideriamo AX1 = λ1 X1 e AX2 = λ2 X2 . Moltiplichiamo la prima


per X2T a sinistra. e la seconda per X1T sempre a sinistra. Otteniamo quindi
X2T AX1 = λ1 X2T X1 (i)
e
X1T AX2 = λ2 X1T X2 .
Di quest’ultima eseguiamo la trasposta di ambo i membri:
X2T AX1 = λ2 X2T X1 (ii).
Sottraiamo ora dalla (i) la (ii), ottenendo 0 = (λ1 −λ2 )X2T X1 , da cui X2T X1 =
0 e grazie alla (i) segue che X2T AX1 = 0. Si noti che la ortogonalitá tra due
vettori implica la loro indipendenza lineare.
2. Tutti gli autovalori di una matrice simmetrica A sono reali.

Dim. Indichiamo di seguito con il simbolo α∗ il coniugato di un numero


complesso α e con X ∗ il trasposto di un vettore X, che abbia inoltre come
componenti i coniugati delle componenti di X.
Sia λ un autovalore di A:
AX = λX, X 6= 0 (iii).
∗ ∗ ∗
Coniughiamo ambo i membri: X A = λ X , e moltiplichiamo ora a destra
ambo i membri per X:
X ∗ AX = λ∗ X ∗ X (iv).
Moltiplichiamo ora ambo i membri della (iii) a sinistra per X ∗ :
X ∗ AX = λX ∗ X (v)
e sottraiamo ora dalla (iv) la (v): 0 = (λ∗ − λ)X ∗ X. Poiché X 6= 0, allora
anche X ∗ X 6= 0, da cui λ = λ∗ é elemento del campo dei reali.
3. Se A é una matrice di ordine n simmetrica e C una matrice dello stesso ordine
ed ortogonale, allora C −1 AC é ancora simmetrica.

Dim. Sia B = C −1 AC. Allora


B T = (C −1 AC)T = (C T AC)T = C T AC = B.
Possiamo ora dimostrare il seguente:
Teorema 1. Sia f : lRn → lRn un endomorfismo. Esiste una base ortonormale
B = {v1 , .., vn } di autovettori di lRn se e solo se f é simmetrico.
In altre parole: Sia data la matrice A ∈ Mn (lR). Esiste una matrice non singolare
P ∈ Mn (lR), costituita da vettori colonna tra loro ortonormali, tale che P −1 · A · P
sia una matrice diagonale se e solo se A é simmetrica.

Dim. Dimostriamo prima che se A é simmetrica allora esiste una base di au-
tovettori ortonormali rispetto alla quale essa é diagonalizzabile. Lo facciamo per
induzione sulla dimensione di lRn (cioé sull’ordine di A). Per n = 1 abbiamo
LE FORME BILINEARI E LE FORME QUADRATICHE REALI. 7

A = [a], la quale é banalmente diagonale ed ottenibile come [1] · [a] · [1], dove [1] é
una matrice ovviamente ortogonale.
Supponiamo che il teorema sia vero per ogni matrice simmetrica di ordine q ≤ n−1.
Sia λ un autovalore di A, sia V l’autospazio relativo a λ e sia V =< X1 , . . . , Xr >.
Se r = n allora λ é l’unico autovalore di A con molteplicitá algebrica e geometrica
coincidenti. Quindi A é diagonalizzabile. Inoltre l’autospazio V coincide con lRn ,
per cui esistono certamente n autovettori relativi a λ che siano tra loro ortogonali
e costituiscano una base di lRn . Sia allora r ≤ n − 1. Quindi per ipotesi induttiva
esiste una base di r autovettori ortogonali per V . Esiste allora almeno un autoval-
ore reale differente da λ. Siano µ1 , . . . , µt tutti gli autovalori di A tra loro diversi e
differenti da λ. La somma delle molteplicitá algebriche di λ, µ1 , . . . , µt é pari a n.
In particolare la somma delle molteplicitá algebriche di µ1 , . . . , µt é pari a n − r.
Indichiano con V1 , . . . , Vt gli autospazi relativi a µ1 , . . . , µt .
Notiamo dapprima che f (Vi ) ⊆ Vi , cioé l’endomorfismo f si puó restringere ad ogni
autospazio Vi relativo ad ogni autovalore µi . Sia quindi (c1 , . . . , ck ) una base di Vi
formata da autovettori e sia Y ∈ Vi . Quindi esistono opportuni scalari βj tali che
Pk
Y = j=1 βj cj . Da cui

Xk k
X k
X k
X k
X
AY = f (Y ) = f ( βj cj ) = βj f (cj ) = βj Acj = βj (µi cj ) = µi βj cj .
j=1 j=1 j=1 j=1 j=1

Constatiamo allora che, per ogni X ∈ V ,


k
X k
X
X T AY = X T (µi βj cj ) = µi βj X T cj = 0
j=1 j=1

vista l’ortogonalitá tra X ed ogni cj . Questo implica che f (Y ) e X sono ancora


ortogonali. A questo punto, grazie all’ipotesi induttiva ed al fatto che f si restringe
ad ognuno degli spazi Vi (ciascuno dei quali di dimensione inferiore a n), possiamo
ammettere che ognuno degli spazi Vi abbia una base formata da autovettori tra loro
ortogonali e che la loro dimensione sia pari alla molteplicitá algebrica dell’autovalore
cui corrispondono. In particolare é facile osservare che la somma delle dimensioni
di V1 , . . . , Vt é pari a n − r. Quindi lRn = V ⊕ (V1 ⊕ . . . ⊕ Vt ) ed una sua base é
fornita da n autovettori tra loro ortogonali.
Diciamo quindi B = {X1 , X2 , . . . , Xr , Y1 , . . . , Yn+r } tale base di lRn formata da
versori normali tra loro (abbiamo indicato con Xi gli autovettori relativi a λ e con
Yj quelli relativi ad un altro qualsiasi autovalore diverso da λ). Sia C la matrice
ortogonale avente in ciascuna colonna rispettivamente le componenti dei vettori
{X1 , X2 , . . . , Xr , Y1 , . . . , Yn+r }. La matrice associata a f rispetto alla base B nel
dominio e nel codominio é A0 = C −1 AC, la quale é ancora simmetrica.
La prima colonna di A0 contiene l’immagine di X1 espressa per componenti rispetto
a B, quindi é esattamente (λ, 0, 0, . . . , 0), e lo stesso dicasi per la prima colonna,
poiché la matrice é simmetrica. Continuando in tal senso otteniamo tutti gli ingressi
nella matrice A0 , la quale é diagonale.
Passiamo ora alla seconda parte della dimostrazione e supponiamo quindi che esista
una base di autovettori ortonormali {X1 , X2 , . . . , Xn } per lRn : proveremo che la
matrice A deve essere simmetrica. Dapprima ricordiamo che la ortonormalitá di
{X1 , X2 , . . . , Xn } vuol dire che XiT AXj = 0 se i 6= j, e XiT AX = 1. Per dimostrare
che A sia simmetrica é sufficiente far vedere che per ogni u, v ∈ lRn si abbia uT Av =
8 LE FORME BILINEARI E LE FORME QUADRATICHE REALI.

v T Au. P P
Siano quindi u = i αi Xi e v = i βi Xi . Allora
X X XX
uT Av = ( αi Xi )T A( βj Xj ) = αi βj XiT AXj =
i j i j
X X
αi βi XiT AXi = αi βi
i i
ed analogamente
X X XX
v T Au = ( βi Xi )T A( αj Xj ) = βi αj XiT AXj =
i j i j
X X X
βi αi XiT AXi = βi αi = αi βi = uT Av.
i i i
Concludiamo allora che se q é una forma quadratica in lRn associata ad una
matrice A, poiché quest’ultima é simmetrica, allora esiste una base di lRn rispetto
alla quale la matrice associata a q é una matrice diagonale.

Teorema 2. Sia q : V −→ K una forma quadratica, con dim(V ) = n e K un


campo. Esiste una base di V rispetto alla quale la matrice associata a q sia di-
agonale, cioé rispetto alla quale q(X) = a1 x21 + a2 x22 + a3 x23 + .... + an x2n , con
a1 , .., an ∈ K. Analogamente, sia f : V × V −→ K una forma bilineare simmetrica.
Esiste una base di V rispetto alla quale la matrice associata a f sia diagonale,
cioé rispetto alla quale f (X, Y ) = b1 x1 y1 + b2 x2 y2 + b3 x3 y3 + .... + bn xn yn , con
b1 , .., bn ∈ K.
P
6. Metodo per la diagonalizzazione. Sia f (X, Y ) = aij xi yj la forma bilin-
eare associata alla base B = {e1 , e2 , .., en } e supponiamo che la matrice A = (aij )
non sia diagonale.
Passo 1. Sia f (e1 , e1 ) = a11 6= 0. Calcoliamo i seguenti coefficienti:
f (e1 , e2 ) f (e1 , e3 ) f (e1 , ei )
c2 = c3 = ..... ..... ci = .
f (e1 , e1 ) f (e1 , e1 ) f (e1 , e1 )
Determiniamo il seguente cambiamento di base
e01 = e1 , e02 = e2 − c2 e1 , e03 = e3 − c3 e1 .... ...e0n = en − cn e1 .
Diciamo C la matrice di cambiamento di base e determiniamo X = CX 0 , Y = CY 0 .
A questo punto sostituiamo X, Y nell’espressione della forma bilineare. La matrice
associata a f dopo aver effettuato tali sostituzioni, avrá la prima riga e la prima
colonna tutte nulle, eccetto eventualmente l’elemento a11 .
Passo 2. Per evitare di introdurre ulteriori simboli, indichiamo la base in cui
ora ci troviamo nuovamente {e1 , e2 , .., en } (ma sappiamo bene che non é quella di
partenza).
Sia ora f (e2 , e2 ) = a22 6= 0. Calcoliamo i seguenti coefficienti:
f (e2 , e3 ) f (e2 , e4 ) f (e2 , ei )
c3 = c4 = ..... ..... ci = .
f (e2 , e2 ) f (e2 , e2 ) f (e2 , e2 )
Determiniamo il seguente cambiamento di base
e01 = e1 , e02 = e2 , e03 = e3 − c3 e2 .... ...e0n = en − cn e2 .
LE FORME BILINEARI E LE FORME QUADRATICHE REALI. 9

Diciamo ancora C la matrice di cambiamento di base e determiniamo X = CX 0 ,


Y = CY 0 . A questo punto sostituiamo X, Y nell’espressione della forma bilineare.
La matrice associata a f dopo aver effettuato tali sostituzioni, avrá la seconda riga
e la seconda colonna tutte nulle, eccetto eventualmente l’elemento a22 .

Ripetiamo tale iter per ogni riga, ogni volta che f (ei , ei ) 6= 0. Alla fine otterremo
la forma diagonale della matrice e quindi della forma bilineare e di quella quadratica
associate.

Passo intermedio. Se dovessimo avere, nel i-esimo passo, il valore f (ei , ei ) = 0,


prima di procedere come giá esposto, dovremo effettuare un cambiamento di base da
{e1 , e2 , .., en } a {e01 , e02 , .., e0n } che garantisca la condizione f (e0i , e0i ) 6= 0. Per farlo,
sará sufficiente individuare il primo indice di colonna j per il quale f (ei , ej ) 6= 0.
Quindi si effettua il cambiamento di base seguente:
e0i = ei + αej , e0k = ek per ogni altro indice k
in modo che la scelta di α ∈ lR garantisca
f (e0i , e0i ) = f (ei + αej , ei + αej ) = 2αf (ei , ej ) + α2 f (ej , ej ) 6= 0.
A questo punto si potrá riprendere come nei passi 1 e 2.

La matrice di cambiamento di base finale (cioé quella che trasforma i vettori


dalle componenti rispetto alla base di partenza alle componenti rispetto all’ultima
base, in cui la matrice é diagonale) é data dal prodotto di tutte le matrici utilizzate
nei passaggi intermedi, moltiplicate nell’ordine in cui esse vengono utilizzate.

Esempio 7. Sia f : lR3 −→ lR la forma quadratica definita da q(X) = x1 x2 +


x1 x3 + x2 x3 cioé
1 1
  
  01 2 21 x1
q(X) = x1 x2 x3  2 0 2   x2  .
1 1
2 2 0 x3

Svolg. Passo 1.
f (e1 , e1 ) = 0 f (e1 , e2 ) 6= 0
e01
= e1 + e2 , e02 = e2 , e03 = e3
La matrice di cambiamento di base é
 
1 0 0
C1 =  1 1 0 
0 0 1
da cui
x1 = x01 , x2 = x01 + x02 , x3 = x03
e sostituendo la nuova espressione del vettore X otteniamo la forma quadratica
espressa nella nuova base
q(X) = x21 + x1 x2 + 2x1 x3 + x2 x3 =
1
  
  11 2 11 x1
x1 x2 x3  2 0 2   x2  .
1 12 0 x3
10 LE FORME BILINEARI E LE FORME QUADRATICHE REALI.

Passo 2.
f (e1 , e2 ) 1 f (e1 , e3 )
f (e1 , e1 ) 6= 0, c2 = = , c3 = =1
f (e1 , e1 ) 2 f (e1 , e1 )
1
e01 = e1 ,
e02 = e2 − e1 , e03 = e3 − e1
2
La matrice di cambiamento di base é
1 − 12 −1
 

C2 =  0 1 0 
0 0 1
da cui
1
x1 = x01 − x02 − x03 , x2 = x02 , x3 = x03
2
e sostituendo la nuova espressione del vettore X otteniamo la forma quadratica
espressa nella nuova base
1
q(X) = x21 − x22 − x23 =
4
  
  1 01 0 x1
x1 x2 x3  0 − 4 0   x2  .
0 0 −1 x3
La matrice finale del cambiamento di base é
1 − 21 −1
 
1
C = C1 · C2 =  1 2 −1 
0 0 1
ed infatti si ha che
1 1
1 − 12
       
1 1 0 0 2 2 −1 1 0 0
 −1 1
0 · 1 0 1 · 1 1
−1  =  0 − 14 0 .
2 2 2 2 2
1 1
1 −1 1 2 2 0 0 0 1 0 0 −1

Esempio 8. Sia f : lR3 −→ lR la forma quadratica definita da q(X) = 5x21 + 3x22 +


x1 x3 cioé
1
  
  5 0 2 x1
q(X) = x1 x2 x3  0 3 0   x2  .
1
2 0 0 x3

Svolg. Passo 1.
f (e1 , e2 ) f (e1 , e3 ) 1
f (e1 , e1 ) 6= 0, c2 = = 0, c3 = =
f (e1 , e1 ) f (e1 , e1 ) 10
1
e01 = e1 , e02 = e2 , e03 = e3 − e1
10
La matrice di cambiamento di base é
1
 
1 0 − 10
C= 0 1 0 
0 0 1
da cui
1 0
x1 = x01 − x , x2 = x02 , x3 = x03
10 3
LE FORME BILINEARI E LE FORME QUADRATICHE REALI. 11

e sostituendo la nuova espressione del vettore X otteniamo la forma quadratica


espressa nella nuova base
1 2
q(X) = 5x21 + 3x22 − x =
20 3
  
  5 0 0 x1
x1 x2 x3  0 3 0   x2  .
1
0 0 − 20 x3
La matrice finale del cambiamento di base é data dalla sola C ed infatti si ha che
5 0 12 1
       
1 0 0 1 0 − 10 5 0 0
 0 1 0 · 0 3 0 · 0 1 0 = 0 3 0 .
1 1 1
− 10 0 1 2 0 0 0 0 1 0 0 − 20

Il metodo precedentemente esposto permette quindi di individuare una base B


rispetto alla quale la forma quadratica assume una forma
q(X) = a11 x21 + a22 x22 + ... + arr x2r
con aii ∈ lR, dove r é il rango della matrice associata alla forma quadratica q, detto
anche rango di q. Esso vale in qualsiasi caso, indipendentemente dal campo K sul
quale V é spazio vettoriale, quindi non necessariamente per K = lR.
Poiché noi ci occupiamo prevalentemente di spazi vettoriali reali, possiamo ot-
tenere risultati piú precisi:
Teorema 3. Sia V uno spazio vettoriale reale, dim(V ) = n ≥ 1, q : V → lR
una forma quadratica reale su V . Esistono una base B di V , un numero intero
0 ≤ p ≤ r, dove r é il rango di q, tali che la matrice associata alla q rispetto alla
base B sia  
Ip 0 0
 0 −Ir−p 0  (a)
0 0 0
dove Ip é il blocco costituito dalla matrice identica di ordine p, Ir−p é il blocco
costituito dalla matrice identica di ordine r − p e gli altri sono tutti blocchi nulli.
In altre parole la forma quadratica si presenta nella forma
q(X) = x21 + x22 + ... + x2p − x2p+1 − ... − x2r .
Si noti che in modo equivalente possiamo dire che ogni matrice simmetrica A ∈
Mn (lR) é congruente ad una matrice diagonale della forma (a).
Esempio 9. La forma quadratica in lR3 , q(X) = x1 x2 + x1 x3 + x2 x3 ha una forma
canonica q(X) = x21 − 41 x22 − x23 , costruita rispetto alla base
1 1
B = {(1, 1, 0), (− , , 0), (−1, −1, 1)}.
2 2

Svolg. Poniamo allora a1 = 1, a2 = − 14 , a3 = −1 e scegliamo


(1, 1, 0) (− 12 , 12 , 0) (−1, −1, 1)
e01 = √ , e02 = √ , e03 = √
a1 −a2 −a3
e01 = (1, 1, 0), e02 = (−1, 1, 0), e03 = (−1, −1, 1).
12 LE FORME BILINEARI E LE FORME QUADRATICHE REALI.

Esprimiamo la forma quadratica rispetto a tale base, con matrice di passaggio


 
1 −1 −1
C =  1 1 −1 
0 0 1
cioé con matrice associata A0 = C T · A · C,
0 12 12
     
1 1 0 1 −1 −1
A0 =  −1 1 0  ·  12 0 12  ·  1 1 −1  =
1 1
−1 −1 1 2 2 0 0 0 1
 
1 0 0
 0 −1 0  .
0 0 −1

Gli interi p e r − p sono detti rispettivamente indice di positivitá e indice di


negativitá di q e la coppia (p, r − p) é detta segnatura di q.
Una forma quadratica q : V → V , con dim(V ) = n, é detta definita positiva se
q(X) > 0, per ogni X ∈ V e la sua segnatura é (n, 0); é detta definita negativa
se q(X) < 0, per ogni X ∈ V , e la sua segnatura é (0, n); é detta semidefinita
positiva se q(X) ≥ 0, per ogni X ∈ V , e la sua segnatura é (r, 0) con r ≤ n; é
detta semidefinita negativa se q(X) ≤ 0, per ogni X ∈ V , e la sua segnatura é (0, r)
con r ≤ n. Nel caso la segnatura di q sia (p, r − p) con 0 < p < r ≤ n, la forma
quadratica é detta indefinita.

Siano f : lRn × lRn → lR una forma bilineare simmetrica e q : lRn → lR la forma


quadratica associata. La f é detta definita positiva, quando lo é q. Una forma
bilineare definita positiva é anche chiamata prodotto scalare. Per quanto detto in
precedenza, per ogni forma quadratica q definita positiva esiste una base B di lRn
rispetto alla quale q sia esprimibile
q(X) = x21 + x22 + ..... + x2n
e quindi
f (X, Y ) = x1 y1 + x2 y2 + ... + xn yn
il quale é ben noto come prodotto scalare standard. Uno spazio vettoriale V in cui
sia introdotto un prodotto scalare é detto spazio vettoriale euclideo.

7. Esercizi svolti sulla diagonalizzazione.


Esercizio 1. Sia q : lR3 −→ lR la forma quadratica definita da q(X) = x21 −2x1 x2 +
x1 x3 cioé
1 −1 21
  
  x1
q(X) = x1 x2 x3  −1 0 0   x2  .
1
2 0 0 x3
Determinarne una forma diagonale.

Svolg.
La matrice associata alla forma quadratica é
1 −1 21
 

A =  −1 0 0 
1
2 0 0
LE FORME BILINEARI E LE FORME QUADRATICHE REALI. 13

Poiché q(e1 ) 6= 0 allora possiamo effettuare il seguente cambiamento di base:


1
−1 1
e01 = e1 , e02 = e2 − e1 = e2 + e1 , e03 = e3 − 2 e1 = e3 − e1
1 1 2
la cui matrice associata é
1 1 − 12
 

C1 =  0 1 0 
0 0 1
da cui
1
x1 = x01 + x02 − x03 , x2 = x02 , x3 = x03 .
2
L’espressione della forma quadratica nelle nuove coordinate é
1
q(x1 , x2 , x3 ) = x21 − x22 − x23 + x2 x3
4
con matrice associata  
1 0 0
A0 =  0 −1 1 
2 .
0 2 − 14
1

Poiché q(e2 ) 6= 0 allora possiamo effettuare il seguente cambiamento di base:


1
e01 = e1 , e02 = e2 , e03 = e3 − 2
e2 = e3 + e2
−1
la cui matrice associata é  
1 0 0
1
C2 =  0 1 2

0 0 1
da cui
1
x1 = x01 ,
x2 = x02 + x03 , x3 = x03 .
2
L’espressione della forma quadratica nelle nuove coordinate é
q(x1 , x2 , x3 ) = x21 − x22
con matrice associata  
1 0 0
A00 =  0 −1 0  .
0 0 0
La base ortogonale B rispetto a cui la forma quadratica si esprime in forma diago-
nale é data dalle colonne della matrice C = C1 × C2 :
 
1 1 0
C =  0 1 21 
0 0 1
per cui B = {(1, 0, 0), (1, 1, 0), (0, 12 , 1)} e la matrice associata alla forma quadratica
in tale base é A00 =T CAC.
Esercizio 2. Sia q : lR3 −→ lR la forma quadratica definita da q(X) = 2x21 −x1 x2 +
2x1 x3 cioé
2 − 12 1
  
x1
q(X) = x1 x2 x3  − 12
 
0 0   x2  .
1 0 0 x3
Determinarne una forma diagonale.
14 LE FORME BILINEARI E LE FORME QUADRATICHE REALI.

Svolg. La matrice associata alla forma quadratica é


2 − 12 1
 

A =  − 12 0 0 
1 0 0
Poiché q(e1 ) 6= 0 allora possiamo effettuare il seguente cambiamento di base:
− 12 1 1
e01 = e1 , e02 = e2 − e1 = e2 + e1 , e03 = e3 − e1
2 4 2
la cui matrice associata é
1 14 − 12
 

C1 =  0 1 0 
0 0 1
da cui
1 1
x1 = x01 + x02 − x03 , x2 = x02 , x3 = x03 .
4 2
L’espressione della forma quadratica nelle nuove coordinate é
1 1 1
q(x1 , x2 , x3 ) = 2x21 − x22 − x23 + x2 x3
8 2 2
con matrice associata  
2 0 0
A0 =  0 − 18 1 
4 .
1
0 4 − 12
Poiché q(e2 ) 6= 0 allora possiamo effettuare il seguente cambiamento di base:
1
e01 = e1 , e02 = e2 , e03 = e3 − 4
e2 = e3 + 2e2
− 18
la cui matrice associata é  
1 0 0
C2 =  0 1 2 
0 0 1
da cui
x1 = x01 , x2 = x02 + 2x03 , x3 = x03 .
L’espressione della forma quadratica nelle nuove coordinate é
1
q(x1 , x2 , x3 ) = 2x21 − x22
8
con matrice associata  
2 0 0
A00 =  0 − 18 0  .
0 0 0
La base ortogonale B rispetto a cui la forma quadratica si esprime in forma diago-
nale é data dalle colonne della matrice C = C1 × C2 :
1 14 0
 

C= 0 1 2 
0 0 1
per cui B = {(1, 0, 0), ( 14 , 1, 0), (0, 2, 1)} e la matrice associata alla forma quadratica
in tale base é A00 =T CAC.
Per ottenere una forma diagonale in cui compaiano come coefficienti non nulli
solamente +1 e −1 dobbiamo esprimere la forma quadratica rispetto ad una base
LE FORME BILINEARI E LE FORME QUADRATICHE REALI. 15

ortonormale B 0 . Questa si ottiene dalla base ortogonale B, dividendo il vettore che


occupa la colonna i nella matrice C, per la radice quadrata del valore assoluto dello
scalare (se é non nullo) presente nella colonna i della matrice A00 :

(1, 0, 0) ( 14 , 1, 0)
B0 = { √ , q , (0, 2, 1)} =
2 1
8

1 1 √
{( √ , 0, 0), ( √ , 2 2, 0), (0, 2, 1)}.
2 2
Quindi la matrice ortonormale di cambiamento di base
 1
√1

√ 0
2 √ 2
C0 =  0 2 2 2 
0 0 1
ci permette di ottenere la forma diagonale
 
1 0 0
A000 =T C 0 AC 0 =  0 −1 0 .
0 0 0

Esercizio 3. Sia q : lR3 −→ lR la forma quadratica definita da q(X) = 2x1 x2 +


2x1 x3 − 2x23 cioé
  
  0 1 1 x1
q(X) = x1 x2 x3  1 0 0   x2  .
1 0 −2 x3
Determinarne una forma diagonale.

Svolg.
La matrice associata alla forma quadratica é
 
0 1 1
A= 1 0 0 
1 0 −2
Poiché q(e1 ) = 0 allora dobbiamo effettuare il seguente cambiamento di base:
e01 = e1 + e2 , e02 = e2 , e03 = e3
la cui matrice associata é  
1 0 0
C1 =  1 1 0 
0 0 1
da cui
x1 = x01 , x2 = x01 + x02 , x3 = x03 .
L’espressione della forma quadratica nelle nuove coordinate é
q(x1 , x2 , x3 ) = 2x21 + 2x1 x2 + 2x1 x3 − 2x23
con matrice associata  
2 1 1
A0 =  1 0 0 .
1 0 −2
16 LE FORME BILINEARI E LE FORME QUADRATICHE REALI.

Poiché q(e1 ) 6= 0 allora possiamo effettuare il seguente cambiamento di base:


1 1
e01 = e1 , e02 = e2 − e1 , e03 = e3 − e1
2 2
la cui matrice associata é
1 − 12 − 12
 

C2 = 0 1
 0 
0 0 1
da cui
1 1
x1 = x01 − x02 − x03 , x2 = x02 , x3 = x03 .
2 2
L’espressione della forma quadratica nelle nuove coordinate é
1 5
q(x1 , x2 , x3 ) = 2x21 − x22 − x23 − x2 x3
2 2
con matrice associata  
2 0 0
A00 =  0 − 12 − 12  .
0 − 12 − 52
Poiché q(e2 ) 6= 0 allora possiamo effettuare il seguente cambiamento di base:
e01 = e1 , e02 = e2 , e03 = e3 − e2
la cui matrice associata é  
1 0 0
C3 =  0 1 −1 
0 0 1
da cui
x1 = x01 , x2 = x02 − x03 , x3 = x03 .
L’espressione della forma quadratica nelle nuove coordinate é
1
q(x1 , x2 , x3 ) = 2x21 − x22 − 2x23
2
con matrice associata  
2 0 0
A000 =  0 − 12 0 .
0 0 −2
La base ortogonale B rispetto a cui la forma quadratica si esprime in forma
diagonale é data dalle colonne della matrice C = C1 × C2 × C3 :
1 − 12
 
0
1
C= 1 2 −1 
0 0 1
per cui B = {(1, 1, 0), (− 12 , 12 , 0), (0, −1, 1)} e la matrice associata alla forma quadrat-
ica in tale base é A000 =T CAC.
Per ottenere una forma diagonale in cui compaiano come coefficienti non nulli
solamente +1 e −1 dobbiamo esprimere la forma quadratica rispetto ad una base
ortonormale B 0 . Questa si ottiene dalla base ortogonale B, dividendo il vettore che
occupa la colonna i nella matrice C, per la radice quadrata del valore assoluto dello
scalare (se é non nullo) presente nella colonna i della matrice A000 :
LE FORME BILINEARI E LE FORME QUADRATICHE REALI. 17

(1, 1, 0) (− 14 , 12 , 0) (0, −1, 1)


B0 = { √ , q , √ }=
2 1 2
2
1 1 1 1 1 1
{( √ , √ , 0), (− √ , √ , 0), (0, − √ , √ )}.
2 2 2 2 2 2
Quindi la matrice ortonormale di cambiamento di base
 1
− √12


2
0
C 0 =  √12 √1 − √12 

2 
0 0 √1
2
ci permette di ottenere la forma diagonale
 
1 0 0
Aiv =T C 0 AC 0 =  0 −1 0  .
0 0 −1
Esercizio 4. Sia q : lR3 −→ lR la forma quadratica definita da q(X) = x21 + 2x22 +
2x1 x3 + x23 cioé
  
  1 0 1 x1
q(X) = x1 x2 x3  0 2 0   x2  .
1 0 1 x3
Determinarne una forma diagonale.

Svolg. La matrice associata alla forma quadratica é


 
1 0 1
A= 0 2 0 
1 0 1
Poiché q(e1 ) 6= 0 allora possiamo effettuare il seguente cambiamento di base:
e01 = e1 , e02 = e2 , e03 = e3 − e1
la cui matrice associata é  
1 0 −1
C= 0 1 0 
0 0 1
da cui
x1 = x01 − x03 , x2 = x02 , x3 = x03 .
L’espressione della forma quadratica nelle nuove coordinate é
q(x1 , x2 , x3 ) = x21 + 2x22
con matrice associata  
1 0 0
A0 =  0 2 0  .
0 0 0
La base ortogonale B rispetto a cui la forma quadratica si esprime in forma
diagonale é data dalle colonne della matrice C, cioé:
B = {(1, 0, 0), (0, 1, 0), (−1, 0, 1)}
e la matrice associata alla forma quadratica in tale base é A0 =T CAC.
18 LE FORME BILINEARI E LE FORME QUADRATICHE REALI.

Per ottenere una forma diagonale in cui compaiano come coefficienti non nulli
solamente +1 e −1 dobbiamo esprimere la forma quadratica rispetto ad una base
ortonormale B 0 . Questa si ottiene dalla base ortogonale B, dividendo il vettore che
occupa la colonna i nela matrice C, per la radice quadrata del valore assoluto dello
scalare (se é non nullo) presente nella colonna i della matrice A0 :
(0, 1, 0)
B 0 = {(1, 0, 0),
√ , (−1, 0, 1)}.
2
Quindi la matrice ortonormale di cambiamento di base
1 0 −1
 
1
C 0 =  0 √2 0 
0 0 1
ci permette di ottenere la forma diagonale
 
1 0 0
A00 =T C 0 AC 0 =  0 1 0 
0 0 0
che individua una forma quadratica semidefinita positiva di segnatura (2, 0).

Risolviamo adesso lo stesso esercizio con un metodo alternativo, tramite l’utilizzo


degli autovalori della matrice A.
Dopo aver risolto l’equazione caratteristica associata alla matrice A, si otten-
gono come autovalori λ1 = 0 con molteplicitá algebrica 1, λ2 = 2 con molteplicitá
algebrica 2.
L’autospazio relativo a λ1 é generato dall’autovettore (1, 0, −1), il cui versore é
( √12 , 0, − √12 ).
L’autospazio relativo a λ2 é generato dagli autovettori (1, 0, 1), (0, 1, 0), i cui
versori sono rispettivamente ( √12 , 0, √12 ) e (0, 1, 0).
La base rispetto a cui la forma quadratica é diagonalizzabile, con gli autovalori
sulla diagonale principale della matrice ad essa associata, é formata proprio da tali
autoversori:
1 1 1 1
B = {( √ , 0, − √ ), ( √ , 0, √ ), (0, 1, 0)}.
2 2 2 2
La matrice di cambiamento di base é allora
 √1 √1 0

2 2
C= 0 0 1 
− √12 √12 0
tale che  
0 0 0
T
CAC =  0 2 0  .
0 0 2
Al solito, per ottenere una forma diagonale in cui compaiano come coefficienti
non nulli solamente +1 e −1 dobbiamo esprimere la forma quadratica rispetto ad
una base ortonormale B 0 che si ottiene dalla base B, dividendo ogni vettore per la
radice quadrata del valore assoluto dell’autovalore (se é non nullo) ad esso relativo:

1 1 1 1 1
B 0 = {( √ , 0, − √ ), ( , 0, ), (0, √ , 0)}.
2 2 2 2 2
LE FORME BILINEARI E LE FORME QUADRATICHE REALI. 19

La matrice di cambiamento di base é allora


 1 1


2 2 0
C0 = 
 0 0 √1 
2 
− √12 1
2 0
tale che  
0 0 0
T 0
C AC 0 =  0 1 0 .
0 0 1

Esercizio 5. Sia q : lR2 −→ lR la forma quadratica definita da q(X) = 3x21 + 3x22 +


4x1 x2 cioé   
  3 2 x1
q(X) = x1 x2 .
2 3 x2
Determinarne una forma diagonale.

Svolg. La matrice associata alla forma quadratica é


 
3 2
A=
2 3
Poiché q(e1 ) 6= 0 allora possiamo effettuare il seguente cambiamento di base:
2
e01 = e1 , e02 = e2 − e1
3
la cui matrice associata é
1 − 23
 
C=
0 1
da cui
2
x1 = x01 − x02 , x2 = x02 .
3
L’espressione della forma quadratica nelle nuove coordinate é
5
q(x1 , x2 ) = 3x21 + x22
3
con matrice associata  
3 0
A0 = .
0 53
La base ortogonale B rispetto a cui la forma quadratica si esprime in forma
diagonale é data dalle colonne della matrice C, cioé:
2
B = {(1, 0), (− , 1)}
3
e la matrice associata alla forma quadratica in tale base é A0 =T CAC.
Per ottenere una forma diagonale in cui compaiano come coefficienti non nulli
solamente +1 e −1 dobbiamo esprimere la forma quadratica rispetto ad una base
ortonormale B 0 . Questa si ottiene dalla base ortogonale B, dividendo il vettore che
occupa la colonna i nella matrice C, per la radice quadrata del valore assoluto dello
scalare (se é non nullo) presente nella colonna i della matrice A0 :
(1, 0) (− 2 , 1)
B 0 = { √ , q3 }=
3 5
3
20 LE FORME BILINEARI E LE FORME QUADRATICHE REALI.


1 2 3
{( √ , 0), (− √ , √ )}.
3 15 5
Quindi la matrice ortonormale di cambiamento di base
" 1
√2
#
√ −
C0 = 3 √ 15
0 √3
5

ci permette di ottenere la forma diagonale


 
1 0
A00 =T C 0 AC 0 =
0 1
che individua una forma quadratica definita positiva.

Risolviamo adesso lo stesso esercizio tramite l’utilizzo degli autovalori della ma-
trice A.
Dopo aver risolto l’equazione caratteristica associata alla matrice A, si ottengono
come autovalori λ1 = 5 e λ2 = 1, entrambi con molteplicitá algebrica 1.
L’autospazio relativo a λ1 é generato dall’autovettore (1, 1), il cui versore é
( √12 , √12 ).
L’autospazio relativo a λ2 é generato dall’autovettore (1, −1), il cui versore é
( √12 , − √12 ).
La base rispetto a cui la forma quadratica é diagonalizzabile, con gli autovalori
sulla diagonale principale della matrice ad essa associata, é formata proprio da tali
autoversori:
1 1 1 1
B = {( √ , √ ), ( √ , − √ )}.
2 2 2 2
La matrice di cambiamento di base é allora
" #
√1 √1
C= 2 2
√1 − √12
2

tale che  
T 5 0
CAC = .
0 1
Al solito, per ottenere una forma diagonale in cui compaiano come coefficienti
non nulli solamente +1 e −1 dobbiamo esprimere la forma quadratica rispetto ad
una base ortonormale B 0 che si ottiene dalla base B, dividendo ogni vettore per la
radice quadrata del valore assoluto dell’autovalore (se é non nullo) ad esso relativo:

1 1 1 1
B 0 = {( √ , √ ), ( √ , − √ )}.
10 10 2 2
La matrice di cambiamento di base é allora
" #
√1 √1
C0 = 10
√1
2
10
− √12

tale che  
T 0 0 1 0
C AC = .
0 1
LE FORME BILINEARI E LE FORME QUADRATICHE REALI. 21

Esercizio 6. Sia q : lR3 −→ lR la forma quadratica definita da q(X) = x21 + 6x22 +


4x1 x3 + x23 cioé
  
  1 2 0 x1
q(X) = x1 x2 x3  2 6 0   x2  .
0 0 1 x3
Determinarne una forma diagonale.

Svolg. La matrice associata alla forma quadratica é


 
1 2 0
A= 2 6 0 
0 0 1
Poiché q(e1 ) 6= 0 allora possiamo effettuare il seguente cambiamento di base:
e01 = e1 , e02 = e2 − 2e1 , e03 = e3
la cui matrice associata é  
1 −2 0
C= 0 1 0 
0 0 1
da cui
x1 = x01 − 2x02 , x2 = x02 , x3 = x03 .
L’espressione della forma quadratica nelle nuove coordinate é
q(x1 , x2 , x3 ) = x21 + 2x22 + x23
con matrice associata  
1 0 0
A0 =  0 2 0 .
0 0 1
La base ortogonale B rispetto a cui la forma quadratica si esprime in forma
diagonale é data dalle colonne della matrice C, cioé:
B = {(1, 0, 0), (−2, 1, 0), (0, 0, 1)}
e la matrice associata alla forma quadratica in tale base é A0 =T CAC.
Per ottenere una forma diagonale in cui compaiano come coefficienti non nulli
solamente +1 e −1 dobbiamo esprimere la forma quadratica rispetto ad una base
ortonormale B 0 . Questa si ottiene dalla base ortogonale B, dividendo il vettore che
occupa la colonna i nella matrice C per la radice quadrata del valore assoluto dello
scalare (se é non nullo) presente nella colonna i della matrice A0 :
(−2, 1, 0)
B 0 = {(1, 0, 0), √ , (0, 0, 1)}.
2
Quindi la matrice ortonormale di cambiamento di base
1 − √22 0
 

C0 =  0 √1
2
0 
0 0 1
22 LE FORME BILINEARI E LE FORME QUADRATICHE REALI.

ci permette di ottenere la forma diagonale


 
1 0 0
A00 =T C 0 AC 0 =  0 1 0 
0 0 1
che individua una forma quadratica definita positiva di segnatura (3, 0), che in base
B 0 si presenta in forma di prodotto scalare standard.
Esercizio 7. Sia q : lR3 −→ lR la forma quadratica definita da q(X) = 4x1 x2 −
x21 − 5x22 − 4x23 cioé
  
  −1 2 0 x1
q(X) = x1 x2 x3  2 −5 0   x2  .
0 0 −4 x3
Determinarne una forma diagonale.

Svolg. La matrice associata alla forma quadratica é


 
−1 2 0
A =  2 −5 0 
0 0 −4
Poiché q(e1 ) 6= 0 allora possiamo effettuare il seguente cambiamento di base:
e01 = e1 , e02 = e2 + 2e1 , e03 = e3
la cui matrice associata é  
1 2 0
C= 0 1 0 
0 0 1
da cui
x1 = x01 + 2x02 , x2 = x02 , x3 = x03 .
L’espressione della forma quadratica nelle nuove coordinate é
q(x1 , x2 , x3 ) = −x21 − x22 − 4x23
con matrice associata  
−1 0 0
A0 =  0 −1 0  .
0 0 −4
La base ortogonale B rispetto a cui la forma quadratica si esprime in forma
diagonale é data dalle colonne della matrice C, cioé:
B = {(1, 0, 0), (2, 1, 0), (0, 0, 1)}
e la matrice associata alla forma quadratica in tale base é A0 =T CAC.
Per ottenere una forma diagonale in cui compaiano come coefficienti non nulli
solamente +1 e −1 dobbiamo esprimere la forma quadratica rispetto ad una base
ortonormale B 0 . Questa si ottiene dalla base ortogonale B, dividendo il vettore che
occupa la colonna i nella matrice C per la radice quadrata del valore assoluto dello
scalare (se é non nullo) presente nella colonna i della matrice A0 :
(0, 0, 1)
B 0 = {(1, 0, 0), (2, 1, 0), √ }=
4
(0, 0, 1)
{(1, 0, 0), (2, 1, 0), }.
2
LE FORME BILINEARI E LE FORME QUADRATICHE REALI. 23

Quindi la matrice ortonormale di cambiamento di base


 
1 2 0
C0 =  0 1 0 
0 0 12
ci permette di ottenere la forma diagonale
 
−1 0 0
A00 =T C 0 AC 0 =  0 −1 0 
0 0 −1
che individua una forma quadratica definita negativa di segnatura (0, 3).

8. Criteri di positivitá. Ci occupiamo infine di come stabilire se una forma


quadratica (o bilineare simmetrica) reale sia definita positiva, cioé quando essa
é un prodotto scalare.
Ricordiamo (ne faremo un continuo utilizzo) che, poiché le matrici associate alle
forme quadrate sono simmetriche, le basi rispetto a cui esse sono esprimibili in
forma diagonale, sono composte da vettori ortonormali.
Teorema 4. Sia f : lRn × lRn → lR una forma bilineare simmetrica e sia A la
matrice che la rappresenta rispetto ad una base B = {e1 , .., en } di lRn . Allora f é
definita positiva (é un prodotto scalare) se e solo se tutti gli autovalori di A sono
positivi.

Dim. Supponiamo che f sia un prodotto scalare, quindi per ogni 0 6= X ∈ lRn si
ha X t AX > 0. In particolare sia X0 = (a1 , .., an ) ∈ lRn − {0} un autovettore. Ne
segue che, per ogni autovalore λ relativo a x0 ,
λ(a21 + a22 + .... + a2n ) = λX0t X0 = X0t λX0 = X0t AX0 = f (X0 , X0 ) > 0
per cui λ > 0.
Viceversa supponiamo che ogni autovalore λi sia positivo. Essendo A simmetrica,
esiste una base di autovettori ortonormali C = {c1 , ..., cn } di lRn . Sia X ∈ lR − {0},
con X = a1 c1 + ... + an cn , cioé siano (a1 , .., an ) le componenti di X rispetto a C.
Segue che
f (X, X) = (a1 c1 + .... + an cn )t A(a1 c1 + ... + an cn ) =
XX XX
ai aj (cti Acj ) = ai aj cti λj cj =
i j i j
XX X
λj ai aj (cti cj ) = λi a2i > 0.
i j i

Teorema 5. Sia p(x) = an xn + an−1 xn−1 + .... + ar xr un polinomio di grado n a


coefficienti ai reali, con 0 ≤ r ≤ n e ar 6= 0. Supponiamo che p(x) abbia tutte le
radici reali. Allora
1. 0 é radice di p(x) se e solo se r ≥ 1 (in tale caso la molteplicitá dello 0 come
radice del polinomio é esattamente r);
2. p(x) ha tante radici positive quante sono le variazioni di segno nella succes-
sione dei suoi coefficienti non nulli.
Il precedente teorema é noto come criterio di Cartesio. Nel caso degli autovalori,
esso é utile per determinarne il segno, come radici del polinomio caratteristico.
24 LE FORME BILINEARI E LE FORME QUADRATICHE REALI.

Definizione 1. Sia A = Mn (K) una matrice quadrata di ordine n a coefficienti in


un campo K, indichiamo
A = (aij )i=1,..,n j=1,..,n .
Indichiamo con Ak la sottomatrice ottenuta da A prendendo ordinatamente la prime
k righe e k colonne. Ogni Ak é detta minore principale di ordine k. Chiaramente
A1 = (a11 ) e An = A.
Teorema 6. Se A ∈ Mn (lR) allora tutti i suoi autovalori sono positivi se e solo se
ogni sottomatrice Ak ha determinante positivo.
LE FORME BILINEARI E LE FORME QUADRATICHE REALI. 25

9. Esercizi.
Esercizio 8. Sia f : lR2 × lR2 −→ lR la forma bilineare definita da
f ((x1 , x2 ), (y1 , y2 )) = x1 (y1 + y2 ) + x2 (y1 − y2 ).
Determinare la matrice associata rispetto alle basi canoniche.
 
1 3
Esercizio 9. Sia data la matrice . Determinare la forma bilineare f :
5 2
lR × lR −→ lR associata a tale matrice rispetto alle basi canoniche di lR2 .
2 2

Esercizio 10. Sia f : lR3 × lR3 −→ lR la forma bilineare definita da


f ((x1 , x2 , x3 ), (y1 , y2 , y3 )) = x1 y1 − x3 y1 + 2x1 y2 + 2x2 y2 + x1 y3 .
Determinare la matrice associata rispetto alle basi canoniche.
Esercizio 11. Sia f : lR2 × lR3 −→ lR la forma bilineare definita da
f ((x1 , x2 ), (y1 , y2 , y3 )) = x1 (y1 + y2 ) + x2 (y2 + y3 )
rispetto alle basi canoniche. Determinare la matrice associata rispetto alle basi
B2 = {(1, 1), (2, 1)} di lR2 e B3 = {(1, 1, 0), (1, 3, 1), (0, 0, 1)} di lR3 .
 
1 0 1
Esercizio 12. Sia la matrice associata alla forma bilineare f : lR2 ×
1 1 0
lR3 −→ lR rispetto alle basi B2 = {(1, 1), (1, 0)} di lR2 e B3 = {(1, 1, 1), (2, 1, 0), (0, 0, 1)}
di lR3 . Determinare la matrice associata rispetto alle basi canoniche di lR2 e lR3 .
 
1 2
Esercizio 13. Sia la matrice associata alla forma bilineare f : lR2 ×
2 2
lR2 −→ lR rispetto alle basi canoniche di lR2 . Determinare la matrice associata
a f rispetto alla base {(1, 3), (1, 5)} sia nel primo che nel secondo fattore lR2 del
prodotto cartesiano.
Esercizio 14. Sia f : lR3 × lR3 −→ lR la forma bilineare definita da
f ((x1 , x2 , x3 ), (y1 , y2 , y3 )) = x1 y1 − x1 y2 + x1 y3 − x2 y1 + x3 y1 .
Determinare la forma quadratica associata alla f e determinarne una forma diag-
onale.
Esercizio 15. Sia f : lR3 × lR3 −→ lR la forma bilineare definita da
f ((x1 , x2 , x3 ), (y1 , y2 , y3 )) = x1 y1 + x1 y2 + 2x1 y3 + x2 y1 + x2 y3 + 2x3 y1 + x3 y2 .
Determinare la forma quadratica associata alla f e determinarne una forma diag-
onale.
Esercizio 16. Sia q : lR3 −→ lR la forma quadratica definita da
f ((x1 , x2 , x3 )) = 3x21 + 2x1 x2 − 4x2 x3 + 7x22 − x23 .
Determinare la forma bilineare associata alla q e determinarne una forma diago-
nale.
Esercizio 17. Sia q : lR3 −→ lR la forma quadratica definita da
f ((x1 , x2 , x3 )) = 2x21 − 2x1 x2 + x22 + 3x2 x3 + 4x23 .
Determinare la forma bilineare associata alla q e determinarne una forma diago-
nale.
26 LE FORME BILINEARI E LE FORME QUADRATICHE REALI.

Esercizio 18. Sia q : lR3 −→ lR la forma quadratica definita da


f ((x1 , x2 , x3 )) = x1 x2 + x1 x3 + x2 x3 .
Determinare la forma bilineare associata alla q e determinarne una forma diago-
nale.
Esercizio 19. Sia q : lR3 −→ lR la forma quadratica definita da
f ((x1 , x2 , x3 )) = 5x21 + 3x22 + x1 x3 .
Determinare la forma bilineare associata alla q e determinarne una forma diago-
nale.
GEOMETRIA NEL PIANO AFFINE ED EUCLIDEO.

Siano V = lR2 e A lo spazio affine associato a V . Indichiamo con {i, j} una base
di V e con (O, i, j) un riferimento in A. Ogni punto P ∈ A é individuato dalle
coordinate (x, y) rispetto al riferimento dato, tali che OP = xi + yj. Diremo assi
coordinati quelle rette passanti per O, concordi e parallele ai vettori i, j.

1. Equazioni di una retta. Ogni retta del piano puó essere individuata da un
suo punto P0 = (x0 , y0 ) e da un vettore v = (l, m) ad essa parallelo. Se P = (x, y)
é un qualsiasi punto della retta, ne segue che il vettore P0 P sará proporzionale
al vettore v, in quanto paralleli. In altri termini avremo che P0 P = tv, per un
opportuno scalare t, dipendente dalla scelta di P sulla retta. Al variare di t ∈ lR, si
ottiene la proporzionalitá di v con ogni vettore scelto sulla retta, cioé si ottengono
tutti i punti della retta:

x − x0 = tl
y − y0 = tm
note come le equazioni parameriche di una retta e generalmente espresse da:

x = x0 + tl
.
y = y0 + tm
Gli elementi della coppia (l, m) sono detti parametri direttori della retta. In parti-
colare se si conoscono due punti della retta P1 = (x1 , y1 ) e P2 = (x2 , y2 ), il vettore
P1 P2 é parallelo alla retta e quindi (l, m) = (x2 − x1 , y2 − y1 ) e l’equazione si puó
ottenere nel modo seguente:

x = x1 + t(x2 − x1 )
y = y1 + t(y2 − y1 )
da cui
x − x1 y − y1
t= =
x2 − x1 y 2 − y1
x − x1 y − y1
t= =
l m
che é detta equazione a catena di una retta.
Da tali espressioni, eliminando il parametro t, otteniamo
mx − mx0 − ly + ly0 = 0
che possiamo riscrivere
ax + by + c = 0
che é l’equazione lineare (implicita) che rappresenta la retta in coordinate affini.
Diremo che due rette sono parallele se esse hanno i parametri direttori proporzionali
(in particolare identici). Si noti che quando la retta é espressa in forma implicita,
i suoi parametri direttori sono dati dalla coppia (l, m) = (b, −a).

1
2 GEOMETRIA NEL PIANO AFFINE ED EUCLIDEO.

Dalla forma implicita ax + by + c = 0 di una retta, ricaviamo la forma detta


”esplicita”: y = mx + q, per m = − ab e q = − cb .

Abbiamo visto come l’equazione della retta passante per i due punti P1 = (x1 , y1 )
e P2 = (x2 , y2 ) si scriva
x − x1 y − y1
=
x2 − x1 y2 − y1
che equivale alla

(x − x1 )(y2 − y1 ) − (y − y1 )(x2 − x1 ) = 0

cioé

x − x1 y − y1

x2 − x1 = 0.
y2 − y1
Quest’ultima si puó riscrivere anche nel modo seguente:

x y 1

x1 y1 1 = 0.

x2 y2 1

Diremo allora che il punto P3 = (x3 , y3 ) é allineato con i punti P1 e P2 se



x3 y3 1

x1 y1 1 = 0.

x2 y2 1

2. Reciproca posizione di due rette. Siano r : ax + by + c = 0 e r0 : a0 x +


b0 y + c0 = 0 due rette. I punti in comune alle due rette sono le soluzioni del sistema
lineare

ax + by + c = 0
a0 x + b0 y + c0 = 0
nelle incognite x, y. Le matrici associate al sistema sono
   
a b a b −c
A= , C= .
a0 b0 a0 b0 −c0
Se rango(A) = rango(C) = 1, allora le due rette sono coincidenti poiché ax + by +
c = α(a0 x + b0 y + c0 ), per un opportuno α ∈ lR.
Se rango(A) = rango(C) = 2, allora il sistema ammette una sola soluzione cioé
le due rette sono incidenti.
Se rango(A) = 1 e rango(C) = 2, allora il sistema é incompatibile e le due rette
non hanno punti in comune, cioé sono parallele. Ció si verifica quando
a b c
= 0 6= 0 .
a0 b c
0
Allora possiamo dire che le due rette sono parallele quando ab = ab0 . Il rapporto − ab
é detto coefficiente direttore (o angolare) della retta, quindi due rette sono parallele
se hanno lo stesso coefficiente direttore.
GEOMETRIA NEL PIANO AFFINE ED EUCLIDEO. 3

3. Fascio di rette. Siano r : ax + by + c = 0 e r0 : a0 x + b0 y + c0 = 0 due rette


distinte. La totalitá delle rette di equazione
λ(ax + by + c) + %(a0 x + b0 y + c0 ) = 0
al variare dei parametri reali λ e %, é detta fascio di rette. Si possono verificare due
casi: r e r0 sono incidenti in un punto, ed allora tutte le rette del fascio hanno in
comune quel punto, si parla di fascio proprio. Oppure r e r0 sono tra loro parallele,
ed allora tutte le rette del fascio sono tra loro parallele, si parla di fascio improprio.
Supponiamo allora di avere una terza retta r00 : a00 x+b00 y +c00 = 0 ed analizziamo
in quale casi essa appartiene al fascio individuato da r e r0 . In pratica si deve
studiare il sistema

 ax + by + c = 0
a0 x + b0 y + c0 = 0
 00
a x + b00 y + c00 = 0
nelle incognite x, y. Le matrici associate al sistema sono
   
a b a b c
A =  a0 b0  , C =  a0 b0 c0  .
00 00
a b a00 b00 c00
Se rango(A) = rango(C) = 2, allora il sistema ammette una sola soluzione, cioé
le tre rette hanno un punto in comune: esse appartengono ad un fascio proprio.
Se rango(A) = 1 e rango(C) = 2, allora il sistema é incompatibile ed le tre rette
sono parallele: esse appartengono ad un fascio improprio.
Possiamo concludere allora che la condizione necessaria e sufficiente affinché le tre
rette appartengano allo stesso fascio (proprio o improprio che sia) é che la matrice
 
a b c
 a0 b0 c0 
a00 b00 c00
abbia rango ≤ 2.
4 GEOMETRIA NEL PIANO AFFINE ED EUCLIDEO.

4. Angoli e distanze nel piano euclideo. Fissiamo nello spazio euclideo un


riferimento cartesiano ortogonale OXY di centro O e versori i, j, rispettivamente
per gli assi X, Y .
Chiameremo coseni direttori di una retta r, i coseni degli angoli che la retta forma
con gli assi coordinati. Se la retta é individuata dai parametri direttori (l, m), i
suoi coseni direttori saranno:
l l
α = cos(r, X) ∈ {+ √ , −√ }
l2 + m2 l2 + m2
m m
β = cos(r, Y ) ∈ {+ √ , −√ }
2
l +m 2 l + m2
2

Consideriamo ora due rette ed individuiamole tramite i rispettivi parametri di-


rettori: r = (l, m) e r0 = (l0 , m0 ). Indichiamo con v e v 0 due vettori paralleli
rispettivamente a r e r0 , uno di componenti (l, m) e l’altro (l0 , m0 ). L’angolo tra le
due rette é lo stesso formato dai due vettori:
ll0 + mm0 ll0 + mm0
cos(r, r0 ) = cos(v, v 0 ) ∈ {+ √ √ , −√ √ }.
l2 + m2 · l02 + m02 l2 + m2 · l02 + m02
Quindi le due rette sono ortogonali se ll0 + mm0 = 0.

Siano P1 = (x1 , y1 ) e P2 = (x2 , y2 ) due punti del piano. La distanza tra i punti
P1 e P2 é il modulo del vettore P1 P2 :
p
δ(P1 , P2 ) = (x2 − x1 )2 + (y2 − y1 )2 .

Consideriamo ora il punto P0 = (x0 , y0 ) e la retta r : ax + by + c = 0. Ci


proponiamo di calcolare la distanza del punto dalla retta (disegno 9). Se P0 ∈ r
chiaramente diremo che essa é nulla. poniamoci allora nel caso in cui P0 ∈/ r. La
distanza di P1 da r é pari alla lunghezza δ(P0 H) del segmento P0 H, dove H é la
proiezione ortogonale di P0 su r. Scegliamo un qualsiasi punto Q1 = (x1 , y1 ) ∈ r.
Allora δ(P0 H) é la proiezione ortogonale del vettore P0 Q1 lungo la direzione del
vettore P0 H. Per determinare tale proiezione abbiamo bisogno del versore u di
P0 H, che é il versore normale alla retta r:
a b
u = (√ ,√ ).
a2+b 2 a + b2
2

Quindi:
a b
δ(P0 H) = |(P0 Q1 ) × ( √ i+ √ j)| =
a2 + b2 a2 + b2
a b
|((x1 − x0 )i + (y1 − y0 )j) × ( √ i+ √ j)| =
a2+b 2 a + b2
2

|ax1 − ax0 + by1 − by0 |


√ .
a2 + b2
Dal fatto che Q1 ∈ r segue che ax1 + by1 = −c, per cui
|ax0 + by0 + c|
δ(P0 H) = √ .
a2 + b2
GEOMETRIA NEL PIANO AFFINE ED EUCLIDEO. 5

5. Simmetrie. Due punti P1 = (x1 , y1 ) e P2 = (x2 , y2 ) sono simmetrici rispetto


al punto Q = (a, b), se Q é il punto medio del segmento P1 P2 , cioé se
x1 + x2 y1 + y2
a= , b= .
2 2
Quindi, dato un punto P1 = (x1 , y1 ), per determinare le coordinate (x2 , y2 ) del suo
simmetrico P2 rispetto al punto Q = (a, b), é sufficiente calcolare
x2 = 2a − x1 , y2 = 2b − y1 .

Consideriamo ora il punto P1 = (x1 , y1 ) e la retta r : y = mx + q. Consideriamo


una qualsiasi retta del fascio di centro P1 :
y − y1 = k(x − x1 )
al variare del parametro reale k otteniamo tutte le rette passanti per P1 . Fissiamo
una retta r0 di tale fascio e sia Q = r ∩ r0 . Il punto P2 , simmetrico di P1 rispetto a
Q, é detto il simmetrico di P1 rispetto alla retta r, lungo la direzione individuata
dalla retta r0 . Quindi uno stesso punto puó avere infiniti simmetrici rispetto ed una
retta che non lo contenga.
Esempio 1. Siano P = (1, 1) e r : x − y = 4. Determiniamo il simmetrico di P
rispetto a r lungo la direzione di coefficiente k = 3.

Svolg. La retta r0 del fascio di centro P e coefficiente angolare 3 é


r0 : y − 1 = 3(x − 1) → y − 3x + 2 = 0.
Il punto Q = r ∩ r0 é dato dalle soluzioni del sistema

y − 3x + 2 = 0
x−y−4=0
dal quale otteniamo Q = (−1, −5). Il simmetrico di P rispetto a Q é

0 x = −2 − 1 = −3
P = (x, y) → → P 0 = (−3, −11).
y = −10 − 1 = −11

Concludiamo con la definizione di asse di un segmento AB: esso é la retta


perpendicolare ad AB e passante per il suo punto medio:
Esempio 2. Siano A = (1, −1) e B = (2, 3). Calcoliamo l’asse del segmento AB.

Svolg. Il vettore AB ha componenti (−1, −4), quindi una retta ad esso ortogonale
ha come parametri direttori, la coppia (b, −a) = (−4, 1).
Il punto medio di AB é Q = ( 23 , 1). Quindi l’asse del segmento AB ha equazione:
1 3
y − 1 = − (x − )
4 2
cioé
2x + 8y + 11 = 0.
6 GEOMETRIA NEL PIANO AFFINE ED EUCLIDEO.

6. Coordinate omogenee nel piano. Sia P = (x, y) un punto del piano. Diremo
che (x1 , x2 , x3 ) sono le coordinate omogenee di P se xx13 = x e xx23 = y.
Nel caso x3 6= 0 le precedenti scritture hanno evidentemente un senso, e diremo
che il punto é proprio. Ogni punto proprio (x, y) puó banalmente essere individ-
uato da una terna di coordinate omogenee (x, y, 1). Inoltre due terne tra loro
proporzionali individuano lo stesso punto nel piano.
Nel caso il punto P sia individuato dalla terna (x, y, 0) esso verrá detto improprio.
L’insieme dei punti impropri del piano forma una retta detta ”retta impropria”, la
cui equazione é x3 = 0.

Consideriamo ora la retta r : ax + by + c = 0, nel passaggio alle coordinate


omogenee, la sua equazione diventa
x1 x2
a +b +c=0
x3 x3
cioé
ax1 + bx2 + cx3 = 0.
Tale retta avrá uno ed un solo punto di intersezione con la retta impropria, e sará
detto il punto improprio di r.
É noto che tutti e soli i punti di r sono quelli le cui coordinate soddisfino
l’equazione ax1 + bx2 + cx3 = 0. Tra questi punti c’é anche il punto improprio
di coordinate omogenee (b, −a, 0). Ma similmente ogni punto improprio del tipo
(αb, −αa, 0) soddisfa la precedente equazione, quindi un punto improprio di una
retta é unico a meno di un fattore di proporzionalitá. In generale, direttamente
dalla definizione di coordinate omogenee, é di facile verifica che due qualsiasi terne
(x, y, 0) e (z, t, 0) tra loro proporzionali, individuino il medesimo punto improprio.
In particolare il punto improprio (b, −a, 0) di una retta r non parallela all’asse
delle Y (cioé con b 6= 0) si puó riscrivere come (1, − ab , 0) (dividendo la terna per b).
Mentre il punto improprio di una retta parallela all’asse Y é (0, 1, 0).
Consideriamo ora l’equazione in coordinate omogenee di due rette tra loro parallele:
r : ax1 + bx2 + cx3 = 0 s : ax1 + bx2 + c0 x3 = 0.
Certamente esse non hanno punti propri in comune. Osserviamo cosa accade alla
loro intersezione: il sistema formato dalle loro equazioni

ax1 + bx2 + cx3 = 0
ax1 + bx2 + c0 x3 = 0
é omogeneo, con 3 incognite ma di rango 2. Una soluzione é quindi una qualsiasi
terna proporzionale ai minori della matrice associata, cioé la seguente:
     
b c a c a b
α ,− , =
b c0 a c0 a b
∼ (l, m, 0)
α (b(c0 − c), −a(c0 − c), 0) = α(c0 − c) (b, −a, 0) =
dove (l, m, 0) indicano al solito i parametri direttori che le rette (in quanto parallele)
hanno in comune. Concludiamo allora che rette parallele hanno lo stesso punto
improprio.
GEOMETRIA NEL PIANO AFFINE ED EUCLIDEO. 7

7. La circonferenza. Sia C = (α, β) un punto del piano e sia r un numero reale


positivo. Diciamo circonferenza di centro C e raggio r, il luogo dei punti del piano
la cui distanza da C é pari a r. Sia P = (x, y) un punto qualsiasi della circonferenza:
p
δ(P, C) = (x − α)2 + (y − β)2 = r
da cui otteniamo (quadrando e riordinando):
(x − α)2 + (y − β)2 = r2

x2 + y 2 + ax + by + c = 0.
In quest’ultima equazione compaiono i coefficienti legati alle coordinate del centro
e alla lunghezza del raggio r:
a b 1p 2
α=− , β=− , r= a + b2 − 4c
2 2 2
Esempio 3. Determinare la circonferenza di centro C = (1, 1) e raggio r = 3.

Svolg. Applicando la definizione, l’equazione é


(x − 1)2 + (y − 1)2 = 9

x2 + y 2 − 2x − 2y − 7 = 0
Esempio 4. Determinare centro e raggio della circonferenza x2 +y 2 −4x−2y +2 =
0.

Svolg. Il centro C ha coordinate α = − −4


2 , β = − −3
2 , da cui C = (2, 1). Il
1
√ 1
√ √
raggio é r = 2 16 + 4 − 8 = 2 12 = 3.

Osservazione 1. La circonferenza é l’unica curva di secondo grado nel piano che


contenga i due seguenti punti impropri: (1, i, 0) e (1, −i, 0) detti punti ciclici.

Circonferenza per tre punti. Consideriamo tre punti P1 = (x1 , y1 ), P2 =


(x2 , y2 ) e P3 = (x3 , y3 ) non allineati cioé

x1 y1 1

x2 y2 1 6= 0.

x3 y3 1

Per ottenere la circonferenza passante per i tre punti dobbiamo sostituire le co-
ordinate dei punti alla generica equazione di una circonferenza. In tale caso la
circonferenza contenente i tre punti é unica. Infatti il sistema lineare che dobbiamo
risolvere  2
 x1 + y12 + ax1 + by1 + c = 0
x2 + y22 + ax2 + by2 + c = 0
 22
x3 + y32 + ax3 + by3 + c = 0
nelle incognite (a, b, c) ha rango 3.
Esempio 5. Determinare la circonferenza contenente i punti (1, 2), (1, 8), (5, 0).
8 GEOMETRIA NEL PIANO AFFINE ED EUCLIDEO.

Svolg. Il sistema lineare da risolvere é



 1 + 4 + a + 2b + c = 0
1 + 64 + a + 8b + c = 0
25 + 5a + c = 0


 a + 2b + c = −5
a + 8b + c = −65
5a + c = −25

le cui soluzioni sono a = −10, b = −10, c = 25 e l’equazione della circonferenza é
x2 + y 2 − 10x − 10y + 25 = 0.

Esiste un altro metodo per determinare l’unica circonferenza passante per i tre
punti e si basa sul fatto che l’asse di un segmento che ha per estremi due punti di
un circonferenza, certamente contiene il centro della circonferenza. Lo esponiamo
riproponendo l’esempio precedente:
Esempio 6. Determinare la circonferenza contenente i punti A = (1, 2), B = (1, 8),
C = (5, 0).

Svolg. Consideriamo l’asse del segmento AB, che ha equazione y − 5 = 0 e passa


per il centro della circonferenza.
Calcoliamo ora l’asse del segmento AC, esso é y − 2x + 5 = 0. Quindi il centro
della circonferenza é il punto comune a tali due rette:

y−5=0
y − 2x + 5 = 0
la cui soluzione é O = (5, 5).
Il raggio della circonferenza é pari alla distanza del centro da uno qualsiasi dei
tre punti noti, per esempio:

r = δ(OC) = 25 = 5
per cui l’equazione della circonferenza é
(x − 5)2 + (y − 5)2 = 25
x2 + y 2 − 10x − 10y + 25 = 0.
GEOMETRIA NEL PIANO AFFINE ED EUCLIDEO. 9

8. Esercizi.
Esercizio 1. Nel piano euclideo determinare il punto simmetrico di P (4, −3) rispetto
al punto Q(1, −1).
Esercizio 2. Nel piano euclideo determinare il punto simmetrico di P (1, 1) rispetto
alla retta r : x − 2y + 4 = 0 lungo la direzione ortogonale a r.
Esercizio 3. Nel piano euclideo determinare il punto simmetrico di P (1, 3) rispetto
alla retta r : x − y + 1 = 0 lungo la direzione (2, 1, 0) (cioé lungo la retta di
parametri direttori (2, 1) e passante per P ).
Esercizio 4. Siano A(2, −1) e B(3, 2) punti del piano euclideo. Determinare l’asse
del segmento AB.
Esercizio 5. Determinare la circonferenza passante per i punti (0, 2), (1, 1), (2, −1).
Esercizio 6. Determinare il centro ed il raggio della circonferenza x2 + y 2 − 3x +
2y − 1 = 0.
Esercizio 7. Siano r : 2x − y + 3 = 0 e s : x = 2t, y = t + 2 due rette
nel piano. Determinare i coseni direttori di entrambe le rette ed inoltre il coseno
dell’angolo tra di esse compreso.
Esercizio 8. Siano r : x = 3t − 1, y = 2t + 5 l’equazione in forma parametrica
di una retta nel piano e P (1, 1) un punto esterno ad essa. Determinare:
1. L’equazione in forma implicita (equazione affine) della retta.
2. La distanza tra il punto P e la retta r. La retta passante per P ed ortogonale
a r.
3. La retta passante per P e parallela a r.
Figura 9
P0

H Q1

u = versore perpendicolare alla retta


P0H = proiezione di [P0 Q1] lungo la direzione di u
TRASFORMAZIONI NEL PIANO EUCLIDEO.

1. Traslazioni. In un riferimento cartesiano OXY del piano euclideo, siano (x, y)


le coordinate del punto P . Supponiamo di considerare un secondo riferimento
O0 X 0 Y 0 in cui gli assi X 0 , Y 0 siano paralleli rispettivamente a X e Y ed il punto O0
abbia coordinate (a, b) rispetto a OXY .
Denotiamo (x0 , y 0 ) le coordinate di P rispetto al riferimento O0 X 0 Y 0 . La relazione
che intercorre tra le due coppie di coordinate di P é la seguente:
 0
x =x−a
y0 = y − b

e le formule inverse sono


x = x0 + a

.
y = y0 + b

Esercizio 1. Sia P (−1, 3) nel sistema di riferimento cartesiano OXY , O0 (1, 2)


e X 0 , Y 0 assi paralleli rispettivamente a X e Y e passanti per O0 . Sia inoltre r :
3x + y − 1 = 0 rispetto a OXY . Determiniamo le coordinate di P e l’equazione di
r rispetto al riferimento O0 X 0 Y 0 .

Svolg. Le equazioni di traslazione sono


 0
x =x−a
y0 = y − b

dove (a, b) sono le coordinate del nuovo centro del riferimento, ed applicandole nel
nostro caso otteniamo le coordinate di P 0
 0
x = −1 − 1 = −2
.
y0 = 3 − 2 = 1

Le formule inverse sono


x = x0 + a

y = y0 + b
da cui otteniamo l’equazione della retta

r: 3(x0 + 1) + (y 0 + 2) − 1 = 0

3x0 + y 0 + 4 = 0.

1
2 TRASFORMAZIONI NEL PIANO EUCLIDEO.

2. Rotazioni. Siano (x, y) le coordinate del punto P nel riferimento OXY . Con-
sideriamo ora un secondo riferimento OX 0 Y 0 in cui gli assi X 0 e Y 0 siano ruotati in
senso antiorario di un angolo φ rispetto agli assi X e Y . Per determinare le coordi-
nate (x0 , y 0 ) di P nel secondo riferimento, ricordiamo che esse sono le componenti
del vettore OP . Esprimiamo tali componenti rispetto alle due coppie di versori, i, j
in OXY , i0 , j 0 in OX 0 Y 0 :
OP = x0 i0 + y 0 j 0 OP = xi + yj.
Quindi possiamo richiamare quanto detto in relazione al cambiamento di base in
uno spazio vettoriale:
   0 
x x
=A·
y y0
dove A é la matrice di cambiamento di base. Nel nostro caso abbiamo che
 
cos(φ) −sen(φ)
A=
sen(φ) cos(φ)
cioé      0 
x cos(φ) −sen(φ) x
= ·
y sen(φ) cos(φ) y0
e le formule inverse sono
 0     
x cos(φ) sen(φ) x
= ·
y0 −sen(φ) cos(φ) y
da cui
x0 = xcos(φ) + ysen(φ)

.
y 0 = −xsen(φ) + ycos(φ)
Si noti che le due matrici usate per il cambiamento di base sono l’una la trasposta
dell’altra, ma anche l’una l’inversa dell’altra, infatti sono entrambe matrici ortogo-
nali.
Esercizio 2. Siano P (1, −1) in OXY e X 0 , Y 0 assi passanti per O e ruotati di π4
in senso antiorario rispetto a X, Y . Sia inoltre r : 2x + 3y − 2 = 0 rispetto al
riferimento OXY . Determiniamo le coordinate di P e l’equazione di r rispetto a
OX 0 Y 0 .

Svolg. Le equazioni di rotazione sono


 0     
x cos(φ) sen(φ) x
= ·
y0 −sen(φ) cos(φ) y
da cui
x0 = xcos(φ) + ysen(φ)

.
y 0 = −xsen(φ) + ycos(φ)
Le formule inverse sono
     0 
x cos(φ) −sen(φ) x
= ·
y sen(φ) cos(φ) y0
da cui
x = x0 cos(φ) − y 0 sen(φ)

.
y = x0 sen(φ) + y 0 cos(φ)
TRASFORMAZIONI NEL PIANO EUCLIDEO. 3

Nel nostro caso le coordinate di P 0 sono


( √ √
x0√= 22 (x + y) =√ 22 (1 − 1) = 0

y 0 = 22 (−x + y) = 22 (−1 − 1) = − 2
e l’equazione della retta é
√ √
2 0 2 0
r: 2 (x − y 0 ) + 3 (x + y 0 ) − 2 = 0
2 2
√ √
r : 5 2x0 + 2y 0 − 4 = 0.

3. Rototraslazioni. Siano (x, y) le coordinate di P in OXY e consideriamo un


riferimento O00 X 00 Y 00 in cui O00 abbia coordinate (a, b) rispetto a OXY e tale che
gli assi X 00 e Y 00 siano ruotati in senso antiorario di un angolo φ rispetto a X e Y .
Determiniamo le coordinate (x00 , y 00 ) di P nel secondo riferimento.
Effettuiamo prima una traslazione
OXY −→ O00 X 0 Y 0
 0
x =x−a
.
y0 = y − b
Successivamente operiamo con una rotazione
O00 X 0 Y 0 −→ O00 X 00 Y 00

x00
     
cos(φ) sen(φ) x−a
= ·
y 00 −sen(φ) cos(φ) y−b
ed inversamente abbiamo che:
     00 
x−a cos(φ) −sen(φ) x
= ·
y−b sen(φ) cos(φ) y 00
cioé
     00   
x cos(φ) −sen(φ) x a
= · + .
y sen(φ) cos(φ) y 00 b
Esercizio 3. Siano P (3, −1) e O0 (1, 2) in OXY e X 0 , Y 0 assi passanti per O0 e
ruotati di π4 in senso antiorario rispetto a X, Y . Sia inoltre r : 2x + y − 3 = 0
rispetto al riferimento OXY . Determiniamo le coordinate di P e l’equazione di r
rispetto a O0 X 0 Y 0 .

Svolg. Le equazioni di rototraslazione sono


 0     
x cos(φ) sen(φ) x−a
= ·
y0 −sen(φ) cos(φ) y−b
da cui
x0 = (x − a)cos(φ) + (y − b)sen(φ)

.
y 0 = −(x − a)sen(φ) + (y − b)cos(φ)
Le formule inverse sono
     0   
x cos(φ) −sen(φ) x a
= · +
y sen(φ) cos(φ) y0 b
4 TRASFORMAZIONI NEL PIANO EUCLIDEO.

da cui
x = x0 cos(φ) − y 0 sen(φ) + a

.
y = x0 sen(φ) + y 0 cos(φ) + b
Nel nostro caso le coordinate di P 0 sono
( √ √ √
x0 =
√ 2
2
(x − a + y − b) = √22 (2 − 3) = − 22√
y 0 = 22 (−x + a + y − b) = 22 (−2 − 3) = −5 22
e l’equazione della retta é
√ √
2 0 0 2 0
r : 2( (x − y ) + 1) + ( (x + y 0 ) + 2) − 3 = 0
2 2
√ √
r : 3 2x0 − 2y 0 + 2 = 0.

4. Esercizi.
Esercizio 4. Siano OXY un sistema di riferimento ortogonale nel piano euclideo,
P un punto di coordinate (−1, 3) rispetto a OXY e r una retta di equazione 2x −
3y+1 = 0 in OXY . Determinare le coordinate di P e l’equazione di r in un secondo
sistema di riferimento O0 X 0 Y 0 nei seguenti casi:
1. O0 ha coordinate (1, 2) rispetto a OXY e gli assi X 0 , Y 0 sono paralleli agli
assi X, Y (traslazione).
2. O0 = O e gli assi X 0 , Y 0 sono ruotati di un angolo α = π3 in senso antiorario
rispetto a X, Y (rotazione).
3. O0 ha coordinate (1, −1) rispetto a OXY e gli assi X 0 , Y 0 sono ruotati di un
angolo α = π6 in senso antiorario rispetto a X, Y (rototraslazione).
Esercizio 5. Siano OXY un sistema di riferimento ortogonale nel piano euclideo e
γ : x2 +y 2 −3x+y −1 = 0 una circonferenza la cui equazione é espressa rispetto a
OXY . Determinare l’equazione di γ in un secondo sistema di riferimento O0 X 0 Y 0
nel caso i cui O0 abbia coordinate (2, 1) rispetto a OXY e gli assi X 0 , Y 0 siano
ruotati di un angolo α = π4 in senso antiorario rispetto a X, Y .
CURVE ALGEBRICHE PIANE E PUNTI MULTIPLI.

1. Intersezione di due curve. Due polinomi f (x, y) e g(x, y) nelle variabili x, y a


coefficienti reali, sono detti proporzionali se esiste a ∈ lR tale che f (x, y) = ag(x, y).
Tale proporzionalitá é una relazione di equivalenza. Una curva algebrica γ é una
classe di equivalenza di polinomi cioé se f (x, y) é un polinomio rappresentante della
classe, allora l’equazione f (x, y) = 0 é rappresentativa della curva γ.
Il grado di una curva é il grado del polinomio che la rappresenta.
Teorema 1. Siano γ : f (x, y) = 0 e δ : g(x, y) = 0 due curve algebriche rispettiva-
mente di gradi n e m. Allora esse hanno n · m punti in comune eccetto il caso in
cui hanno infiniti punti in comune.

Come caso particolare consideriamo quello in cui γ : f (x, y) = 0 sia una curva
di grado n e δ : g(x, y) = 0 sia una curva di grado 1, cioé una retta. Allora
γ e δ hanno n punti in comune eccetto quando δ sia una componente di γ cioé
f (x, y) = g(x, y) · h(x, y), dove h(x, y) é un polinomio di grado n − 1.
3
Per esempio
√ γ : x√ − 3xy 2 = 0 e δ : y − 1 = 0 hanno in comune i seguenti 3 punti
: (0, 1), ( 3, 1), (− 3, 1).
Al contrario γ : x3 + x2 y − xy + x − y 2 + y = 0 e δ : x + y = 0 hanno in comune
infiniti punti, cioé tutti quelli di δ, quindi la retta δ é una componente di γ, infatti
x3 + x2 y − xy + x − y 2 + y = (x + y) · (x2 − y + 1).

2. Molteplicitá di un punto. Sia γ : f (x, y) = 0 una curva algebrica di grado n


e sia P0 (x0 , y0 ) un punto di γ.
Consideriamo una generica retta r : y −y0 = m(x−x0 ) passante per P0 . La retta
r e la curva γ hanno in comune n punti, non necessariamente tutti distinti, almeno
uno dei quali é proprio P0 . Diciamo che r e γ hanno molteplicitá di intersezione M
nel punto P0 se (x0 , y0 ) é una soluzione di molteplicitá M per il sistema

f (x, y) = 0
.
y − y0 = m(x − x0 )

Esempio 1. Sia γ : x3 − y 2 = 0 e siano P0 = (0, 0) e r : x − y = 0. Determiniamo


la molteplicitá di intersezione tra γ e r nel punto P0 .

Svolg. Il sistema
x3 − y 2 = 0

x−y =0
ha le tre soluzioni (1, 1) con molteplicitá 1, (0, 0) con molteplicitá 2. Quindi nel
punto P0 le due curve hanno molteplicitá di intersezione M = 2 (e nel punto (1, 1)
hanno molteplicitá di intersezione 1).

1
2 CURVE ALGEBRICHE PIANE E PUNTI MULTIPLI.

Consideriamo ora una generica retta ri del fascio di centro P0 . Ciascuna delle
rette ri ha una molteplicitá di intersezione Mi con γ in P0 . Diremo molteplicitá di
P0 per la curva γ, il minimo di tali Mi .
Il punto P0 é detto semplice se min{Mi } = 1, é detto punto multiplo se min{mi } ≥
2, in particolare é detto doppio se min{Mi } = 2, triplo se min{Mi } = 3, quadruplo
se min{Mi } = 4, etc. etc.

Esercizio 1. Sia γ : x3 − x2 + y 2 = 0 una cubica in OXY . Determiniamo la


molteplicitá di P (0, 0) ∈ γ.

Svolg. Sia r la generica retta passante per P : y = mx.


Per ottenere la molteplicitá di P dobbiamo intersecare r con γ.
 3
x − x2 + y 2 = 0
 3
x − x2 + m2 x2 = 0

y = mx y = mx
in cui la soluzione (0, 0) é doppia, quindi P é un punto doppio per γ.
Esercizio 2. Sia γ : (x2 + y 2 )2 + 3x2 y − y 3 = 0 una quartica in OXY . Deter-
miniamo la molteplicitá di P (0, 0) ∈ γ.

Svolg. Sia r la generica retta passante per P : y = mx.


Per ottenere la molteplicitá di P dobbiamo intersecare r con γ.
 2
(x + y 2 )2 + 3x2 y − y 3 = 0
 2
(x + m2 x2 )2 + 3mx3 − m3 x3 = 0
→ →
y = mx y = mx
 3
x (x + m4 x + 2m2 x + 3m − m3 ) = 0
y = mx
in cui la soluzione (0, 0) é tripla, quindi P é un punto triplo per γ.
Esercizio 3. Sia la curva di sesto grado γ : (x2 + y 2 )3 − 4x2 y 2 = 0 in OXY .
Determiniamo la molteplicitá di P (0, 0) ∈ γ.

Svolg. Sia r la generica retta passante per P : y = mx.


Per ottenere la molteplicitá di P dobbiamo intersecare r con γ.
 2
(x + y 2 )3 − 4x2 y 2 = 0
 2
(x + m2 x2 )3 − 4m2 x4 = 0
→ →
y = mx y = mx
 4 2
x (x + m6 x2 + 3m2 x2 + 3m4 x2 − 4m2 ) = 0
y = mx
in cui la soluzione (0, 0) é quadrupla, quindi P é un punto quadruplo per γ.

3. Calcolo dei punti doppi di una curva. Per terminare questo capitolo, esponi-
amo a brevi linee (e certamente non nella sua forma completa) il metodo general-
mente utilizzato per una prima analisi degli eventuali punti doppi di una curva
algebrica. Esso prevede la conoscenza da parte degli studenti della definizione e del
metodo di calcolo delle derivate parziali di funzioni a due variabili, ed é qui espresso
solo per un senso di completezza.
Sia P0 = (x0 , y0 ) un punto della curva rappresentata dal polinomio f (x, y), per cui
CURVE ALGEBRICHE PIANE E PUNTI MULTIPLI. 3

avremo che f (x0 , y0 ) = 0. Diremo che P0 é un punto doppio della curva se la coppia
di coordinate (x0 , y0 ) é una soluzione del sistema formato dalle equazioni:

 f (x, y) = 0
f 0 (x, y) = 0
 x0
fy (x, y) = 0
00 00 00
ed almeno una delle derivate seconde fxx (x, y), fyy (x, y), fxy (x, y) non si annulla,
quando é calcolata in (x0 , y0 ).
Esempio 2. Sia γ : 2x4 − 3x2 y + y 2 − 2y 3 + y 4 = 0. Determiniamo gli eventuali
punti doppi.

Svolg. Risolviamo dapprima il sistema



 f (x, y) = 2x4 − 3x2 y + y 2 − 2y 3 + y 4 = 0
fx0 (x, y) = 8x3 − 6xy = 0
0
fy (x, y) = −3x2 + 2y − 6y 2 + 4y 3 = 0

Esso é soddisfatto dalla coppia (0, 0). Quindi il punto P0 = (0, 0) é il candidato ad
00
essere un punto doppio per la curva. In effetti la derivata seconda fyy = 2 − 12y +
2
12y non si annulla in P0 , ed esso é un punto doppio.
LE CONICHE.

1. Definizione. Una curva algebrica γ : f (x, y) = 0 di secondo grado é detta


conica, quindi si puó rappresentare tramite l’equazione:
f (x, y) = ax2 + bxy + cy 2 + dx + ey + f = 0
dove a, b, c, d, e, f ∈ lR. In coordinate omogenee la curva é rappresentata da un
polinomio omogeneo e l’equazione diventa:
f (x1 , x2 , x3 ) = a11 x21 + 2a12 x1 x2 + a22 x22 + 2a13 x1 x3 + 2a23 x2 x3 + a33 x23 = 0
dove aij ∈ lR, per ogni i e j.
Si noti che anche il prodotto di due polinomi di primo grado determina un
polinomio rappresentante di una conica. In tale caso la conica é l’unione delle due
rette rappresentate dai due polinomi di primo grado.
Consideriamo ora la matrice simmetrica di ordine 3, formata con i coefficienti
dell’equazione della conica in coordinate omogenee:
 
a11 a12 a13
A =  a12 a22 a23 
a13 a23 a33
 
x1
ed indichiamo X =  x2  il vettore delle coordinate omogenee, allora l’equazione
x3
della conica puó essere riscritta in forma compatta:
f (x1 , x2 , x3 ) = X T · A · X = 0

ed il polinomio f (x1 , x2 , x3 ) é una forma quadratica in lR3 , di matrice associata A.

Esempio 1. L’equazione x2 + 2y 2 − xy + x − 1 = 0 rappresenta una conica nel


piano.

Svolg. La matrice associata alla conica é


1 − 21 1
 
2
A = − 12
 2 0 .
1
2 0 −1

Diremo che una conica γ : f (x, y) = 0 é riducibile se essa é composta da due


rette cioé se esistono due polinomi di primo grado g(x, y) e h(x, y) tali che f (x, y) =
g(x, y) · h(x, y).

1
2 LE CONICHE.

2. Intersezione di una conica ed una retta. Siano γ : X t AX = 0 l’equazione


di una conica e r una retta passante per i punti P = Y e P 0 = Y 0 . Un generico
punto della retta é allora rappresentabile tramite il suo vettore di coordinate X,
come combinazione lineare dei vettori di coordinate Y e Y 0 : X = λY +µY 0 é quindi
l’equazione della retta. Se vogliamo determinare l’intersezione tra γ e r, sostituiamo
X nell’equazione della conica:

(λY + µY 0 )t A(λY + µY 0 ) = 0

da cui
µλ(Y 0t AY ) + µ2 (Y 0t AY 0 ) + λ2 (Y t AY ) + λµ(Y t AY 0 ) = 0.
Osserviamo che, poiché Y 0t AY é un numero reale, Y 0t AY = (Y 0t AY )t = Y t AY 0 ,
quindi segue che

λ2 (Y t AY ) + 2µλ(Y t AY 0 ) + µ2 (Y 0t AY 0 ) = 0

che é una equazione di secondo grado nelle incognite (λ, µ). Le due soluzioni for-
niscono le coppie di valori (λ1 , µ1 ), (λ2 , µ2 ) che, sostituiti alternativamente nell’equazione
della retta, individuano i due punti di intersezione P1 = λ1 Y + µ1 Y 0 , P2 =
λ2 Y + µ2 Y 0 . Il discriminante dell’equazione di secondo grado é

(Y t AY 0 )2 − (Y t AY )(Y 0t AY 0 ) = ∆.

Si puó concludere allora che la retta r é esterna, tangente o secante la conica, in


relazione al valore del ∆ (rispettivamente < 0, = 0, > 0.) In particolare possiamo
sfruttare quanto detto per dimostrare il seguente:

Teorema 1. Sia γ una conica con matrice associata A. Le seguenti affermazioni


sono equivalenti:
1. La conica γ é riducibile.
2. La conica γ ha almeno un punto doppio.
3. det(A) = 0.

Dim. Supponiamo che la conica sia riducibile.


Essa é allora composta da due rette, incidenti in un punto (proprio o improprio)
oppure coincidenti. In entrambi i casi i punti comuni alle rette sono punti doppi
per la conica in base alla definizione di punto doppio per una curva.
Viceversa se una conica possiede un punto doppio P0 , e detto Q1 un qualsiasi
altro punto della conica, segue che la retta contenente i punti P0 , Q1 dovrá avere
almeno 3 punti di intersezione con la conica. Ció vuol dire che la retta appartiene
completamente alla conica, la quale é perció riducibile.
Infine dimostriamo che la conica é riducibile se e solo se det(A) = 0. Infatti,
per quanto detto sopra, la conica é riducibile ⇔ esiste P 0 = Y 0 punto doppio ⇔
una qualsiasi retta r per P 0 ha 2 intersezioni con la conica γ in P 0 ⇔ intersecando
γ ed r si ottiene un sistema con ∆ = 0, dove nel nostro caso (vedi argomento
precedente) ∆ = (Y t AY 0 )2 − (Y t AY )(Y 0t AY 0 ) = (Y t AY 0 )2 (poiché P 0 ∈ γ e
quindi (Y 0t AY 0 ) = 0 ). Siamo allora nella seguente situazione: (Y t AY 0 ) = 0 al
variare di Y vettore generico del piano. Quindi AY 0 = 0, cioé det(A) = 0, visto che
la soluzione Y 0 = (0, 0, 0) non rappresenta alcun punto del piano.
LE CONICHE. 3

3. Polaritá rispetto ad una conica. Sia γ : X T · A · X = 0 una conica non


riducibile. Definiamo polaritá rispetto a γ la corrispondenza biunivoca che associa
ad ogni punto X 0 = (x01 , x02 , x03 ) del piano, una retta r0 la cui equazione é data da
X 0T · A · X = 0. La retta r0 é detta retta polare di X 0 rispetto a γ, il punto X 0 é
detto polo della retta r0 rispetto a γ.
Consideriamo r0 la retta polare del punto X 0 rispetto alla conica γ. La sua
equazione é data in forma compatta da X 0T AX = 0. Sia X 00 un qualsiasi punto di
r0 . Quindi le sue coordinate soddisfano l’equazione algebrica che individua la retta
r0 , cioé X 0T AX 00 = 0. Trasponendo ambo i membri di tale uguaglianza, otteniamo
X 00T AX 0 = 0, che ha un significato ben preciso: il punto di coordinate X 0 appar-
tiene alla retta r00 polare di X 00 (tale proprietá é detta reciprocitá). Riassumendo
potremmo dire che se un punto X 00 appartiene alla polare di un altro punto X 0 ,
allora X 0 appartiene alla polare di X 00 . Diremo che i punti X 0 e X 00 sono tra loro
coniugati, ed analogamente che le rette r0 e r00 sono tra loro coniugate.
Un punto é detto autoconiugato se appartiene alla propria polare.
Consideriamo ora un punto X 0 appartenente alla conica. Sia t la retta tangente
alla conica in X 0 . Se scegliamo un qualsiasi altro punto X 00 ∈ t, l’equazione della
tangente si puó esprimere in forma compatta con X = αX 0 + βX 00 . Se ora inter-
sechiamo la retta t con la conica, ovviamente ci aspettiamo un discriminante del
sistema pari a zero, vista la tangenza in X 0 . Come giá visto nel paragrafo relativo
all’intersezione di una retta con una conica, il discriminante di tale sistema sará:
∆ = (X 00T AX 0 )2 − (X 00T AX 00 )(X 0T AX 0 ) = 0
e poiché X 0 ∈ γ, segue che X 0T AX 0 = 0 e quindi ∆ = (X 00T AX 0 )2 = 0. Inoltre
(X 00T AX 0 ) é uno scalare, il suo quadrato é nullo solo se esso stesso é nullo ed il
suo valore trasposto deve essere ancora nullo; in definitiva X 0T AX 00 = 0, cioé X 00
(generico punto della tangente t) é un punto della retta polare di X 0 . Questo é
sufficiente per affermare che se un punto appartiene alla conica, allora la sua retta
polare coicide con la tangente in esso alla conica. Tale punto é autoconiugato, piú
in generale, tutti e soli i punti autoconiugati rispetto ad una conica sono quelli della
conica stessa. Ed ancora, una retta é detta autoconiugata se contiene il proprio
polo. Tutte e sole le rette autoconiugate rispetto ad una conica, sono le tangenti
alla conica stessa.
Concludiamo fornendo un significato geometrico della retta polare di un punto
esterno alla conica. Sia P 0 = X 0 il punto scelto. Da esso si possono condurre
due rette tangenti alla conica, siano esse r1 e r2 . Indichiamo con P1 il punto di
tangenza per la retta r1 , con P2 il punto di tangenza per la retta r2 . Per quanto
detto sopra, tali rette tangenti alla conica sono proprio le rispettive polari di P1
e P2 , per cui il punto P 0 appartiene sia alla polare di P1 , che alla polare di P2 .
Grazie alla reciprocitá, entrambi P1 e P2 devono appartenere alla retta r polare di
P 0 . Tale retta r é quindi esattamente la retta congiungente i punti P1 e P2 .

4. Classificazione di una conica. Sia γ : X T · A · X = 0 una conica nel piano.


Se intersechiamo γ con la retta impropria x3 = 0 otteniamo ovviamente 2 soluzioni,
cioé i due punti impropri della conica.
Una conica con due punti impropri reali e distinti é detta Iperbole. Una conica
con due punti impropri reali e coincidenti é detta Parabola. Una conica con due
punti impropri complessi coniugati é detta ellisse. In altre parole la retta impropria
é secante all’iperbole, tangente alla parabola, esterna all’ellisse. Consideriamo il
4 LE CONICHE.

sistema
XT · A · X = 0 a11 x21 + 2a12 x1 x2 + a22 x22 = 0
 
→ .
x3 = 0 x3 = 0
Il discriminante del sistema é ∆ = a212 − a11 a22 , da cui

 ∆ > 0 → Iperbole
∆ = 0 → Parabola .
∆<0 → Ellisse

In particolare se indichiamo con A33 il complemento algebrico dell’elemento a33


della matrice A, notiamo che A33 = −∆, quindi

 A33 > 0 → Ellisse
A33 = 0 → Parabola .
A33 < 0 → Iperbole

Osservazione 1. Tutte e sole le coniche che contengono i punti ciclici (1, i, 0) e


(1, −i, 0) sono le circonferenze (che sono particolari ellissi).

5. Fascio di coniche. Siano γ1 : f1 (x1 , x2 , x3 ) = 0 e γ2 : f2 (x1 , x2 , x3 ) = 0 due


coniche distinte. Poiché sono curve di secondo grado, escludendo il caso in cui
siano entrambe riducibili con una retta in comune, esse hanno 4 punti A, B, C, D
in comune (distinti o no, reali o no).
Il caso generale é quello in cui A, B, C, D sono tutti tra loro distinti.
Nel caso in cui si abbia A = B e C, D distinti, allora le due coniche hanno una
tangente comune ad entrambe (γ1 e γ2 sono chiamate coniche tangenti), ed é la
retta tangente nel punto A = B.
Nel caso si abbia A = B e C = D allora le due coniche hanno due rette tangenti
in comune (γ1 e γ2 sono chiamate coniche bitangenti), e sono le tangenti nel punto
A = B ed in C = D.
Nel caso si abbia A = B = C allora le due coniche hanno una retta tangente
in comune (γ1 e γ2 sono chiamate coniche osculatrici), ed é la tangente nel punto
A = B = C.
Nel caso si abbia A = B = C = D allora le due coniche hanno una retta tangente
in comune (γ1 e γ2 sono chiamate coniche iperosculatrici), ed é la tangente nel punto
A = B = C = D.

Diciamo fascio di coniche la totalitá delle coniche che si ottengono da


a1 · f1 (x1 , x2 , x3 ) + a2 · f2 (x1 , x2 , x3 ) = 0
al variare di a1 , a2 ∈ lR. Tutte le coniche del fascio hanno in comune gli stessi 4
punti A, B, C, D, i quali sono chiamati punti base del fascio. Inoltre per individuare
un fascio di coniche é sufficiente conoscere due qualsiasi coniche di esso (ad esempio
anche due coniche riducibili che appartengono al fascio).

Il caso generale é quello in cui A, B, C, D sono tutti tra loro distinti, si parla
quindi di fascio generale.
Nel caso in cui si abbia A = B e C, D distinti, allora le coniche del fascio hanno
una tangente comune (si parla di fascio di coniche tangenti), ed é la retta tangente
nel punto A = B.
LE CONICHE. 5

Nel caso si abbia A = B e C = D allora le coniche del fascio hanno due rette
tangenti in comune (si parla di fascio di coniche bitangenti), e sono le tangenti nel
punto A = B ed in C = D.
Nel caso si abbia A = B = C allora le coniche del fascio hanno una retta
tangente in comune (si parla di fascio di coniche osculatrici), ed é la tangente nel
punto A = B = C.
Nel caso si abbia A = B = C = D allora le coniche del fascio hanno una retta
tangente in comune (si parla di fascio di coniche iperosculatrici), ed é la tangente
nel punto A = B = C = D.

Ogni fascio di coniche contiene 3 coniche riducibili (distinte o no a seconda del


tipo di fascio). Nel seguente prospetto indichiamo con tA e tC rispettivamente le
tangenti comuni a tutte le coniche di un fascio nei punti A e C:

Tipo di fascio Coniche riducibili


generale AB · CD
AD · BC
AC · BD
tangenti A = B tA · CD
AD · AC contata due volte
bitangenti A = B, C = D tA · t C
AC · AC contata due volte
osculatrici A = B = C tA · AD contata tre volte
iperosculatrici A = B = C = D tA · tA contata tre volte

6. Diametri e centro di una conica. Definiamo diametro di una conica γ non


riducibile, la retta polare, rispetto alla conica, di un qualsiasi punto improprio
(h, k, 0). Sia A la matrice associata alla conica, allora un diametro é dato da
   
  a11 a12 a13 x1
h k 0 ·  a12 a22 a23  ·  x2  = 0
a13 a23 a33 x3
(a11 h + a12 k)x1 + (a12 h + a22 k)x2 + (a13 h + a23 k)x3 = 0
o meglio
h(a11 x1 + a12 x2 + a13 x3 ) + k(a12 x1 + a22 x2 + a23 x3 ) = 0.
Al variare del punto (h, k, 0) si ottiene un diverso diametro, e quindi al variare dei
parametri h, k si ottengono tutte le rette di un fascio. Il centro di tale fascio é detto
”centro della conica” (il quale per la reciprocitá é il polo della retta impropria).
Le coordinate del centro sono allora la soluzione del sistema

a11 x1 + a12 x2 + a13 x3 = 0
a12 x1 + a22 x2 + a23 x3 = 0
le cui soluzioni sono date dai complementi algebrici A13 , A23 , A33 della matrice A
associata alla conica.
Il caso in cui A33 = 0 é quello della parabola, la quale é detta conica a centro
improprio (o conica senza centro). Il fascio dei diametri é un fascio improprio,
cioé tutti i diametri sono tra loro paralleli ed hanno la direzione data dal punto
improprio della parabola.
6 LE CONICHE.

Iperbole ed ellisse sono dette coniche a centro (proprio). Tutti i diametri sono
tra loro a due a due coniugati.
In particolare i diametri tra loro coniugati ed ortogonali sono detti assi della
conica (2 assi per iperbole ed ellisse, 1 asse per la parabola). I diametri che siano
autoconiugati (cioé tangenti alla conica) sono detti asintoti della conica (2 reali per
l’iperbole, 2 immaginari per l’ellisse, 1 improprio per la parabola).
Per determinare assi e asintoti é sufficiente operare come segue: sappiamo bene
che il generico diametro della conica é dato da:
   
  a11 a12 a13 x1
h k 0 ·  a12 a22 a23  ·  x2  = 0 (∗).
a13 a23 a33 x3
In definitiva il nostro compito é quello di determinare le coppie di valori (h, k) da
utilizzare nella (*), sia per individuare gli asintoti che gli assi.
Affinché (h, k, 0) sia esattamente il polo di un asintoto, dovrá accadere che esso
sia un punto autoconiugato (l’asintoto sará tangente in esso alla conica), quindi
imporremo che
   
  a11 a12 a13 h
h k 0 ·  a12 a22 a23  ·  k  = 0.
a13 a23 a33 0
Svolgendo si ottiene a11 h2 + a22 k 2 + 2a12 hk = 0. Le soluzioni di tale equazione
biquadraica forniscono le due coppie di valori (h, k) e (h0 , k 0 ) che, sostituite nella
(*) serviranno per il calcolo effettivo dei due asintoti (stiamo ovviamente parlando
di una conica che sia una iperbole).
Al contrario affinché (h, k, 0) sia esattamente il polo di un asse, dovrá accadere che
sia un punto coniugato con la direzione ad esso ortogonale, quindi imporremo che
   
  a11 a12 a13 k
h k 0 ·  a12 a22 a23  ·  −h  = 0.
a13 a23 a33 0
In questo caso si ottiene a12 (k 2 − h2 ) − (a11 − a22 )hk = 0. Le soluzioni di tale
equazione biquadraica forniscono le due coppie di valori (h, k) e (h0 , k 0 ) che, sosti-
tuite nella (*) serviranno per il calcolo dei due assi.
Nel caso della parabola la ricerca dell’asse é facilitata. Infatti il polo dell’asse
della parabola é dato dalla direzione ortogonale a quella individuata dal punto
improprio della parabola stessa.
Osserviamo infine che le direzioni dei due asintoti di un’iperbole si possono ot-
tenere ricordando che esse non sono altro che i punti impropri dell’iperbole stessa.

7. Classificazione delle coniche proiettive. Sia γ : X T ·A·X = f (x1 , x2 , x3 ) =


0 l’equazione di una conica,
f (x1 , x2 , x3 ) = a11 x21 + 2a12 x1 x2 + a22 x22 + 2a13 x1 x3 + 2a23 x2 x3 + a33 x23 = 0.
Poiché f é una forma quadratica in lR3 , allora vale il seguente:

Teorema 2. Esiste un sistema di riferimento ortonormale in lR3 rispetto al quale


l’equazione della conica γ é una delle seguenti:
1. x21 + x22 + x23 = 0 (conica generale a punti non reali);
2. x21 + x22 − x23 = 0 (conica generale a punti reali);
LE CONICHE. 7

3. x21 + x22 = 0 (conica semplicemente degenere a punti non reali);


4. x21 − x22 = 0 (conica semplicemente degenere a punti reali);
5. x21 = 0 (conica doppiamente degenere).
Per ottenere la forma ridotta congruente a quella di partenza, si applica esatta-
mente il metodo di diagonalizzazione delle forme quadratiche.

8. Classificazione delle coniche euclidee. Consideriamo ora l’equazione affine


di una conica
γ : f (x, y) = a11 x2 + 2a12 xy + a22 y 2 + 2a13 x + 2a23 y + a33 = 0
dove aij ∈ lR, per ogni i e j. Indichiamo q(x, y) = a11 x2 + 2a12 xy + a22 y 2 la forma
quadratica in lR2 , con matrice associata A33 .
Poiché q(x, y) é riferita ad un sistema otonormale in lR2 , ogni cambiamento del
sistema di riferimento é individuato da una matrice ortogonale C, per cui C T =
C −1 . In tale caso la diagonalizzazione della forma quadratica q(x, y) puó essere
effettuata tramite la semplice diagonalizzazione della matrice simmetrica A33 ad
essa associata.
Sia quindi    
  a11 a12 x
q(x, y) = x y · ·
a12 a22 y
e siano λ1 e λ2 gli autovalori di A33 . Indichiamo con w1 = (b11 , b21 ) l’autovettore
che genera l’autospazio relativo a λ1 e w2 = (b12 , b22 ) quello
 che genera
 l’autospazio
b11 b12
relativo a λ2 . Allora la matrice che diagonalizza A33 é con
b21 b22
 
λ1 0
C T · A33 · C = C −1 · A33 · C = .
0 λ2
Il cambiamento di variabili che permette tale diagonalizazione é
     0 
x b11 b12 x
= ·
y b21 b22 y0
dopo il quale la conica si presenta nella seguente forma:
f (x0 , y 0 ) = λ1 x2 + λ2 y 2 + 2a013 x + 2a023 y + a033 = 0.
In pratica la f (x0 , y 0 ) é la conica che otteniamo facendo ruotare i suoi assi fino a
renderli paralleli agli assi coordinati.

Supponiamo che la conica sia un’iperbole o una ellisse.


Determiniamo ora una traslazione che riporti il centro della conica nel centro
degli assi coordinati:  0
x = x00 − c
y 0 = y 00 − d
λ1 (x00 − c)2 + λ2 (y 00 − d)2 + 2a013 (x00 − c) + 2a023 (y 00 − d) + a033 = 0
cioé
λ1 x002 + λ2 y 002 + (−2λ1 c + 2a013 )x00 +
(−2λ2 d + 2a023 )y 00
+ (λ1 c2 + λ2 d2 + 2a013 c + 2a023 d + a033 ) = 0 (B).
00 00
Poiché dopo la rototraslazione scompaiono i termini in x e y , allora imponiamo
che
−2λ1 c + 2a013 = 0
8 LE CONICHE.

−2λ2 d + 2a023 = 0.
I valori di c e d che risolvono le precedenti equazioni sono i valori che determinano
la traslazione. É sufficiente sostituirli nell’equazione (B) per ottenere la conica in
forma ridotta:
λ1 x002 + λ2 y 002 + λ3 = 0
con λ3 = λ1 c2 + λ2 d2 + 2a013 c + 2a023 d + a033 .

Consideriamo ora il caso in cui la conica sia una parabola.


Per prima cosa osserviamo che la prima rotazione viene effettuata utilizzando
come riferimento quello formato dall’asse della parabola e dalla retta tangente nel
suo vertice.
Inoltre uno dei due autovalori λ1 , λ2 di A33 é nullo, per cui, dopo il primo
cambiamento di riferimento (rotazione), l’equazione della conica si presenta in una
delle due seguenti forme:

λ1 x02 + 2a023 y 0 + a033 = 0 (C)


oppure
λ2 y 02 + 2a 130 x0 + a033 = 0 (D).

Cominciamo con il caso (C). Determiniamo una traslazione che riporti il vertice
della parabola nel centro degli assi coordinati:
 0
x = x00 − c
y 0 = y 00 − d
l’equazione diventa
λ1 (x00 − c)2 + 2a023 (y 00 − d) + a033 = 0 (C0 )
Poiché dopo la rototraslazione scompaiono il termine noto ed il termine in x00 ,
dobbiamo imporre che
2λ1 c = 0
λ1 c2 − 2a023 d + a033 = 0.
I valori di c e d che risolvono le precedenti equazioni sono i valori che determinano
la traslazione. É sufficiente sostituirli nell’equazione (C’) per ottenere la conica in
forma ridotta:
λ1 x002 + 2a023 y 00 = 0.

Passiamo ora al caso (D). Dopo la traslazione, l’equazione della parabola é

λ2 (y 00 − c)2 + 2a013 (x00 − d) + a033 = 0 (D0 )


Poiché dopo la rototraslazione scompaiono il termine noto ed il termine in y 00 ,
dobbiamo imporre che
2λ2 d = 0
λ2 d2 − 2a013 c + a033 = 0.
I valori di c e d che risolvono le precedenti equazioni sono i valori che determinano
la traslazione. É sufficiente sostituirli nell’equazione (D’) per ottenere la conica in
forma ridotta:
λ2 y 002 + 2a013 x00 = 0.
LE CONICHE. 9

Quanto detto fin’ora si puó riassumere nel seguente:


Teorema 3. Ogni conica del piano euclideo reale é congruente ad una delle seguenti
forme, dette forme canoniche:
x2 y2
1. a2 + b2 = 1, ellisse;
x2 y2
2. a2 + b2 = −1, ellisse a punti non reali;
x2 y2
3. a2 + b2 = 0, ellisse degenere;
x2 y2
4. a2 − b2 = 1, iperbole;
x2 y2
5. a2 + = 0, iperbole degenere;
b2
2
6. y − ax = 0, parabola;
7. y 2 − a2 = 0, parabola degenere;
8. y 2 + a2 = 0, parabola degenere a punti non reali;
9. y 2 = 0, conica doppiamente degenere.

Riassumendo, abbiamo visto che una rototraslazione degli assi di una conica
(dell’asse e della retta tangente al vertice, nel caso della parabola) ci permette di
ottenere una ulteriore e piú semplice forma della conica stessa, riferita ad un sistema
di riferimento opportuno. Tali forme sono dette ’ridotte’ o ’canoniche’ .
Nel cambiamento del sistema di riferimento ortogonale, vi sono alcune quantitá
(reali) che non mutano, cioé si mantengono invarianti nel passaggio da una forma
della conica all’altra. Tali quantitá vengono dette invarianti ortogonali:
Teorema 4. Sia γ : X T ·A·X = 0 una conica del piano euclideo e sia X 0T ·A0 ·X 0 =
0 la sua equazione in forma ridotta, cioé dopo una cambiamento ortonormale del
sistema di riferimento. Siano aij gli elementi della matrice A e a0ij quelli della
matrice A0 . Allora valgono le seguenti:
1. det(A) = det(A0 ).
2. A33 = A033 .
3. a11 + a22 = a011 + a022 .
Possiamo sfruttare il precedente teorema per ottenere la forma ridotta di una
conica. Sia A la matrice associata alla conica, operiamo nel modo seguente:

Coniche a centro. La forma canonica alla quale si vuole arrivare é la seguente


a011 x21 + a022 x22 + a033 x23 = 0
la cui matrice associata é
a011 0
 
0
 0 a022 0  .
0 0 a033
0 0 0
Per ottenere i valori di a11 , a22 , a33 é sufficiente applicare i tre punti del teorema e
risolvere le equazioni:
det(A) = a011 · a022 · a033
A33 = a011 · a022
a11 + a22 = a011 + a022 .

Parabola. Una forma canonica alla quale si puó arrivare é la seguente


a011 x21 + 2a023 x2 x3 = 0
10 LE CONICHE.

la cui matrice associata é


a011
 
0 0
 0 0 a023  .
0 a023 0
Per ottenere i valori di a011 , a023 é sufficiente applicare i tre punti del teorema e
risolvere le equazioni:
det(A) = −a011 · a02
23
a11 + a22 = a011 .
L’altra forma canonica puó essere
a022 x22 + 2a013 x1 x3 = 0
la cui matrice associata é
a013
 
0 0
 0 a022 0 .
a013 0 0
Per ottenere i valori di a022 , a023 é sufficiente applicare i tre punti del teorema e
risolvere le equazioni:
det(A) = −a022 · a02
13
a11 + a22 = a022 .
Esercizio 1. Classificare la conica x2 + 4xy − y 2 + x − y + 1 = 0, determinarne
eventuali assi e asintoti, una sua forma canonica ed il polo della retta 2x−y +1 = 0
rispetto ad essa.

Svolg. La matrice associata alla conica é


1
 
1 2 2
A =  2 −1 − 12 
1
2 − 12 1
con det(A) = −6, A33 = −5, I = a11 + a22 = 0, quindi é una iperbole equilatera.
Determiniamo il generico diametro:
1
   
1 2 2 x1
1 h 0 ·  2 −1 − 12  ·  x2  = 0.
 
1
2 − 12 1 x3
Il generico diametro é
1 h
(1 + 2h)x1 + (2 − h)x2 + ( − )x3 = 0
2 2
0 1+2h
il cui coefficiente angolare é h = h−2 .
√ √
Per ottenere gli asintoti imponiamo h = h0 , da cui h ∈ {2 + 5, 2 − 5} e quindi
gli asintoti sono √ √ √
(10 + 4 5)x − 2 5y + (− 5 − 1) = 0
e √ √ √
(10 − 4 5)x + 2 5y + ( 5 − 1) = 0.
√ √
Per ottenere gli assi imponiamo hh0 = −1, da cui h ∈ { −1+2 5 −1− 5
, 2 } e quindi
gli assi sono √ √ √
4 5x + (10 − 2 5)y + (3 − 5) = 0
LE CONICHE. 11

e √ √ √
−4 5x + (10 + 2 5)y + (3 + 5) = 0.
Una forma canonica é data da a11 x21 + a22 x22 + a33 x23 = 0, con matrice associata
 
a11 0 0
0
A =  0 a22 0  .
0 0 a33
I suoi invarianti ortogonali sono allora
−6 = det(A0 ) = a11 a22 a33 , −5 = A033 = a11 a22 , 0 = I = a11 + a22
dai quali otteniamo
√ √ 6
a11 = 5, a22 = − 5, a33 =
5
oppure
√ √ 6
a11 = − 5, a22 = 5, a33 =
5
e le due forme canoniche ottenute sono:
√ 2 √ 2 6
5x − 5y + = 0
5
e
√ √ 6
− 5x2 + 5y 2 + = 0.
5
Infine determiniamo il polo della retta 2x1 − x2 + x3 = 0 rispetto alla conica. Esso
avrá coordinate (a, b, c) tali che
1
   
1 2 2 x1
a b c ·  2 −1 − 12  ·  x2  = α(2x1 − x2 + x3 ).
 
1
2 − 12 1 x3
Quindi dobbiamo risolvere il sistema

 2a + 4b + c = 4α
4a − 2b − c = −2α
a − b + 2c = 2α

le cui soluzioni sono


1 5 5
(α , α , α )
8 8 4
che é una terna proporzionale a (1, 5, 10) = (a, b, c). t
u
Esercizio 2. Determinare l’asse ed una forma canonica della parabola 2x2 + 2y 2 −
4xy + 2x − 1 = 0.

Svolg. La matrice associata alla conica é


 
2 −2 1
A =  −2 2 0 
1 0 −1
con det(A) = −2, A33 = 0, I = a11 + a22 = 4.
Determiniamo il punto improprio della parabola:
2x21 + 2x22 − 4x1 x2 = 0

x3 = 0
12 LE CONICHE.

da cui x1 = 1, x2 = 1, x3 = 0 sono le coordinate di tale punto. La direzione ad esso


ortogonale é Q = (1, −1, 0). Tale punto Q é il polo dell’asse:
   
  2 −2 1 x1
1 −1 0 ·  −2 2 0  ·  x2  = 0
1 0 −1 x3
4x1 − 4x2 + x3 = 0.
Una forma canonica é data da a11 x21 + 2a23 x2 x3 = 0, con matrice associata
 
a11 0 0
A0 =  0 0 a23  .
0 a23 0
I suoi invarianti ortogonali sono allora
−2 = det(A0 ) = −a11 a223 , 4 = I = a11 + a22
dai quali otteniamo
1
a11 = 4, a23 = √
2
oppure
1
a11 = 4, a23 = − √
2
e le due forme canoniche ottenute sono:
2
4x21 + √ x2 x3 = 0
2
e
2
4x21 − √ x2 x3 = 0
2
cioé √ √
y = −2 2x2 e y = 2 2x2 .
t
u
2 2
Esercizio 3. Determinare una forma canonica dell’ellisse x −xy+y −5x+7y+1 =
0.

Svolg. Utilizziamo il metodo degli autovalori. Quindi determiniamo gli autovalori


1 − 12
della matrice A33 = . Essi sono h1 = 12 e h2 = 32 . L’autospazio
− 12 1
relativo all’autovalore h1 é generato dall’autovettore v = (1, 1) che ha come versore
( √12 , √12 ).
L’autospazio relativo all’autovalore h2 é generato dall’autovettore v = (−1, 1)
che ha come versore (− √12 , √12 ).
Allora il primo cambiamento di variabili, con il quale annulliamo il termine in
xy é dato da
 " √1 # 
− √12 x0
 
x 2
= √1 ·
y 2
√1
2
y0
da cui (
x= √1 (x0 − y0 )
2
y= √1 (x0 + y0 )
2
LE CONICHE. 13

e l’equazione della conica diventa


1 0 1 1 5 7
(x − y 0 )2 − (x0 − y 0 )(x0 + y 0 ) + (x0 + y 0 )2 − √ (x0 − y 0 ) + √ (x0 + y 0 ) + 1 = 0
2 2 2 2 2
cioé
1 02 3 02 2 12
x + y + √ x0 + √ y 0 + 1 = 0.
2 2 2 2
Per annullare i termini in x0 e y 0 operiamo la seguente traslazione:
x0 = x00 − a y 0 = y 00 − b
1 00 3 2 12
(x − a)2 + (y 00 − b)2 + √ (x00 − a) + √ (y 00 − b) + 1 = 0
2 2 2 2
da cui
1 002 3 2 2 12 12
(x + a2 − 2ax00 ) + (y 002 + b2 − 2by 00 ) + √ x00 − √ a + √ y 00 − √ b + 1 = 0.
2 2 2 2 2 2
I coefficienti di x00 e y 00 si annullano per a = √22 e b = √42 e l’equazione della conica
diventa:
1 00 2 3 4 2 2 12 4
(x − √ )2 + (y 00 − √ )2 + √ (x00 − √ ) + √ (y 00 − √ ) + 1 = 0
2 2 2 2 2 2 2 2
cioé
1 002 3 002
x + y = 12
2 2
x002 y 002
+ = 1.
24 8
Vogliamo ora ripetere l’esercizio utilizzando il metodo di rototraslazione degli assi:
gli assi dell’ellisse sono
a1 : x−y−4=0 che scegliamo come X0
a2 : x + y + 2 = 0 che scegliamo come Y 0 .
I versori degli assi sono
1 1 1 1
a1 = ( √ , √ ) a2 = (− √ , √ )
2 2 2 2
ed il centro dell’ellisse é C = (1, −3). Allora le formule del cambiamento del riferi-
mento (rototraslazione) sono
 " √1 # 
− √12 x0
   
x 2 1
= √1 · +
y 2
√1
2
y0 −3
da cui (
x= √1 (x0 − y0 ) + 1
2
y= √1 (x0 + y0 ) − 3
2
e l’equazione della conica diventa
1 1 1 1
( √ (x0 − y 0 ) + 1)2 − ( √ (x0 − y 0 ) + 1)( √ (x0 + y 0 ) − 3) + ( √ (x0 + y 0 ) − 3)2
2 2 2 2
1 0 1
−5( √ (x − y 0 ) + 1) + 7( √ (x0 + y 0 ) − 3) + 1 = 0
2 2
cioé
1 02 3 02
x + y − 12 = 0
2 2
14 LE CONICHE.

x02 y 02
+ = 1.
24 8
t
u
Esercizio 4. Determinare una forma canonica della parabola x2 − 4xy + 4y 2 + 2x +
y − 5 = 0.

Svolg. Utilizziamo il metodo


 degli
 autovalori. Quindi determiniamo gli autovalori
1 −2
della matrice A33 = . Essi sono h1 = 0 e h2 = 5. L’autospazio
−2 4
relativo all’autovalore h1 é generato dall’autovettore v = (2, 1) che ha come versore
( √25 , √15 ).
L’autospazio relativo all’autovalore h2 é generato dall’autovettore v = (1, −2)
che ha come versore ( √15 , − √25 ).
Allora il primo cambiamento di variabili, con il quale annulliamo i termini in xy
e y é dato da
 " √2 √1
# 
− x0
 
x 5 5
= √1 ·
y 5
√2
5
y0
da cui (
x= √1 (2x0 − y 0 )
5
y= √1 (x0 + 2y 0 )
5
e l’equazione della conica diventa
1 4 4 1 1
(2x0 −y 0 )2 − (2x0 −y 0 )(x0 +2y 0 )+ (x0 +2y 0 )2 + √ (2x0 −y 0 )+ √ (x0 +2y 0 )−5 = 0
5 5 5 5 5
cioé
5
5y 02 + √ x0 − 5 = 0
5
02 1 0
y + √ x − 1 = 0.
5
Per annullare il termine noto operiamo la seguente traslazione:
x0 = x00 − a y 0 = y 00 − b
1
(y 00 − b)2 + √ (x00 − a) − 1 = 0
5
1 1
y 002 + b2 − 2by 00 + √ x00 − √ a − 1 = 0.
5 5
Per avere la forma canonica dovremmo avere

b=0 a=− 5
da cui
1 1 √
(y 00 − b)2 + √ (x00 − a) − 1 = y 002 + √ (x00 + 5) − 1 =
5 5
1
y 002 + √ x00 = 0.
5
t
u
Esercizio 5. Sia dato il fascio di coniche x2 + kxy − 1 = 0, al variare di k ∈ lR.
Determinare i punti base, le coniche riducibili e le tangenti comuni a tutte le coniche
del fascio.
LE CONICHE. 15

Svolg. In coordinate omogenee l’equazione del fascio é data da


x21 + kx1 x2 − x23 = 0.
Per determinare i punti base del fascio scegliamo due coniche qualsiasi, per esempio
x21 − x23 = 0 e x1 x2 = 0 ed intersechiamole:

x1 x2 = 0
x21 − x23 = 0
che ha come soluzione i punti (0, 1, 0) con molteplicitá 2, (1, 0, 1) e (1, 0, −1) en-
trambi con molteplicitá 1. Siamo in presenza di un fascio di coniche tangenti, cioé vi
é una tangente comune a tutte le coniche del fascio. Per determinarla, scegliamo una
conica irriducibile del fascio, per esempio per k = 2, e calcoliamo la tangente a tale
conica nel punto (0, 1, 0): per il valore k = 2, la conica ottenuta é x2 + 2xy − 1 = 0.
La tangente ad essa in (0, 1, 0) é
   
  1 1 0 x1
0 1 0 ·  1 0 0  ·  x2  = 0 cioé x1 = 0.
0 0 −1 x3
Infine calcoliamo le coniche riducibili del fascio: esse si ottengono annullando il
determinante della matrice associata alla generica conica del fascio:

1 k

0
k 2 k2
0 = 2 0 0 =
0 0 −1 4

quindi per k = 0, con molteplicitá 2, si ottiene la conica riducibile del fascio x2 −1 =


0. La terza conica riducibile si ottiene osservando che il parametro k é associato ad
essa nell’espressione del fascio: xy = 0, con molteplicitá 1. t
u
Esercizio 6. Sia dato il fascio di coniche kx2 + 2xy − k = 0, al variare di k ∈ lR.
Determinare i punti base, le coniche riducibili e le tangenti comuni a tutte le coniche
del fascio.

Svolg. In coordinate omogenee l’equazione del fascio é data da


kx21 + 2x1 x2 − kx23 = 0.
Per determinare i punti base del fascio scegliamo due coniche qualsiasi, per esempio
x21 − x23 = 0 e 2x1 x2 = 0 ed intersechiamole:

2x1 x2 = 0
x21 − x23 = 0
che ha come soluzione i punti (0, 1, 0) con molteplicitá 2, (1, 0, 1) e (1, 0, −1) en-
trambi con molteplicitá 1. Siamo in presenza di un fascio di coniche tangenti,
cioé vi é una tangente comune a tutte le coniche del fascio. Per determinarla,
scegliamo una conica irriducibile del fascio, per esempio per k = 2, e calcoliamo la
tangente a tale conica nel punto (0, 1, 0): per il valore k = 2, la conica ottenuta é
2x2 + 2xy − 2 = 0. La tangente ad essa in (0, 1, 0) é
   
  2 1 0 x1
0 1 0 ·  1 0 0  ·  x2  = 0 cioé x1 = 0.
0 0 −2 x3
16 LE CONICHE.

Infine calcoliamo le coniche riducibili del fascio: esse si ottengono annullando il


determinante della matrice associata alla generica conica del fascio:

k 1 0

0 = 1 0 0 = k
0 0 −k
quindi per k = 0, con molteplicitá 1, si ottiene la conica riducibile del fascio x1 x2 =
0. L’altra conica riducibile (il fascio ne contiene solo 2 distinte) si ottiene osservando
che il parametro k é associato ad essa nell’espressione del fascio: x2 − 1 = 0, con
molteplicitá 2. t
u
Esercizio 7. Sia dato il fascio di coniche x2 + ky 2 + xy − 1 = 0, al variare di
k ∈ lR. Determinare i punti base, le coniche riducibili e le tangenti comuni a tutte
le coniche del fascio.

Svolg. In coordinate omogenee l’equazione del fascio é data da


x21 + kx22 + x1 x2 − x23 = 0.
Per determinare i punti base del fascio scegliamo due coniche qualsiasi, per esempio
x21 + x1 x2 − x23 = 0 e x22 = 0 ed intersechiamole:
x22 = 0


x1 + x1 x2 − x23 = 0
2

che ha come soluzione i punti (1, 0, 1) e (1, 0, −1) entrambi con molteplicitá 2. Siamo
in presenza di un fascio di coniche bitangenti, cioé vi sono due tangenti comuni a
tutte le coniche del fascio. Per determinarle, scegliamo una conica irriducibile del
fascio, per esempio per k = 1, e calcoliamo la tangente a tale conica nei punti
(1, 0, 1) e (1, 0, −1): per il valore k = 1, la conica ottenuta é x2 + y 2 + xy − 1 = 0.
La tangente ad essa in (1, 0, 1) é

1
   
1 0 x1
  1
2 1
1 0 1 · 2 1 0  ·  x2  = 0 cioé x1 + x2 − x3 = 0.
2
0 0 −1 x3
La tangente in (1, 0, −1) é

1
   
1 0 x1
  1
2 1
1 0 −1 · 2 1 0  ·  x2  = 0 cioé x1 + x2 + x3 = 0.
2
0 0 −1 x3
Infine calcoliamo le coniche riducibili del fascio: esse si ottengono annullando il
determinante della matrice associata alla generica conica del fascio:

1 1

0
1 2 1
0 = 2 k 0 = −k +
0 0 −1 4

quindi per k = 14 , con molteplicitá 1, si ottiene la conica riducibile del fascio x1 +


1 2 2
4 x2 + x1 x2 − x3 = 0. Le altre coniche riducibili si ottengono osservando che il
parametro k é associato ad esse nell’espressione del fascio: y 2 = 0, con molteplicitá
2 (infatti la retta y = 0 passa per i due base che hanno molteplicitá 2). t
u
LE CONICHE. 17

9. Fuochi di una conica. Siano C1 = (1, i, 0) e C2 = (1, −i, 0) i punti ciclici della
retta impropria. Le rette appartenenti ai due fasci di centri C1 e C2 , sono dette
rette isotrope. Quindi una qualsiasi retta isotropa avrá equazione y = ix+q oppure
y = −ix + p, per opportuni q, p elementi del campo dei numeri complessi.
Indichiamo con f1 : y = ix + q il fascio delle rette isotrope di centro C1 e con
f2 : y = −ix + p il fascio di centro C2 . Osserviamo che se una retta r appartiene
al fascio f1 allora la sua coniugata r appartiene al fascio f2 . Inoltre r ∩ r = P é un
punto reale.
Consideriamo quindi una conica non riducibile γ. Dal fascio f1 si possono con-
durre due rette r, t che siano tangenti a γ. Analogamente dal fascio f2 si possono
condurre due rette che siano tangenti a γ: esse sono esattamente le coniugate r e t
delle rette r, t.
Definizione 1. I fuochi di una conica non riducibile γ sono i 4 punti (di cui 2 reali)
che scaturiscono dall’intersezione delle rette r, t con le rette r e t. In particolare i
due fuochi reali sono F1 = r ∩ r e F2 = t ∩ t.
Osservazione 2. Sottolineamo che la parabola ha un solo fuoco reale, poiché da
ciascun punto ciclico si puó condurre una sola retta tangente alla curva.
Anche nel caso della circonferena vi é un solo fuoco, esattamente il centro della
circonferenza: il motivo risiede nel fatto che, poiché i punti ciclici appartengono
alla circonferenza, anche in questo caso da ciascuno di essi si puó condurre una
sola tangente alla curva.
Esempio 2. Determinare il fuoco della parabola γ : 2x2 − y + x + 1 = 0.

Svolg. Consideriamo il fascio di rette isotrope di centro C1 = (1, i, 0): f1 : y =


ix + q. Costruiamo il sistema che determini l’intersezione tra il fascio e la conica:
2x2 − y + x + 1 = 0

y = ix + q

2x2 − ix − q + x + 1 = 0

y = ix + q

2x2 + (1 − i)x + (1 − q) = 0

.
y = ix + q
Poiché siamo interessati alla retta del fascio che sia tangente alla parabola, imponi-
amo che il discriminante del sistema sia nullo:
(1 − i)2 − 8(1 − q) = 0
la cui soluzione é q = 41 i + 1. Quindi la retta del fascio f1 che sia tangente alla
parabola ha equazione y = ix + 14 i + 1. Inoltre la retta che appartenga al fascio
f2 di centro C2 = (1, −i, 0) e che sia tangente alla parabola, é la coniugata della
precedente : y = −ix − 41 i + 1. Il fuoco della parabola é l’intersezione di tali due
rette:
r : y = ix + 41 i + 1


r : y = −ix − 14 i + 1
da cui F = r ∩ r = (− 14 , 1).
Esempio 3. Determinare i fuochi dell’iperbole γ : xy + x − y + 1 = 0.
18 LE CONICHE.

Svolg. Consideriamo il fascio di rette isotrope di centro C1 = (1, i, 0): f1 : y =


ix + q. Costruiamo il sistema che determini l’intersezione tra il fascio e la conica:

xy + x − y + 1 = 0
y = ix + q

x(ix + q) + x − (ix + q) + 1 = 0
y = ix + q
 2
ix + (q − i + 1)x + (1 − q) = 0
.
y = ix + q
Poiché siamo interessati alle rette del fascio che siano tangenti all’iperbole, imponi-
amo che il discriminante del sistema sia nullo:
(q − i + 1)2 − 4i(1 − q) = 0
le cui soluzioni sono
√ √
q = −1 − i + 2 2i q = −1 − i − 2 2i.
Quindi le rette del fascio f1 che siano tangenti all’iperbole hanno equazioni:
√ √
y = ix − 1 − i + 2 2i y = ix − 1 − i − 2 2i.
√ p
Osserviamo che 2i = (1 + i)2 ∈ {+(1+i), −(1+i)} per cui possiamo considerare
le tangenti
r : y = ix − 1 − i + 2(1 + i) t : y = ix − 1 − i − 2(1 + i).
Inoltre le rette appartenenti al fascio f2 di centro C2 = (1, −i, 0) e che siano tangenti
all’iperbole, sono le coniugate delle precedenti :
r : y = −ix − 1 + i + 2(1 − i) t : y = −ix − 1 + i − 2(1 − i).
I fuochi sono le intersezioni di tali coppie di rette, in particolare i fuochi reali
scaturiscono da: 
r : y = ix − 1 − i + 2(1 + i)
r : y = −ix − 1 + i + 2(1 − i)
da cui F1 = r ∩ r = (−1, 1) e da

t : y = ix − 1 − i − 2(1 + i)
t : y = −ix − 1 + i − 2(1 − i)
da cui F2 = t ∩ t = (3, −3).

10. Esercizi.
Esercizio 8. Nel piano ampliato si determinino i punti base del fascio x2 + (λ −
1)y = 0 con la relativa molteplicitá .
Esercizio 9. Nel piano ampliato si determinino i punti base del fascio y 2 + (λ +
1)x = 0 con la relativa molteplicitá .
Esercizio 10. Nel piano ampliato si determinino le coniche degeneri del fascio
λxy + x2 + y = 0 con la relativa molteplicitá .
Esercizio 11. Nel piano ampliato si determinino le coniche degeneri del fascio
xy + λxy + 3 = 0 con la relativa molteplicitá .
Esercizio 12. Nel piano ampliato si determinino le coordinate del polo della retta
3x − 4y + 1 = 0 rispetto alla conica x2 + 2y 2 − 3xy + 2x − y = 0.
LE CONICHE. 19

Esercizio 13. Nel piano ampliato si determinino le coordinate del polo della retta
x + y = 0 rispetto alla conica 2x2 + y 2 − 1 = 0.
Esercizio 14. Nel piano ampliato si determinino le coordinate del polo della retta
2x − 3y + 2 = 0 rispetto alla conica x2 + 2y 2 − 3xy + 2x − y = 0.
Esercizio 15. Data la conica x2 + 2xy + 4y 2 − 4x − 10y + 4 = 0, determinare una
coppia di diametri coniugati, uno dei quali parallelo all’asse X.
Esercizio 16. Determinare l’equazione della circonferenza che passa per i punti
(4, −2), (2, 3) ed ha centro sulla retta x − y − 1 = 0.
Risposta : (x − 12 )2 + (y + 12 )2 = 29
2
Esercizio 17. Determinare l’equazione del cerchio di centro (6, 7) che sia tangente
alla retta 5x − 12y − 24 = 0.
Risposta : (x − 6)2 + (y − 7)2 = 36
Esercizio 18. Data la circonferenza x2 + y 2 + 2x + y + 1 = 0, determinare la
tangente nel punto (−1, 0).
Risposta : y = 0
Esercizio 19. L’equazione 3x2 − 5xy − 2y 2 + x + 5y − 2 = 0 rappresenta una conica
riducibile. Determinare le equazioni delle rette in cui essa si decompone.
Risposta : x − 2y + 1 = 0 e 3x + y − 2 = 0.
Esercizio 20. Data l’iperbole di equazione x2 −4xy +y 2 −2x−4y −1 = 0, di centro
(− 53 , − 43 ) e di assi 3y − 3x − 1 = 0 e y + x + 3 = 0, determinare le sue equazioni
ridotte. 2 2
2 2
Risposta : x10 − y10 = 1 e − x10 + y10 = 1.
3 9 9 3

Esercizio 21. Data la conica γ : 3xy − 4y 2 − 3x + 2y − 3 = 0, determinarne il


centro, eventuali asintoti, assi ed il vertice.
Esercizio 22. Determinare centro, assi, asintoti della conica: γ : 2x2 −y 2 +4x−1 =
0
Esercizio 23. Nel piano euclideo ampliato sia dato il fascio di coniche
4x2 + 4xy + ky 2 + 4x − 10y − 1 − k = 0.
Di tale fascio si determinino i punti base, le coniche riducibili, eventuali tangenti
comuni a tutte le coniche.
Esercizio 24. Date le coniche di equazione
x2 − y 2 + 2x − 2y − 2 = 0, x2 + 8y 2 + 2x + 16y + 3 = 0
determinare nel fascio da esse individuato:
1. le coniche degeneri;
2. la conica passante per il punto (1, 1);
3. le ellissi, le iperboli e le parabole in esso contenute.
Esercizio 25. Scrivere l’equazione del fascio di coniche che ha come punti base
(0, 0), (2, −1), (1, 2), (−3, −2).
Esercizio 26. Scrivere l’equazione del fascio di coniche che ha come punti base
(−1, −2), (3, −2), (1, −1, 0)
e tangenti in quest’ultimo punto alla retta x + y = 0.
20 LE CONICHE.

Esercizio 27. Scrivere l’equazione del fascio di coniche che ha come punti base
A(−1, −3), B(2, 5) e tangenti in A alla retta x + 1 = 0 ed in B alla retta y − 5 = 0.
Esercizio 28. Data la conica γ : x2 +2xy + y 2 − 4y + 6 = 0, determinarne il centro,
il fuoco, il vertice e l’asse (é una parabola).
Scrivere l’equazione della conica γ in forma ridotta.
Esercizio 29. Data la conica γ : 3x2 − 4xy + 16x − 8y − 60 = 0, determinarne il
centro, il vertice, gli assi e gli asintoti.
Scrivere l’equazione della conica γ in forma ridotta.
Esercizio 30. Determinare la tangente comune a tutte le coniche del fascio xy −
λ(x2 − y 2 − 2y) = 0
Esercizio 31. Determinare la tangente comune a tutte le coniche dela fascio xy −
λ(x − y 2 ) = 0
Esercizio 32. Determinare il polo della retta 7x−10y+5 = 0 nella polaritá rispetto
alla conica x2 + 2y 2 − 3xy + 2x − y = 0.
Esercizio 33. Nel piano ampliato determinare le coniche degeneri del fascio λxy +
y2 − λ = 0
Esercizio 34. Nel piano ampliato determinare le coordiante del polo della retta
7x − 9y + 1 = 0 nella polaritá rispetto alla conica x2 + 2y 2 − 3xy + 2x − y = 0.
Esercizio 35. Nel piano ampliato determinare le coniche degeneri del fascio λxy +
xy − 1 = 0
Esercizio 36. Nel piano ampliato determinare il polo della retta 4x − 5y = 0 nella
polaritá rispetto alla conica x2 + 2y 2 − 3xy + 2x − y = 0.
Esercizio 37. Nel piano ampliato quali sono le coniche degeneri del fascio xy +
λx2 − λy = 0
Esercizio 38. Determinare le forme ridotte dell’iperbole 2x2 − 3xy − 2y 2 + 5x = 0
Esercizio 39. Determinare le forme ridotte della parabola x2 − 2xy + y 2 + 2x = 0.
Esercizio 40. Studiare la seguente conica 2x2 − y + x + 1 = 0.
Esercizio 41. Studiare la conica 2x2 − y 2 + 4x − 1 = 0.
Esercizio 42. Determinare l’equazione ridotta dell’iperbole equilatera x2 − y 2 = 4
riferita ai suoi asintoti.
Esercizio 43. Determinare le equazioni ridotte dell’ellisse x2 −xy+y 2 −5x+7y+1 =
0 di centro (1, −3) ed assi x − y − 4 = 0, x + y + 2 = 0.
RETTE E PIANI NELLO SPAZIO AFFINE ED EUCLIDEO.

1. Equazioni di un piano. Sia π un piano nello spazio affine A3 . Per indi-


viduare π abbiamo bisogno di un punto P0 = (x0 , y0 , z0 ) ∈ π e di un vettore
v = (l, m, n) ∈ π. In particolare, se v1 = (l1 , m1 , n1 ) e v2 = (l2 , m2 , n2 ) sono
due vettori indipendenti del piano, essi generano ogni altro vettore del piano, cioé
v = tv1 + t0 v2 , con t, t0 parametri reali.
Sia ora P = (x, y, z) un generico punto del piano. Esso dipende allora da P0 e
da v, cioé OP = f (OP0 , v), da cui
 x = x0 + tl1 + t0 l2

y = y0 + tm1 + t0 m2
z = z0 + tn1 + t0 n2

che sono dette equazioni parametriche di un piano. Da queste otteniamo


 x − x0 = tl1 + t0 l2

y − y0 = tm1 + t0 m2
z − z0 = tn1 + t0 n2

cioé il vettore P P0 dipende linearmente dai vettori v1 , v2 , per cui



x − x0 y − y0 z − z0


l1 m1 n1 = 0
l2 m2 n2
svolgendo la quale si ha l’equazione affine del piano
ax + by + cz + d = 0.

Supponiamo ora di avere tre punti non allineati nello spazio P0 = (x0 , y0 , z0 ),
P1 = (x1 , y1 , z1 ), P2 = (x2 , y2 , z2 ). Essi individuano un piano, in particolare i
vettori P0 P1 e P0 P2 sono tra loro indipendenti ed hanno componenti
P0 P1 = (x1 − x0 , y1 − y0 , z1 − z0 )
P0 P2 = (x2 − x0 , y2 − y0 , z2 − z0 ).
Per ogni altro punto P = (x, y, z) del piano, il vettore P0 P = (x − x0 , y − y0 , z − z0 )
é dipendente dai precedenti due vettori, cioé

x − x0 y − y0 z − z0

x1 − x0 y1 − y0 z1 − z0 = 0

x2 − x0 y2 − y0 z2 − z0
che é equivalente alla

x y z 1


x0 y0 z0 1
=0

x1 y1 z1 1

x2 y2 z2 1

1
2 RETTE E PIANI NELLO SPAZIO AFFINE ED EUCLIDEO.

che ci riporta ancora alla forma affine dell’equazione del piano.

2. Reciproca posizione di due piani. Siano π : ax + by + cz + d = 0 e π 0 :


a0 x + b0 y + c0 z + d0 = 0 due piani. I punti in comune ai due piani sono le soluzioni
del sistema lineare 
ax + by + cz + d = 0
a0 x + b0 y + c0 z + d0 = 0
nelle incognite x, y, z. Le matrici associate al sistema sono
   
a b c a b c d
A= , C= .
a0 b0 c0 a0 b0 c0 d0
Se rango(A) = rango(C) = 1, allora i due piani sono coincidenti poiché ax + by +
cz + d = α(a0 x + b0 y + c0 z + d0 ), per un opportuno α ∈ lR.
Se rango(A) = rango(C) = 2, allora il sistema ammette ∞1 soluzioni, cioé i due
piani hanno ∞1 punti in comune e quindi sono incidenti in una retta.
Se rango(A) = 1 e rango(C) = 2, allora il sistema é incompatibile ed i due piani
non hanno punti in comune, cioé sono paralleli. Ció si verifica quando
a b c d
0
= 0 = 0 6= 0
a b c d

3. Fascio di piani. Siano π : ax + by + cz + d = 0 e π 0 : a0 x + b0 y + c0 z + d0 = 0


due piani distinti. La totalitá dei piani di equazione
λ(ax + by + cz + d = 0) + %(a0 x + b0 y + c0 z + d0 ) = 0
al variare dei parametri reali λ e %, é detta fascio di piani. Si possono verificare due
casi: π e π 0 sono incidenti in una retta, ed allora tutti i piani del fascio hanno in
comune quella retta, si parla di fascio proprio. Oppure π e π 0 sono tra loro paralleli,
ed allora tutti i piani del fascio sono tra loro paralleli, si parla di fascio improprio.
Supponiamo allora di avere un terzo piano π 00 : a00 x + b00 y + c00 z + d00 = 0 ed
analizziamo in quale casi esso appartiene al fascio individuato da π e π 0 . In pratica
si deve studiare il sistema

 ax + by + cz + d = 0
a0 x + b0 y + c0 z + d0 = 0
 00
a x + b00 y + c00 z + d00 = 0
nelle incognite x, y, z. Le matrici associate al sistema sono
   
a b c a b c d
A =  a0 b0 c0  , C =  a0 b0 c0 d0  .
a00 b00 c00 a00 b00 c00 d00
Se rango(A) = rango(C) = 2, allora il sistema ammette ∞1 soluzioni, cioé i
tre piani hanno ∞1 punti in comune e quindi sono incidenti in una retta: essi
appartengono ad un fascio proprio.
Se rango(A) = 1 e rango(C) = 2, allora il sistema é incompatibile ed i tre piani
non hanno punti in comune, cioé sono paralleli: essi appartengono ad un fascio
improprio.
RETTE E PIANI NELLO SPAZIO AFFINE ED EUCLIDEO. 3

4. Stella di piani. Siano π : ax + by + cz + d = 0, π 0 : a0 x + b0 y + c0 z + d0 = 0 e


π 00 : a00 x + b00 y + c00 z + d00 = 0 tre piani. Consideriamo il sistema

 ax + by + cz + d = 0
a0 x + b0 y + c0 z + d0 = 0
 00
a x + b00 y + c00 z + d00 = 0
nelle incognite x, y, z. Le matrici associate al sistema sono
   
a b c a b c d
A =  a0 b0 c0  , C =  a0 b0 c0 d0  .
a00 b00 c00 a00 b00 c00 d00
Abbiamo giá visto che:
se rango(A) = rango(C) = 1 allora i tre piani coincidono tra loro;
se rango(A) = rango(C) = 2 allora i tre piani formano un fascio proprio;
se rango(A) = 1 e rango(C) = 2 allora i tre piani formano un fascio improprio.
Poniamoci ora nel caso in cui i tre piani siano distinti e non appartengano ad
uno stesso fascio. Quindi dovremo avere necessariamente rango(C) = 3.
Se anche rango(A) = 3, allora il sistema ammette una sola soluzione, cioé i tre
piani hanno in comune un punto, si dice che essi formano una stella propria di piani.
Se rango(A) = 2, allora il sistema é incompatibile, i tre piani non hanno punti
in comune, in pratica uno dei tre piani é parallelo alla retta che gli altri due hanno
in comune: si dice che i tre piani formano una stella impropria.
In ogni caso, la totalitá dei piani che appartengono alla stella é data dall’equazione:

α(ax + by + cz + d = 0) + β(a0 x + b0 y + c0 z + d0 ) + γ(a00 x + b00 y + c00 z + d00 ) = 0


al variare dei parametri reali α, β e γ.
Supponiamo di avere un quarto piano π 000 : a000 x + b000 y + c000 z + d000 = 0. Esso
appartiene alla stella formata dai piani π, π 0 e π 00 (propria o impropria che sia) se
la matrice  
a b c d
 a0 0 0 0 
 00 b00 c00 d00 
 a b c d 
a000 b000 c000 d000
ha rango 3.

5. Equazioni di una retta. Definiamo retta nello spazio, l’insieme di tutti i punti
comuni a due piani non paralleli, quindi la rappresentiamo con le equazioni:

ax + by + cz + d = 0
.
a0 x + b0 y + c0 z + d0 = 0
Ogni retta dello spazio puó essere individuata da un suo punto P0 = (x0 , y0 , z0 ) e
da un vettore v = (l, m, n) ad essa parallelo. Quindi ogni punto P = (x, y, z) della
retta é tale che il vettore P0 P sia proporzionale al vettore v cioé P0 P = tv, per
un opportuno parametro t dipendente dalla scelta di P sulla retta. Al variare del
parametro t si ottengono tutti i punti della retta:

 x = x0 + tl
y = y0 + tm
z = z0 + tn

4 RETTE E PIANI NELLO SPAZIO AFFINE ED EUCLIDEO.

che sono dette equazioni parametriche della retta. La terna (l, m, n) rappresenta i
parametri direttori della retta. In particolare se si conoscono due punti della retta
P1 = (x1 , y1 , z1 ) e P2 = (x2 , y2 , z2 ), il vettore P1 P2 é parallelo alla retta e quindi
(l, m, n) = (x2 −x1 , y2 −y1 , z2 −z1 ) e l’equazione si puó ottenere nel modo seguente:

 x = x1 + t(x2 − x1 )
y = y1 + t(y2 − y1 )
z = z1 + t(z2 − z1 )

da cui
x − x1 y − y1 z − z1
t= = =
x2 − x1 y 2 − y1 z2 − z1
che é detta equazione a catena di una retta.
Due rette sono parallele se e solo se esse hanno i tre parametri direttori pro-
porzionali (in particolare identici).

6. Rette complanari. Siano date le due rette



ax + by + cz + d = 0
r:
a0 x + b0 y + c0 z + d0 = 0
di parametri direttori (l, m, n) e
 00
0 a x + b00 y + c00 z + d00 = 0
r :
a000 x + b000 y + c000 z + d000 = 0
di parametri direttori (l0 , m0 , n0 ).
Diremo che r e r0 sono complanari se esiste un piano che le contenga entrambe.
In tal caso, le due rette possono essere incidenti in un punto, cioé i quattro piani che
le formano appartengono ad un stella propria, oppure le due rette sono parallele,
cioé i quattro piani che le formano appartengono ad una stella impropria. Possiamo
quindi dedurre che due rette sono complanari se la matrice
 
a b c d
 a0 0 0 0 
 00 b00 c00 d00 
 a b c d 
a000 b000 c000 d000
ha rango ≤ 3, cioé quando

a b c d
a0 b0 c0 d0


= 0.
a00 b00 c00 d00


a000 b000 c000 d000

Analogamente possiamo determinare una ulteriore condizione di complanaritá tra


le due rette: siano scelti due punti P = (x, y, z) ∈ r e P 0 = (x0 , y 0 , z 0 ) ∈ r0 . Se le
due rette fossero complanari, i vettori (l, m, n), (l0 , m0 , n0 ) e (x − x0 , y − y 0 , z − z 0 )
sarebbero tra loro dipendenti cioé
x − x0 y − y 0 z − z 0


l m n = 0.
l0 m0 n0

Se due rette non sono complanari sono dette ”sghembe”.


RETTE E PIANI NELLO SPAZIO AFFINE ED EUCLIDEO. 5

7. Reciproca posizione tra una retta ed un piano. Siano dati il piano


π: ax + by + cz + d = 0
e la retta
π 0 : a0 x + b0 y + c0 z + d0 = 0

r: .
π 00 : a00 x + b00 y + c00 z + d00 = 0
Se i tre piani appartengono allo stesso fascio, allora la retta r appartiene al piano
π (essa é l’asse del fascio).
Se i tre piani appartengono ad una stella propria allora la retta ed il piano π
hanno un punto in comune (che é il centro della stella).
Se i tre piani appartengono ad una stella impropria allora la retta ed il piano
non hanno punti in comune, cioé la retta é parallela al piano.
Poniamoci in quest’ultimo caso, retta e piano siano paralleli. Siano P1 =
(x1 , y1 , z1 ) e P2 = (x2 , y2 , z2 ) due punti della retta. Allora il vettore (l, m, n) =
(x2 − x1 , y2 − y1 , z2 − z1 ) é parallelo a π. In altre parole esiste un piano σ :
ax + by + cz + div , parallelo a π, che contiene il vettore P1 P2 . Quindi le coordinate
dei punti P1 e P2 soddisfano all’equazione di σ:
ax2 + by2 + cz2 + div = 0

ax1 + by1 + cz1 + div = 0
e sottraendo le due equazioni otteniamo
a(x2 − x1 ) + b(y2 − y1 ) + c(z2 − z1 ) = 0
al + bm + cn = 0
che é la condizione di parallelismo tra una retta ed un piano.

8. Calcolo dei parametri direttori di una retta. Sia data la retta



π : ax + by + cz + d = 0
r:
π 0 : a0 x + b0 y + c0 z + d0 = 0
con parametri direttori (l, m, n). Essi sono le componenti di un vettore parallelo alla
retta. Tale vettore deve allora essere parallelo sia al piano π che a π 0 , ed applicando
la condizione di parallelismo otteniamo:

al + bm + cn = 0
.
a0 l + b0 m + c0 n = 0
Questo é un sistema omogeneo di rango 2, nelle tre incognite (l, m, n), le cui
soluzioni sono proporzionali ai minori
     
b c c a a b
, , .
b0 c0 c0 a0 a0 b0
Esercizio 1. Determinare l’equazione del piano contenente i punti (1, 1, 0), (1, 0, 1),
(3, −1, 0).

Svolg. Sia (x, y, z) il generico punto del piano. L’equazione in forma cartesiana si
ottiene allora da:
x y z 1

1 1 0 1
1 0 1 1 =0


3 −1 0 1
cioé x + y + z − 2 = 0.
6 RETTE E PIANI NELLO SPAZIO AFFINE ED EUCLIDEO.

Esercizio 2. Determinare l’equazione del piano contenente il punto (1, 1, 2) e par-


allelo ai vettori di componenti (−1, 2, 3) e (1, 2, −1).

Svolg. In forma parametrica l’equazione del piano é


 x = −t + t0 + 1

y = 2t + 2t0 + 1
z = 3t − t0 + 2

ed in forma cartesiana
x−1 y−1 z−2

−1 2 3 =0

1 2 −1
cioé 4x − y + 2z − 7 = 0.
Esercizio 3. Sia dato il fascio di piani (3x + y − 2z) + a(x − 4y + 2z − 1) = 0.
Verificare se il piano π : 2x + 5y − 4z + 2 = 0 appartiene o no al fascio.

Svolg. Le matrici associate al fascio sono


   
3 1 −2 3 1 −2 0
A= , C= .
1 −4 2 1 −4 2 −1
Hanno entrambe rango 2, quindi il fascio é proprio. Consideriamo ora le matrici
associate ai tre piani (aggiungiamo una riga con i coefficienti di π):
   
3 1 −2 3 1 −2 0
A0 =  1 −4 2  , C 0 =  1 −4 2 −1  .
2 5 −4 2 5 −4 2
La matrice A0 ha rango 2, ma la matrice C 0 ha rango 3, quindi il piano π non
appartiene al fascio dato. In particolare i tre piani formano una stella impropria.
Esercizio 4. Sia dato il fascio di piani (x + y − 3z + 2) + a(2x + 2y − 6z + 1) = 0.
Verificare se il piano π : 5x + 5y − 15z + 4 = 0 appartiene o no al fascio.

Svolg. Le matrici associate al fascio sono


   
1 1 −3 1 1 −3 2
A= , C= .
2 2 −6 2 2 −6 1
La matrice A ha rango 1, e la matrice C ha rango 2: é un fascio improprio. Consid-
eriamo ora le matrici associate ai tre piani (aggiungiamo una riga con i coefficienti
di π):
   
1 1 −3 1 1 −3 2
A0 =  2 2 −6  , C0 =  2 2 −6 1 .
5 5 −15 5 5 −15 4
La matrice A0 ha rango 1 e la matrice C 0 ha rango 2, quindi il piano π appartiene
al fascio dato.
Esercizio 5. Sia data la stella di piani (3x+y−2z)+a(x−4y+2z−1)+b(x−1) = 0.
Verificare se i piani π : 3x + 5y − 4z = 0 e σ : 2x + y − 1 = 0 appartengono o no
alla stella.
RETTE E PIANI NELLO SPAZIO AFFINE ED EUCLIDEO. 7

Svolg. Le matrici associate alla stella sono


   
3 1 −2 3 1 −2 0
A =  1 −4 2 , C =  1 −4 2 −1  .
1 0 0 1 0 0 −1
Hanno entrambe rango 3, quindi la stella é propria. Consideriamo ora le matrici
associate ai quattro piani (aggiungiamo una riga con i coefficienti di π):
   
3 1 −2 3 1 −2 0
 1 −4 2   1 −4 2 −1 
A0 =  , C0 =  .
 1 0 0   1 0 0 −1 
3 5 −4 3 5 −4 0
Le matrici A0 e C 0 hanno entrambe rango 3, quindi il piano π appartiene alla stella.
Consideriamo ora le matrici ottenute aggiungendo i coefficienti del piano σ:
   
3 1 −2 3 1 −2 0
 1 −4 2   1 −4 2 −1 
A00 =  , C 00 =  .
 1 0 0   1 0 0 −1 
2 1 0 2 1 0 −1
La matrice C 00 ha rango 4, quindi il piano σ non appartiene alla stella.
Esercizio 6. Sia data la stella di piani (2x+y+z)+a(x+y+z)+b(3x+2y+2z−1) =
0. Verificare se i piani π : 2x + 2y + 2z − 3 = 0 e σ : x + 2y − z = 0 appartengono
o no alla stella.

Svolg. Le matrici associate alla stella sono


   
2 1 1 2 1 1 0
A =  1 1 1 , C= 1 1 1 0 .
3 2 2 3 2 2 −1
La matrice A ha rango 2 e la matrice C ha rango 3, quindi la stella é impropria.
Consideriamo ora le matrici associate ai quattro piani (aggiungiamo una riga con i
coefficienti di π):
   
2 1 1 2 1 1 0
0
 1 1 1  0
 1 1 1 0 
A = , C = .
 3 2 2   3 2 2 −1 
2 2 2 2 2 2 −3
Le matrici A0 e C 0 hanno rispettivamente rango 2 e rango 3, quindi il piano π
appartiene alla stella.
Consideriamo ora le matrici ottenute aggiungendo i coefficienti del piano σ:
   
2 1 1 2 1 1 0
 1 1 1   1 1 1 0 
A00 =  3 2 2 ,
 C 00 = 
 3 2 2 −1  .

1 2 −1 1 2 −1 0
La matrice A00 ha rango 3, quindi il piano σ non appartiene alla stella impropria.
Esercizio 7. Determinare la retta passante per i punti (1, 0, 1) e (2, 3, 1).
8 RETTE E PIANI NELLO SPAZIO AFFINE ED EUCLIDEO.

Svolg. I parametri direttori della retta sono dati dalle differenze delle coordi-
nate omologhe dei due punti: (l, m, n) = (1, 3, 0), quindi una forma parametrica
dell’equazione della retta é: 
 x=t+1
y = 3t .
z=1

In forma cartesiana, eliminando il parametro t dalle precedenti, otteniamo:

3x − y − 3 = 0
.
z=1

Esercizio 8. Determinare la retta passante per il punto (2, 1, 1) e parallela alla


retta s : 2x + y − z = x + y − 2z + 1 = 0.

Svolg. I parametri direttori della retta s sono (−1, 3, 1), e quindi essi possono
essere considerati anche come i parametri direttori della retta da determinare.
Quindi una forma parametrica dell’equazione della retta é:

 x = −t + 2
y = 3t + 1 .
z =t+1

In forma cartesiana, eliminando il parametro t dalle precedenti, otteniamo:



x+z−3=0
.
3x + y − 7 = 0

Esercizio 9. Determinare se le seguenti rette sono complanari:


r: x − 3y + 2 = x + y + z − 1 = 0
s: x = −t + 2, y = 5t + 3, z = −t.

Svolg. Ps = (2, 3, 0) é un punto di s. Qr = (−2, 0, 3) é un punto di r. Il vettore


che li congiunge ha componenti (4, 3, −3). Inoltre i parametri direttori di r sono
(−3, −1, 4) e quelli di s sono (−1, 5, −1). Calcoliamo il determinante della matrice
 
4 3 −3
A =  −3 −1 4  .
−1 5 −1
Poiché det(A) 6= 0, le due rette non sono complanari (si dice che sono sghembe).
Esercizio 10. Determinare se le seguenti rette sono complanari:
r: x−y =z−1=0
s: x − y + 2z − 3 = 2x − 2y + 3z − 4 = 0

Svolg. Per verificare se sono complanari consideriamo la matrice formata dai co-
efficienti dei quattro piani che le compongono:
 
1 −1 0 0
 0 0 1 −1 
A=  1 −1 2 3  .

2 −2 3 −4
RETTE E PIANI NELLO SPAZIO AFFINE ED EUCLIDEO. 9

Poiché det(A) = 0, le due rette sono complanari. Calcoliamo allora il piano π che
la contiene entrambe: determiniamo dapprima il fascio di piani che abbia come
sostegno la retta s:
F : (x − y + 2z − 3) + a(2x − 2y + 3z − 4) = 0.
Il piano π appartiene a tale fascio. Inoltre π deve contenere ogni punto di r.
Scegliamo P = (0, 0, 1) ∈ r (la scelta di tale punto deve essere tale da essere certi
che P non sia il punto comune alle due rette). Imponiamo ora che il generico piano
del fascio contenga P :
2 − 3 + a(3 − 4) = 0 → a = −1
quindi
π: x − y + z − 1 = 0.
t
u
Esercizio 11. Si verifichi che il piano π : 2x − y − 4z + 2 = 0 e la retta
r: x = t, y = 1 − 2t, z=t
sono paralleli.

Svolg. I parametri direttori della retta sono (l, m, n) = (1, −2, 1) ed i coefficienti
del piano sono (a, b, c) = (2, −1, −4), per cui é verificata la condizione al+bm+cn =
0.
Esercizio 12. Determinare il punto comune alla retta
r: 2x − y + 1 = x + y − z = 0
ed al piano π : 3x − y + 2z = 0.

Svolg. I parametri direttori di r sono (1, 2, 3) ed un suo punto é P0 = (0, 1, 1).


Una forma parametrica della retta é allora:

 x=t
y = 2t + 1 .
z = 3t + 1

Sostituiamo il generico punto di r, che ha coordinate (t, 2t + 1, 3t), nell’equazione


del piano. Otterremo il valore del parametro t per il quale il punto appartiene sia
alla retta che al piano:
1
3t − (2t + 1) + 2(3t + 1) = 0 → t = −
7
quindi il punto comune a retta e piano é dato da
 x = − 17

y = 57 .
z = 47

Esercizio 13. Determinare il piano passante per il punto (2, 1, 1) e parallelo al


piano π : 2x − y + 3z + 5 = 0.

Svolg. Il generico piano parallelo a π ha equazione


2x − y + 3z + k = 0
10 RETTE E PIANI NELLO SPAZIO AFFINE ED EUCLIDEO.

al variare di k ∈ lR si ottengono tutti i piani paralleli a π. Imponiamo il passaggio


per il punto (2, 1, 1): → k = −6, cioé l’equazione del piano richiesto é 2x − y +
3z − 6 = 0.

9. Coordinate omogenee nello spazio. Sia P = (x, y, z) un punto nello spazio.


Diremo che (x1 , x2 , x3 , x4 ) sono le coordinate omogenee di P se
x1 x2 x3
x= , y= , z= .
x4 x4 x4
Se x4 6= 0, il punto é detto proprio e puó essere individuato dalla quaterna di
coordinate omogenee (x, y, z, 1).
In caso x4 = 0, il punto é detto improprio. L’insieme di tutti i punti impropri
dello spazio é individuato dall’equazione x4 = 0, che rappresenta un piano, il piano
improprio.
Tutte le quaterne di coordinate omogenee che siano tra loro proporzionali, indi-
viduano il medesimo punto.
L’equazione di un piano in coordinate omogenee é
π: ax1 + bx2 + cx3 + dx4 = 0
ed i suoi punti impropri si ottengono intersecandolo col piano improprio:

ax1 + bx2 + cx3 + dx4 = 0
x4 = 0
Questa é una retta, detta retta impropria (o giacitura) di π. Tutti i piani tra loro
paralleli hanno la stessa giacitura.

L’equazione di una retta in coordinate omogenee é data dall’intersezione dei due


piani 
ax1 + bx2 + cx3 + dx4 = 0
r:
a0 x1 + b0 x2 + c0 x3 + d0 x4 = 0
I punti impropri di r si ottengono intersecandola col piano improprio:

 ax1 + bx2 + cx3 + dx4 = 0
a0 x1 + b0 x2 + c0 x3 + d0 x4 = 0
x4 = 0

Quindi ogni retta contiene un solo punto improprio (l, m, n, 0), dove (l, m, n) sono
esattamente i parametri direttori di r. Per cui rette tra loro parallele hanno lo
stesso punto improprio.
Se una retta ed un piano sono paralleli, allora il punto improprio della retta
appartiene alla giacitura del piano.

10. Angoli nello spazio euclideo. Fissiamo nello spazio euclideo un riferimento
cartesiano ortogonale OXY Z di centro O e versori i, j, k, rispettivamente per gli
assi X, Y, Z.
Chiameremo coseni direttori di una retta r, i coseni degli angoli che la retta forma
con gli assi coordinati. Se la retta é individuata dai parametri direttori (l, m, n), i
suoi coseni direttori saranno:
l l
α = cos(r, X) ∈ {+ √ , −√ }
2
l +m +n2 2 l + m2 + n2
2

m m
β = cos(r, Y ) ∈ {+ √ , −√ }
2
l +m +n 2 2 l + m2 + n2
2
RETTE E PIANI NELLO SPAZIO AFFINE ED EUCLIDEO. 11

n n
γ = cos(r, Z) ∈ {+ √ , −√ }
l2 2
+m +n 2 l + m2 + n2
2

Consideriamo ora due rette ed individuiamole tramite i rispettivi parametri di-


rettori: r = (l, m, n) e r0 = (l0 , m0 , n0 ). Indichiamo con v e v 0 due vettori paralleli
rispettivamente a r e r0 , uno di componenti (l, m, n) e l’altro (l0 , m0 , n0 ). L’angolo
tra le due rette é lo stesso formato dai due vettori:
ll0 + mm0 + nn0
cos(r, r0 ) = cos(v, v 0 ) = ± √ √ .
l2 + m2 + n2 · l02 + m02 + n02
Quindi le due rette sono ortogonali se ll0 + mm0 + nn0 = 0.

Sia π : ax + by + cz + d = 0 un piano. Un vettore v di componenti (vx , vy , vz )


é parallelo al piano se avx + bvy + cvz = 0. Ció vuol dire anche che il vettore
v é ortogonale ad ogni vettore w di componenti proporzionali alla terna (a, b, c).
In particolare questo implica che ogni vettore w = %(a, b, c) é ortogonale anche al
piano ax + by + cz + d = 0. Diciamo allora che una retta é ortogonale al piano π
se i suoi coseni direttori sono
a a
α = cos(r, X) ∈ {+ √ , −√ }
a2 + b2 + c2 a2 + b2 + c2

b b
β = cos(r, Y ) ∈ {+ √ , −√ }
a2 + b2 + c2 a2 + b2 + c2
c c
γ = cos(r, Z) ∈ {+ √ , −√ }.
a2 + b2 + c2 a2 + b2 + c2

Siano ora π : ax + by + cz + d = 0 e π 0 : a0 x + b0 y + c0 z + d0 = 0 due piani distinti.


L’angolo formato dai due piani é uguale a quello formato dai due versori normali
ai due piani:
aa0 + bb0 + cc0 aa0 + bb0 + cc0
cos(π, π 0 ) ∈ {+ √ √ , −√ √ }.
a2 + b2 + c2 · a02 + b02 + c02 a2 + b2 + c2 · a02 + b02 + c02
Da ció concludiamo anche che i due piani sono ortogonali se aa0 + bb0 + cc0 = 0.

Consideriamo infine una retta r di parametri direttori (l, m, n) ed un piano π :


ax + by + cz + d = 0. Il versore normale al piano é
a b c
n = (√ ,√ ,√ ).
a2 2
+b +c2 2 2
a +b +c2 a + b2 + c2
2

π
Sia ϕ = (r, n) l’angolo formato dalla retta r e dal versore n. Definiamo 2 −ϕ
l’angolo formato dalla retta r e dal piano π. Quindi segue che:
al + bm + cn
sen(r, π) = cos(r, n) = ± √ √
a2 + + c2 · l2 + m2 + n2
b2
da cui otteniamo che la retta ed il piano sono paralleli se al + bm + cn = 0.
12 RETTE E PIANI NELLO SPAZIO AFFINE ED EUCLIDEO.

11. Distanze nello spazio euclideo. Siano P1 = (x1 , y1 , z1 ) e P2 = (x2 , y2 , z2 )


due punti dello spazio. La distanza tra i punti P1 e P2 é il modulo del vettore P1 P2 :
p
δ(P1 , P2 ) = (x2 − x1 )2 + (y2 − y1 )2 + (z2 − z1 )2 .

Consideriamo ora il punto P0 = (x0 , y0 , z0 ) ed il piano π : ax + by + cz + d = 0.


Ci proponiamo di calcolare la distanza del punto dal piano. Se P0 ∈ π chiaramente
diremo che essa é nulla. Poniamoci allora nel caso in cui P0 ∈ / π. La distanza di
P0 da π é pari alla lunghezza δ(P0 H) del segmento P0 H, dove H é la proiezione
ortogonale di P0 su π. Scegliamo un qualsiasi punto Q1 = (x1 , y1 , z1 ) ∈ π. Allora
δ(P0 H) é la proiezione ortogonale del vettore P0 Q1 lungo la direzione del vettore
P0 H. Per determinare tale proiezione abbiamo bisogno del versore u di P0 H, che
é il versore normale al piano π:
a b c
u = (√ ,√ ,√ ).
a2 2
+b +c2 2 2
a +b +c2 a + b2 + c2
2

Quindi:
a b c
δ(P0 H) = |(P0 Q1 ) × ( √ i+ √ j+√ k)| =
a2 + b2 + c2 a2 + b2 + c2 a2 + b2 + c2
a b c
|((x1 −x0 )i+(y1 −y0 )j+(z1 −z0 )k)×( √ i+ √ j+ √ k)| =
a2 2
+b +c2 2 2
a +b +c2 a + b2 + c2
2

|ax1 − ax0 + by1 − by0 + cz1 − cz0 |


√ .
a2 + b2 + c2
Dal fatto che Q1 ∈ π segue che ax1 + by1 + cz1 = −d, per cui
|ax0 + by0 + cz0 + d|
δ(P0 H) = √ .
a2 + b2 + c2

Scegliamo ora una retta r di parametri direttori (l, m, n) ed il punto P1 =


(x1 , y1 , z1 ) non appartenente alla retta. Costruiamo il piano π ortogonale alla
retta e passante per P1 : tale piano é unico. Indichiamo con Q1 = π ∩ r, esso é
la proiezione ortogonale di P1 su r. La distanza del punto P1 dalla retta r é la
distanza δ(P1 , Q1 ).
Esiste comunque una formula che consente di calcolare direttamente la distanza
tra il punto P1 e la retta r. Indichiamo come sopra Q1 il punto che sia la proiezione
di P1 su r. Scegliamo arbitrariamente un punto Q0 = (x0 , y0 , z0 ) ∈ r. Il segmento
h = P1 Q1 é l’altezza del trapezio avente come lati l1 = Q1 Q0 e l2 = P1 Q0 , inoltre
h é l’altezza relativa al lato l1 . Per cui h = lA1 , dove A é l’area del trapezio. Quindi:
|Q1 Q0 ∧ P1 Q0 |
δ(P1 , r) = h = .
|Q1 Q0 |
In particolare, pur supponendo di non conoscere le coordinate di Q1 , é certo che
esiste un opportuno α ∈ lR, tale che Q1 Q0 = α(l, m, n), da cui


i j k

Q1 Q0 ∧ P1 Q0 = αl αm αn .

(x1 − x0 ) (y1 − y0 ) (z1 − z0 )
RETTE E PIANI NELLO SPAZIO AFFINE ED EUCLIDEO. 13

Allora il modulo del vettore Q1 Q0 ∧ P1 Q0 é


s
x − x0 y1 − y0 2 x1 − x0 z1 − z0 2 y1 − y0
2
z1 − z0
α 1 + +
l m l n m n
p
ed il modulo di Q1 Q0 é α l2 + m2 + n2 ). Concludiamo che
|Q1 Q0 ∧ P1 Q0 |
δ(P1 , r) = h = =
|Q1 Q0 |
v 2 2 2
u
u x1 − x0 y1 − y0 x1 − x0 z1 − z0 y − y0 z1 − z0
+ 1

u +
t l m l n m n
.
l 2 + m2 + n 2

Siano ora π : ax + by + cz + d1 = 0 e π 0 : ax + by + cz + d2 = 0 due piani paralleli.


La distanza tra di essi é pari alla distanza di un qualsiasi punto P0 = (x0 , y0 , z0 ) di
π dall’altro piano π 0 :
|ax0 + by0 + cz0 + d2 |
δ(π, π 0 ) = √ =
a2 + b2 + c2
|d2 − d1 |

a2 + b2 + c2
poiché ax0 + by0 + cz0 = −d1 .

Infine siano r = (l, m, n) e r0 = (l0 , m0 , n0 ) due rette sghembe. Esisteranno quindi


due piani π e π 0 tra loro paralleli tali che r ∈ π e r0 ∈ π 0 . La distanza tra r e r0 é
pari alla distanza tra π e π 0 .
Un altro metodo per calcolare la distanza tra le rette sghembe r e r0 é quello di
sfruttare la proprietá per la quale esiste una ed una sola retta s perpendicolare ad
entrambe r e r0 e con esse incidente. Dopo aver individuato la retta s, si costruiscono
i punti P = r ∩ s e P 0 = r0 ∩ s. La distanza tra le due rette sghembe é pari alla
distanza tra P e P 0 . Esponiamo nei particolari questo procedimento: Siano scelti i
punti Q = (x0 , y0 , z0 ) ∈ r e Q0 = (x00 , y00 , z00 ):
 x = x00 + t0 l0
 
 x = x0 + tl
0
r: y = y0 + tm , r : y = y00 + t0 m0 .
z = z0 + tn z = z00 + t0 n0
 

I generici punti P ∈ r e P 0 ∈ r0 sulle due rette hanno coordinate


P = (lt + x0 , mt + y0 , nt + z0 ), P 0 = (l0 t0 + x00 , m0 t0 + y00 , n0 t0 + z00 )
ed il vettore w che li congiunge ha componenti
w = (lt − l0 t0 + x0 − x00 , mt − m0 t0 + y0 − y00 , nt + n0 t0 + z0 − z00 ).
Al variare dei parametri t e t0 dobbiamo determinare i punti P e P 0 tali che il
vettore w sia ortogonale sia con r che con r0 , cioé devono annullarsi i seguenti
prodotti scalari:
w×(l, m, n) = (lt−l0 t0 +x0 −x00 )·l+(mt−m0 t0 +y0 −y00 )·m+(nt+n0 t0 +z0 −z00 )·n = 0
w×(l0 , m0 , n0 ) = (lt−l0 t0 +x0 −x00 )·l0 +(mt−m0 t0 +y0 −y00 )·m0 +(nt+n0 t0 +z0 −z00 )·n0 = 0.
Le soluzioni t, t0 , del precedente sistema, permettono di individuare i punti P, P 0
sulle rette r e r0 .
14 RETTE E PIANI NELLO SPAZIO AFFINE ED EUCLIDEO.

Esercizio 14. Determinare il piano contenente la retta di equazioni r : 3x+6y −


z − 2 = x + y − 1 = 0 e ortogonale alla retta s : x = t + 2, y = 4t, z = −t + 1.

Svolg. Determiniamo il fascio di piani contenente r:


3x + 6y − z − 2 + h(x + y − 1) = 0
(3 + h)x + (6 + h)y − z − 2 − h = 0
ed imponiamo la ortogonalitá con s:
3 + h = 1, 6+h=4
da cui h = −2 e quindi il piano é
x + 4y − z = 0.
Esercizio 15. Determiniamo il piano contenente i punti (1, 0, 1), (−1, 1, 1) e par-
allelo alla retta r : 2x + y − 1 = x − y − 2 = 0.

Svolg. I parametri direttori della retta r sono (0, 0, 1), per cui il piano richiesto
contiene i tre punti in coordinate omogenee (1, 0, 1, 1), (−1, 1, 1, 1), (0, 0, 1, 0):

x1 x2 x3 x4

1 0 1 1
π : =0
−1 1 1 1
0 0 1 0
π: x1 + 2x2 − x4 = 0
x + 2y − 1 = 0.
Esercizio 16. Determinare la distanza tra il punto P = (0, 2, 1) ed il piano π di
equazione 3x + 2y − z + 2 = 0.

Svolg.
|3 · 0 + 2 · 2 + (−1) · 1 + 2| 5
δ(P, π) = √ =√ .
9+4+1 14
Esercizio 17. Determinare la distanza tra i due piani π : 2x + y + 2z − 1 = 0 e
π 0 : 4x + 2y + 4z + 7 = 0.

Svolg. Riscriviamo π 0 : 2x + y + 2z + 7
2 = 0.
| 7 − (−1)| 9
3
δ(π, π 0 ) = √2 = √2 = .
4+1+4 9 2
Esercizio 18. Determinare la distanza tra le rette 2x + 3y − 2 = x − 1 = 0 e
y = x − z + 1 = 0.

Svolg. Verifichiamo se le rette sono complanari o sghembe:



2 3 0 −2

1 0 0 −1
=0
0 1 0 0

1 0 −1 1
RETTE E PIANI NELLO SPAZIO AFFINE ED EUCLIDEO. 15

quindi le rette sono complanari. Inoltre le terne di parametri direttori delle rette
sono
(0, 0, 1) (−1, 0, −1)
per cui le rette sono incidenti e non parallele e la loro distanza é nulla.
Esercizio 19. Determinare la distanza tra le rette r : 2x + 3y − 1 = x − 1 = 0 e
s : y = x − z + 1 = 0.

Svolg. Verifichiamo se le rette sono complanari o sghembe:



2 3 0 −1

1 0 0 −1

0 1 0
=6 0
0
1 0 −1 1
quindi le rette sono sghembe. Inoltre le terne di parametri direttori delle rette sono
r = (0, 0, 1) s = (1, 0, 1).
Costruiamo il fascio di piani che ha come sostegno r
Fr : 2x + 3y − 1 + h(x − 1) = 0
Fr : (2 + h)x + 3y − 1 − h = 0
Il piano πr di Fr parallelo alla retta s deve contenere il punto improprio di s, s∞ =
(1, 0, 1, 0). Imponiamo il passaggio per s∞ : 2+h = 0 ed il piano é πr : 3y +1 = 0.
Analogamente costruiamo il fascio di piani che ha come sostegno s
Fs : y + k(x − z − 1) = 0
Fs : kx + y − kz − k = 0
Il piano πs di Fs parallelo alla retta r deve contenere il punto improprio di r,
r∞ = (0, 0, 1, 0). Imponiamo il passaggio per r∞ : k = 0 ed il piano é πs : y = 0.
Allora
1
δ(r, s) = δ(πr , πs ) = .
3
Calcoliamo ora la retta di minima distanza, cioé quella ortogonale ed incidente ad
entrambe le rette date:
In coordinate parametriche il generico punto di r é Pr = (1, − 13 , t) e quello di s é
Qs = (t0 , 0, t0 + 1). Il segmento Pr Qs ha componenti (t0 − 1, 13 , t0 − t + 1). Inponiamo
che tale vettore sia ortogonale ad entrambe le rette:
ortogonalitá con r : t0 − t + 1 = 0
ortogonalitá con s : t0 − 1 + t0 − t + 1 = 0
da cui t0 = 1 e t = 2.
Quindi i punti contenuti nella retta di minima distanza sono
1
Pr = (1, , 2) Qs = (1, 0, 2)
3
e la retta di minima distanza é x − 1 = y − 2 = 0.
Esercizio 20. Determinare la distanza tra il punto P = (1, 1, 3) e la retta r di
equazioni x = 2t, y = 1 − 2t, z = t.
16 RETTE E PIANI NELLO SPAZIO AFFINE ED EUCLIDEO.

Svolg. Scegliamo Q = (0, 1, 0) sulla retta r. Il segmento P Q ha componenti


(1, 0, 3). Consideriamo la matrice
   
1 0 3 1 0 3
A= = .
l m n 2 −2 1
La distanza tra punto P e la retta r é
p
A2 + A212 + A213
δ(P, r) = √11
l2 + m2 + n2
in cui Aij sono i minori di ordine 2 di A.
√ √
4 + 25 + 36 65
δ(P, r) = √ = .
4+4+1 3

Un altro metodo potrebbe essere il seguente:


Determiniamo la stella di piani passante per P :
a(x − 1) + b(y − 1) + c(z − 3) = 0
e tra tutti questi piani scegliamo l’unico ortogonale alla retta r, cioé a = 2, b = −2,
c = 1:
π : 2x − 2y + z − 3 = 0.
Il punto di intersezione tra r e π é Q = ( 10 1 5
9 , − 9 , 9 ). La distanza tra r e P é pari
alla distanza tra P e Q:
r √
1 100 484 65
δ(P, r) = δ(P, Q) = + + = .
81 81 81 3

Esercizio 21. Determinare la distanza tra le rette r : x − 2z + 1 = y − 3z = 0 e


s : x − z − 2 = y − 2z + 3 = 0.

Svolg. Verifichiamo se le rette sono complanari o sghembe:



1 0 −2 −1

0 1 −3 0
1 0 −1 −2 6= 0


0 1 −2 3
quindi le rette sono sghembe. Inoltre la terna di parametri direttori della retta s é
(1, 2, 1). Costruiamo il fascio di piani che ha come sostegno r
Fr : x − 2z + 1 + h(y − 3z) = 0
Fr : x + hy + (−2 − 3h)z + 1 = 0.
Il piano πr di Fr parallelo alla retta s deve contenere il punto improprio di s,
s∞ = (1, 2, 1, 0). Imponiamo il passaggio per s∞ : 1 + h = 0 ed il piano é πr :
x − y + z + 1 = 0.
Scegliamo ora un punto Q a piacere su s: Q = (2, −3, 0). La distanza tra s e r
é pari alla distanza tra Q e πr :
|2 + 3 + 1| 6 √
δ(r, s) = δ(Q, πr ) = √ = √ = 2 3.
3 3
Calcoliamo ora la retta di minima distanza, cioé quella ortogonale ed incidente
ad entrambe le rette date:
RETTE E PIANI NELLO SPAZIO AFFINE ED EUCLIDEO. 17

In coordinate parametriche il generico punto di r é Pr = (2t − 1, 3t, t) e quello


di s é Qs = (t0 + 2, 2t0 − 3, t0 ). Il segmento Pr Qs ha componenti (2t − t0 − 3, 3t −
2t0 + 3, t − t0 ). Inponiamo che tale vettore sia ortogonale ad entrambe le rette:

ortogonalitá con r : 14t − 9t0 + 3 = 0

ortogonalitá con s : 9t − 6t0 + 3 = 0


da cui t = 3 e t0 = 5.
Quindi i punti contenuti nella retta di minima distanza sono

Pr = (5, 9, 3) Qs = (7, 7, 5)

e la retta di minima distanza é



 x=t+5
y = −t + 9 .
z =t+3

Esercizio 22. Determinare il piano passante per i punti P = (1, 0, 1), Q =


(−1, 1, 1) e parallelo alla retta r : 2x + y − 1 = x − y − 2 = 0.

Svolg. I parametri direttori della retta r sono (0, 0, 1) e quelli della retta contenente
il segmento P Q sono (2, −1, 0). Il piano richiesto é quello contenente i vettori
(0, 0, 1), (2, −1, 0) e passante per il punto P (oppure Q):

x−1 y z−1

0 0 1 = 0

2 −1 0
oppure analogamente é il piano contenente i tre punti, in coordinate omogenee,
(0, 0, 1, 0), (2, −1, 0, 0), (1, 0, 1, 1):

x1 x2 x3 x4

0 0 1 0
=0
2 −1 0 0

1 0 1 1

cioé x1 + 2x2 − x4 = 0, o equivalentemente x + 2y − 1 = 0. t


u

Esercizio 23. Determinare il piano passante per il punto P = (0, 1, 1) e parallelo


alle rette r : x = 3t + 1, y = t − 2, z = 3 e s : x − 2z = y − 3z + 1 = 0.

Svolg. I parametri direttori delle due rette sono:

r : (3, 1, 0) s : (2, 3, 1).

Il piano richiesto é quello contenente i vettori (3, 1, 0), (2, 3, 1) e passante per il
punto P :

x y−1 z−1

3 1 0 = 0

2 3 1
18 RETTE E PIANI NELLO SPAZIO AFFINE ED EUCLIDEO.

oppure analogamente é il piano contenente i tre punti, in coordinate omogenee,


(3, 1, 0, 0), (2, 3, 1, 0), (0, 1, 1, 1):

x1 x2 x3 x4

3 1 0 0
=0
2 3 1 0

0 1 1 1
cioé x1 − 3x2 + 7x3 − 4x4 = 0, o equivalentemente x − 3y + 7z − 4 = 0. t
u
Esercizio 24. Determinare la retta contenente il punto P = (0, 0, 3) e parallela
alla retta r : x = 1 − t, y = 2, z = 1 + 2t.

Svolg. I parametri direttori della retta sono identici a quelli della retta r, quindi
la sua equazione in forma parametrica é data da:

 x = −t
y=0
z = 2t + 3

ed in forma cartesiana, eliminando t dalle precedenti equazioni:



y=0
.
2x + z − 3 = 0
t
u
Esercizio 25. Determinare la retta passante per i punti P = (1, 1, 0, 0) e Q =
(0, 0, 1, 0).

Svolg. Sia (x1 , x2 , x3 , x4 ) il generico punto appartenente alla retta richiesta. La


condizione di allineamento di tale punto con i punti P e Q é che la matrice
 
x1 x2 x3 x4
 1 1 0 0 
0 0 1 0
abbia rango 2, cioé

x1 x2 x3

1
1 0 =0
x1 − x2 = 0.
0 0 1
Inoltre la retta certamente appartiene al piano improprio x4 = 0, quindi una sua
espressione cartesiana é la seguente:

x4 = 0
.
x1 − x2 = 0
Esercizio 26. Determinare la retta s passante per il punto P = (−1, 2, 3) e tale
da essere complanare ed ortogonale alla retta r : 2x + y = 2x − z − 1 = 0.

Svolg. Determiniamo la retta in forma cartesiana. Il primo piano é quello apparte-


nente al fascio di asse r e passante per P :
Fr : 2x + y + h(2x − z − 1) = 0
ed imponendo il passaggio per P si ottiene h = 0, quindi il primo piano é 2x+y = 0.
RETTE E PIANI NELLO SPAZIO AFFINE ED EUCLIDEO. 19

Il secondo piano é quello appartenente alla stella di centro P e ortogonale alla


retta r:
SP : a(x + 1) + b(y − 2) + c(z − 3) = 0
ed imponendo l’ortogonalitá con r, i cui parametri direttori sono (−1, 2, −1), si
ottiene a = −1, b = 2, c = −1, quindi il secondo piano é x − 2y + 2z − 1 = 0. La
retta s é allora: 
2x + y = 0
.
x − 2y + 2z − 1 = 0

12. Elementi complessi nello spazio. Ogni punto dello spazio é individuato da
una quaterna di coordinate omogenee (x1 , x2 , x3 , x4 ) non tutte nulle. Due qualsiasi
quaterne tra loro proporzionali rappresentano lo stesso punto nello spazio.
Un punto é detto reale se tra le infinite quaterne proporzionali che lo rappresen-
tano, ce ne é almeno una composta da soli numeri reali.
Analogamente sia π : ax1 + bx2 + cx3 + dx4 = 0 l’equazione di un piano in
coordinate omogenee. Il piano é detto reale se esiste una sua rappresentazione
π : α(ax1 + bx2 + cx3 + dx4 ) = 0, α ∈ lR
in modo tale che αa, αb, αc, αd ∈ lR.
Consideriamo ora la retta

ax1 + bx2 + cx3 + dx4 = 0
r:
a0 x1 + b0 x2 + c0 x3 + d0 x4 = 0
Essa é reale se tra gli infiniti piani del fascio
α(ax1 + bx2 + cx3 + dx4 ) + β(a0 x1 + b0 x2 + c0 x3 + d0 x4 ) = 0
ne esiste almeno uno che sia reale.

Se P é un punto non reale, esiste una sola retta reale che lo contiene: é la retta
congiungente P con il punto ad esso coniugato.
Se π é un piano non reale, esso contiene una sola retta reale: tale retta scaturisce
dall’intersezione di π con il piano ad esso coniugato.
Se r é una retta non reale, essa contiene al piú un punto reale: tale punto é
l’intersezione di r con la retta ad essa coniugata. Si noti che non é detto che una
retta immaginaria contenga un punto reale. Infatti essa potrebbe essere sghemba
con la propria coniugata e non avere con questa alcun punto di intersezione.
Esercizio 27. Quanti punti reali possiede la retta r : ix − y − i + 2 = x + 2y −
iz − 2 − i = 0?

Svolg. La retta coniugata a r é data dalle equazioni r : −ix − y + i + 2 =


x + 2y + iz − 2 + i = 0. Verifichiamo se r e r sono complanari o sghembe:

i −1 0 2 − i

1 2 −i −2 − i

−i −1 0 6= 0
2 + i
1 2 i −2 + i
quindi le due rette sono sghembe, non hanno punti in comune ed allora non con-
tengono alcun punto reale.
Esercizio 28. Quanti punti reali possiede la retta r : (1+i)x−y −i = iy −1 = 0?
20 RETTE E PIANI NELLO SPAZIO AFFINE ED EUCLIDEO.

Svolg. La retta coniugata a r é data dalle equazioni r : (−i − 1)x − y + i =


−iy − 1 = 0. Verifichiamo se r e r sono complanari o sghembe:

1 + i 1 0 −i

0 i 0 −1
1−i 1 0 i =0


0 −i 0 −1
quindi le due rette sono complanari. Inoltre la matrice
 
1+i 1 0 −i
 0
 i 0 −1 

 1−i 1 0 i 
0 −i 0 −1
ha rango 3, quindi le due rette hanno un punto in comune (i 4 piani che le individ-
uano appartengono ad una stella), cioé la retta r possiede un punto reale.
Esercizio 29. Quanti punti reali possiede la retta r : x + y − i = (1 + i)x + (1 +
i)y − 1 + i = 0?

Svolg. La retta coniugata a r é data dalle equazioni r : x + y + i = (1 − i)x +


(1 − i)y − 1 − i = 0. Verifichiamo se r e r sono complanari o sghembe:

1
1 0 −i
1 + i 1 + i 0 −1 + i
=0
1 1 0 i

1 − i 1 − i 0 −1 − i
quindi le due rette sono complanari. Inoltre la matrice
 
1 1 0 −i
 1 + i 1 + i 0 −1 + i 
 
 1 1 0 i 
1 − i 1 − i 0 −1 − i
ha rango 2, quindi le due rette hanno infiniti punti in comune (i 4 piani che le
individuano appartengono ad un fascio), cioé la retta r é tutta reale.
Esercizio 30. Quanti punti reali possiede la retta r : x−iy −i = ix+2y + 2 = 0?

Svolg. La retta coniugata a r é data dalle equazioni r : x+iy+i = −ix+2y+2 =


0. Verifichiamo se r e r sono complanari o sghembe:

1 −i 0 −i

i 2 0 2

1 =0
i 0 i
−i 2 0 2
quindi le due rette sono complanari. Inoltre la matrice
 
1 −i 0 −i
 i 2 0 2 
 
 1 i 0 i 
−i 2 0 2
ha rango 2, quindi le due rette infiniti punti in comune (i 4 piani che le individuano
appartengono ad un fascio), cioé la retta r é tutta reale.
RETTE E PIANI NELLO SPAZIO AFFINE ED EUCLIDEO. 21

I parametri direttori di r sono (0, 0, 1) ed un suo punto é P = (0, −1, 0). Allora
una sua rappresentazione reale é data da

 x=0
y = −1
z=t

ed in forma cartesiana: 
x=0
.
y = −1

Esercizio 31. Determinare la retta passante per i punti P = (1, 1 − i, 0, i) e Q =


(1, 1 + i, 0, −i).

Svolg. Poiché i due punti sono l’uno il coniugato dell’altro, la retta richiesta é cer-
tamente tutta reale. Sia (x1 , x2 , x3 , x4 ) un generico punto della retta. La condizione
di allineamento di tale punto con i punti P e Q é che la matrice
 
x1 x2 x3 x4
 1 1−i 0 i 
1 1 + i 0 −i
abbia rango 2, cioé
x1 x2 x3

1
1 − i 0 = 0 x3 = 0
1 1+i 0

x1 x2 x4

1 1−i i =0 − 2ix1 + 2ix2 + 2ix4 = 0.

1 1 + i −i
L’equazione cartesiana della retta é:
 
x3 = 0 z=0
→ .
x1 − x2 − x4 = 0 x−y−1=0
22 RETTE E PIANI NELLO SPAZIO AFFINE ED EUCLIDEO.

13. Esercizi.
Esercizio 32. Si determinino i valori del parametro k in modo che i tre piani di
equazioni x = 0, x + ky + z + 1 = 0, x + y + kz − 1 = 0 appartengano alla stessa
stella.
Esercizio 33. Dati i piani α : x + y − z = 0 e β : 2x + y + z = 0 ed il punto
P (0, 2, 1), scrivere l’equazione del piano passante per P ed ortogonale ad α e β.
Esercizio 34. Scrivere l’equazione del piano passante per P (0, 0, 6) che taglia il
piano z = 0 secondo la retta 2x − 3y − 6 = z = 0.
Esercizio 35. Siano dati il punto P (1, 2, −1) e la retta x−y+2z−1 = 2x+y+z = 0.
Calcolare la distanza tra P ed r.
Esercizio 36. Determinare l’equazione della retta r passante per P (1, −1, 2) e
parallela ai piani 3x + 2y + z + 1 = 0 e 6x + 4y − z + 3 = 0.
Esercizio 37. Si considerino le rette r : x = 1, y = 2t, z = t−1 e s : x−y+2 =
x + y + z − 3 = 0. Verificare che r e s sono sghembe. Scrivere l’equazione del piano
contenente r e parallelo a s e quella del piano perpendicolare a r e passante per il
punto (0, 2, 1).
Esercizio 38. Si considerino le rette r : x − 3y + 2 = x + y + z + 1 = 0 e s :
x = 2 − t, y = 3 + 5t, z = −t. Dimostrare che non sono complanari. Determinare
le equazioni della retta passante per P (2, 0, −2) e complanare (separatamente ma
contemporaneamente) con r e s.
Esercizio 39. Si determini il piano del fascio (x + y − z) + k(x − 4y + z − 1) = 0
che sia perpendicolare al piano x = 1.
Esercizio 40. Determinare i parametri direttori di una qualsiasi retta perpendico-
lare al piano x − 2y + z − 1 = 0 e, tra tali rette, le equazioni cartesiane di quella
passante per il punto (1,2,1).
Esercizio 41. La retta r : x − i = x − iy + z − 2 = 0 é reale?
Esercizio 42. Si scriva l’equazione del piano passante per i punti (0, 3, 5, 0), (0, 1, 1, 0),
(1, 0, 0, 1).
Esercizio 43. Si scriva l’equazione del piano passante per i punti (0, 2, 2, 0), (0, 0, 4, 0),
(2, 0, 0, 1).
Esercizio 44. Si considerino le rette r : x − z + 1 = y − 2 = 0 e s : y − z − 1 =
x − y = 0. Si determinino le equazioni cartesiane e quelle parametriche della retta
appartenente al piano y = z che sia incidente con r ed ortogonale a s.
Esercizio 45. Sia π il piano ortogonale alla retta r : x + y − 4 = 2x − z − 4 = 0 e
passante per q(0, 1, 1). Si determinino le equazioni cartesiane e parametriche della
retta parallela a π, passante per P (1, 0, 1) ed incidente la retta t : x−y = z−2 = 0.
Esercizio 46. Date le rette r : 2x−y+z = 2x+z−1 = 0 e s : x−2z+1 = y+z−
1 = 0 determinare: (i) le equazioni della retta passante per P (1, −2, 1) ed incidente
entrambe le rette r e s; (ii) le equazioni della retta t incidente ortogonalmente
entrambe le rette r e s; (iii) la minima distanza tra r e s.
RETTE E PIANI NELLO SPAZIO AFFINE ED EUCLIDEO. 23

Esercizio 47. Si considerino le rette r : x − y = z − 1 = 0 e s : x − y + 2z − 3 =


y − 2z = 0. Nel caso siano complanari si determini il piano che le contiene; nel
caso siano sghembe determinare le equazioni dei piani paralleli a cui appartengono.
Esercizio 48. Si considerino la retta r : x − 1 = y = z e la retta s passante per
il punto P (1, 2, 0) e parametri direttori (1, −1, 1). Nel caso r e s siano complanari,
determinare il piano che le contiene. Nel caso r e s siano sghembe, determinare la
retta di minima distanza.
LE SUPERFICI QUADRICHE.

1. Definizione. Una quadrica é una superficie nello spazio, luogo dei punti P =
(x, y, z) le cui coordinate soddisfano ad un’equazione del tipo
a11 x2 + a22 y 2 + a33 z 2 + a44 + 2a12 xy + 2a13 xz + 2a23 yz+
+2a14 x + 2a24 y + 2a34 z = 0.
In coordinate omogenee l’equazione di una quadrica é data da
a11 x21 + a22 x22 + a33 x23 + a44 x24 + 2a12 x1 x2 + 2a13 x1 x3 + 2a23 x2 x3 +
+2a14 x1 x4 + 2a24 x2 x4 + 2a34 x3 x4 = 0.
Osserviamo immediatamente che se π : ax+by+cz+d = 0 e π 0 : a0 x+b0 y+c0 z+d0 = 0
sono due piani, allora
π ∪ π0 : (ax + by + cz + d) · (a0 x + b0 y + c0 z + d0 ) = 0
é l’equazione di una quadrica, che é detta ”ridotta” nell’unione dei due piani.
Indichiamo con  
x1
 x2 
X=  x3 

x4
il generico vettore delle coordinate omogenee e sia
 
a11 a12 a13 a14
 a12 a22 a23 a24 
A=  a13 a23 a33 a34


a14 a24 a34 a44
la matrice simmetrica formata dai coefficienti che compaiono nell’equazione della
quadrica. Allora l’equazione puó essere riscritta in forma compatta come segue:
X T · A · X = 0.
Si noti che ogni retta dello spazio incontra una quadrica in due punti, eccetto
il caso in cui la retta appartenga completamente alla quadrica (nel qual caso i
punti di incontro tra la retta e la quadrica sono infiniti, tutti quelli della retta).
Analogamente a quanto detto per i punti di una curva algebrica nel piano affine o
euclideo, un punto di una quadrica é detto doppio se una qualsiasi retta passante
per esso ha una intersezione doppia con la quadrica nel punto stesso.
Determiniamo una prima classificazione delle quadriche in base alla presenza di
punti doppi:
Teorema 1. Sia Q : X T · A · X = 0 una quadrica. Essa contiene almeno un punto
doppio se e solo se det(A) = 0.

1
2 LE SUPERFICI QUADRICHE.

In particolare diciamo ”quadriche generali” quelle che non presentano punti


doppi (det(A) 6= 0); diciamo ”quadriche specializzate” quelle che presentano un
solo punto doppio; diciamo ”quadriche riducibili” quelle che presentano infiniti
punti doppi.

Concludiamo questa prima sezione osservando che l’intersezione tra un piano


π : ax + by + cz + d = 0 ed una quadrica Q : X T · A · X = 0 determina una curva
nello spazio, piú precisamente una conica che giace sul piano π, la cui equazione si
scrive

ax + by + cz + d = 0
X T · A · X = 0.
Tale conica é riducibile solo quando il piano π é tangente alla quadrica, oppure nel
caso la quadrica sia ridotta nell’unione di due piani.

2. Quadriche generali. Queste sono le quadriche prive di punti doppi. In pre-


senza di una quadrica generale Q : X T · A · X = 0, si definisce una corrispondenza
biunivoca tra l’insieme di tutti i punti dello spazio e l’insieme di tutti i piani dello
spazio, tale che ad ogni punto P = (a1 , a2 , a3 , a4 ) corrisponda il piano di equazione
   
a11 a12 a13 a14 x1
   a12 a22 a23 a24   x2 
π: a1 a2 a3 a4 ·   a13 a23 a33 a34  ·  x3  = 0.
  

a14 a24 a34 a44 x4

Il punto P é detto polo del piano π rispetto alla quadrica Q, il piano π é detto
piano polare di P rispetto alla quadrica Q.
In particolare, se P ∈ Q é un punto della quadrica, il suo piano polare, rispetto
a Q, coincide col piano tangente in P alla quadrica.

Fissiamo ora una quadrica Q : X T · A · X = 0 ed un punto P ∈ Q. Tramite la


polaritá appena definita, é possibile costruire il piano tangente π in P alla quadrica.
L’intersezione tra Q e π genera una conica γ riducibile. Se γ é ridotta in due rette
reali e distinte, il punto P é detto iperbolico. Se γ é ridotta in due rette reali e
coincidenti, il punto P é detto parabolico. Se γ é ridotta in due rette immaginarie
e coniugate, il punto P é detto ellittico.
Una quadrica generale non possiede punti parabolici, inoltre vale il seguente:

Teorema 2. Sia Q una quadrica generale. Se essa possiede un punto iperbolico


(rispettivamente ellittico) allora ogni punto della quadrica é iperbolico (rispettiva-
mente ellittico).

Le quadriche generali a loro volta si suddividono in tre classi. Tale classificazione


viene fatta in base allo studio della conica che scaturisce dall’intersezione tra la
quadrica ed il piano improprio x4 = 0.
LE SUPERFICI QUADRICHE. 3

2.1. Iperboloide. É una quadrica generale che interseca il piano improprio in una
conica irriducibile e reale (si dice che il piano improprio e l’iperboloide sono secanti
l’un l’altro).
Se i punti dell’iperboloide sono tutti iperbolici, esso é detto iperboloide iperbol-
ico, se i punti sono tutti ellittici esso é detto iperboloide ellittico.
Teorema 3. Un iperboloide é iperbolico se e solo se det(A) > 0, é ellittico se e
solo se det(A) < 0.
L’iperboloide possiede tutte le sezioni piane, cioé si possono generare sia iperboli
che parabole che ellissi, intersecando un iperboloide con i piani dello spazio.

2.2. Paraboloide. É una quadrica generale che interseca il piano improprio in una
conica riducibile (si dice che il piano improprio é tangente al paraboloide).
Teorema 4. Una quadrica generale é tangente al piano improprio (quindi é un
paraboloide) se e solo se il complemento algebrico A44 , dell’elemento a44 della ma-
trice A, é nullo.
Se i punti del paraboloide sono tutti iperbolici, esso é detto paraboloide iperbol-
ico, se i punti sono tutti ellittici esso é detto paraboloide ellittico.
Teorema 5. Un paraboloide é iperbolico se e solo se det(A) > 0, é ellittico se e
solo se det(A) < 0.
Teorema 6. Un paraboloide iperbolico possiede iperboli o parabole come sezioni
piane, cioé si possono generare sia iperboli che parabole, intersecando un paraboloide
iperbolico con i piani dello spazio.
Teorema 7. Un paraboloide ellittico possiede ellissi o parabole come sezioni piane,
cioé si possono generare sia ellissi che parabole, intersecando un paraboloide ellittico
con i piani dello spazio.

2.3. Ellissoide. É una quadrica generale che interseca il piano improprio in una
conica irriducibile immaginaria (si dice che il piano improprio é esterno all’ ellis-
soide).
I punti di un ellissoide sono tutti ellittici.
Teorema 8. Un ellissoide possiede punti tutti reali se e solo se det(A) < 0, ed é
detto ellissodie reale, in caso contrario é detto ellissoide immaginario.
Teorema 9. Un ellissoide possiede solamente ellissi come sezioni piane, cioé si pos-
sono generare solamente ellissi, intersecando un ellissoide con i piani dello spazio.
Esercizio 1. Classificare la quadrica di equazione x2 + y 2 − z 2 + xy + x − 3 = 0.

Svolg. La matrice associata alla quadrica é


1 21 1
 
0 2
 1 1 0 0 
A= 2 .
 0 0 −1 0 
1
2 0 0 −3
Otteniamo che det(A) > 0 con A44 6= 0, dove A44 é il complemento algebrico
dell’elemento a44 della matrice A.
4 LE SUPERFICI QUADRICHE.

Inoltre la conica impropria della quadrica é data da:


 2
x1 + x22 − x23 + x1 x2 = 0
x4 = 0

ed é a punti reali. Allora la superficie é un iperboloide a punti iperbolici.

Esercizio 2. Classificare la quadrica di equazione 2x2 + y 2 + z 2 − x − 2y + 1 = 0.

Svolg. La matrice associata alla quadrica é

0 − 21
 
2 0
 0 1 0 −1 
A=  .
0 0 1 0 
− 21 −1 0 1

Otteniamo che det(A) < 0 con A44 6= 0.


Inoltre la conica impropria della quadrica é data da:

2x21 + x22 + x23 = 0



x4 = 0

ed é a punti immaginari. Allora la superficie é un ellissoide a punti reali.

Esercizio 3. Classificare la quadrica di equazione x2 +y 2 +2xy+2xz+2yz−2x+2 =


0.

Svolg. La matrice associata alla quadrica é


 
1 1 1 −1
 1 1 1 0 
A=  1 1
.
0 0 
−1 0 0 2

Otteniamo che det(A) > 0 con A44 = 0.


Allora la superficie é un paraboloide a punti iperbolici.

3. Quadriche specializzate. Sono le quadriche con un solo punto doppio, il quale


é detto vertice. Per quanto riguarda i punti delle superfici specializzate, vale il
seguente:

Teorema 10. Se Q é una quadrica specializzata, allora tutti i punti della superficie
Q sono parabolici.

Anche le quadriche specializzate si suddividono in sottoclassi, e tale classifi-


cazione si riferisce alla conica impropria della quadrica, cioé la conica che scaturisce
con l’intersezione col piano improprio.
LE SUPERFICI QUADRICHE. 5

3.1. Cono. Il cono é la quadrica specializzata la cui conica impropria é irriducibile


(il cono ed il piano improprio sono secanti). Il vertice V del cono é un punto proprio
e le sue coordinate (x1 , x2 , x3 , x4 ) sono la soluzione del sistema lineare
   
a11 a12 a13 a14 x1
 a12 a22 a23 a24   x2 
 a13 a23 a33 a34  ·  x3  = 0.
   

a14 a24 a34 a44 x4


Si riconosce che la superficie é un cono utilizzando il seguente risultato:
Teorema 11. Sia A la matrice associata alla quadrica Q. La superficie é un cono
se e solo se det(A) = 0, rango(A) = 3 e A44 6= 0.

Un cono possiede tutte le sezioni piane, cioé si possono generare sia iperboli che
parabole che ellissi, intersecando un cono con i piani dello spazio.
Inoltre tutti i piani tangenti alla superficie di un cono devono contenerne il
vertice.

3.2. Cilindro. Il cilindro é la quadrica specializzata la cui conica impropria é


riducibile (il cilindro ed il piano improprio sono tangenti). Il vertice V del cilindro
é un punto improprio e le sue coordinate (x1 , x2 , x3 , 0) sono la soluzione del sistema
lineare    
a11 a12 a13 a14 x1
 a12 a22 a23 a24   x2 
 a13 a23 a33 a34  ·  x3  = 0.
   

a14 a24 a34 a44 0


Si riconosce che la superficie é un cilindro utilizzando il seguente risultato:
Teorema 12. Sia A la matrice associata alla quadrica Q. La superficie é un
cilindro se e solo se det(A) = 0, rango(A) = 3 e A44 = 0.

Ogni cilindro possiede un solo tipo di sezione piana indipendentemente dal piano
col quale si effettua l’intersezione. Cioé, fissato il cilindro Q, si possono generare
solo iperboli o parabole o ellissi, intersecando il cilindro con i piani dello spazio.
In particolare, poiché il piano improprio é tangente al cilindro, la conica impropria
ridotta dovrá rispettare il seguente prospetto:

Conica impropria Generica sezione del cilindro


2 rette reali e distinte Iperbole
2 rette reali e coincidenti Parabola
2 rette immaginarie coniugate Ellisse

Inoltre tutti i piani tangenti alla superficie di un cilindro devono contenerne il


vertice.
Esercizio 4. Classificare la quadrica di equazione x2 +y 2 +2xy+2yz−2x+y+1 = 0.
6 LE SUPERFICI QUADRICHE.

Svolg. La matrice associata alla quadrica é


 
1 1 0 −1
 1 1 1 12 
A=  0 1
.
0 0 
−1 21 0 1
Otteniamo che det(A) = 0 con A44 6= 0.
La superficie é un cono di vertice X = (x1 , x2 , x3 , x4 ) tale che AX = 0, da cui

 x1 + x2 − x4 = 0
x1 + x2 + x3 + x24 = 0


 x2 = 0
−x1 + x22 + x4 = 0

e quindi X = (1, 0, − 32 , 1).


Esercizio 5. Classificare la quadrica di equazione x2 + y 2 + 2xy − 2x + y + 1 = 0.

Svolg. La matrice associata alla quadrica é


 
1 1 0 −1
 1 1 0 12 
A=  0 0
.
0 0 
−1 21 0 1
Otteniamo che det(A) = 0 con A44 = 0 e rango 3.
La superficie é un cilindro di vertice X = (x1 , x2 , x3 , x4 ) tale che AX = 0, da cui

x1 + x2 = 0
−x1 + x22 = 0
e quindi X = (0, 0, 1, 0).
Inoltre la conica impropria della quadrica é data da:
 2
x1 + x22 + 2x1 x2 = 0
x4 = 0
(x1 + x2 )2 = 0


x4 = 0
cioé sono due rette reali e coincidenti, quindi la conica generatrice del cilindro é
una parabola.

4. Quadriche riducibili. Sono le quadriche di equazione


a11 x2 + a22 y 2 + a33 z 2 + a44 + 2a12 xy + 2a13 xz + 2a23 yz+
+2a14 x + 2a24 y + 2a34 z = 0
tali che
a11 x2 + a22 y 2 + a33 z 2 + a44 + 2a12 xy + 2a13 xz + 2a23 yz+
+2a14 x + 2a24 y + 2a34 z = (ax + by + cz + d) · (a0 x + b0 y + c0 z + d0 )
cioé sono unione di due piani π e π 0 di equazioni:
π : ax + by + cz + d = 0 π 0 : a0 x + b0 y + c0 z + d0 = 0.
Tali quadriche hanno infiniti punti doppi, sono i punti della retta comune ai due
piani π e π 0 .
In particolare si possono verificare i seguenti 3 casi:
LE SUPERFICI QUADRICHE. 7

1. i due piani sono distinti ed incidenti, ed allora det(A) = 0, rango(A) = 2, vi


sono ∞1 punti doppi propri;
2. i due piani sono distinti e paralleli, ed allora det(A) = 0, rango(A) = 2, vi
sono ∞1 punti doppi impropri;
3. i due piani sono coincidenti, ed allora det(A) = 0, rango(A) = 1, vi sono ∞2
punti doppi propri.
Aggiungiamo come caso ’molto particolare’ quello in cui la quadrica ha equazione
x24 = 0, cioé vi sono ∞2 punti doppi impropri.

Consideriamo π 00 : a00 x + b00 y + c00 z + d00 = 0 un qualsiasi altro piano dello spazio.
L’intersezione della quadrica riducibile con π 00 determina una conica ridotta nelle
due rette:

a0 x + b0 y + c0 z + d0 = 0
 
ax + by + cz + d = 0
r1 : r2 :
a00 x + b00 y + c00 z + d00 = 0 a00 x + b00 y + c00 z + d00 = 0

5. Piani ed assi di simmetria delle quadriche generali.


Definizione 1. Il cerchio assoluto é una conica irriducibile priva di punti reali che
giace sul piano improprio. Essa é identificata dalle equazioni:
 2
x1 + x22 + x23 = 0
.
x4 = 0
Ricordando che i punti ciclici di un qualsiasi piano sono i punti che la propria
giacitura ha in comune con ogni circonferenza che giace sul piano stesso, il cerchio
assoluto é in definitiva il luogo dei punti ciclici di tutti i piani dello spazio.
Definizione 2. Sia A = (aij ) ∈ M4 (lR) la matrice non singolare associata all’equazione
di una quadrica generale. Si definisce centro di simmetria della quadrica Q il punto
C ∈ Q tale che, se P ∈ Q é un punto della superficie, allora anche P 0 , il simmet-
rico di P rispetto a C appartenga anche alla superficie. Analogamente a quanto
visto per le coniche, le coordinate del centro di simmetria si ottengono direttamante
dalla matrice A: C = (A41 , A42 , A43 , A44 ), dove ogni Aij é il complemento algebrico
dell’elemento aij ∈ A considerato con l’opportuno segno (derivante dalla posizione
che esso occupa nella matrice). Nel caso di un paraboloide, essendo A44 = 0, si
parla di quadrica a centro improprio o priva di centro.
Definizione 3. I piani di simmetria (piani principali) di una quadrica generale sono
quei piani dello spazio che, rispetto alla quadrica definita, hanno come polo il punto
improprio della direzione ad essi ortogonale. Gli assi di simmetria della quadrica
sono le rette che si ottengono dall’intersezione dei piani di simmetria. Nel caso di
iperboloide ed ellissoide (quadriche a centro) si determinano 3 assi di simmetria
(in corrispondenza di 3 piani di simmetria): tali assi si possono ottenere anche
congiungendo il centro di simmetria con ciascun polo di ogni piano di simmetria. Nel
caso del paraboloide (senza centro di simmetria) si determina 1 asse di simmetria
(in corrispondenza di 2 piani di simmetria).
Teorema 13. Le prime 3 coordinate dei poli dei piani di simmetria sono le com-
ponenti degli autovettori della sottomatrice A44 della matrice A rappresentante la
quadrica generale.
8 LE SUPERFICI QUADRICHE.

Dim. Indichiamo con P0 = (x0 , y0 , z0 , 0) il polo di un piano di simmetria e sia


 
a11 a12 a13 a14
 a21 a22 a23 a24 
A=  a31 a32

a33 a34 
a41 a42 a43 a44
la matrice che rappresenta la quadrica. Il piano polare di P0 rispetto alla quadrica
ha equazione
(a11 x0 + a21 y0 + a31 z0 )x1 + (a12 x0 + a22 y0 + a32 z0 )x2 +

(a13 x0 + a23 y0 + a33 z0 )x3 + (a14 x0 + a24 y0 + a34 z0 )x4 = 0


e per sua definizione esso deve essere ortogonale alla direzione (x0 , y0 , z0 , 0), cioé:
(a11 x0 + a21 y0 + a31 z0 ) = λx0

(a12 x0 + a22 y0 + a32 z0 ) = λy0


(a13 x0 + a23 y0 + a33 z0 ) = λz0
per un opportuno λ ∈ lR. Ne segue che

 (a11 − λ)x0 + a21 y0 + a31 z0 = 0
a12 x0 + (a22 − λ)y0 + a32 z0 = 0
a13 x0 + a23 y0 + (a33 − λ)z0 = 0

il quale ha soluzioni non banali solo quando



(a11 − λ) a21 a31


a12 (a22 − λ) a32 = 0.

a13 a23 (a33 − λ)
In sostanza λ é un autovalore della matrice A44 e le coordinate (x0 , y0 , z0 ) del
punto P0 = (x0 , y0 , z0 , 0) individuano le componenti di un autovettore relativo a λ.
Al variale di λ si ottengono tutti gli autovettori e quindi tutti i poli dei piani di
simmetria.
Oltre all’utilizzo degli autovalori della matrice A44 (come appena visto), esiste
un secondo metodo per determinare i poli dei piani di simmetria: esso consiste nel
determinare inizialmente un fascio di coniche, utilizzando, come coniche di base, il
cerchio assoluto e la conica impropria della quadrica. Quindi si individuano le tre
coniche riducibili di tale fascio ed i rispettivi punti doppi. Tali punti sono i poli dei
piani di simmetria.
Verifichiamo col seguente esempio l’equivalenza dei due metodi:
Esempio 1. Consideriamo il paraboloide iperbolico Q di equazione 6xz+8yz−5x =
0. Determiniamo piani ed asse di simmetria utilizzando entrambi i metodi.

Svolg. Metodo del fascio di coniche improprie:


La conica impropria di Q é 6x1 x3 + 8x2 x3 = 0, x4 = 0, determiniamo il fascio con
il cerchio assoluto:
6x1 x3 + 8x2 x3 + λ(x21 + x22 + x23 ) = 0

F : .
x4 = 0
LE SUPERFICI QUADRICHE. 9

La matrice associata al fascio é



λ 0 3

 0 λ 4

3 4 λ
il cui determinante é λ(λ2 − 25). Quindi si ottengono le coniche riducubili del fascio
per i valori λ = 0, 5, −5.
Per λ = 0 la conica é x3 (6x1 + 8x2 ) = 0, x4 = 0, il cui punto doppio é P1 =
(4, −3, 0, 0). Il piano polare di P1 rispetto al paraboloide Q é x4 = 0 (tale piano
risulterá sempre nel caso di un paraboloide, chiaramente non verrá mai preso in
considerazione).
Per λ = 5 la conica é 5x21 + 5x22 + 5x23 + 6x1 x3 + 8x2 x3 = 0, x4 = 0 il cui punto
doppio é P2 = (3, 4, −5, 0). Il piano polare di P2 é 6x + 8y + 10z − 3 = 0.
Infine per λ = −5 la conica é −5x21 − 5x22 − 5x23 + 6x1 x3 + 8x2 x3 = 0, x4 = 0 il cui
punto doppio é P2 = (3, 4, 5, 0). Il piano polare di P2 é 6x + 8y − 10z + 3 = 0.
L’asse di simmetria é quindi

6x + 8y + 10z − 3 = 0
6x + 8y − 10z + 3 = 0
inoltre possiamo calcolare il punto di sella del paraboloide iperbolico, intersecando
l’asse con la sua superficie:

 6x + 8y + 10z − 3 = 0
6x + 8y − 10z + 3 = 0
6xz + 8yz − 5x = 0

3
ottenendo le coordinate (0, 0, 10 ).

Metodo degli autovalori.


É sufficiente verificare che gli autovalori della matrice A44 sono esattamente λ =
0, 5, −5 e che i rispettivi autovettori rispetto ad A44 sono: (− 43 , 1, 0) per λ = 0,
( 53 , 45 , 1) per λ = 5, (− 35 , − 45 , 1) per λ = −5. Quindi i poli dei piani si simmetria
sono esattamente (− 34 , 1, 0, 0), ( 35 , 45 , 1, 0), (− 35 , − 45 , 1, 0).
Osservazione 1. Il calcolo del punto di sella di un paraboloide iperbolico é analogo
al calcolo del punto del parabolide ellittico che potremmo definire impropriamente
’vertice del parabolide’. In entrambi i casi tali punti ricopriranno un ruolo fonda-
mentale per la riduzione a forma canonica delle equazioni dei paraboloidi.

6. Forma canonica di una quadrica generale con centro di simmetria.


Sia X t AX = 0 l’equazione di una quadrica generale con centro di simmetria (iper-
boloide o ellissoide). Una rototraslazione degli assi di simmetria che riporti l’origine
del sistema di riferimento nel centro di simmetria e faccia coincidere gli assi coordi-
nati con gli assi di simmetria della quadrica, trasforma l’equazione della superficie
nella sua forma canonica:

Ellissoide:
2 2
z2
1. xa2 + yb2 + c2 = 1 ellissoide reale;
2 2
x y z2
2. a2 + b2 + c2 = −1 ellissoide immaginario.
10 LE SUPERFICI QUADRICHE.

Iperboloide:
2
x2 y z2
1. a2 + b2 − c2 = 1 iperboloide iperbolico;
2 2
x y z2
2. a2 − b2 + c2 = 1 iperboloide iperbolico;
2 2 2
3. - xa2 + yb2 + zc2 = 1 iperboloide iperbolico;
x2 y2 z2
4. a2 − b2 − c2 = 1 iperboloide ellittico;
2 2 2
5. - xa2 + yb2 − zc2 = 1 iperboloide ellittico;
2 2 2
6. - xa2 − yb2 + zc2 = 1 iperboloide ellittico.
t
Diciamo X BX = 0 l’equazione della quadrica nella sua forma ridotta. Come
accadeva per le coniche, si puó dimostrare (non lo facciamo!) che esistono alcune
quantitá che non cambiano dopo aver effettuato un cambiamento di base tramite
vettori ortonormali: vengono detti invarianti ortogonali e sono
det(A) = det(B)
det(A44 ) = det(B44 )
la somma degli autovalori di A44 é pari alla somma degli autovalori di B44 .
In altre parole, se indichiamo con αx2 +βy 2 +γz 2 = δ l’equazione di una quadrica a
centro, é sufficiente calcolare gli autovalori di A44 per ottenere i coefficienti α, β, γ,
inoltre gli autoversori relativi ad α, β, γ compongono la matrice di rotazione. Quindi
si puó effettuare una traslazione rispetto alle coordinate del centro o equivalente-
mente applicare l’invarianza del det(A) = det(B) per ottenere il valore del coeffi-
ciente δ.
Esempio 2. Consideriamo l’iperboloide ellittico di equazione 2x2 − 2y 2 − 2yz −
2z 2 − 3 = 0 di matrice associata:
 
2 0 0 0
 0 −2 −1 0 
A=  0 −1 −2 0  .

0 0 0 3
Determiniamone una forma canonica e le formule di rototraslazione che ci perme-
ttono di ottenerla.

Svolg. Una forma canonica é ax2 + by 2 + cz 2 = d, dove a, b, c sono gli autovalori


della sottomatrice A44 . Eseguendo i calcoli, essi sono 2, −3, −1, quindi l’equazione
sará 2x2 − 3y 2 − z 2 = d con matrice associata
 
2 0 0 0
 0 −3 0 0 
B=  0 0 −1 0 

0 0 0 −d
di determinante −6d. Inoltre é facile calcolare che det(A) = −18 e grazie all’invarianza
ortogonale sappiamo che 6d = 18, quindi d = 3. Infine l’equazione in forma ridotta
sará 2x2 − 3y 2 − z 2 − 3 = 0.
Vediamo ora quale é stata la trasformazione effettuata per ottenere tale forma
canonica.
L’autospazio relativo all’autovalore 2 rispetto alla matrice A44 é generato dal ver-
sore (1, 0, 0).
LE SUPERFICI QUADRICHE. 11

L’autospazio relativo all’autovalore −3 rispetto alla matrice A44 é generato dal vet-
tore (0, 1, 1) e quindi anche dal versore (0, √12 , √12 ).
Infine l’autospazio relativo all’autovalore −1 rispetto alla matrice A44 é generato
dal vettore (0, 1, −1) e quindi anche dal versore (0, √12 , − √12 ).
Inoltre il centro della quadrica é (A41 , A42 , A43 , A44 ) = (0, 0, 0, 6), cioé in coordinate
non omogenee (0, 0, 0).
Quindi la rototraslazione che ci permette di concludere é la seguente:

1 0 0 x01
       
x1 0
 x2  =  0 √1 √1  ·  x02  +  0  .
2 2
x3 0 √1
2
− √12 x03 0

7. Forma canonica di un paraboloide. Sia X t AX = 0 l’equazione di un paraboloide.


Come in precedenza una trasformazione ortonormale del sistema di riferimento
trasforma l’equazione della superficie nella sua forma canonica. Facciamo rifer-
imento ai paraboloidi che abbiano nella loro forma ridotta, l’asse di simmetria
coincidente con l’asse Z, in modo del tutto identico il discorso puó essere riportato
agli altri 2 casi:

x2 y2
1. a2 + b2 = 2dz paraboloide ellittico;
x2 y2
2. −
a2 = 2dz paraboloide iperbolico.
b2
t
Diciamo X BX = 0 l’equazione della quadrica nella sua forma ridotta. Come in
precedenza i seguenti sono invarianti ortogonali:
det(A) = det(B)
det(A44 ) = det(B44 )
la somma degli autovalori di A44 é pari alla somma degli autovalori di B44 .
Esempio 3. Consideriamo il paraboloide iperbolico di equazione 6xz +8yz −5x = 0
di matrice associata:
0 0 3 − 52
 
 0 0 4 0 
A=  3 4 0 0 .

− 52 0 0 0
Determiniamone una forma canonica e le formule di rototraslazione che ci perme-
ttono di ottenerla.

Svolg. Una forma canonica con asse di simmetria coincidente con l’asse Z é ax2 +
by 2 + 2cz = 0, dove a, b sono gli autovalori non nulli della sottomatrice A44 (ri-
cordiamo che per un paraboloide l’autovalore nullo esiste sempre ma non fornisce
indicazioni, poiché l’autovettore ad esso relativo é il polo del piano improprio).
Eseguendo i calcoli, gli autovalori non nulli sono 5, −5, quindi l’equazione sará
5x2 − 5y 2 + 2cz = 0 con matrice associata
 
5 0 0 0
 0 −5 0 0 
B=  0 0 0 c 

0 0 c 0
12 LE SUPERFICI QUADRICHE.

di determinante 25c2 . Inoltre é facile calcolare che det(A) = 100 e grazie all’invarianza
ortogonale sappiamo che 25c2 = 100, quindi c ∈ {−2, 2}. Infine le equazioni in
forma ridotta saranno
5x2 − 5y 2 − 4z = 0
oppure
5x2 − 5y 2 + 4z = 0.

Vediamo ora quale é stata la trasformazione effettuata per ottenere tali forme canon-
iche.
L’autospazio relativo all’autovalore 0 rispetto alla matrice A44 é generato dal ver-
sore ( 54 , − 35 , 0).
L’autospazio relativo all’autovalore 5 rispetto alla matrice A44 é generato dal ver-
3 4
sore ( 5√ , √
2 5 2
, √12 ).
Infine l’autospazio relativo all’autovalore −5 rispetto alla matrice A44 é generato
3 4
dal versore (− 5√ 2
, − 5√ 2
, √12 ).
3
Inoltre il punto di sella della quadrica é (0, 0, 10 ).
Quindi la rototraslazione che ci permette di concludere é la seguente:
  √ 3 3 4
 
− 5√ x01
   
x1 5 2 2 5 0
4 4 3
 x2  =  √ 
5 2
− 5√2 − 5  ·  x02  +  0  .

x3 √1 √1 0 x03 3
10
2 2
Osserviamo che nella rotazione, possiamo scegliere quale asse coordinato di-
venterá l’asse di simmetria, sará sufficiente nella matrice di rotazione posizionare
l’autoversore relativo all’autovalore nullo esattamente nella colonna 1, se si sceglie
l’asse X, nella colonna 2 se si sceglie l’asse Y , nella colonna 3 (come nel nostro
esempio) se si sceglie l’asse Z.
Si verifichi infatti che, se si scegli come rototraslazione la seguente:
  4 3 3
 
√ − 5√ x01
   
x1 5 5 2 2 0
 x2  =  − 35
 4
√ − 5√4 
·  x02  +  0 
5 2 2 
x3 0 1
√ √1 x03 3
10
2 2
in cui l’autoversore relativo all’autovalore nullo é posto nella prima colonna, la
forma canonica finale sará 5y 2 − 5z 2 − 4x = 0, cioé un paraboloide iperbolico con
asse di simmetria coincidente con l’asse X.

8. Forma canonica di un cono. Sia X t AX = 0 l’equazione di un cono, con


matrice associata A e vertice V di coordinate XV = (xv , yv , zv ). Poiché il vertice
del cono si comporta esattamente come un centro di simmetria, quanto detto sulle
quadriche a centro proprio (iperboloidi ed ellessoidi) puó essere applicato anche
per la riduzione a forma canonica dell’equazione di un cono. Si tratta allora di
determinare gli autovalori della sottomatrice A44 e quindi di costruire una base di
autoversori, tramite la quale riempire la matrice di rotazione P . Infine si effettua
la traslazione rispetto al vettore di coordinate XV . Dopo la rototraslazione,
   0 
x1 x1
 x2  = P ·  x02  + XV
x3 x03
l’equazione ridotta assume una delle seguenti forme:
LE SUPERFICI QUADRICHE. 13

1. a2 x2 + b2 y 2 + c2 z 2 = 0 nel caso di un cono immaginario (l’unico punto reale


é il vertice);
2. a2 x2 + b2 y 2 − c2 z 2 = 0 nel caso di un cono reale (con asse perpendicolare al
piano z = 0).
I coefficienti a2 , b2 , c2 sono gli autovalori della A44 .
Esempio 4. Consideriamo il cono immaginario di equazione 3x2 +2y 2 +2xz+3z 2 =
0 di matrice associata:  
3 0 1 0
 0 2 0 0 
A=  1 0 3 0 .

0 0 0 0
Determiniamone una forma canonica e le formule di rototraslazione che ci perme-
ttono di ottenerla.

Svolg. Il vertice del cono ha coordinate (0, 0, 0). Gli autovalori della sottomatrice
A44 sono λ1 = 2, con molteplicitá algebrica 2, e λ2 = 4. Una forma canonica é
2x2 + 2y 2 + 4z 2 = 0.
Vediamo ora quale é stata la trasformazione effettuata per ottenere tali forme canon-
iche.
L’autospazio relativo all’autovalore 2 rispetto alla matrice A44 é generato dai ver-
sori ( √12 , 0, − √12 ), (0, 1, 0).
L’autospazio relativo all’autovalore 4 rispetto alla matrice A44 é generato dal ver-
sore ( √12 , 0, √12 ).

Quindi la rototraslazione che ci permette di concludere é la seguente:


  √1 √1
0 x01
     
x1 2 2 0
 x2  =  0 1 0  ·  x02  +  0  .
x3 − √12 0 √1
2
x03 0

9. Forma canonica di cilindri a base ellittica e iperbolica. Sia X t AX = 0


l’equazione di un cilindro con base ellittica o iperbolica, con matrice associata
A. Anche in questo caso si tratta dapprima di determinare gli autovalori della
sottomatrice A44 : uno di essi é certamente λ = 0. Si costruisce una base di au-
toversori, tramite la quale riempire la matrice di rotazione P : l’autospazio relativo
all’autovalore nullo ha dimensione 1, il suo versore generatore é parallelo alle gen-
eratrici del cilindro; gli altri autoversori individuano un piano perpendicolare alla
direzione della generatrici. Una volta effettuata la rotazione si osserva l’equazione
ottenuta: se non sono presenti termini di primo grado, allora non vi é alcun bisogno
di traslazioni. In caso contrario, si effettua una sostituzione x0 = x + a, y 0 = y + b,
z 0 = z + c, imponendo che i termini di primo grado si annullino. I valori a, b, c che
verificano l’annullarsi dei termini lineari, sono esattamente le coordinate del vet-
tore di traslazione (esso é il centro C = (a, b, c) di una qualsiasi conica direttrice del
cilindro, cioé una conica che giace su un qualsiasi piano ortogonale alle generatrici
del cilindro). L’equazione ridotta finale assume una delle seguenti forme:
1. a2 x2 + b2 y 2 + c2 = 0 nel caso di un cilindro immaginario;
14 LE SUPERFICI QUADRICHE.

2. a2 x2 + b2 y 2 − c2 = 0 nel caso di un ellittico (con generatrici parallele all’asse


Z);
3. a2 x2 − b2 y 2 − c2 = 0 nel caso del cilindro iperbolico (con generatrici parallele
all’asse Z).
I coefficienti a2 , b2 sono gli autovalori non nulli della A44 . Si osservi che in modo
analogo si ottengono le equazioni canoniche dei cilindri con generatrici parallele agli
assi X o Y .

Esempio 5. Consideriamo il cilindro a base iperbolica di equazione 6xz +8yz −5 =


0 di matrice associata:
 
0 0 3 0
 0 0 4 0 
A=  3 4 0 0 .

0 0 0 −5
Determiniamone una forma canonica e le formule di rototraslazione che ci perme-
ttono di ottenerla.

Svolg. Il vertice del cilindro ha coordinate (4, −3, 0, 0), infatti l’autospazio relativo
all’autovalore 0 rispetto alla matrice A44 é generato dal versore ( 45 , − 35 , 0).
Gli altri due autovalori di A44 sono 5, −5.
L’autospazio relativo all’autovalore 5 rispetto alla matrice A44 é generato dal versore
3 4
( 5√ , √
2 5 2
, √12 ).
Infine l’autospazio relativo all’autovalore −5 rispetto alla matrice A44 é generato
3 4
dal versore (− 5√ 2
, − 5√ 2
, √12 ).
Quindi la rotazione é la seguente:

  3 3 4
 
√ − 5√ x01
 
x1 5 2 2 5
4 4
 x2  = 
 √
5 2
− 5√ 2
− 53
  0 
 · x2 .
x3 √1 √1 0 x03
2 2

La forma che si ottiene dopo la sostituzione delle variabili é x2 − y 2 − 1 = 0, con


generatrici parallele all’asse Z. Essa non presenta termini di primo grado, quindi
non necessita di alcuna traslazione.

10. Forma canonica di cilindri a base parabolica. Discorso a parte per i cilin-
dri a base parabolica: essi presentano un autovalore nullo per la matrice A44 , con
molteplicitá algebrica 2. L’autospazio associato é un piano parallelo alle generatrici
del cilindro, quindi contiene il vertice. La scelta dei tre vettori per la rotazione deve
in tale caso essere piú oculata che non nei precedenti.
Il primo versore é quello relativo alla direzione individuata dal vertice.
Il secondo, anche esso appartenente all’autospazio V0 relativo all’autovalore nullo,
é il versore della retta che si ottiene dall’intersezione del piano V0 e di un qualsiasi
piano ortogonale alla direzione del vertice.
L’ultimo versore é il generatore dell’autospazio relativo all’autovalore non nullo
della matrice A44 , che é quindi ortogonale ai precedenti due.
Una volta effettuata la rotazione si osserva l’equazione ottenuta: se non sono pre-
senti termini di grado zero (termini noti), allora non vi é alcun bisogno di traslazioni.
LE SUPERFICI QUADRICHE. 15

In caso contrario, si eleminano i termini noti procedendo con una traslazione im-
posta, come per i cilindri a base iperbolica o ellittica. Alla fine si ottiene la seguente:
a2 x2 + by = 0
in cui i ruoli delle variabili x, y, z sono intercambiabili.
Esempio 6. Consideriamo il cilindro a base parabolica di equazione x2 − 2xy +
y 2 − 4x − 4y − 4z + 4 = 0 di matrice associata:
 
1 −1 0 −2
 −1 1 0 −2 
A= .
 0 0 0 −2 
−2 −2 −2 4
Determiniamone una forma canonica e le formule di rototraslazione che ci perme-
ttono di ottenerla.

Svolg. Gli autovalori della sottomatrice A44 sono λ1 = 0, con molteplicitá alge-
brica 2, e λ2 = 2. Il vertice del cilindro ha coordinate (1, 1, −2, 0), quindi il primo
versore per la rotazione é ( √16 , √16 , − √26 ).
Inoltre l’autospazio relativo all’autovalore 0 rispetto alla matrice A44 é dato dal
piano di equazione x − y = 0.
Scegliamo ora un qualsiasi piano perpendicolare alla direzione individuata dal ver-
tice, ad esempio quello passante per l’origine: x + y − 2z = 0.
Il secondo vettore per la rotazione é quindi individuato dall’intersezione

x−y =0
x + y − 2z = 0
per cui ha componenti (1, 1, 1), con versore ( √13 , √13 , √13 )
L’ultimo vettore é l’autovettore relativo all’autovalore λ2 = 0, cioé (1, −1, 0), con
versore ( √12 , − √12 , 0). Quindi la rotazione é la seguente:
  √1 √1 √1
 
x01
 
x1 6 3 2
1 √1
 x2  =   √6 3
− √12 
 ·  x02  .
x3 − 6
√2 √1
0 x03
3

La forma che si ottiene dopo la sostituzione delle variabili é 2z 2 − 4 3y + 4 = 0.
La presenza del termine di grado zero, richiede una ulteriore traslazione: poniamo
x = x0 , y = y 0 + b, z = z 0 tali che:

2z 02 − 4 3(y 0 + b) + 4 = 0
√ √
2z 02 − 4 3y 0 + (−4 3b + 4) = 0.

Imponendo (−4 3b + 4) = 0 otteniamo b = √13 . La trasformazione finale sará
  √1 √1 √1
 
x01
   
x1 6 3 2 0
1 1 1 1
 x2  =  √ 
6

3
− √2  ·  x02  +  √3 

x3 2
− √6 √3 1
0 x03 0

che porta alla forma ridotta 2z 2 − 4 3y = 0.
16 LE SUPERFICI QUADRICHE.

11. Esercizi.
Esercizio 6. Classificare la seguente quadrica x2 −y 2 +2z 2 +6yz −4xz −2x−3 = 0
e determinare la conica intersezione con il piano z = 1.
Esercizio 7. Classificare la quadrica x2 +y 2 −z 2 +2xy−2x−2y = 0 e determinarne
gli eventuali punti doppi.
Esercizio 8. Classificare la quadrica x2 + 2y 2 − z 2 + 2xy − 2x − 2y + 1 = 0 e
determinarne gli eventuali punti doppi.
Esercizio 9. Classificare la quadrica x2 − y 2 + z 2 + 2xz − 2x − 4y − 2z − 3 = 0,
determinarne gli eventuali punti doppi e la conica all’infinito.
Esercizio 10. Classificare la quadrica x2 + 2y 2 + z 2 + 2xy − 2x − 3 = 0 ed i suoi
punti.
Esercizio 11. Classificare la quadrica xy + yz + xz − 2x + 1 = 0 ed i suoi punti.
Esercizio 12. Classificare la quadrica y 2 − z 2 + 4xy − 4xz − 6x + 4y + 2z + 8 = 0
ed i suoi punti.
Esercizio 13. Determinare la conica intersezione tra la quadrica x2 + y 2 + z 2 +
2xy − 2x = 0 ed il piano x + y − z = 0; ripetere l’esercizio nel caso in cui il piano
sia x = 0.
2 2 2
Esercizio 14. Determinare la conica intersezione tra la quadrica √ 2x −3y −6z = 0
ed il piano x = 0; ripetere l’esercizio nel caso il piano sia x = 3z + 1.
Esercizio 15. Classificare la quadrica 2x2 + 2xy + z = 0 e determinare la conica
intersezione con il piano 4x + 2y + z − 2 = 0.
Esercizio 16. Classificare la quadrica x2 − 2y 2 − z 2 = 1 e determinare le coniche
intersezione con i piani α : x = z − 1 e β : x = 1.
Esercizio 17. Classificare la quadrica x2 − z 2 − 1 = 0 e determinare la conica
intersezione col piano z = 1.
Esercizio 18. Classificare la quadrica −4x2 + y 2 + z 2 − 2 = 0 e determinare la
conica intersezione col piano y + z − 2 = 0.
Esercizio 19. Classificare la quadrica x2 + y 2 − 2xz + z 2 − 1 = 0 e determinare la
conica intersezione col piano x − z = 0.
Esercizio 20. Classificare la quadrica x2 + y 2 + 2xy = 0 e determinare le coniche
intersezione coi piani x = 1 e z = 0.
Esercizio 21. Classificare la quadrica 2x2 +y 2 +3z 2 +4x−2y+2 = 0 e determinare
le coniche intersezione coi piani x = 0 e y = 0.
Esercizio 22. Classificare la quadrica x2 − z 2 + 2y = 0 e determinare la conica
intersezione col piano x − 2z = 0.
Esercizio 23. Classificare la quadrica x2 − y 2 + z 2 = 1 e determinare le coniche
intersezione coi piani x − y = 1 e z = 1.
LA SFERA.

1. Definizione. Siano C = (α, β, γ) un punto dello spazio Euclideo e r ∈ lR con


r > 0. La sfera di centro C e raggio r é una superficie, luogo dei punti P = (x, y, z)
dello spazio la cui distanza da C é pari a r. Da tale definizione ricaviamo che la
sua equazione é: p
(x − α)2 + (y − β)2 + (z − γ)2 = r
da cui
(x − α)2 + (y − β)2 + (z − γ)2 = r2
x2 + y 2 + z 2 + ax + by + cz + d = 0
dove
a b c
C = (α, β, γ) = (− , − , − )
2 2 2
1p 2
r= a + b2 + c2 − 4d > 0.
2
Esempio 1. Determinare centro e raggio della sfera x2 +y 2 +z 2 −3x+5y−z+1 = 0.

Svolg. Il centro della sfera data é:


3 5 1
C = ( ,− , )
2 2 2
ed il raggio: √ √
9 + 25 + 1 − 4 31
r= = .
2 2
L’equazione della sfera si puó infatti scrivere anche:
3 5 1 31
(x − )2 + (y + )2 + (z − )2 = .
2 2 2 4
t
u

2. Sezioni piane. Siano S : x2 + y 2 + z 2 + ax + by + cz + d = 0 una sfera di centro


C e raggio r e π : ex + f y + gz + h = 0 un piano. Indichiamo con δ = δ(C, π) la
distanza tra il centro della sfera ed il piano π.
Se δ > r allora il piano é esterno alla sfera. La loro intersezione genera una
circonferenza non riducibile ed immaginaria.

Se δ = r allora il piano é tangente alla sfera. La loro intersezione genera una


circonferenza ridotta in due rette immaginarie coniugate.

Se δ < r allora il piano é secante la sfera. La loro intersezione genera una


circonferenza reale, le cui equazioni sono date dal sistema:
S : x2 + y 2 + z 2 + ax + by + cz + d = 0

π : ex + f y + gz + h = 0.

1
2 LA SFERA.

Il centro C 0 della circonferenza é dato dall’intersezione s ∩ π, dove s é la retta


passante
√ per il centro della sfera ed ortogonale a π; il raggio della circonferenza é
r0 = r2 − δ 2 .

Poiché la sfera é un particolare ellissoide, possiamo utilizzare la polaritá definita


per le quadriche irriducibili per calcolare un qualsiasi piano tangente ad una sfera
in un suo punto.

3. Fascio di sfere. Siano S : x2 + y 2 + z 2 + ax + by + cz + d = 0 una sfera di


raggio r e centro C e S 0 : x2 + y 2 + z 2 + a0 x + b0 y + c0 z + d0 = 0 una di raggio r0 e
centro C 0 .
Indichiamo con δ = δ(C, C 0 ) la distanza tra i due centri. Supponiamo che
|r − r0 | < δ < r + r0
allora le due sfere si intersecano in una circonferenza reale. Tale circonferenza giace
su di un piano la cui equazione si ricava sottraendo membro a membro le equazioni
delle due sfere:
x2 + y 2 + z 2 + ax + by + cz + d = 0

x + y 2 + z 2 + a0 x + b0 y + c0 z + d0 = 0.
2

cioé
π : (a − a0 )x + (b − b0 )y + (c − c0 )z + (d − d0 ) = 0.
Da cui otteniamo l’equazione della circonferenza che le due sfere hanno in comune:
x2 + y 2 + z 2 + ax + by + cz + d = 0

γ:
(a − a0 )x + (b − b0 )y + (c − c0 )z + (d − d0 ) = 0.
Definiamo fascio di sfere secanti in γ e di piano radicale π, tutte le sfere che si
ottengono, al variare del parametro reale λ, dall’equazione
(x2 + y 2 + z 2 + ax + by + cz + d) + λ(x2 + y 2 + z 2 + a0 x + b0 y + c0 z + d0 ) = 0
o equivalentemente dall’equazione
(x2 + y 2 + z 2 + ax + by + cz + d) + λ((a − a0 )x + (b − b0 )y + (c − c0 )z + (d − d0 )) = 0.
Se P1 é un punto non appartenente a π allora esiste una ed una sola sfera apparte-
nente al fascio e passante per P1 .

Supponiamo ora che


|r − r0 | = δ
allora le due sfere sono tangenti internamente l’un l’altra. Viceversa, se r + r0 = δ
allora le due sfere sono tangenti esternamente. In entrambi i casi le due sfere si
intersecano in una circonferenza ridotta in due rette immaginarie coniugate, con un
unico punto reale P0 , che é quello di tangenza tra le due sfere.
Indichiamo con π : ex + f y + gz + h = 0 il piano tangente ad entrambe le sfere
S e S 0 in P0 .
Definiamo fascio di sfere tangenti in P0 e di piano radicale π, tutte le sfere che
si ottengono, al variare del parametro reale λ, dall’equazione
(x2 + y 2 + z 2 + ax + by + cz + d) + λ(x2 + y 2 + z 2 + a0 x + b0 y + c0 z + d0 ) = 0
o equivalentemente dall’equazione
(x2 + y 2 + z 2 + ax + by + cz + d) + λ(ex + f y + gz + h) = 0.
LA SFERA. 3

Anche in questo caso, se P1 é un punto non appartenente a π allora esiste una ed


una sola sfera appartenente al fascio e passante per P1 .
Esempio 2. Determinare il piano tangente alla sfera x2 +y 2 +z 2 −2x+2y−2z−1 =
0 nel punto P = (1, 1, 1).

Svolg. Il piano tangente puó essere visto come il piano polare del punto P rispetto
alla quadrica (in questo caso la sfera) data:
   
1 0 0 −1 x1
  0 1 0 1   x2 
  
1 1 1 1 ·  0 0 1 −1  ·  x3  = 0
−1 1 −1 −1 x4
che si riduce a 2x2 − 2x4 = 0, cioé y − 1 = 0.

Analogamente possiamo seguire il seguente procedimento:


Il centro della sfera data é C = (1, −1, 1), quindi il vettore CP ha componenti
proporzionali alla terna (0, 1, 0). Il piano tangente deve essere ortogonale a tale
vettore quindi la sua equazione é:
(0)(x − 1) + (1)(y − 1) + (0)(z − 1) = 0
cioé y − 1 = 0. t
u
Esempio 3. Determinare la sfera contenente la circonferenza di equazioni
γ: S∩π : x2 + y 2 + z 2 − 2x + 2z = y + z = 0
ed il punto P = (1, 1, 1).

Svolg. Per prima cosa verifichiamo che √ la circonferenza γ é reale; infatti il centro
di S é C = (1, 0, −1), il suo raggio é 2, e la distanza δ(C, π) = √12 é minore del
raggio di S.
Costruiamo il fascio di sfere contenenti la circonferenza γ:
Fγ : x2 + y 2 + z 2 − 2x + 2z + h(y + z) = 0
ed imponiamo il passaggio di tali sfere per il punto P : otteniamo h = − 32 . Quindi
la sfera richiesta é ottenuta dal fascio per tale valore di h:
2x2 + 2y 2 + 2z 2 − 4x + z − 3y = 0.
t
u
Esempio 4. Determinare la sfera tangente al piano π : x + y − z − 1 = 0 nel
punto P = (1, 1, 1) e passante per il punto Q = (2, 1, 0).

Svolg. La retta ortogonale a π e passante per P ha equazioni:



 x=t+1
y =t+1 .
z = −t + 1

Su tale retta si trovano i centri di ogni sfera che sia tangente a π in P . Per esempio
scegliamo t = 1, il centro della sfera S1 ottenuta é C = (2, 2, 0) ed il suo raggio é
4 LA SFERA.


la distanza δ(P, C) = 3. La sfera ha equazione (x − 2)2 + (y − 2)2 + z 2 = 3 cioé
x2 + y 2 + z 2 − 4x − 4y + 5 = 0. Costruiamo il fascio di sfere tangenti a π in P :
F : x2 + y 2 + z 2 − 4x − 4y + 5 + h(x + y − z − 1) = 0.
Imponiamo il passaggio per il punto Q, sostituendo le sue coordinate nell’equazione
del fascio F : h = 1. Quindi la sfera richiesta ha equazione
x2 + y 2 + z 2 − 3x − 3y − z + 4 = 0.
t
u
LA SFERA. 5

4. Esercizi.
Esercizio 1. Determinare centro e raggio della sfera 3x2 + 3y 2 + 3z 2 − 2x = 0,
x2 + y 2 + z 2 − 3y + z + 2 = 0.
Esercizio 2. Determinare centro e raggio della circonferenza x2 + y 2 + z 2 = 3x −
4y − 5 = 0.
Esercizio 3. Determinare il piano tangente alla sfera (x − 2)2 + y 2 + z 2 = 4 nel
suo punto (0, 0, 0).
Esercizio 4. Determinare la retta tangente alla circonferenza (x−2)2 +y 2 +z 2 −4 =
x + y − z = 0 nel suo punto (0, 0, 0).
Esercizio 5. Determinare i piani tangenti alla sfera x2 +y 2 +z 2 −2x−2y−6z+8 = 0
e contenenti la retta x + x − 1 = z = 0.
Esercizio 6. Determinare il piano comune alle due sfere (x − 1)2 + y 2 + z 2 = 4,
(x − 3)2 + y 2 + z 2 = 1.
Esercizio 7. Determinare la sfera passante per il punto (0, 0, 0) e per la circon-
ferenza x2 + y 2 + z 2 − 2x − 3 = 4x − 11 = 0.
Esercizio 8. Determinare la sfera tangente al piano x−y +z = 0 nel punto (0, 0, 0)
e passante per il punto (1, 2, 0).
Esercizio 9. Determinare la circonferenza per i punti (−1, 0, 0), (0, 2, 0), (0, 0, 1).
Esercizio 10. Determinare centro e raggio della circonferenza passante per i punti
A(1, 2, 1), B(3, −1, 2) e tangente in B alla retta x + z − 5 = y − 2z + 5 = 0.

Potrebbero piacerti anche